Download as docx, pdf, or txt
Download as docx, pdf, or txt
You are on page 1of 584

ТНЕ

MATHEMATICS
ОР
INVESTMENT

ВУ

WILLIAM Ь. НАВТ, Рн.Т


АВВООИТЕ рвопввов О»
мдтнвм-АТIОВ ТО
пм•уввмтх о»
PREFACE

book provides an elementary course in the theory and the


application of annuities certain and in the mathematical aspects of
life insurance. The book is particularly adapted to the needs of
students in colleges of business administration, but it is also fitted
for study by college students of mathematics who are not
specializing in business. Annuities certain and their applications
are considered in Part I, life insurance is treated in Part Il, and •a
treatment of logarithms and of progreesions is given in Part IIL
The prerequisites for the study •of the book are three semestere of
high school algebra and an acquaintance with progressions and
logarithmic computation. Very complete preparation would be
furnished by three semesters of high school and a course in college
algebra including logarithms.
The material in the book has been thoroughly tested by the
author through the teaching of it, in mimeographed form, for two
years in claeges at the University of Minnesota. It has been aimed
in this book to present the Bubject in such a way that its beautiful
simplicity and great' usefulness will be thoroughl%
appreciated by all of the BtudentB to whom it is taught. Features
of the book which will appeal to teachers of the subject are as
follows :
1. Illustrative examples are consistently used throughout to illuminate new
theory, to illustrate new methods, and to supply models for the solution of problems by
the students.
2. Large groups of problems are supplied to illustrate
each topic, and, in addition, sets of miscellaneous problems are
given Bt the close of each important chapter, while a review Bet
is placed at the end of éaeh of the major parts of the book.
3. Flexibi.lityinthelengthofthe course is provided for; the
teacher can conveniently choose from this book either a one or
two semester, three-hour course, on account of the latitude
afforded by (a) the large number of problems, (b) the segregation
of optional methods and diffcult topics into Supplementary
Sedons whose omission does not break the continuity of the
remainder of the.
PREFACE
4. The concept of an equation of value is emphasized as
B unifying principle throughout.
5. Formulas are simplified and reduced to as small a
number as tha author considers possible, if the classical
notation of the subject is to be preserved. In part I, a
simplification is effected by the use of the interest period
instead of the year as a time unit in a final pair of formulas
for the amount and for the present value of an annuity certain.
By use of these two formulas, the present values and the amountg
of most annuities met in practice can be conveniently computed
with the aid of the standard tables. In the applications of
annuities certain, very few new formulas are introduced. The
student is called upon to recognize all usual problems involving
amortization, sinking funds, bonds, etc., as merely different
instances of a Bingle algebraic problem; that is, the finding of
one unknown quantity by the solution of one of the fundamental
pair of annuity formulas.

6. Interpolation methods are used to a


very great extent and their logical and
practical completeness is emphasized. Some
problems solved by interpolation are treated
by other methods, as well, and such optional
methods are found segregated into
Supplementary Sections.
7. Practical a9pects of the subject are
emphasized throughout.
8. Very complete tables are provided,
including five-place table of logarithms, the
values of the interest and annuity functions
for twenty-five interest rates, tables of the
most essential insurance functione, ond table
of• squares, square roots, and •reciprocals.
The tables may be obtained either bound with
the book or bound separately.
9. In the discussion of life annuities
and life insurance, the emphasis is placed on
methods and on principles rather than on
manipulative proficiency. It is aimed to give
the student a clear conception of the
mathematical foundations of the subject. No
attempt ig made to prepore the student as an
insurance actuary, but the treatment in this
book iB an excellent introduction to more
advanced courses in actuarial science.
The interest and annuity tables prepared in
connection with this book make possible the
solution of most problems accurately to cents,
if ordinary arithmetic is used. Results can be
obtained with suffcient accuracy for most
class purposes if the five-place table of
logarithms is used in the computations. If the
teacher considers it desirable to use seven-
place logarithms, the author recommends the
use of Glover's Tables from Applied
Mathematics. These excellent tables contain
the
PREFACE

values, and the seven-place logarithms of the values, of the interest


and annuity functions, a standard seven-place table of-logarithms,
and a variety of other useful tables dealing with insurance and
statistics.
The author acknowledges his indebtedness to Professor James
Glover for his permission to publish in the tables of this book
certain extracts from Glover's Table8 which were published, for the
first time, in that book.
UNIVERSITY or MINNESOTA, January 1,
1924.
CONTENTS

PART 1 — ANNUITIES CERTAIN


CHAPTER 1

SIMPLE INTEREST AND SIMPLE DISCOUNT


BECFION PAGE

1.
Definition of
interest1
2.
Simple
interest1
3.
Ordinary and exact interest 3
4. Algebr8ic problems in simple interest 5
5. Simple discount6
6. Banking use of Aimple discount 9
7. Discounting notes10

CHAPTER 11

COMPOUND INTEREST

* Supplementary
CONTENTS

8. Definition of
compound interest 14
9. Compound
interest formula15
10. Nominal and effective rates18 Il.
Interest for parts of a period20 12.
Graphical comparison of. simple and compound
interest .
13, Values of
obligations24
14.

Equations of value .27


15. Interpolation of methods 29 Logarithmic methods
32
Equated time .33
Interest, converted continuously 35
Miscellaneous problems 36
section, vii

* Supplementary
CONTENTS
CHAPTER 111

ANNUITIES CERTA-N
PAGE

19. Definitions 39
20. Special cases 41
21.42
Formulas
22.45 inthe
Annuitie
23.49 paid
Most
s
24.50 genera formula
Summaryl of annuity
s
25. Annuities due . 56
26. Deferred annuities 59

CHAPTER IV
PROBLEMS IN ANNUITIES
29. Outline of problems 67
Continuous annuities 62

28. * Computations
of high accuracy 63
Miscellaneous problems 64

30. Determination of payment 67


31. Determination of term70
32. Determination of interest rate 71
CHAPTER V
* Supplementary
CONTENTS
PAYMENT OF DEBTS BY PERIODIC INSTALLMENTS
34. Amortization of debt 78
35. Amortization of a bonded debt 81
36. Problems where the payment is known 83
37. Sinking fund method 85
38. Comparison of the amortization and the sinking fund 88
methods
Miscellaneous problems 89
39. * Funds invested with building and loan 92
associations
Retirement of loans made by building and 94
loan ossociations
33. * Diffcult cases and exact methods 74

Ivfrcellaneous problems76

section.
CHAPTER VI

DEPRECIATION, PERPETUITIES, AND CAPITALIZED COST


smcr10N PAGE 41. Depreciation, sinking fund pla,n 96
42. Straight line method98
43. Composite life .99
44. Valuation of a mine101
45. Perpetuities103
46. Capitalized cost105

* Supplementary
CONTENTS
47.* Diffcult cases under
perpetuities 108
48. * Constant percentage
method of depreciation
109
Miscellaneous problems111

CHAPTER Vll
BONDS
49. Terminology113
50. Meaning of the investment rate113
51. Purchase price of bond at. 8 given yield114
52. Changes ih book value117
53. Priöé at a given yield between interest
dates121
54. Professional practices in bond
transactions124
55. Approximate bond yields126
66. Yield on a dividend date by interpolation128
57. Special types of bonds130
58.* Yield of a bond bought between interest dates .131
Miscellaneous problems133 Review problems on Part I135

PART 11 —LIFE INSURANCE


CHAPTER vm
LIFE ANNUITIES
59. Probability 147
60. Mortality Table . 148
* Supplementary
CONTENTS
61. Formulas for probabilities of living and dying 150
aeotion.
x
62.
Mathematical

expectation 152
63. Present value of pure endowment 153
64. Whole life annuity 155
65. Commutation symbols 157
66. Temporary and deferred life annuities
CHAPTER IX
LIFE INSURANCE
68. Terminology 165

159
67. Annuities due . 162
Miscellaneous problems
163

Net single premium, whole life insurmce


166
* Supplementary
CONTENTS
70. Term insurance
168
71. Endowment insurance 170
72. Annual premiums 171
Net single premiums ag present values 175

of expectations
74.* Policies of irregular type . 176

CHAPTER X
POLICY RESERVES
•75. Policy reserves

178
76. Computation of the reserve 180

Supplementary Exercise 183

Miscellaneous Problems on Insurance 184


PART 111 —AUXILIARY SUBJECTS
CHAPTER Xl
LOGARITHMS
77. Definition of logarithms . 187

78. Properties of logarithms . 188

* Supplementary
CONTENTS
79. Common logarithms 190

80. Properties of the mantissa and the .


characteristic 191
81. Use of tables of mantissas
193
82. Logarithms of numbere with five significant 194
digits
section,

* Supplementary
CONTENTS

PAGE
83. T6 find the number when the logarithm iB
given196
84. Computation of products and of quotients197
85. Computation of powers and of roots 198
86. Problems in computation
87. Exponential equations
201
88.* Logarithms to bases different from 10
203

CHAPTER Xll
PROGRESSIONS
89. Arithmetical progresaions
90. Geometrical progressions
205
91. Infinite geometrical progressions

Note Approxi.mate determination of the time to


1. double money

* Supplementary
CONTENTS

ot compound interest 211

Note 2. Approximate determination of the equated 211


time
Note 3. Solution of equations by interpolation 212
Note 4. Abridged multiplication 213
Note 5. Accuracy of interpolation for finding the
time, under com-
pound intereat 214

Note 6. Accuracy of interpolation for finding the 214


term of an annuity
INDEX 217

section.

* Supplementary
MATHEMATICS OF INVESTMENT
PART 1 — ANNUITIES CERTAIN
CHAPTER 1
sn,fPLE INTEREST AND snaPLE DISCOUNT
1. Definition of interest. — Interest is the income received from
invested capital. The invested capital is called the principal ; at any
time bfter the investment of the principal, the sum of the principal
plus the interest due is called the amount. The interest charge is
usually stated as a rate per cent of the principal per year. . If $P iB
the principal, r the rate expressed as a decimal, and i the interest for
I year, then by definition i = Pr, or
i
(1)
36.60
Thus, if $1000 earns $36.60 interest in one year, r — = .0366, or
the 1000 rate ig 3.66%. Also, if P = $1 in equation 1, then r i, or the rate r
equals the interest on $1 for 1 year.

2. Simple interest. —If is computed on the original principal


during the whole life of 8 transaction, simple interest is being
charged. The simple interest on principal P is proportional to the
time P is invested. Thus, if the simple interest for 1 year is $1000,
the interest for 5.7 years is $5700.
Let I be the interest earned by P in t years, and let A be the
amount due at the end of t years ; since amount = principal plus
interest, A - P+l. (2) Since the interest earned by P in 1 year Pr, the
interest earned in t years iB t(Pr) or 1 = Prt. (3)
Hence P + I = P 4- Prt, so that, from equation 2,
A = PCI + rt). (4)
2 MATHEMATICS OF

1
INVESTMENT

It is important to realize that equation 4 relatæ two sums, P and A,


which are equally desirable if money can be invested at, or is worth, the
simple interegt rate r. The possession of P at any instant iB as desirable as
the possession of at a time t years later, because if P iB invested at the rate
r, it will grow to the amount A in t years. We shall call P the present value
or present worth of the amount A, due at the end of t years.
3. Ordinary and exact interest. — Simple interest iB computed by
equation 3, where t is the time in years. If the time is given in days, there
are two varieties of interest used, ordinary and exact simple interest. In
computing ordinary interest we assume one year to have 360 days, while
for exact interest we assume 365 days.
Example 1. — Find the ordinary and the exact interest at 5% on $5000
for 59 days.
Sotution. — For the ordinary intaeBt 10 use equation 3 with t = and for the exact
interest Ie use t =
10 = 5000(.05) $40.97; = 5000(.05) = $40.41.

A relation between the ordinary interest 10 and the


exact interest on a principal P for D days at the rate r.
1 —PrD,• 1, = Pr—D = L PrD. (5)
360 360 365 365
From equations 5 it iB seen that PrD = 360 10 - — 365 or
10 73
(6)
SIMPLE NTEREST AND SBIPLE DISCOUNT 3
which shows that ordinary interest iB greater than exact interest.
From equation 6,

(7)

(8)
Thug, if the exact interest I. $40.41, we obtain from equation 7,
40.41
10 40.41 + — $40.97.
72
EXERCISE 1
In the first five problems find the interest by use of equation 3.

RATE TEE INTEREST


PRINOIPAI,

1. $ 5,570. 75
2. 8,000. days
3 115,380. 3.5% 93 exact
. 4,838.70 .045 days ordinary
4 2,500. .0625 80 exact
. 7.5% days ordinary
5. .055 35 dsys exact
27
days
4 MATHEMATICS OF
6. The exact interest on a certain principal for a certa,in number
of dayg is $50.45. Find the ordinary interest for the same period of time.
7. The ordinary interest on a certain principal for certain number
of days is $35.67. Find the exact interest for the same period.
8. In problem 1 find the ordinary interest by use of the result of problem 1.

When the rate is 6%, the ordinary interest for 60 days is (.01)P, which iB
obtained by moving the decimal point in P two places to the left, while the
interest for 6 days is (.001)P. These facts are the basis of the 6% method
for computing ordinary interest at or at rates conveniently related to 6%.
EcampZi 2. — By the 6% method find the ordinary interest on $1389.20
for 88 days at 6%, and at 4.5%.
SoZulion. — $13.892 ig at 6% for 60 days
4.168 is interest at 6% for 18 days (3 times 6 days)
1.158 is interegt bt 6% for 5 days (+1 of 60 days)
$19.218 iB interegt at 6% for 83 days
4.805 is interest for 83 days at 1.5% (i of 6%)
$14.413 iB interest for 83 days at 4.5%
The extensive interest •tables used in banks make it unnecessary to perform

multiplications or divisions computing simple teregt. Table IV in this book makes it


unnecessary to perform divisions.
Example 3. —Find the exact interest at 5% on $8578 for 96
days.
Solution. — From Table IV the interest at 1% on $10,000 for 96
days is $26.3013699. The interest on$8578iB (.8578) (5) (26.3014) $112.81.
INUsT1vfENT

To find the time between two dates, it is sometimes assumed


that each month has 30 days. For example;
February 23, 1922, is 1922 : 2 : 23 or 1921 : 14:23 (9)
June 3, 1921, is 1921: 6: 3
Elapsed time — O: 8:20 260 dayg
SIMPLE NTEREST AND SBIPLE DISCOUNT 5
The enct time can be found from Table Ill. February 23, 1922, is the
54th day of 1922 or the 419th day from January 1, 1921. June 3, 1921, is
the 154th day from January 1, 1921. The elapsed time is (419 — 154) 265
days.
Norm. — In this book, for the Bake of uniformity, proceed as follows, unless
otherwise. directed, in problems involving simple interest: (a) use ordinary interest if
the time interval iB given in days; (b) in computing the number of days between dates,
find the exact number of days; (c) if the time ig given in months, reduce it to B fraction
of a year on the basis of 12 months to the without changing to days. Methods in the
business world are lacking in uniformity in these respecu, and, in any practical
application, explicit informaåon should be obtained as to the procedure to be followed.

EERCISE
Find the ordinary in the first five problems by use of the 6% method.
$3957.50, t —— 170 days, r .06.
$3957.50, t = 170 days, r = .07 "893.75, t —
53 days, r
4. P $13,468.60, t = 41 days, r = .03.
$9836.80, t = 134 days, r = .05
6. Find the exact interest in problem 4 by use of Table IV.
7. Find the ordinary interest on $8500 at 6% from August 11,
1921, to March 13, 1922. Use the approximate number of days, aa in
expression 9 above.
8. Find the ordinary interest in problem 7, but uge the exact num.
ber of days.
9. Find the ordinary and exact interest on $1750 at 5% from
April 3, 1921, to October 13, 1921, using the exact number of days.
10. (c) Find the ordinary and exact interest in problem 9, using the
approximate number of days. (b) Which of the four methoda of problems 9
and 10 is the most favprable to a creditor?
4. Algebraic problems.—lf a suffcient number of the quantities
(A, P, I, r, t) are given, the others can be determined by equations
6 MATHEMATICS OF
2, 3, and 4. When the rate r, or the time t, is unknown, equation 3
should be used. When the present value P is unknown, equation 4 is
most useful.
Ecample 1. —If a $1000 principal increases to $1250- when
invested at simple interest for 3 years, what iB the interest rate?
Solution. —P = $1000, A - $1250, t = 3. The interest 1 -$260. From r = Prt,
250 - 3000 r, or r = .0833.
Example 2. — What principal invested at 5.5% simple interest will
amount to $1150 after 2 years, 6 months?

Solution. — Use equation 4. 1150 PCI + 2.5(.055)] = 1.1375 P; P


1150
= $1010.99. An equivalent statement of Example 2 would be,

1.1375
Find the present value of $1150, due at the end of 2 years, 6 months, if money
is worth 6.6% simple interest."

EXERCISE

Find the missing quantities in the table below :


PROB RATE
. 1

1. $3500. $ 750. $15 .04 3 yr., 6 mo. 2


2. 3500.
0 .055 yr.
8. .058
2500.
.075
4. 30 2 yr., 3 mo.
.035
5. 1800.5 0 2
.06 yr.,
6. 0 65 6 mo.
.055
9850.5 0 3 yr.
8. .03
0 1680. ; 9 mo. 11 mo.
.037
9. 5 1 yr., 6 mo.
10.
SIMPLE NTEREST AND SBIPLE DISCOUNT 7
.072
5

11. Find the present value of $6000, due •after 8 months, if money is worth 9%.
12. W borrowed $360 from B and agreed to repay it at the end of
8 months, with simple interest at the rate 5.25%. What did W pay
at the end of 8 months?
8 MATHEMATICS OF INVESTMENT
13. Find the present worth of $1350, due Bt the end of 2
months, i! money is worth 5% simple interest.

14. At the end of 3 months I must pay


$1800 to B. To cancel this obligation
immediately, what should I pay B if he iB
willing to accept payment and is able to
reinvest money at 6% simple interest ?
Example 3. —A merchant is offered $50 discount for cash
payment of $1200 bill due after 60 days. If he pays cash, at what
rate may he consider big money to be earning interest for the
next 60 days?
Solution. — He would pay $1150 now in place of $1200 at the end
of 60 days. To find •the interest rate under which $1150 is the
present value of $1200, due
1150r in 60 days, use I — Prt;
50 = , or r = .26087. His money earns in6 terest Bt
the rata 26.087%; he could afford to borrow at any
smaller rate in order to be able to pay caeh.
16. A merchant is offered a $30 discount for
cash payment of a $1000 bill due at the end of
30 days. What is the largest rate at which he
could afford to borrow money in order to pay
cash?
16. A 3% discount is offered for cash
payment of a $2500 bill, due at the end of 90
days. At what rate is intereat earned over the
90 days if CBSh pgyment is made ?
17. The terms of payment for a certain
debt are: net cash in 90 days or 2% discount
for cesh in 30 days. At what rate ig interest
earned if the discount is taken advantage of ?
HINT. —For a $100 bill, $100 paid after 90 dBYB, or
$98 at the of 30 . daye, would cancel the debt.

6. Simple discount. — The process of finding the present value


P of an amount A, due at the end of t years, is called
discounting A. The difference between A and its present value P,
or A P, is called the discount on A. From A = P + r, I = A — P;
thus I, which is the intereät on P, also is the discount on A.
If $1150 is the diacounted value of $1250, due at the end of 7
months, the discount on the $1250 ig $100; the interest on
$1150 for 7 months iB $100.
SIMPLE NTEREST AND SBIPLE DISCOUNT 9
In considering the interest on a known
principal P we computed I at certain rate per
cent of P. In considering 1 8B the discount on
a hiown amount A, it iB convenient to compute
I at a certain rate per cent per year, Of A,
If iB the,diBcount on

A, due at the end of 1 year, and if d iB thediscountrateexpreaaed


a_decim212 then by definition = Ad, or
(10)

Simple discount, like simple interest, is discount which is


proportional to the time. If Ad is the discount on A, due in 1
year, the discount on an amount A due at the end of t years is
t(Ad), or
1- Adt. (11)
From P = A — I, P = A — Adt, or
(12)
If the time is given in days, we may use either exact or
ordinary simple discount, according as we use one year as equal
to 365 or to 360 days, in finding the value of t.
Non. — In simple discount problems in this book, for the sake of
uniformity, proceed follow: unless otherwise directed: (a) if the time is
given (b) in the number of days between dates, find the exact number of
days ; (c) if the time is given in months, redücüiöto a procffcü are not
uniform, a.nd hence in any practical application of discount one should
obtain explicit information to the procedure to follow.
Ecampte 1. —Find the discount rate if $340 is the present value
of $350, due after 60 days.
Solution. — From I — Adt, 10 = 350 d. d = .17143, or 17.143%.
Ecample 2. — If the discount rate iB 6%, find the present value
of $300, due at the end of 3 months.
10 MATHEMATICS OF INVESTMENT
Solution. —From I = Adt, 1 = 300(.06) = 4.50. P - A — r 300
4
— 4.50 = $295.50.
Nom. — If A is known and P is unknown, it is easier to find P when the
discount rate is given than when the interest rate is To appreciate thia fact
compare the solution of Example 2 above •with that of Example 2 of
Section 4, where a quotient had to be com#uted. This simplifying property
of discount rates ig responsible for their wide use in banking and •business.

The use of a discount rate in finding the present value P of a


known amount A ig equivalent to the use of some interest rate,
which is always different from the discount rate.
SIMPLE NTEREST AND SBIPLE DISCOUNT 11
Example 3. — (a) If a 6% discount rate iB charged in discounting
amounts due after one year, what equivalent interest rate could be
used ? (b) What would be the interest rate if amounts due after 3
months were being discounted ?
Suppose A = $100, due after 1 year. Than I = 100(.06)
= $6,.and P = $94. Let r be the equivalent rate, and use I = Prt.
6 94r; r .063830. (b) If A — $100, 3 months, 1 -_ 100(.06) 4
- $1.50, and P $98.50. From 1 Prt, 1.50 98.bOr.
4
Compare the results of Example 3. When discount rate is being
used, the equivalent interest rate is larger for long-term than for
short-term transactions and in both cases iB larger than the

The brief methods available for computing simple interest apply as


well to the computation of simple discount because both operationg
involve multiplication by a small decimal. Thug, we may use the 6%
method fer computing discount, and simple interest tables may be
used a.s simple discount tables.

EnRCISE Iv
1. Find the discount rate if the discount on $1500, due after one year,

2. Find the discount rate if the present value of $1250, due after
$1193.75.

Find the missing quantities in the table by use of equations 11 and 12.
I, DISCOüNT ON DISCOUNT
OR INT. ON P RAE, d
A 1B Dun
VALUE, P

8. $145.50 $1200 I yr. $260 .05


4. 150 6
mo. a .07
5. 2000 .046
mo.
6. 867.75 800 .06
90 do.
7. 875 120 dB.
750. 85 .08
8. 9 mo.
9. 6 mo. 100 .0626
1500
10. 72 dB. .06
7500
12 MATHEMATICS OF INVESTMENT

11. 6 mo.
12.
18. Write in words problems equivalent to problems 3, 4, 10, and 12 above.
14. (a) What simple interest rate would be equivalent to the charge made
in problem 3 ; (b) in problem 7?
16. If d = .045, (a) what is the equivalent interest rate for I-year
transaction; (b) for one whose term is 4 months ?
7/16. What discount rate would be equivalent to the use of a 6%
interest rate in a I-year transaction? HINT.—Let P = $100; find A and
I, and use I = Adt.
17. discount rate would be equivalent to the use of a 6% interest rate in a 6-
month transaction ?
SUPPLEMENTARY Non. — Formula.8 can be obtained relating the discount
rate d and the equivalent interest rate r on I-year transactions. Suppose that $1 iB
due at the end of 1 year. Then, in equations 11 and 12, A — $1, I — d, and P = 1
— d. From I = Prt with' = I, d = (1 — d)r, or

(13)
From equation 13, r — rd d, orr = d(l + r), so that

It must be remembered that equations 13 and 14 connect the discount and the interest
rates only in the case of I-year transactions.

SUPPLEMENTARY EnRCISE V
1. By use of equations 13 and 14, solve problems 14, 15; 16, and 17 of Exercise
IV.

6. Ban-king use of simple discount in lending money. — If X asks for a $1000


loan for six months from a bank B which is charg-* ing 6% discount, B will
cause X to sign a note promising to pay B $1000 at the end of 6 months. Then, B
will give X the present value of the $1000 which he has promised to pay. The
bank computes this present value by IEe of its discount rate. P 1000 — 30 =
$970, which X receives. The transaction is • equivalent to B lending X $970 for
6 months. Thö interest rate which X is paying is that which is equivalent to the
6% discourit rate. The banker would X that he ig paying 6% interest in
advance, but thig would merely be a cdlloquial manner-of stating that
the discount rote iB In this book the phrase interest in advance ig always used in
this customary colloquial sense.
SIMPLE NTEREST AND SBIPLE DISCOUNT 13
Ecample 1. — X requests a loan of $9000 for 3 months from a
bank B charging 5% discount. Find the immediate proceeds of the
loan and the interest rate Which X is paying.
Solidion. — X promises to pay $9000 at the end of 3 months. Discount on
000 for 3 months at 5% ig $112.n Immediate proceeds, which X receives, are
$9000 — $112.50 = $8887.50. Todeterminetheinterestrate, user = Prt. 112.5
8887.5 re), or r = .M)633.
Example 2. — X wishes to receive $9000 as the immediate
proceeds of a 90-day loan from a bank B which is charging 5%
interest payable in advance. For what sum will X draw the note
which he will give to B?
Solution. — P = $9000, t = 90 days, d .05, and A iB unknown. From equation 12, 9000 = A (I — .0125). A =
$9113.92, for which X will draw the note.

EnRCISE VI
Determine how much X receives from the bank B. In the first
tb.ree problems, also determine the interæt rate which X is paying.

LOAN REQUESTED BY X ron D1300üNT or B

$5,000 6 months .065


1,760 75 days .07
8. 3,570 90 dayg .06
4. 190 30 days .05
5. 7,500 45 days .055
e. 3,800 3 months .0625

PROB. Pnoanng TEEM or DIB00üNT or B

7. $ 3,500 80 days .06


8. 4 months
1,300 8 months
.05
9. 150,000 .07
60 days
10. 4,300 .055
90 days
11. 9,350 6 months
12. .05
14 MATHEMATICS OF INVESTMENT
Determine the size of the loan which X would request from B if X desired
the immediate proceeds given in the table.
7. Discounting notes. — The discounting of promissory notes gives rise to problems similar to those of
Exercise VI. Congider the following

notes :
SIMPLE NTEREST AND SBIPLE DISCOUNT 15

NOTE (a)

Minneapolis, June 1, 1922.


Sic months after date I promise to pay to Y cr crder $5000
uithout interest. Value received. Signed X.

NOTE (b)

Chicago, June 1, 1922.


One hundred and eighty days after date I promise to pay to Y
or order $5000 together with simple interest from date at the rate
7%. Value received. Signed X.

On August 1, Y sells note (a) to a bank B. The sale is


accomplished by Y indorsing the note, transferring his rights to B,
who win receive the $5000 on the maturity date. The transaction is
called discounting the note because B pays Y the present or
discounted value of $5000, due on December 1, 1922.
Eæamptz l. —If B discounts notes at 5%, what will Y receive on August 1 ?
Solution. — B is using the discount rate 5% in computing present values. The discount on
due after 4 months, is $83.33; B will pay Y $4916.67.
Eæample 2. — On July 31, Y discounts note (b) at the bank B of
Example I. What are Ys proceeds from the sale of the note?
Solution. — B first computes the mBturity value of the note, or what X will
pay on the maturity date, which iB November 28. The transaction is equivalent
to discounting th.i8 maturity value. Term of the discount is 120 days (July 31 to
November 28). From equation 4, the maturity value of the note is 5000(1
+ .035) = $5175.OO. Discount for 120 days 8t on $5175 is $86.25. Proceeds -
$5175 - $86.25 = *5088.75.
EXERCISE vr
I. X paid Y for an order of goods with the following note :

Chicago, June 1, (922.


SIMPLE NTEREST AND SBIPLE DISCOUNT 16

Sicty days after date I promise to pay to Y or order $375 at the


Continental Trust Company. Value received. Signed X.
17 MATHEMATICS OF INVESTMENT

Thirty days later, Y discounted this note at a bank charging


5.5% discount. Find Y'B proceeds from the sale of the note.
2. Find the proceeds in problem 1, if the discount rate iB 6%.

3. The bank B of problem 1, after buying the note from Y,


immediately rediscounted it at a Federal Reserve Bank i whose
rediscount rate. was .035. What did B receive for the note?
4. The holder of a non-interest-bearing note dated October
1, 1911, payable 4 months. after date, discounted it at a bank on
October 1, at the rate 4%. The bank's discount on the note wag
$20. What wag the face of the note?

5. X owes a firm Y $800, due immediately. Inpdyment X


draws a 90day non-interest-bearing note for such a Bum that, if Y
immediately discountsitat a bank charging 6% discount, the
proceeds will be $800. What ig the face value of the note?
HINT. — In equation 12, P $800 and A is unknown.
6. X draws 60-day non-interest-bearing note in payment of
a bill for $875, due now. What should be the face of the note so
that the immediate proceeds to the creditor will be $875 if he
discounts it immediately at B bank whose discount rate is 6.5%?
Find the proceeds from the sale of the following Kotes :

Dan OF Non Buna DIBO. or


Non INT.
BOLD ON BOER
Nom

6/10/17 80 days .06 6/20/17


$ 460 6/12/18 120 days .05 6/26/18
1200 90 days .07 .06
3/25/20 4/24/20 .04
8. 375 60 days .00
470
11/20/21 .06 12/ 5/21
9. 3 months .08
325 4/30/20 6 months .08 6/ 1/20
10. .06
11.2 3000 8/14/19 12/16/19
12.8 -
1
A Federol Reserve Bank disgountB commero!ßl notee brought to it by banks belonging to the Federal Reaerve
System. The rate of theFederal ReserveBank is called B rediscount rate beoause notes discounted by it have been
18 MATHEMATICS OF INVESTMENT

discounted previously, py some other bgnk. This previous discounting hag no effect BO far the computation of the
preaent value by the Federal B4n.k is oonoemed.
1
The note is due on 7/31/ 20, the læt day of the third month from April. mnd the exact number
Of dayg between 6/1/20 and 7/31/20.
Find the exact number of dayg between 12/16/19 and 2/14/20.
SIMPLE NTEREST AND SBIPLE DISCOUNT 19

13. Y owes W $5000 due now. In payment Y draws a 45-


day noninterestrbearing note, which W discounta immediately at a
bank charging 6% interest in advance. What is the face of the note
if W'8 proceeds are $5000?
14. W degires $2500 as the immediate proceeds of a 6-month loan from a bank
which charges 7% interest in advance. What loan,wi.ll W quest ?
15. A bank B used the rate 6% in discounting a 90-day note for
$1000. The note waa immediately rediscounted by B at a Federal Reserve
Bank whose rate was 4%. Find Bs profit on the transacåon,
CHAPTER 11
COMPOUND INTEREST
8. Definition of compound interest. — If, Bt
stated intervals during the term of an
investment, the interest due is added to the
principal and thereafter earns interest, the sum
by which the original principal has increased
by the end of the term of the investment is
called compound At the end of the term, the
total amount due, which consists of the original
principal plus the compound interest, is called
the compound amount.
We speak of interest being compounded, or
payable, or converted into principal. The time
between successive conversions of interest into
principal iB called the conversion period. In a
compound interest transaction we must know (a) the
conversion period and (b) the rate at which
interest iB earned during a conversion
period. Thus, if the rate is 6%, compounded
quarterly, the conversion period is 3 months and
ülterest iB earned at the rate 6% per year during each
period, or at the rate 1.5% per con-
version period.

Ezampze 1. —Find the compound amount after 1


year if $100 is in, vested at the rate 8%, compounded
quarterly.
Solution.— The rate per conversion period is .02. Oriänal
principal is $100.
At end of 3 mo. $2.000 interest is due; new principal iB
$102.000.
At end of 6 mo. $2.040 interest is due ; new principal ig $104.040.
At end of 9 mo. $2.081 intæest is due; new principal iB $106.121.
At end of I yr. $2.122 interest is due; new principal is $108.243.
The compound interest earned in 1 year is 8.248. The rate of increase of
principal per year is 8243 - .08243, or 8.243%. 100
EXERCISE vm
1. By the method of Example I find the compound amount
after 1 year if $100 is invested at the rate 6%, payable quarterly.
What wag the compound amount after 6 months? At what rate per
year does principal inareaae in this case ?
14
MATHEMATICS OF NVESTMENT
INTEREST

2. Find the annual rate of growth of


principal under the rate .04, converted
quarterly.
N0E. — Hereafter, the unqualified word intereet
will alWbYS to mmpound interegt. If a transaction
extends over more than I year, compound intereat
should be used. If the time involved is less than 1
year, simple interest generally is used.

9. The compound interest formula. —Let


the interest rate per conversion period be
r, expressed as a decimal. Let P be the
original principal and let be the compound
amount to Which P accumulates by the end of
Ic conversion periods. Then; we shall prove
that A = PCI + r)k. (15)
The method of Example 1, Section 8, applies in
establishing equation 15.
Original principal invested iB
Interest due at end of 1st period is Pr.
New principal at end of 1st period is P + Pr = PCI -f- r).
Interest due at end•of 2d period iB P(l -F r)r.
New principal at end of 2d period ig PCI + r) + PCI +
r)r = P(l +
By the end of each period, the principal on
hand at the beginning of the period has been
multiplied by (1 -l- r). Hence, by the end
of lc periods, the original principal P has
been multiplied successiye times by (1 -l-
r) or by (1 -F r) h. Therefore, the compound
amount after k periods is P(l + r) k.
If money can be invested at the rate r per
period, the sums P and A, connected by
equation 15, are equally desirable, because
COMPOUND 23
if P iB invested now it will grow to the
value A by the end of periods. We shall call
P the present value of A, due at the end of
periods.
The fundamental problems under compound interest are the
following :
(a) The accumulation problem, or the
determination of the amount A When we Imow the
principal P, the interest rate, and the time
for which P is invested. To accumulate P, means
to find the compound amount A resulting from
the investment of P.
(b) The discount problem, or the
determination of the present value P of a known
amount A, when we know the interest rate and
16
the date on which A is due. To discount A
means to find its present value P. The
discount on A is (A — P).
The accumulation problem is solved by equation 15.

Dcample 1. — Find the compound amount


after 9 years and 3 months on a principal
P = $3000, if the rate is 6%, compounded
quarterly.
Solution. —The rate per period iB r — .015; the
number of periods is — 4(9.25) = 37.
A - — 3000(1.73477663) 35204.33. (Using
Table V) The compound interest earned iB $5204.33
— $3000 = $2Ä)4.33.
Non. — If interest is converted m times per
year, •find the numba of conversion periods in n
years from the equation = mn.
To solve the discount problem we first solve equation 15 for

P, ob (16)
MATHEMATICS OF NVESTMENT
Example 2. — Find the present value of $5000,
due at the end of 4 years a,nd 6 months, if money
earns 4%, converted semi-annually.
Solution. —Rateperperiodisr = .02; number of
periods is k = 2(4.5) = 9. P 5000(1.02)-•
5000(.83676527) $4183.78. Table VI) The discount
on ig "000 — $418.78 $816.22.
N0E. —Recognize that Example 2 involvea the
formation of a product when solved by Table VI. A
problem iB solved incorrectly if available tablea
are not used to aimplify the work. Since products
are to compute than quotients, the following
solution of Example 2 should be considered
mcorrect, although mathematically flawless,
because 8 quotient is computed.
5000
- 84183.78. (UAingTab1e V)
(1.02)9 1.19509257

In describing interest rates in the


future, a standard abbreviation will be
used. When we state the rate to be (.05, m
— 2), the " m = 2 " signifies that
interest iB compounded twice per year, or
semi-annually. The rate (.08, m = 1) means
8%, compounded annually; (.07, m 12) means
7%, converted monthly; (.06, m = 4) means
6%, compounded quarterly.
Non. —The quantity (I + r) in equation 15 iB
sometimæ caned the accumuladon factor, while the
quantity 1 or (I + in equation 16 ig
called the discount factor. In many books the

letter v is used to denote the discount factor, or v (I +


r)-1. Thus, at the rate 7%, v4 — (1.07) *
COMPOUND 25
NTEREST
EXERCISE IX
4
I. By use of the binomial theorem verify all digits of the entry for (1.02) in
Table V.
2. In Table VI verify all digits of the entry for by using the
entry for (1.02) d in Table V and by completing the long division
involved.
1 1
— (1.02)-6 - (1.02)' ¯ 1.12616242'
3. Find the compound amount on $3,000,000 after 16 years
a.nd 3 months, if the rate is (.06, m = 4).
4. Accumulate o $40,000 principal for 15 years under the rate (.05, m =
4). What compound interest is eamed ?
6. Find the present value of $6000, due after 4} years, if money can
earn interest at the rate (.08, m = 4). What ig the discount on the
$6000?
6. Discount $5000 for 19 years and 6 months, at the rate t.05, m = 2).
In the table below, find that quantity, P or A, which is not given. In the first four problems,
before doing the numerical work, write equivalent problems in words.

PRINOIP&L, AMOUNT,
PR0B. OR Dun RAE

$4000 5 yr., 6 mo.


8. $1000. .04,
10 yr., 3 mo.
9. 12 yr. .07,
10. 8 yr.,.6 mo. .06,
11. 2500. 13 yr., 9 mo. .03,
MATHEMATICS OF NVESTMENT
12. 576.50 1500 7 yr., 6 mo.
18. 1398.50 3 yr., 6 mo.
14. 15 yr., 3 mo.
15. 8300 14 yr., 6 mo.
1300. 9500 5 yr.
16. 1. 2 yr., 9 mo.
17.
18. 75 yr.
100. 100 100 yr.
19.
20. 100 175 yr.
1 1. 173 yr.
21. 1 30 int. periods
1 35 int. periods
22.
as.
.08,
.06,
.06, 12
.05,
:055,
.045,
.03,
.05,
.035,
.045,
.065,
.04, per period
.05, per period
i
In problem 20 In problem 21
COMPOUND 27
18
24. (a) If the rate is i, compounded annually, and if the original
principal is P, derive the formula for the compound amount after 10
years. (b) After n years.
25. If $100 hod been invested in the year 1800 A.D. at the rate .03, m = 1),
what would be the compound amount now?
26. (a) If the rate is j, compounded m times per year, derive a
formula for the compound amount of a principal P after 10 years.
S
(b) After n years.
10. Nominal and effective rates. — Under a given type of compound interest _the rate per year
at which principal grows is

called the rate. The fér cen-t• quoted in -toting a type Of compound
interest is called the nominal rate ; it is the rate per 'f' year it which
money earns interest during a conversion period. F In the illustrative
Example 1 of Section 8 it was seen that, when !ate.. was 8%,
converted quarterly, the effective rate

8.24%. We shall say thé rote is (j, m) to abbreviate the nominal


rate is j, converted m times per year." Let i represent the effective
rate.
The effective rate corresponding to the nominal rate j, converted m
times per year, gati*fies the equation

(17)
To prove this, consider investing P = $1 for 1 year at the rate
(j, m). The rate per period is and the number of periods in
1 year is m. The amount A after 1 year and the interest I
earned in that time are

1
The rate of increase of principal per year iB i = = r, because
MATHEMATICS OF NVESTMENT
P = $1. Hence
— 1. (18)
Transpose the 1 in equation 18 and equation 17 iB obtained.
Example 1. — What ig the affecffve rate corresponding to the rate (.05, m =
4) ? i/
n.(TERDST

Solution. — Use equation 17. I i = (1.0125)' = 1.05094534.

Example 2. — What nominal rate, if converted 4 times per year, will


yield the effective rate 6%?

Solution. From equation 17, 1.06

1 + i = (1.06* = 1.01467385, from Table X. (19)


j = 4(.01467385) - .05869540.
Table XI furnishes an easier solution. From equation 19,

— — 1; j = — 1] - .05869538. (Table Xl)

Example 3. What nominal rate, converted quarterly, will give the same
yield as (.05, m = 2) ?
5Zution. — Let j be the unknown nominal rate. The effective rate i cor-

ræponding to (.05, m = 2) must equal the effective rote corræponding to the nominal rate
j, compounded quarterly. From equation 17,

. (1.025)' = (1 + 204
1 + ä (1.025)} - 1.01242284. j = 4(.01242284) = .04969136.
EXERCISE X

l. (a) In Table X verify the entry for (1.05)} by use of Table 11.
(b) In Table Xlverifytheentryforp = 4 andi = .05, by using Table X.
2. Find the effective rates corresponding to the rates (.06, m = 2) and
(.06, m = 4).
3. Find the nominal rate which, if converted semi-annually, yields the effective
rate .05. (a) Solve by Table X. (b) Read the result out of Table Xl.
COMPOUND 29
Solve the problems in the table orally by the aid of Tables V and XI. State equiva,lent
problems in words.
PROB.

4.
5. .04
6. .0275
7. .025
8. .06
9.
MATHEMATICS OF NVESTMENT

16. Derive a formula for the nominal rate which, if converted p


times per year, gives the effective rate i. N0TD. — The resulting value
of j is denoted by the notation jp, 88 in Table XI.
17. Which interest rate is the better, 5% compounded monthly
or
5.5% compounded semi-annually?
18. Which rate is the better, (.062, m — 1) or (.06, m = 2)?

19. Determine the nominal rate which, if converted semi-annually, V/


may be used in place of the rate 5%, compounded quarterly. S63
20. What nominal rate compounded quarterly could equitably
replace the rate (.04, m = 2)?
Non. — When interest iB compounded annually, the nominal and the
effective rates are equal because in this case both represent the rate of increase
of principal per year. This equality is Been also by placing m = 1 in equation
17. Hence, to that money 18 worth the effective rate 5% is equivalent to saying
that money 18 worth •the nominal rate 5%, compounded annuaUy.

If m, the number of conversion periods per year of. B nominal


rate, is increased, the corresponding effective rate is also increased.
If j = .06, the effective rates i for different values of m are:
2 4 12 52 365
1
.06000 .06090 .06136 .06168 .0618 .06183
0
We could consider interest converted every day or every moment or
every second, or, as a limiting cage, converted continuously (m =
infinity). The more frequent the conversions, the more just is the interest
method from the standpoint of a lender, so that interest, converted
MATHEMATICS OF NVESTMENT

continuously, iB theoretically the most ideal. The effective rate does not
increase enormously as we increase the frequency of compounding.
When j = .06, in the extreme case of continuous conversion (see Section
18, below), i = .06184, only slightly in excess of .06183, which ig
the•effective rate when m = 365.
11. hterest for fractional parts of a period. —In deriving equation 15
we assumed to be an integer. Let us agree ag a new
32 MATHEMATICS OF INVESTMENT
INTEREST

definition that the compound amount A


shalLbe_giypn by equation 15 also¯WhZn lc is-
nTt- an integer.
Example 1. — Accumulate $1000 for 2 years and 2 months,
at the rate

/ I Solution. —The rate per - .02, and E - 4(29 = 8}.


A = — = 1000( . .02)} log' 1.02 - 0.006734.
log 1171.66 — 3.068801.
log A = 3.074536; = *1187.23.
Example 2. — Find the present volue of $3500, d e at the end of
2 years and 10 months, if tnoney is worth (.07, m = 2).
Solution. — The rate per period is T .035, and = 5}.

P - = $2880.09.i (Tables VI andX) The


methods of Examples 1 and 2 are complicated
unless Ic is a, convenient number. Approximate
practical methods are described below.
Rule 1. — To find the compound amount after periods
when is not an integer, (a) compute the compound
amount after the largest number of whole conversion
periods contained in the given time. (b) Accumulate
this amount for the remaining time at simple
interest at the given nominal rate.
Ecampte 3. — Find the a,mount in Example 1 by use of Rule 1.
Solution. — Compound amount after 2 years is = $1171.66. Simple
interest at the rate 8% for 2 months on $1171.66 ig $15.62. Amount at end of 2 years
and. 2 months iB 1171.66 + 15.62 = $1187.28, slightly greater than the regult of
Example 1. The use of Rule 1 is always slightly in favor of the creditor.

Rule 2. — To find the present value of A,


due at the end of periods, when Jc iB not an
integer, (a) discount the amount A for the
smallest number of whole periods containing the
given time. This gives the discounted volue of
A at a time before the present.
1
If five-place logarithms are used in multiplications or divisions, the
will be accurate to only four significant figures. Hence, in Example 2, if we desire
P accurately to cents, ordinary multiplioaåon must be used (unless logarithm tables
With even or more plaoeg are available). In performing the ordinaw multiplica— tion,
COMPOUND 33

ih finding P in Example 2, the student ig advised to uge the abridged method


described in the Appendix, Note 4.
(b) Accumulate this result up to the present time at simple interest at
the given nominal rate.
Ecampte 4. — Solve Example 2 by use of Rule 2.
Solution. — The smallest number of conversion periods
containing 2 yean and 10 months is 6 periods, or 3
years. Diacounted value of $3600, 3 years before
due, or 2 months before the present, is =
$2847.25. Simple interat on $2847.25 at 7% for 2 months
is $33.22. Present value iB 2847.25 + 33.22 — $2880.47,
which is greater than the result of Example 2. Results
computed by use of Rule 2 are always slightly larger
than the true present values as found from equation 16.

Norm. — Unless otherwise directed, use the methods of Examples


1 and 2 when Vis not am integer. Compute the time between dates
approximately, as in expreæion 9, of Chapter I, and reduce to
years on the basis of 360 days to the year.
EXERCISE Xl
Find P or A, whichever is not given. Use Table X whenever
possible.
AMOUNT, P ACCUMULATES FOR, INÆRBBT
OR IS Dan RAE
VALUE, P

$2000 3 years, 3 mo. 5


1. years, 1 mo.
2. 16 years, 8 mo.
8000 13 yecrs, .07, m =
8. 4000 7 mo. Il
4. 1000 4
years, 5 .05,
5. 1000 mo. 6
6. 5000 years, 4
7. 1500 mo.
8. 1500 7 years, 10
mo.
7 years, 10 mo.
m
=

9. Find the amount in problem 1 by use of Rule 1.


34 MATHEMATICS OF INVESTMENT
10. Find the present value in problem 5 by use of Rule 2.

11. Find the amount in problem 4 by use of Rule 1.


12. Find the present value in problem 7 by use of Rule 2.

18. On June 1, 1920, X borrowe $2000 from Y and. agrees to


pay the compound amount on whatever date he settles his
account. By use of Rule 1, determine what X shouupay on
August 1, 1922, if interest iB at the rate 6%, compounded
quarterly.+
14, At the end of 5 years and 3 monthB, $10,000 is due. Discount it
to the present time if money is worth (.05, m = 2). Use Rule 2.

12. Graphical comparison of simple and


compound
interest. — In
Figure 1 the
straight line EF
is the graph of
the equation
1 .06 t,
the amount after
t years if $1 is
invested at
simple intereet
at the rate .06.
The curved line
EH is the graph
of the equation
A = (1.06) t,
the amount after t years if $1 is
invested at the rate .06 compounded
annually. This curve was sketched through
the points correspondi_ng to the
following table of values : FIG. 1
COMPOUND 35
1 - 1.0296 1.06 1.124 1.338 1.791
1.0147
1 2 5 10
The entries for t = and t = are from Table X. In
Figure 2, that part of the curves for which t = O
to t = 1 has been magnified (and distorted
vertically, for
A—I.06 emphasis). Figure 2
shows that, when the
time is less than one
conversion period, the
amount at simple
interest is greater than the amount at compound
interFIG. 2 est. The two amounts are the same when
t = 1, and, thereafter, the compound amount rapidly
grows greater than the amount at simple interest.
The ratio, (compound amount) + (amount at simple
interest), approaches infinity as t approaches
infinity.
36 OF INVESTMENT

EXERCISE
1. (a) Draw graphs on the same coördinate system, of the amount at simple
interest, rate 5%, and of the amount at (.05, m = 1), for a principal of $1, from t
= O-to t = 10 years. (b) Draw a second graph of that part of the curves for
which t = 0 to t — 1 with your original scales magnified 10 times.

13. Values of obligations. — A financial obligation is a promise


to pay, or, an obligation is equivalent to promissory note. Consider
the following obligations or notes :
(a) Three years and 9 months after date, X promises to pay
$1000 to Y or order.
(b) Three years and 9 months after date, X promises to pay $1000
together with an accumulated interest at the rate 6%, corn— pounded
quarterly, to Y or order.
Ecample 1. — One year after date of note (a), what does Y
receive on discounting it with a banker B to whom money is worth
(.05, m = 4) ?
Solution. B pays the present value of $1000, due after 2 years and 9 months,
or = $872.28.
Ecample 2. — One year after date of note (b), what is its value to
a man W to whom money is worth (.07, m = 4) ?
Solution. — Mbturity value Of obligation (b)
is Ite value to W, 2 yeors and 9 months before
due, is = $1033.03.

Under a stipulated rate of interest, the value of an obligation, n


years after' its maturity date, iB the compound amount which would
be on hand if the maturity value had been invested for n years at the
stipulated interest rate.
Ecample 3. — Note (b) was not paid when due. What should X pay at the
end of 5 years to cancel the obligation if money is worth (.07, m 4)

Solution. Maturity value of note iB Value at the


rate (.07, m = 4) to be paid by X, 1 year und 3 months after maturity date, is
$1363.52.
37 OF INVESTMENT

N0TE. — In all succeeding problems in compound intereat, reckon elapsed time between
dates apßroximmtely, in expression 9 of Chapter I. If it is stated that a -sum

due on a cerwa dB-


tea the gum understood to be due
COMPOUND NTERDST

( 25 )
tDithout interest. If B sum is with accumulated interest, this fact will be mentioned explicitly.
EXERCISE
1. If money iB worth (.07, m — 2) to W, what would he
pay to Y for note (a), above, 3 months after date of the note?
2. If money is worth (.06, m = 2) to W, what should he pay
to Y for note (b), above, 3 months after date of the note?
3. X borrows $1500 from Y and gives him the following note :
BOSTON, July 15, 1922.
Three years and 6 months after date, I promise to pay to Y or order at the
Firgt Nationål Bank, $1500 together with accumulated interest at the rate
(.07, m = 2)
Value received. Signed X.

On January 15, 1923, what does Y receive on selling this note to bank which
uses the rate (.06, m = 2) in discounting?
4. What would Y receive for the note in problem 3 if he
discounted it on July 15, 1922, at a bank using the rate (.055, m =
2) ?
6. X owes $300, due with accumulated interest at therate (.04, m = 4) at
the end of 5 years-and 3 months. What iB the value of this obligation two
yean before it iB due to a man to whom money iB worth (.06, m = I) ?
6. At the end of 4} years, 87000 is due, together with accumulated
interest at. the rate (.045, m = 2). (a) Find the value of this obligation
2} years before it ig due if money is worth (.05, m = 2). (b) What iB
its value then under the rate (.045, m = 2) ?
7. On May 15, 1918, $10,000 was borrowed. It wag to be repaid on August 15,
1921, with accumulated interest at the rate (.08, m = 4). No payment wag made
until August 15, 1923. What was due then if money was considered worth (.07, m =
2) after August 15, 1921 ?S/4
COMPOUND NTERDST

8: On May 15, 1922, what was the volueof the obligation of problem 7 if
money was worth (.07, m = 4) after August 15, 1921 ?
9. Find the value of the obligation of problem 7 on November 15, 1923, if money
is worth (.05, m = 4), commencing on August 15, 1921.
10. X Y (a) $2000, due in 2 years, and (b) $1000, due in 3} years with
accumulated interest at the rate (.05, m = 2). At the end of one year what should
X pay to cancel the obligations if money is worth (.04, m 2) to Y?
I-IINT. —X should pay the.sum of the values of big obligbtions.
40 MATHEMATICS OF

n.rvEsT1v1ENT

11. At the endof3yearsand3months, $10,000isduewith


accumulated interest at (.05, m = 4). (a) What is the value of this
obligation at the end of 5 years if money is worth (.07, m = 4)? (b)
What is its value then if money is worth (.04, m = 4) ?
12. The note of problem 3 is sold by Y on October 15, 1924, to a
banker to whom money is worth 6%, effective. By use of Rule 2 of Scction 11
find the amount the banker will pay.

The valu•e of an obligation depends on when it is due. Hence, to


compare two obligations, due on different dates, the values of the
obligafions must be compared on some common date.
EcampZe 4. —If money iB worth (.05, m = 1), which is the more
valuable obligation, (a) $1200 due at the end of 2 years, or (b)
$1000 due at the end of 4 years with accumulated interest at (.06, m
= 2) ?
Solution. — Compare values at the end of 4 yearg under thc rate (.05, m =
1). 'Ihe value of (a) after 4 years iB - $138.00. Tho value of O after
4 years is = $1266.77. Hence, (a) is tho moro valuable.
Nom. —The value of an obligation on any date, the present for example, is
the sum of money which if possegaed to-day iB as desirable as the payment
promised in the obligation, If the present values of two obligations are the
same, their values at any future time must likewise be equal, because theso
future values are the compound amounts of the two equal present values.
Similarly, if the present values are equal, the valueg at any previous date must
have been equal, because these former values would be the results obtained on
discounting the two equal present valuee to the previous date. Hence, any
comparison date may be used in comparing the values of two obligations,
because if their values are equal on one date they are equal on all other dates,
both past and future. If the value of one obligation ia greater than that of
another on one date, it will be the greater on all datæ. For instance, in Example
4 above, on comparing values at the end of 3 years, the value of (a) ig
1200(1.05) - *1260.00; the value of (b) is - 31206.45.
Hence, in the original solution, (u) is seen to be tho more valuable. The
comparison date Bhould be selected BO aa to minimize the computation
required. the original solution of Example 4 was the moat desirable.
41 MATHEMATICS OF

EXERCISE

1. If money iB worth (.04, m = 2), which obligation is the more


valuable : (a) $1400 due after 2 years, or (b) $1500 due after 3 years?
2. If money is worth (.05, m = 2), which obligation is the
more valuable : (a) $1400 due after 5 yem, or (b) $1,000 due after 4
years with
COMPOUND INTEREST 42

accumulated interest at (.07, m = 2)? Use 4


years from now aa the comparison date.
8. Solve problem 2 with 6 years from now as the comparison
date.
4. If money is worth (.06, m = 2), compare the
value of (a) $5000 due after 4 years with (b) an
obligation to pay $4000 after 3 yean with
accumulated interest at (.05, m = 1).
5. Compare the set of obligations (a)
with set (b) if money is worth (.06, m = 2) :
(a) $1600 due after 3 •years; $1000 due after 2 years
with accumulated interest at the rate (.04, m = 2).
(b) $1200 due after 2 years; $1400 due after 2} years

with accumulated interest at the rate (.05, m = 2).


6. Which obligation is the more valuable if money
is worth (.06, m = 4) : (a) $8000 due after 3* years
with accumulated interest at (.05, m = 4), or (b)
$8500 due after 3 years?
14. Equations of value. — An equadon of value is an equation
stating that the sum of the values, on a certain comparison
date, of one set of obligations equals the sum of the values on
this date of another set. Equations of value are the most
powerful tools available for solving problems throughout the
mathematics of investment.

Norm — In writing an equation of value, the


comparison date must be mendoned, and every
term in the equation must represent the value of
some obligation on this date. To avoid urors, make
preliminary list* of the sets of obligations being
compared.
EcampZe 1. —W owes Y (a) $1000 due after 10 years,
(b) $2000 due after 5 years with accumulated
interest at (.05, m = 2), and (c) $3000 due after 4
COMPOUND INTEREST 43

years with accimulated interest at (.04, m — 1). W


wishes to pay in full by making two equal payments
at the ends of the 3d and 4th years. If money is
worth (.06, m = 2) to Y, find the Bize of W's
payments. Sdution. —Let $$ be the payment. to
replace his old obligations by two new ones. Let 4
years from now be the comparison date.
OLD OBLIGATIONB Ngw OBLIGATIONB

$1000 due in 10 years. $c due in 3 years.


$$ due in 4 years.
(b)
due in 5
yeaas. due in
4 years.
44 MATHEMATICS OF INVESTMENT

In the following equation of value the left member iB the sum of the values of
the old obligations on the comparison d%te. This sum must equal the sum of the
values of the new obligations given in the right member.

+ z. (20)
If 5 years from the præent were used aa the comparison dote, the equation
would be
(21)
from which, of course, the same value of c is obtained because equation 21
could be obtained by multiplying both Bides of equation 20 by (1.03) 2. All
obligations were accumulated for one more year in writing. equation 21 as
compared with equation 20.
EnRCISE XV
Solve each problem by writing an equation of value. List the
obligations being compared.
1. W owes Y $1000 due after 4 years and $2000 due after 3 yeare and 3 months.
What sum paid now will discharge these debts if money is worth (.08, m = 4) to Y?
2. W desires to discharge his obligations in problem 1 by
two equal made at the ends of 1 year and of 1 year and 6
months, respectivelj. Find the payments if money ig worth (.06, m =
4) to Y.
3. W desires to pay hig obligatio»s in problem 1 by three
equal paymentg made after 1, 2, and 3 years. Find the payments if
money ig worth (.06, m 4) Y.
4. What payment made at the end of 2 years will discharge
the following obligations if money is worth (.05, m = 2) : (a)
$10,000 due after 4 years, and (b) $2000 due after 3* years with
accumulated interest it (.07, m = 2)?
6. If money is worth (.06, m = 2), determine the size of the equal pay-
menu which, if made at the ends of the 1st and 2d years, will
discharge the obligations of problem 4.
e. What sum, paid at the end of 2 years, will complete payment of the obligations of problem
4 if twice that sum was previously paid at the end pf the first year? Money is worth (.08, m = 2).
45 MATHEMATICS OF INVESTMENT

7. W owed Y $1000 due after 3 years, and $3000 due with


accumulated interest at (.06, m 2) after years. W , paid $1500 after 2
years.
46 MATHEMATICS OF INVESTMENT
8. A man, owing the obligations (a) a,nd (b) of problem 4, paid $8000 at
the end of 3 years. What Bingle additional payment should he make at the end of 5
years to cancel his obligations if money is worth (.04, m = 2) to his creditor?
9. Determine whether it would be to the creditor's advantage in
problem 8 to stipulate that money is worth (.05, m = 2) to him.
15. Interpolation methods. — The usual problem in compound
interest, where the rate or the time is the only unknown quantity,
may be solved approximately by interpolating in Table V. The
method is the same as that used in finding a number N from a
logarithm table when log N is known.
EcampZe I. — Find the nominal rate under which $2350 will accumulate to
$3500 by the end of 4 years and 9 months, if interest is compounded quarterly.

Solutioh. — Let r be the unknown rate per period. The nominal rate wül be 4
r. From equation 15,

3500
3500 - 2350(1 + ,— = 1.4894.
2350
The first and third entries in, the table are from the row
in Table V for n = 19. In finding r by interpolation we
assume that is the same proportion of the way from .02
.02 1.4568 to .0225 1.4894 is of the way from 1.4568 to
1.4894 1.5262. Since 1.5262 — 1.4568 = .0694, and 1.4894 —
.022 1.5262 1.4568 = .0326, then 1.4894 is of the way from
6

1.4568 to 1.5262.The distance from .02 to .0225 is .0225 — .02 = .0025. =


Hence r = .02 + .0212. The nominal rate is 4 r .089.
Interest rates per period determined as above are usually in error by
not more thon l k. of the difference between the table rates used. Thus,
the value of r above is probably in error by not more than or
about .0001. The error happens to be much less, because a solution by
exact methods gives r = .02119. Re8ults obtained by interpolation
should be computed to one more than the numba of decimal places
which are expected to be accurate.
COMPOUND INTEREST 47
Non. — When interpolating, it ig suffcient to use only four placæ Of the in
Table V. Use of more places does not the accuracy of the final results and
causes unnecessary computation.

The author gives no theoretical justificatiön for this statement. He verified its truth for. numerous

examples distributed over the complete range of

Table V. 332 13841

( (g) N 24 156
Dcample 2. —How long will it take $050 to accumulate to
$7375 if invested at (.06, m = 4) ?
Solution. — Let k be the necessary number of interest periods.
7375
7875 (1.015) - — - 1.4048.
5250
The and third entries in the table are from Table V.
(1.015)N
Since 1.4084 - 1.3876 - .0208, and 1.4048 — 1.3876
= .0172,. then k ig of the way from 22 to 23, or
1.3876 = 22 + = 22.83 periods of 3 months. The timo

23 1.4084
2.83 obtained as above
is = 5.71 years. A value of 4 ig in error by not more than df the interest rate 1 per
period. The error in Example 2 is much less, because an exact solution of the problom
gives = 22.831.

Example 3. — X owes Y $1000 due after 1 year, and $2000 due after 3
years with accumulated interest at (.05, m = 2). When would the payment
of $4000 balance Ys account if money iB worth (.06, m 4) to Y?
Solution. be the number of convergion periods of Q,he rate (.06, m = 4) between
the present and the date when $4000 should be paid. With the present as comparison
date, the equation of value for the obligations iB 4000(1.015)?

— 2882.09.
From interpolation in Table VI, k — 22 + = 22.02. X should pay $4000
22.02
after = 5.50 years.
4
48 MATHEMATICS OF INVESTMENT
EcampZe 4. — How long will it take for money to double itself if left
to accumulate at (.06, m = 2) ?
Solution. —Let P — $1 and A = $2. If k represents the neceBEary number of conversion periods, a solution by interpolation
gives 23.44; the time iB 11.72 years. Another approximate method iB furnished by the following rule.

Rule 1.2 — To determine the time necessary for money to double itself at compound interest : (a)
Divide .693 by the rate per period. (b) Add .35 to this result. The sum is the time in conversion periods.
The error of this approximate result generally ig less than few hundredths of i period.
On solving Example 4 by this rule, —.693 + .36 = 23.45. .03

For justification of thi8 staterqent see A.ppendix, Nota 5. A knowledge of the calculue is neoeeery in recd.ing this note.
* For proof of this rule see Appendix, Note 1.
COMPOUND 49

)
EÆRCISE XVI I

Solve all problems by interpolation unless


otherwise directed. In each problem in the table,
find the missing quantity.

PROB. AMOUNT, PRINCIPAL, P Aoaou-ul..ærns FOR, OR


NOMINAL
CONVERSIONS
A 1B DOE PEB Yma.B

1. $2735 $1500 1
2. 2500 2000 .05 2
3. 2 1 .06
1
4. 1000 750
5. 5010 4
4270 .07 2
6. 6575 15 years
7. 3000 1000 3 years, 9 mo. .05 2
7 years, 6 mo. 2
0 8. Find the nominal rate under which 53500 •ig
the present value of $5000, due •at the end of
years. Interest is compounded semiually.
9. How lon•g will it take for money to quadruple
itself if invested at (.06, m = 2)? 2 3.
10. (a) At what nominal rate compounded annually
will money double in 14 years? (b) Solve by use of
Rule 1.
Il. If money is worth (.07, m = 1), when will the
payment of $4000 cancel the obligations (a) $2000
due after 3 years, bnd (b) $2000 due after 7 years?
12. If money is worth (.05, m = 2), when will the
payment of $3000 cancel the obligations (a) $1500
COMPOUND 50

due after 3 years, and (b) $1000 due at the end of


2} years with accumulated interest at (.06, m = 4) ?
18. By use of Rule 1, determine how long it take:
for money to double itself under each of the
following rates: (a) (.06, m = 4) ; (b) (.04, m = 2)
; (c) (.05, m 2) ; (d) (.03, m 1).
14. By use of the result* of problem 13, determine
how long it takes for money to quadruple itself under
each of the four rntes in problem 13.
16. If money ig worth (.04, m = 2), when wül the
payment of $3500 cancel the liabilities (a) $1000
due after 18 months, and (b) $2000 due after years?
The Miscellaneous Problems at the end of the chapter mgy be taken up immediately
afur the completion of Exercise XVI.
COMPOUND NTERDST
SUPPLEMENTARY MATERIAL

16. Logarithmic meüaods. — Problems may


arise to which the tbbles hand do not apply,
or in which more accuracy is desired than is
obtainable by interpolation methods.
Logarithmic methods are available in such
cases.
EcampZe 1. — Find the present value of $350.75, due at the end
of 6 years and 6 months, if interest is at the rate (.0374, m =
2).
350.75
P = 350.75(1.0187)-" - •
¯ (1.0187)
la
log 350.76 = 2.54500
13 log 1.0187 — 13(.0080463) - 0.10460 (Using Table
11) (subtract) log P 2.44040. P = $275.68.
If Table I woe used in obtaining log (1.0187) B, 13
log 1.0187 13(.00804) = 0.10452, in error by ..00008.

Ecampte 2. — If interest is converted


quarterly, find the nominal rate under which
$2350 iB the present value of $2750J due
after 4 years and 9 months.
Solution. — 12t r be the unknown rate per period;
the nominal rate is 4 r.

2750 2350(1 1 + r (2750\+


kääBö)
log 2750 — 2.43938 log 2350 =
2.37107 log quot. — 0.06826.
log quot. = 0.00369.
I r — 1.0083, r = 0083. The nominal rate is 4 r
= .0332, converted quarterly.
Example 3, — How long will it take for $3500
to accumulate to $4708 if iB at the rate
(.08, m = 4)?
52 MATHEMATICS OF INVESTMENT
Solution. — Let be the necessary number of conversion periods.
4708
4708 - — • k log 1.02 =
log— 3500• 3500'
log 4708 3.67284 log 1.02 = 0
0086002 log 3500 - 8.54407 log guot.
= 0.12877. — .12877. 12877 log 12877
— 4.10982
860.02' log 860.02 — 2.93451
(subtract) log = 1.17631
The time ig 14.973 periods of 3 months, or 3.743 years,
EXERCISE XVII
Use exact logarithmic methods in all problems
on this page. Table Il whenever advisable.
I. Find the compound amount after 3 years and 8 months, if $3500
is invested at the rate (.063, m = 4).
2. Find the present value of $3600 which is
due at the end of 8 years and 6 months, if money
is worth (.078, m = 2).
9. At what nominal rate, converted quarterly,
is $5000 the present value of $7300, due at the
end of 2 years and 9 months?
4. Find the length of time necessary for
principal of $2000 to aecumulate to $3600, if
interest is bt the rate (.05, m = 1).
6. Solve problems 2 and 5 of Exercise XVI by exact methods.
6. Solve problems 3 and 4 of Exercise XVI by exact methods.

7. Fidd the nominal rate which, if converted semi-annually,


yields the effective rate .0725. q
8. Find the nominal rate which, if
coliverted semi-annually, is equivalent to the
rate .068, compounded quarterly.
9. (a) Determine how long it will take for money
to double itself at the rate (.06, m = 1). (b) Compare
your answer with the result you obtain on using Rule 1
of Section 15.
10. One dollar ig allowed to accumulate at (.03, m
= 2). A second dollar accumulates at (.06, m = 1). When
COMPOUND NTERDST
will the compound amount on the second dollar be three
times thBt on the first?
HINT. — Take the logarithm of both ädes of the equation
obtained.

17. The equated equated date for a


set of obligations is the date on which they
could be discharged by a single payment equal to
the sum of the maturity values of the
obligations. The time between the present and
the equated date is called the equated time, and
it is found by solving an equation of value.
Example 1. If money is worth (.05, m = 2), find the
equated time for the payment of the obligations (a) $2000 due
after 3 years, and (b) $1000 due a.fter 2 years with
accumulated interest at the rate

—The sum of the maturity values of (a) and (b) iB

2000
+ 1000(1.02)' — $3082.48. Let the equated time be conversion periods of
tberate (.05, m 2). The value of the obligation $3082.43, due after h periods (on
the equated date), must be equal to the sum of the values of the given obligations.
With the present the comparison date, the corresponding equation of value is
= 2705.2.

3082 4
(1.025)E = — 1.1394.
2705.2

By the method Of Section 16, k = 5.286 six-month periodB or, the equated time is
2.643 years. By the interpolation method, — 5.28. The present wag used as the
comparison date above to avoid having k appear on both Bides of the equation.

To obtain the equated time approximately, the following


rule is usually used.
Rule 1.1 — Multiply the maturity value of each obligation by the
time i.n years (or months, or days) to elapse before it is due. Add
54 MATHEMATICS OF INVESTMENT
these products and divide by the sum of the maturity values to obtain
the equated time.
On using this rule in Example 1 above, we obtain
equated time = 3(2000) 4- 2(1082 4) — 2.65 years.
3082.4
Rule 1 iB always used in finding the equated date for short-term
commercial accounts. The equated date for an account is also called
the average date and the process of finding the average date ig called
averaging the account. Since Rule 1 does not involve the interest rate,
it is unnecessary to state the rate when asking for the equated date far
an account.
NOTE. — Results obtained by use of Rule 1 ore always a little too large, so
that a debtor is favored by its use. The accuracy of the rule is greater when the
interest rate i: low than when it is high. The accuracy is greater for shortterm than
for long-term obligations.

EnRClSE xvm
1. If money iB worth (.05, m — 1), find the equated time for the payment of @
$1000 due after 3 years, and (b) $2000 due after 4 years. Solve by Rule 1.

2. Solve problem 1 by the exact method of Example 1 above.

i
For derivation of the rule see Appendix. 2.
COMPOUND NTERDST

6)
9. (I) If money is worth (.07, m = 2), find the equated time
for the payment of (a) $1000 due after 3 years and (b) $2000
due after 4 years with accumulated interest at (.05, m = 2).
Solve by the exact method. (Il) Solve by Rule 1.

4. Find the equated tilne for an account requiring the


payment of $55 ; after 3 months, $170 after 9 months, and
$135 after 7 months. Use Rule 1. i
6. (a) Aman owes four 180-day, non-interest-
bearing notes dated 0B follows: March 9, for
$400; May 24, for $250; August 13, for $525;
Åugust 80, for $500. By use of Rule 1 find the
equated time and the equated date for the
payment of the notes, considering for
convenience that March 9 is the present. (b)
How much must be paid on the equated date to
cancel these obligations, if no other payment
is made ?
8. If money iB worth 6%, simple interest, what
should be paid 30 days after the equated date in
problem 5 in Older to balance the account, if no
other payment is made ?

18. Continuously conveiåble interest; the


force of interest.l — The compound amount on
$1 at the end of one year, if interest iB at
the nominal rate j, converted m times per
year, is A = (1 -f- It was seen at the end of
Section 10 that, as m
COMPOUND NTERDST

increases, the amount A increases. .AB m


increases without bound, or in other words, as m
approaches infinity, the amount A does not
increase without bound but approaches a limiting
value ei, where e = 2.7182818+ iB the bage of the
Naperian, or natural, system of logarithms. To
prove this we use the theory of limits.
lim A = = lim (1 -k
lim (1 +

2
It is known that
= e. Therefore,

(22)

It iB customary to say that this limiting


value e i is the compound amount on $1 at the
end of one year in the ideal where i A knowledge
of the theory of li.m.iu ig advisable in
reading this section. See Granville's CatouZua,
Revised Edition, page 22.
57 MATHEMATICS OF INVESTMENT

interest is converted continuously. For every


value of m, the effective rate i
corresponding to the nominal rate j is given

by — 1]. Hence, in the limiting case


where interest
is converted continuously, it follows from equation 22
that i = lim (1 -F i m — 1
m
(23)
EcampZe 1. —Find the effective rate if the
nominal rate is .05, converted continuously.
Solution. — 1 + = e•0B. log (I + i) — .05 loge, where
e — 2.71828.
.05 log e .05(0.43429) - 0.02171

1 + i — 1.0513; i.0513.
The force of interest, corresponding to a given effective rate
i, is the nominal rate which, if converted continuously, will
yield the effective rote i. Hence, represents the force of
interest, the value j = must satisfy equation 23, or
(2
4) EcampZe 2. — Find the force of interest
if the effective rate is .06.
Solution. — 1.06 = eb. . blog g = log 1.06.
log
1.06 .0253059 - .058269.
log e .43429
Under the effective rate i, the compound
amount of principal P at the end of n is =
PCI + i)n. If the nominal rate is j, converted
continuously, (1 + i) = e i ; hence A = or A =
Pen}.
To Compute A we use logarithms; log A = log P + nj log e.
58 MATHEMATICS OF INVESTMENT

EXERCISE

1. Find the effective rate if the


nominal rate is .06, converted continuouxly.

2. Find the force of interest if the


effective rate is .05. %
3. (a) Find the amount after 20 years if $2000 is
invested at the rate .07, converted continuously. (b) Compare
your answer with the compound amount in cue the fate iB (.07, m
4).
COMPOUND NTERDST

37
MISCELLANEOUS PROBLEMS
1. A man, in buying a house, is offered the option of paying $1000 cash and
$1000 annually for the next 4 years, or $650 cash and $1100 annually for the next 4 years. If
money iB worth (.06, m = 1), which method is the better from the purchaser's standpoint?

2. A merchant desires to obtain $6000 from his banker. (a)


If the loan is to be for 90 days and if the banker charges 6% interest
in advance, for what sum will the merchant make out the note which
he will give to the banker? (b) What simple interest rate is the man
paying?
8. A merchant who originally invested $6000 has $8000 capital at
the end of 6 years. What has been the annual rate of growth of his
capital if the rate iB assumed to have been uniform through the 6
years?
4. If gasoline consumption is to increase at the rate of 5% per year, when
will the consumption be double what it is now? Solve by two methods.
6. When will the payment of $5000 cancel the obligations
$2000 due after 3 years, and $2500due after 6 years? Money is
worth (.05, m 2). e. A certain life insurance company lends money
to policy holders at 6% interest, payable in advgnce, and allows
repayment of all or part of the loan at any time. Six months before
the maturity of a $2000 loan, the policy holder A sends a check
for $800 to apply on his loan. What additional sum will A pay at
maturity?
HINT. — First find the sum, due in 6 months, of which $800 is the presentvLIue.
7. A must pay B $2000 after 2 years, and $1000 after 3 years and
6 months. At the énd of 1 year A paid B $1500. If money is worth
(.05, m = 2), what additional .equal payments at the ends of 2 years
and 6 months and of 3 years will cancel A's liability?
8. (a) If you were o creditor, would you specify that
money is worth a high or a low rate of interest to you, if one of your
debtors desired to pay the value of an obligation on a date before
it is due? Justify your answer in one sentence. (b) If a debtor
COMPOUND NTERDST

desires to discha.rge an obligation by making payment on a date


after it was due, what rate, high or low, should the creditor specify
as the worth of money?
9. At the end of 4 years and 7 months, $3000 is due. Find
its present value by the practical rule if money is worth (.08, m = 4).
10. A man hag his money invested in bonds whiah yield 5%, payable
semi-annually. If he desires to reinvest his money, what is the lowest rate,
payable quarterly, whioh his new securities should yield ?
61 MATHEMATICS OF INVESTMENT

11. One dollar is invested at simple interest, rate 5%. A Becond


dollar is invested at (.05, m = 1). When will the compound amount on the
second dollar be double the simple interest amount on the first?
HINT. —Solve the equation by interpolation; see illustrative Example I in
Appendix, Note 3.
12. After how long a time will the compound amount on $1 at the
rate (.06, m = 2) be double the amount on a second $1 at the rate (.035,

HINT. —Use either interpolation or logarithmic methods. See problem Il.


13. If $100 is invested now, what will be the compound
amount after 20 years if the effective rate of interest for the first 5
years will be 6%, whereas interest will be at the rate (.04, m = 2)
for the last 15 years?
14. A man owes $2000, due at the end of 10 years. Find its
present value if it iB assumed money will be worth 4% effective,
for the first 5 years, and 6% effective, for the last 5 years.
15. If $100 ig due at the end of 5 years, discount it to the present
time, (a) under the rate 5%, compounded annually ; (b) under the
simple interest rate 5% ; (c) under the simple discount rate 5%.
CHAPTER 111
ANNUITIES CERTAIN

19. Deflnidons. — An annuity is a sequence of periodic


payments. An annuity certain iB one whose payments extend over a
fixed term of years. For instance, the monthly payments made in
purchasing a house on the instalment plan, form an annuity certain.
A contingent annuity is one whose payments •last for a period of
time which depends on events whose dates of occurrence cannot be
accurately foretold. For instance, a sequence of paymentB (such as
the premiums on an insurance policy) which ends at the death of
some individual form a contingent annuity. In Part I of this book we
consider only annuities certain.
The sum of the payments of an annuity made in one year is called
the annual rent. The time between successive payment dates is the
payment interval. The time between the beginning of the first
payment interval and the end of the last, is called the term of the
annuity. Unless otherwise stated, an payments of an annuity are
equal, and they are due at the ends of the. payment intervats; the
first payment is due at the end of the first interval, and the last is
due at the end of the term. Thus, for an annuity of $50 per month
for 15 years, the payment interval is 1 month, the annual rent is
$600, and the term is 15 years; the first payment is due after 1
month, and the last, after 15 years.
Under a specified rate of interest, the present value of an annuity is the sum of the .present
values of all payments of the annuity. The amount of an annuity is the sum of the compound
amounts that would be on hand at the end of the term if all payments should accumulate at
interest until then from the dates on which they are due.
Non I. — Conside an annuity of $100, payable annually for 5 years, with
interest at the rate 4%, effective. We obtain the present value A of this annuity
by adding the 2d column in the table below, and the amount S by
63 MATHEMATICS OF INVESTMENT
COMPOUND Auoowr AT END or
PAYMßNT OF $100 VALUE OF TERM IF PAYMENT IR Larr
DUE AT END or PAYMENT TO AT
INTEREST
I year 100(1.04F1 96.15385 =
2 years 92.45562
88.89964 116.98686
3 years = 112.48640
86.48042
4 years 100(1.04)' = 108.16000
82.19271
6 years 100(1.04) = 104.00000
100 100.00000
(add) A - $445.18224 (add) S — $541.63220
The present value A = $45.18 is as desirable as the future possession of all payrnents bf the annuity. amount S =
$541.63, possessed at the end of 5 years, is as desirable aa all of the payments. Hence, A should be the present value of
S, due bt the end of the term, or we should have S = A (1.04) 5. This relation is verified to hold ;

445.182 X 1.21665290 = $541.632 = S. (25)


Nom 2. —In the table below it is verified that, if B fund is formed by investing $446.182 at
4% effective, this fund will provide for all payments of the annuity of Note 1 and, in so doing,
will become exactly exhausted at the time of the last payment. ThiB result can be foreseen
theoretically because $445.182 is the gum of the present values of all of the payments.

INT. AT IN FUND AT
DUE END or END
YEA.B YAA.R END
PATENT
or

1 $446.10 $17.807 $462.989 $100


362.989 14.520 377.509 100
2 11.100 288.609 100
277.509
3 188.609 7.544 196.163 100
4 96.153 3.846 99.999 100
5
EnRCISE XX
1. (a) Form a table as in Note 1 above in order to find the present
value and the amount of an annuity which pays $1000 8t the end of each 6
months for 3 years. Money ig worth 6%, compounded semi-annually. (b) Verify
64 MATHEMATICS OF INVESTMENT
as in equation 25, that is the present value of S, due at the end of the term. (c)
Form B table as in Note 2, to verify that the present value A, if invested at (.06, m
= 2), creates a fund exactly guffcient to

provide thé
payments of the annuity.
ANNUITIES CERTAIN

1
20. The examples below illustrate
methods used later to obtbin fundamental
annuity formulas.
Doamp7,e 1. — If money iB worth (.06, m = 4),
find the present value A and the amount S of an
annuity whose annual rent is $200, payable
semiannually for 15 years.
Sozution. — Each payment is $100. The entries in the
2d and 4th columns below are verified by the principles
of compound interest.
or TIME PROM DATE COMP. AMT. AT
$100 PRESENT OF PAYMENT END OF TERM IF
VALÜE or TO END OF PAYT. 18 LEFT no
END or PAYMENT ACOÜMÜLAE AT
INT.
6 months 14 yr., 6 100(1.015)
I mo. 58
14 years
year etc.
etc. 6 months
14 yr., 6 O months
100
mo.
15 years
Sum = A Sum = S
Hence, S 10011 + (1.015)! -F. • • etc. • + (1.015)" + (1.015) ].
68

The bracket contains a geometrical progression of 30


terms for which the ratio is w = (1.015) 2, the first
term a = 1, and the last term L — (1.015) 58 .
ANNUITIES CERTAIN

By the formula for the gum of a geometrical


progression,a

s — 100 — 100(1.015)" — 1 - 1002.44321978 — 1 —


$4774.918.
(1.015)' — 1 1.03022500 — 1
On adding the 2d column in the table we obtain the
present value
A — + (1.015) -68 +. • • etc. • + (1.015B + (1.015B].
The geometrical progression, in.the bracket hag the
ratio w = (1.015) 2, while a = (1.015) -øo and L =
(1.015) -2. Since = 1,
A = 1001 — (1.015)-0D 100 1 — .40929'597 =

$1954.356. (1.015)2 — 1.03022500 — 1


The preaent va&ue of $4774.918, due at the end of 15 years, should
equal A,
or S A (1.015) B0. We verify that

A (1.015)60 - = $4774.920. (26)


i
Geometrieal Progressions in Part Ill, Chapter XII, should be studied if the
sfudent ha,B not met them previously. Section 20 may be omitted without d iN turbing
the oontinuity of the succeeding geotiong, but geometrical needed in Sections 21
and 22. ' see Part 111, Section 90.
67 MATHEMATICS OF INVESTMENT

Ecample 2. —Find the present value A of an


annuity of $100 pel month for 3 years and 6
months, if money is worth (.05, m = 2).
Solution. —A iB the sum of the entries in the 2d
row below.
Payment
of
$100 due I month 2 months etc. 3 yr., 5 mo. 3 yr., 6 mo.
after
Present value
of payment etc.

— + (1.025)-+ + etc. + (1.025)-}


+ (1.025)-1].
The ratio of the geometrical progression ig w
(1.025)å ; a = an. L — (1.025)-å. Since
WL —a = 1 — (1.025) -7,
- 100 1 — .84126524 = 83849.14. (TablæV1mdX
1.00412392 —
1

EnRCISE

In each problem derive formulas for A and


S for the annuity describec the method of
Examples 1 and 2 above.
An annuity whose annual rent iB $200,
payable quarterly for 1 years. Money is
worth (.08, m = 4). Compute the formulaa for
A and and verify 8B in equation 26 that A iB
present value of S, due at ty end of the
term. A
2. Fifteen successive annual payments of
$1000, the first due aft( 1 year. Money is
worth (.05, m = 2). Compute A and S and
verif that A iB the present value of S, due
at the end of the term.
68 MATHEMATICS OF INVESTMENT

3. Payments of $100, made at the end of each 3


months for 15 year Money is worth (.05, m = 4).
4. (a) The annual rent of the annuity is
$2000, the payment interv iB 3 months, and the
term is 12} years. Money is worth (.06, m = 1
(b) Solve the problem if money is worth (.06,
m = 2).
5. An annuity which pays $100 at the end
of each interest period fi 10 interest periods.
Money is worth .045, per interest period.
21. Formulas for A and S in the most simple case. — Co: sider
the annuity paying $1 at the end of each year for n year I-at
(ail at i) be the present value, and (Sil at i) be the amount
this onnuity when money iB worth the rate i compounded annuall
The entries in the table below are eaaily verified.
ANNUITIES CERTAIN
43
PATENT or FROM Dan COMP. AMT. AT
$1 or or END or
Dun PAYMENT TO TERM PAYT. 1B Inm TO
END or END INT.

I year (n — 1)
2 years yr. (n
etc.
— 2)
(n — I) yr. etc.
yr. n etc.
years 1 year
O years 1
Sum = (50t Sum (5 at i)
i)
Hence,

(5 at i) = 1 + (1 + i) + etc. ... + (1
+ + (1 +
This is B geometrical progression where the
ratio w = (1 -+ i), the first term a 1, and
the last term L = (1 4- . Since
— (1 + — 1, and (w — 1) = i, the formulaQDL — a (QDL
— a) _ gives

(SilatO = (27)
On adding the 2d column of the
table we
obtain (cm at = i)
(1
which is a
geometrical progreesion with the ratio w =
(1
+

i), a = (1 -k and L = (1 + 0¯1. Since


— (1 + in
(a-m (2
8) i
70 MATHEMATICS OF INVESTMENT
If each payment of the annuity had been $R
instead of $1, the present value A and the
amount S would have been A = RG at i) and S =
at i).
It is important to realize that formulas 27
and 28 may be used whenever the payment
interval of the annuity equals the conversion
period of the interest rate, In deriving the
formulas, the interest period was called 1
year, merely for concreteness. Hence, if i is
the interest rate per period, then at i)
represents the amount and at i) the present
value of an annuity which pays $R at the end
of each interest period for n periods. Thug,
1006m at .025) is the present value of an
annuity paying $100 at the end of each
interest period for 18 periods if money is
worth the rate .025 per period.
Ecample I. — Find the amount and the
present value of an annuity paying $150 at
the end of each 3 months for 15 years and 6
months, if money iB worth 6%, compounded
quarterly.
Solution. — Since the payment interval equals the
interest period, formulaa 27 and 28 apply with the
number of payments n 62, and with i = .015.
Amount = 150(8a-n .015) = 150(101.13773956)
= $15170.66. Pr. val. = at .015)
= 150( 40.18080408) $6027.12.
The value of is from Table VII and that of iB from Table VIII,

Norm. Recognize that the solution above makeg r no


use of the explicit expræions for and because their
values are tabulated. ffhe use of the formulas for a
or 8 in such a case would be B complicated, and
therefore an incorrect method.

EnRCISE xm
37
(1.02) — 1
ANNUITIES CERTAIN
I. (a) In Table VII verify the entry for (sa-n at .02) — .02
by use of Table V. (b) Verify the entry for (%-m at .04) in
Table VIII by use of Table VI.

2. Find the present value and the amount


of an annuity which pays $500 at the end of
each year for 20 years, if money is worth
(.05, m = I).
8. If moneyisworth (.05, m = 2), find the
present valueand theamount of an annuity
whose annual rent is $240, payable semi-
annually for 13 years and 6 months.
Find the present values and the amounts of the annuities
below.

ANNUL
PB0B.
RENT

$ 50 4
10,000
500
8 mo.
300
6
4. mo. 14 yr.,
5. 6 9 mo.
6. mo. 18 yr.
7. I 19 yr., 6 mo.

8. yr.
15 yr.
25 yr.
1
9, 6 23 yr. 2
10. mo. 7 yr. 2
I
mo. $1000
1
2
2000 .06, m -
2400 12
ANNUITIES CERTAIN

In purchasing a house a man agrees to pay $1000 cash and


at the end of each 6 monthk for the next 6 years. If money is
worth (.07,.m 2); wha.t would be an equivalent cash valuation
for the house?
HINT. —The cash price is the sum of the present values of
all payments. The present value of the first payment is $1000.
The remaining 12 payments come at the ends of the payment
intervals and hence form b standard annuity whose present
value is at .035).

12. The man of problem Il has just paid the


installment due at the end of 4 year8 and 6 months.
What additional payment, if made immediately,
would cancel his remaining indebtednese if money ig
worth (.08, m = 2) ?
HINT. —His remaining indebtedness at any time, or
the principal outstanding, is the present vulue of
all remaining payments.
18. If you deposit $50 at the end of each 3
months in a savings bank which pays interest
quarterly at the rate 3%, how much will be
to your credit after 20 years and 6
months, if you make no withdrawals?
14. A man in buying a house has agreed to pay $1000 it
the beginning of each 6 months until 29 installments have been
paid. If money is worth 6%, compounded semi-annually, what is
an equivalent cash price for the house?

15. At the end of each year a corporation


places $5000 in a depreciation fund which is to
provide for plant replacement at the end of 12
years. (a) What sum will be in the fund at the end
of 12 years if it accumulates at the effective rate
7%? (b) What sum iß in thefund at the beginning of
the seventh year?
ANNUITIES CERTAIN

16. A man desires to deposit with a trust company a, suffcient sum


to provide his family with $500 at the end of each 3 months for the next 15
years. If the trust company credits interest 8t the rate 6%, quarterly, on all
funds, what should the man deposit?

HINT. — See the table of Note 2 of gecåon 19.


22. Further annuity formulas. — Consider the annuity whose
annual rent is $1, payable p times per year for n years. Each
of the np payments is ; the first is due at the end of —
years, and the others are due at intervals of — years for the
rest of the term.

If money is worth the rate i, compounded annually, let (SR at


represent the amount of the annuity and (afi Qt i) its present
74 MATHEMATICS OF INVESTMENT

value. To derive formulas for and sm we


form the table below.
$1 Coup. AMT. END OF
PAYMENT or DATE
—p or mo END
AT 1B Lum
AT
DUT AT TERM
or TERU
END or TO

1
P
2
P $1
etc.
etc.
$1
P
etc. $1
1 P
n years $1 P
P O years

Hence, on adding the fourth column we obtain

The progression in the bracket has the ratio u = (1 + i)i, the


first term a = 1, bnd the last term L = (1 + O n¯å. Since — a

(29)
The denominator of the last fraction is the
expression we have previously called 1 (j, at
i). On multiplying numerator and inator
of the last fraction by i, we obtain
75 MATHEMATICS OF INVESTMENT

(SR at i) = 3 j
Since, by formula 27, the last fraction
is (Sil at i),
(sf at i) = at O. (30)
See Exercise X, Problem 16. Also see heading of Table XI. The fB0t that (j,

is the nominal rate which, if converted p times per yebr, yieldB the effective
rate iB of importance in applications of equation 29. We use merely a convenient
abbreviation for complicated algebraic expression.
ANNUITIES CERTAIN

From the second column of the table we obtain

(0%) at i) = $1 + + (1 +
+ ... etc. ... + (1 + i) p].
The ratio of the geometrical progression in the bracket is
w = (1 + i)i, the
first term a = (I + i)
-n, and L = (1 + i)-j.
Since WL — a = 1
— (1 +
at i) = (31)

From this expression we derive, as in formula 30,

(a? at = ato. (32)


If the sum of the payments mode in 1 year, or the annual rent,
had been $R instead of $1, the present x, ralue of the annuity would
have been at i) and the amounti at i).
In the discussion above, money was worth the rate i,
compounded once per year, while the annuity was payable p times
per year for n years and the sum of the payments made in 1 year
was $1. The word year was used in this statement and in the proof
of formulas 29 to 32 for the sake of concreteness. All of the
ANNUITIES CERTAIN

reasoning remains valid if the word year is changed throughout to


interest period. Thus, when money is worth the rate i, per interest
period, if an annuity is payable p times per interest period for a term
of n interest periods, and if sum of the payments made in one
interest period is $R, the present value A and the amount S are
given by
A= at i) = ;

(33)
S = R(sØ) at i) = RkSi1 at O.

Example l.—lf money is worth 2), find A and S for an annuity of


fifty quarterly payments of $100 each, the first due at the end of
three months.
78 OF INVESTMENT
MATHmv1ATICS
Sohdion. — Payments occur twice in each interest
period. Hence, use formulna 38 with the data listed
below. Tables XII, VIII, and VIl are used in
computing.
.025
),

(34.15776393
.1
2
),
$6873.9
s

Dzample 2. — If money is worth (.06, m 4),


find and S for an annuity whose annual rent is
$1000, payable monthly for 12 years and 3
months.
Solution. — Use formulae 33, because the payments ocot.u• three
timee in each interæt period.

Non. — When p = l, formulas 29 and 81 reduce to


formulas 27 and 28, or (89 cti) — (8ä1ati) and (09 at
i) (amati). We may obtain the same resulta on placing
p — I in formulas 80 and 32, because at i) 11 (1 +i) —
1] — i and I. These resulte could have bean foretold
because, when
p = I, the payment interval equals the interest period,
and hence formulæ 27 and 28 apply well formulas 29 and
31. Ln the future think of (sm i) as at i), with the
value of p left off and understood to be p I Gust we
omit the exponent 1 in algebra when we write z instead
of $1). Thus, formulas 30 and 32 express the pruent
value and the amount of an annuity payable p times per
interest period in terms of the present value and the
amount of an annuity payable once per interest period.

EÜRCISE xxm
ANNUITIES CERTAIN 79
1. Verify the entry in Table XII for • —- .06 and p = 2.
.06 .06
HINT. — — — from Table XI. Complete the
division. ja '.05912603'

2. money is worth (.06, m —2), find the present value and


the amount of an annuity whose term is 9 years and 6 months, and
whose Annual rent is $1200, payable monthly.

Compute the present values and the amounts of the annuities below.
ANNUAL Eack PAYMENT
PR0B. TBRL
PAYMENT INTERVAL

8. $1000 16 yr.
5000 6 mo. 12 yr.
5. 1 mo.
6. 9 yr., 6 mo.
6 mo. 19 yr. .06,
7.
8. 3 mo. 8 yr., 6 mo. .07,
2000 $500
9. 3 mo. .05, m 1
500 225 10 yr., 6
3 mo. .08,
10. 200 mo. 6
I mo. .055, m 2
3 mo. yr., 3 .06,
750 mo. 8 yr. .04,
11. In beying a farm it has been agreed to pay $100
at the end of each month for the next 25 years. • If money iB
worth the effective rate 7%, what would be an equivalent
cash valuation for the farm 7, C
12. If $50 iB deposited in a bank at the end of every month
for the next 15 years and is. left to accumulate, what will be on hand at
thg end of 15 years if the bank pays 6%, compounded annually on
deposits?'
80 OF INVESTMENT
13. A sinking fund is being accumulated by
payments of $1000, made at the end of each 3 months. Just
after the 48th payment to the fund has been made, how much
is in the fund if it accumulates at (.045, m = 1) ?
14. An investment yields $50 at the end of each 3 months,
and payment.s will continue for 25* years. What is a fair valuation for
the project if money is worth (.05, m = 2) ?
15. How much could a railroad company afford to pay to
eliminate dangerous crossing requiring the attention of two watchmen,
each receiving $75 Ver month, if money ig worth (.04, m = 1)? Asume
that the crossing will be used for 50 years.
16. Prove from formula 29, that (811 at i) — -r. Thus iB the
Bum
$
1 which, if. paid at the end of 1 yea,r, is equivalent to p
payments of — P made at equal intervals during the year.
23. The most general annuity formulas. —Consider the
annuity whose annual rent is $1, payable p times per year for
n years. To find the present value and the amount of this
annuity when money is worth the nominal rate j, compounded
m times per yeBr, we might first compute the corresponding
effective rate i and then use formtdaa 29 and 31. It iB better to
use equation 17 to obtain
81 MATHEMATICS OF INVESTMENT

entirely new formulas in terms of the given quantities j and


m. From equation 17,

(1+1)
On substituting these expressions in formulas 29 and 31 we obtain

(SR at j, m) =

(34)

(afi) atj, m) =

If the annual rent of the annuity above were $R instead of


$1, the present value would be at j, m) a.nd the amount
would be R(sfi) at j, m).
NOTE. — Formulas 34 include all previous formulas aa specia,l cages,
because, when m = I andi i, formulas 34 reduce to formtdaa 29 and 31, fromwhich
we started. Thug, think of (8%) at as being (h at j = i, 1) With the value of m left
out and understood to be m I. Likewise, (am at i) = at

24. — For an annuity under Case 1 below we usually may


compute the present value A and the amount S by means of
our tables. For an annuity under Case 2, the explicit formulas
for and S must be computed with much less aid from the
tables.
Case I. — The annuity is payable p times per interest period, where
p is an integer. The method of Section 22 applies, with additional
simplification when p = 1. If
82 MATHEMATICS OF INVESTMENT

p = the number of payments per interest period, n


= the term, expressed in interest periods, i = the
rate, per interest period, and
$R = the sum of the payments made in one interest period, then
A = R(aon) at i) = at ; S at = at O. (I)
ANNUITIES CERTAIN 83

When P = I, $R is the annuity payment, n is the number-of payments, and


S R(sm at i).
(11)
The values of A and S in I and Il can usually be computed by Tables
Vll, Vlll, and Xll.
N0TE. — One or more of Tables VII, VIII, and XII will not apply if i is not a table interest
rate, or if n is not an integer. In that the explicit formulas 29 and 81 for (aff at i) and (8%) at i)
must be computed.

Case 2. — The annuity iB not payable an integral number of times per interest period. The
general formulas 34 must be used, and if
n = the term in years, p = the number of payments per year,
$R = the annual rent, j = the nominal rate,
and m = the number of conversion
periods per year, then

A = R(aßl) at j, m) = R

S = R(sß) at j, m) =
R

SüPPLEMENTARY Nom. — From formulas Il and Ill it can be proved that (ah) atj, m) =

(%iiäl
These formulas can be used to simplify the computation of the præent
values and the of many annuities coming under Case 2. Other simplifying
formulas could be derived but they would not be of suffcieutly general
application to justify their consideiotion.
Bcample 1. — An annuity will pay $500 semi-annually for 8 years. Find the present value A
if money is worth (.06, m = 4). Solution. — The annuity
comes under Case 2.
Oaae 2 n = 8 —
A—
4),
years, p = 2, j - .06, m — 4,
R 31000.
ANNUITIES CERTAIN 84

A — 1000

1000(1 — .6209929) — $6269.76.


2(1.03022500 — 1)
ANNUITIES CERTAIN 85
Eæampte 2. — In buying a house a man has agreed to pay $1000
cash, and $200 bt the end of each month for 4 years and 3 months.
If money ig worth (.06, m = .2), what would be an equivalent cash
price for the property ?
Solution. — First disregard the cesh payment. The other payments form an
annuity under Cue I whose present value is
A - 12000% at .03)
Case I n = at .03),
8.5 int. periods,
A=
P 6, i = .03, R = 31200.
1200(1.01242816)
.03
-8
(1.03) $8997.33. — .7778280.
.03
The equivalent cash price is $1000 + $8997.30 — $9997.33.

Dcample 3. — At the end of each 3 months a man deposits $50


with a building and loan association. What sum is to his credit bt the
end of 4 years if interest is accumulating at the rate (.075, m = 2),
from the date of each deposit?
Solution. — The amount on hand is the amount of an annuity which comes
under Cue I.
Cage 1 n = S- at .0375), g = 100 (1.0375) 8 — 1
8 int. periods, (Formula 29)
P = 2, i = .0375, R = $100.
log (1.0375) = 0.0079940, from Table 11. _ 100(1.3425 — 1) s

(1.0375)} = 1.018577, from Table 11. 2(.018577)


8 log (1.0375) = 8(.0159881) = 0.12790. 34.25
= *921.8.
.037154
(1.0375)8 = 1.3425, from Table 1.
The answer is not stated to five digits because the numerator 34.25 was
obtainable only to four dWt,s from Table I.
NOTE. — In every problem where the present value or amount of an annuity is to be
computed, first list the case and the elements of the annuity aa in the examples above.
86 MATHEMATICS OF INVESTMENT
Norm — To find A and S for an annuity we could always proceed as under Case 2, even
though the annuity comes under Cage I. Thus, for the annuity of Example 3 above,
the term iB n = 4 years, the annual rent iB R = $200, payable p — 4 times per year, j =
.075, and m 2. Hence, from formulaa 111 of vase 2,
S — 200(8$) at .075, m — 2) 200 (1.0375) 8 — 1

which is the same as obtained above. The only difference in method is that, under Cage
2, the fundamental time unit is the year, yhereas under Caze 1 it iB the interest period.
The classification of annuity computations under two casa would not be advisable if we
were always to compute A and S by the explicit formulas, aa is necessary in Example 3.
But, if we used the general formulas of Case 2, with the year as a time unit, in problems
under Case I to which Tables VII, VIII, and XII apply, unnecessary coyaputationol
confusion would result and other inconvenient auÄLiary formulas would have to be
derived. Hence, use the method of Cage 1 whenever possible.

EXERCISE xmv
Compute A and S for each annuity in the table. Use Table Il when it iB
an aid to accuracy.
ANNoat, EACH
PROB. PAYMENT TERM
INTERVAL
ANNUITIES CERTAIN 87

1. $10,000 .05,
2. .06,
3. .05,
4. 500
.05,
5. 240
.04,
6. 100 .09,
15 years .05,
8. 5,000 I month
12 years .07,
9. I month
2,000 6 months 19 yr., 6 mo.
10. 7 yr., 6 mo.
6 months
11. Il yr., 6 mo.
3 months
12. 18 years
1 year 28 years
13. 6 months
6 yr., 9 mo.
$ 400 3 months
14. 6 months I 10 years
3,000 2500
15. year 15 years
3 months
16. 6 months
9 yr., 3 mo.
17. 1,200 20 years
150 3 months
18. 30 years
4 months 9 years
19. 125 3 months 12 years .0625,
20. 1 year 35 years .05,
21. 6 months 12 years .08,
22. 700 6 months 15 yr., 6 mo. .005,
50 3 months 6 yr., 3 mo. .048,
100 4 months 9 years .04,
6 months 10years .04,
I month .07,
200 17 years
.055,
150 .03,
250 .06,
.04,
.045,
25 .06,
23. To provide for the retirement of a bond issue at the end of 20
years, a city will place $100,000 in a sinking fund at the end of each 6
months, (a) If the fund accumulates at the rate (.05, m = 2), what sum
will be available at the end of 20 years? (b) What sum is in the fund at
the beginning of the 12th year?
88 MATHEMATICS OF INVESTMENT
24. An investment will yield $50 at the end of each
month for the next 15 yea18. If money is worth (.05,
m = 4), what would be a fair present valuation for the
project ?
26. A depreciation fund is being accumulated by
semi-annual deposits of $250 in a bank which pays
5%, compounded quarterly. How much will be in the
fund just after the 30th deposit ?
26. A will decrees that X shall receive $1000 ot
the beginning of each 6 months until 10 payments have been
made. If money is worth (.06, m = 2), on what sum should
Ys inheritance tax be computed, assuming that the payments
will certainly be made ?
27. A certain bond has attached
coupons for $5 each, payable at the end of
each year for the next 25 years. If money
is worth 5% effective, find the present
value of the coupons.
28. The bond of problem 27 will be redeemed
for $100 by the issuing corporation at the end of
25 years. What should an investor pay for the
bond if he deües 5% effective on his investment?
HINT. —He should pay the present value of the coupons
plus the present value of the redemption price.
29, A farm is to be paid for by 10 successive annual
installments of $5000 in addition to a cash payment of
$15,000. What iB an equivalent ca,sh price for the farm if
money iB worth (.05, m = 2) ?
80. (a) At the end of the 5th year in
problem 29, after the payment due has been
made, the debtor wishes to make an
additional payment immediately which will
cancel his remaining liability. The
creditor is willing to accept payment if
money is considered worth 4% effective.
What does the debtor pay? (b) Why should
ANNUITIES CERTAIN 89
the creditor specify the rate 4% effective
instead of a higher rate, (.05, m = 1) for
instance? HINT. — Find the present value
of the remaining payments.
31. A man has been placing
$100 in a bank at the end of
each month for the last 12 years. What is
to his credit if his savings have been
accumulating at the rate 6%, compounded
semi-annually from their dates of deposit
?
82. A man wishes to donate immediately to a
university Buffcient money to provide for the
erection and the maintenance, for the next 50
years, of a budding which will cost $500,000 to
erect and will require $1000 at the end.of each
month to maintain. How much should he donate if
the university is able to invest its funds at 5%,
converted .semi-annually ?

SS. A certain bond has attached coupons for $5 each, payable


semiannually for the next 10 years. At the end of 10 years the bond will be
redeemed for $125. What should an investor pay for the bond if he desires
6%, compounded semi-annually, on his investment?
HINT. — See problem 28.
84. A man, who borrowed a sum of money, iB to discharge the liability
by paying $500 at the end of each 3 months for the next 8 years. What sum
did he borrow if the creditor's interest rate is (.055, m = 2) ?
35. (a) In problem 34, at the end of 4 years, just after the installment due
has been paid, what additional payment would cancel the remaining
liability if money is still worth (.055, m = 2) to the creditor? (b) What
would be the payment if money is worth (.04, m = 4) to the creditor?
90 MATHEMATICS OF INVESTMENT
36. A man X agreed to pay $1000 to his creditor at the end of
each 6 months for 15 years, but defaulted on his first 7 payments. (a)
What should X pay at the end of 4 years, if money is worth (.06, m = 2) to
his oreditor? (b) What should he pay if money is worth (.05, m = 2) ?
87. In problem 36, at the end of 4 years, X desires to make a single
payment which will cancel his liability due to his previous failure to pay,
and also will discharge the liability of the payments due in the future. (a)
What should he pay if money ig worth (.06, m = 2) to his creditor? (b) Find
the payment if the rate is (.05, m 2).
88. A certain bond has attached coupons of $2 each, payable
quarterly for the next 20 years, and at the end of that time the bond itself
will be redeemed for $110. What should a man pay for the bond if he
considers money worth .effective ?
89. Prove by use of formulas 34 that the present value of an
annuity, accumulated at the rate (j, m) for n years, will equal the amount of
tho annuity ; that is, prove algebraically that

4)mn = R(afi) at j, -k R(SFI at i, m) S.


Another Btatement of this result would be that 'CA is the present value of
S, due at the end of the term of the annuity."
40. What is the amount of an annuity whose term ig 14 years, and
whose present value iB $1575, if interest is at the rate (.06, m = 2) ?
HINT. — Use the result of problem 39.
41. What is the effective rote of interest in use if the present value of an annuity is $2500,
the amount $3750, and the term 10 years?
42. A will bequeaths to a boy who is now 10 years old, $20,000
worth of bonds which pay 6% interest semi-annually. The will requires that
half of the interest shall be deposited in a savings bank which pays 4%,
compounded quarterly. The accumulation of the savings account, and the
bonds themselves, are to be given to the boy on his 25th birthday. Find the
value of the property received by him on that date.
43. A man desires to deposit with a trust company a suffcient sum
to provide his family an annuity of $200 per month for 10 years. What
ANNUITIES CERTAIN 91
should he deposit if the trust company will credit interest at the rote 5%
compounded quarterly, on the unexpended balance of the fund ?
44. If you can invest money at (.03, m = 2), what is the least sum
you would take at the present time in return for a contract on your part to
pay $100 at the end of each 6 months for the next 15 years ?
46. If money is worth (.04, m = 1), is it more profitable to pay $100 at the
end of each month for 3 years as rent on motor truck, or to buy one for
$3000, assuming that the truck will be useless after 3 years? Assume in
both cases that you would have to pay the upkeep.
25. due. — The payments of the standard annuities considered
previously were made at the ends of the payment intervals. A.n annuity due
is one whose payments occur at the beginn.ing of each interval, so that the
first payment is due immediately. The definitions of the a,mount and. of the
present value of an annuity as given in Section 19 apply without change of
wording to an annuity due. It must be noticed, however, that the last
payment of an amuity due occurs at the beginning of the last interval,
whereas the end of the term is the end of this interval. Hence, the amount of
an annuity due is the sum of the compound amounts of the payments one
interval after the last payment is made. For an annuity due whose annual
rent is $100, payable quarterly for 6 years, the last $25 payment iB made at
the end of 5 years and 9 months. The amount of this annuity is the sum of
the compound amounts of the payments at the end of 6 years, the •end of
the term.
For the treatment of annuities due and for other purposes in the future, it
is essential to recognize that, regardless of when a sequence of periodic
payments sta,rt, they will form an ordinary annuity if judged from a date
one payment interval before the first payment. Hence, one interval before
the first payment, the
ANNUITIES CERTAIN 92
sum of the discounted values of the payments is the present
value of the ordinary annuity they form. Moreover, the sum of
the accumulated values of the payments on the 188t payment date
is the amount of this ordinary annuity.
Example 1. — If money is worth (.05, m =
2), find the present value A and the amount S
of an annuity due whose annual rent is $100,
payable quarterly for 6 years.
O
increasing Eme
— ropmsents 3 months represents the present
FIG. 3

Solution.— Consider the time scale in Figure 3,


where X repræenta a payment date, T ig the end of the
term, 6 yean from the pregent, and L is the laat
payment date, 3 months before T. Q is 3 months before
the present. Considered from Q the payments form an
ordinary annuity whose term ends at L and whose
present value A and amount S' are
Case 1
12 int. periods,
A' = 50(aB at .025),
n .025, R = S' = at .025).
$50.
Since A ' is the sum of the discounted valueg of the
payments at Q, 3 months before the present, we
accumulate A for 3 months to find A, the present
value of the annuity due.

Since S' is the sum of the accumulated values of the


payments at L, we accumulate S' for 3 months to find
S, which is the sum of the valua at time T.

S = 8'(1.025)} = 506% at
ANNUITIES CERTAIN

Tables VII, VIII, X, und XII would be used to compute A and S.


Second 801ution. — The first $25 payment is cash
and the remaining pay menu form an ordinary annuity,
judged from the present. Ita present
value A' is
Case 1 n = 11.5
int. periods, p = 2, i A' 50(oæat .025).
= .025, R = $50.

Hence, = 25 + .025).
To find S, first consider a new annuity consisting of
all payments of the annuity due, with an additional
$25 due at time T. Since T is the last payment date
of the new sequence of payments, the Bum of their
values 8t time T iB the amount S' Of an ordinary
annuity, or
S' = at .025).
Cage I ne value of the additional $25 payment at time
n = 12.5 int. periods, p - T ig included in S', or S' = S + 25. S = at .025) —
2, i - .025, R = $50. 25. To find the numerical valuæ of A and S , a and
must be computed from formulas 29 and 31.
Hence, the first solution was leas complicated numerically. In some problems,
however, the second sokltion would bc the least complicated.

Two rules may be stated corresponding, respectively, to the two


methods of solution considered above.
Rule 1. — To find A and S for an annuity due, first find tho
present value A' and the amount S' of an ordinary annuity having
the same term, annual rent, and payment interval. Then :
(a) 44 is the compound amount on A' after one payment interval.
(b) S is the compound amount on S' after one payment interval.
Rule 2. — To find A for an annuity due, first find A',
the•presont value of all payments, omitting the first. Then,
if W is the annuity payment, = A' -k W. To find S first
obtain S', the amount of the ordinary annuity having a
94 MATHEMATICS OF INVESTMENT
payment at the end of the term in addition to the payments
of the annuity due. Then,

Nom. —It is customary in actuarial textbooks to use block roman type to


indicate amounts and present valUeB of annuities duo. Thus (BPI) at j, m)
(p)
repræenta the amount, and (an at j, m) the present value of un annuity duo
whoee annual rent is $1, payable p times per year for n years, if money is
worth

EXERCISE XXV
In each problem draw figure similar to Figure 3. Find and S for
each annuity due in the table, by use of the specified rule.
INTERVAL
INTEREST

1. 3 mo. 2
10 yr. 2
2. 6 mo. $ 300 2
7 yr., d mo. .06, m I
3. 6 mo. 2
12 yr., 6 mo. 3600 .03, m 4
12 yr. 3 mo. 1000 .06, tn 1
6. Corry through the solution of problem 2 by Rule 1 far
enough to be able to state why it is inconvenient.
e. A man deposited $100 in a bank at the beginning of each 3
months for 10 years. (a) What sum is to his credit at the end of 10
years if the bank credits 6% interest quarterly from the date of
deposit? (b) What sum is to his credit at the end of 9 years and
9 months, after the deposit hag been made at that time?
•J 7. In purchasingohouse, amanhas agreed topay$100 at the
beginning of each month for the next 5 years. (a) If money is
worth 6% effective, find the present value of the payments. (b)
If mone is worth (.06, m = 12), find their present value.
ANNUITIES CERTAIN

8. A man was loaned $75 on the 1st of each month, for 12


months each year, during the four years of his college course.
(a) If hig creditor considers money worth 3% effective, what is
the liability of the debt at the end of the 4 years? (b) If the
debtor makes no payment until four years after he graduates,
what should he pay then to settle in full ?
9. If money is worth the effective rate i, prove that (a—I
at i), the present value, and (h at i), the amount of an annuity
due of $1 payable

annually for n years, are given by

10. Prove that


1
(SRI at j, m) = (1 + at j, m); (ARI at j, m) = (I + at j, m).
26. Deferred annuiåes. — A deferred annuity is one whose
term does not begin until the expiration of a certain length of
time. Thus, an annuity whose term is 6 years, deferred 4 years,
and whose annual rent is $1000, payable semi-annually,
consists of 12 paymentB of $500, the first due after (4 years -F
6 months) and the last, after (4 years + 6 years).
Ezample 1. — If money is worth (.06, m — 1), find A and S for the deferred annuity
of the last paragraph.

FIG. 4

In Fig. 4, X represents a payment date of the deferred annuity, N, the


present, T, the end of 10 years, the end of the term, and B, the beginning
of the term,
6 MATHEMATICS OF INVESTMENT

Solution. — Consider the time scale in Figure 4. The payments form an


ordinary annuity when judged from B, 6 months before the first payment. S',
the amount, and A the present value (at B), of this ordinary annuity are

Cage 1 n = 6 int. S' = at .05).


periods, p - 2, i = .05, R = A' = at .05).
$1000.
S' is the gum of accumulated values at time T.
Since S, the amount of the deferred annuity, is also equal to the sum of values
at T,
S = S' = 1000(80) at .05).
Since At is the sum of the discounted values at B, we must discount A ' for 4
yean to obtain the present value A.
0
) at .05) (1.05B.
A = 10000
Second solutionfor A. — Consider a new annuity having payments of $500
at the end of each 6 months for the first 4 years as well as for the last 6. The
new payment dates are indicated by circles in Figure 4. The present value A of
the deferred annuity equals the present value A of the new annuity over the
whole 10 years minus the present value A n of the over the fi.rst 4 years, which
are not to be received. Both and are the present values af ordinary annuities.

Case 1 A' = 1000@— at .05). n — 10, and 4, int.


periods,
.05, R - $1000. A" = 10000%) at .05).
A = A' — A" — at .05) — (ah) at .06)],
A= at .05) — (ail at .05)].
From Example 1 it is clear that the amount of a deferred equals ffe a.mount of an ordinary
annuity having the same term. Corresponding to the two methods used above in obtaining A,
we state the two rules below.

Rule 1. — To obtain for an annuity whose term is deferred w


years, first find A', the present value of the ordinary annuity having
the same term. Then, A equals the value of A' discounted for years.
Rule 2. — If the term of the annuity is n years, deferred u years,
then
ANNUITIES CERTAIN

A = [present value of an ordinary annuity


with term (w + n) years] — (present value of
an ordinary annuity vith term w years), where
these new annuities have the same annual rent
and payment interval as the deferred annuity.
Non. — The present values and the amounts of
deferred annuities are indicated in actuarial
writing by the symbols for ordinary annuities with a
nurnber prefixed showing the time for which the term

is deferred. Thus at j, m) and (u18öl at j, m)


represent the present value and the amount when the term is deferred w years.

I
EXERCISE XXVI
In each problem draw a figure similar to Figure 4. Find
the present value of each deferred annuity in the table,
by use of the specified rule.

TERM PAYMENT ANNUAL


PR0B. TERM
DEFERRED INTARVAL
INTEREST Ron

10 yr., 6 3 .05, m 4 2
1. 6 yr. mo. mo. $1000 1
2. 7 yr. 8 yr., 6 1 .07, m 1
mo. 300 .05, m 2
3. 9 yr. mo. 12 1
4. 13 yr. yr. 1 yr. 1200 1
10 yr., 6 mo. 1 mo.
5. Carry through the solution of problem 4 by
Rule 2 until you are able to state why it is
inconvenient.
6. Solve problem 3 if money iB worth (.07, m = 2).
7. A man will receive 8 pension of $50 at the end of each
month for'920.4'
ANNUITIES CERTAIN

10 payment to occur I month after he is 65


years old. Assuming that he will Live to receive 811
payments, find the present value of his expectation
if money is worth (.04, m = 1), and if he is now 50
years old.
8. A certa.in mine will yield a semi-annual
profit of $50,000, the first poyment to come at the
end of 7 years, and the last after 42 years, at
which time the mine will become worthless. What is
a, fair valuation for the mine if money iB worth 5%,
effective ?
9. A recently paved road will require no
upkeep until the end of 3 years, at which time $3000
will be needed for repairs. After that, $3000 will
be used for repairs at the end of each 6 months for
15 years. Find the present value of ail future
upkeep if money is worth (.05, m = 2).
1
The Miscellaneous Problems at the end of the
chapter may be taken up im mediately after the
completion of Exercise XXVI.
ANNUITIES CERTAIN
10. By use of Rules 1 and 2 prove the relations below, for an
annuity whose annual rent is $1, payable p times per year, and
whose term is n years, deferred w years.
at O.
at
i)
— (4 at i).
SUPPLEMENTARY MATERIAL •U
27.1 Condnuous annuities. — If money is worth the effective rate
i, the present value of an annuity whose annual rent iB $1, payable
p times per year, is

(4 at i) = (aäl at i).
We may consider an unuity payable weekly, p = 52, or daily, p =
365, and we may a,sk what value does (OKI at i) approach as p
becomes large without bound ? We obtain
1
lim at i) = i(aäl at i) lim
(jp at i)

But, from Section 18 we have lim jp b, the force of interest


corresponding to the effective rate i. Hence at i) = i(aa-l at i) lim
(CFI

In the same way it can be shown that

lim at i) —
As p o, the annuity approaches the ideal cage of an annuity whose annual rent is payable
continuously. If we let at i) represent the present value and (ki at d) the amount of a continuous
annuity, the results above show that

1 Section 18 is prerequiAite for the reading of this section.


100 MATHEMATICS OF INVESTMENT
(ääl at i) =(85)
Recall that 1 + i =
log(l eb so
that ö — Since log e —
log e log
2.7182818 = 0.4342945, we obtain from equation 85,
63
at i) (.4342945) at 0(.4342945)
log (1 + i) (SRI at i) = log(l + i)
The present value and the amount of an
annuity, which is payable continuously,
differ but slightly from the corresponding
quantities for an annuity which is payable a
very large number of times per year (see
problem 1 below). Hence we may use ööl and
0)
as approximations for. a;] and if p is
very large.
EXERCISE xxvll
1. (a) If money is worth 6%, effective,
find the present value and the amount of an
annuity whose annual rent is $100, and whose
term is 10 years, if the annuity is payable
continuously. (b) Solve the problem if the
annuity is payable monthly.
2. If one year equals 360 days, find
approximately the amount of an annuity of $1 per day
for 20 years if money is worth 4%, effective. HINT. —
Use a continuous annuity as an appropriLtion.
3. A member of a labor union has agreed
to contribute $.20 per day to o benefit fund
for 3 years. Under the rate (.05, m = 1),
ANNUITIES CERTAIN
find approximotely the present value of his
agreement if 8 year has 365 days.
4. An industrial insurance policy for
$100 calls for a premium of 10 cents at the
end of each week. Find approximately, by use
of a continuous annuity, the equivalent
premium which could be paid 8t the end of the
year if money is worth 3å%.
Non. — If the conversion period of an interest rate
is not stated, assume it to be I year.
28.1 Computadons of high accuracy. — he binomial theorem can

be used in interest computations to which the tables do not


apply. As a special case of the binomial theorem, 2 we have n(n —
1)$2 n(n — 1) (n — 2)c8
(1 + C)n I TIC

(36)

When n is a positive integer, series 36


conta,ins (n + 1) terms, the last of which is
cn. If n is a negative integer or fraction,
the series contqins infinitely many terms. In
this case, if lies
A knowledge of the binomial theorem iB

needed in this section. * See page 93 in


R.ietz and Crathpme's CoUegg Algebra.
between — 1 and 4- 1, the infinite series converges, and, if c is very
small, the sum of the first few terms gives a good approximation to
the value of (1 -k z)n. The proof of this statement is too diffcult for
an elementary treotment.
(4)
102 MATHEMATICS OF INVESTMENT
Da;mnpte l. —Find the value of (a—I at .033) accurately to six
significant figures.
Solution. — From formula 31,
(37)
(a;at .033)
From
(1.033* .. (38)
.
.03259930.
I — (1.033B
The next term in series 38, beyond the last one computed, is negligible in the
8th decimal place, and hence our result is accurate to the 7th decimal place
with only a slight doubt aa to the 8th. To compute (1.033B, first compute
(1.033) 6 and then take the reciprocal; (1.033)' = 1.2150718; (1.033B
= .8229966. Hence,
.1770034
at .033) - - 5.429669.
¯ .03259930
The fina,1 9 is not dependable because the final O in the denominator wag
doubtful.
EXERCISE xxvm
1. Compute (s n at .0325) and (a at .0325) accurately to five significant figures.

2. Compute (je at .02) accurately to the 7th decimal place.


3. A ma,n pays $50 to a building and loan association at
the end of each week. If the deposits accumulate at the rate (.06, m
= 2), how much will be to his credit at the end of 3 years? Obtain
the result correct to four significant figures.
MISCELLANEOUS PROBLEMS
1. If $1750 is the present value of an annuity whose term is 12
years, what is its amount if money is worth (.05, m — 4) ?
2. If $100 is deposited in a bank at the beginning of each
month for 10 years, what ig the accumulated amount at the
ANNUITIES CERTAIN
time of the laet payment if interest is at the rate 6%
compounded semi-annually on all monéy from the date of
65
3. In problem 2, what is the amount at the end of 10
years?
4. A inan W has occupied farm for 5 years and, pending decision
of case in court, has paid no rent to B to whom the farm is finally
awarded. What should W pay Lt the end of 5 years if rent of $100 should
have been paid monthly, in advance, and if money ig worth (.06, m =
12) ?
5. What should W pay in problem 4 if the rent is considered due at
the end of each month and if money is worth 6%, compounded annually?
6. A building will cost $500,000. It will require, at the beginning
Of each year, $5000 for heat and light, $5000 for janitor service , and, 8t
the end of each year, $3000 for smallrepnirs. It is to be completely
renovated, at cost of $20,000 Lt the end of each 15 years. If the cost of
the annual repai1E is included in the cost of renovation at the end of each
15 years, and if the building iB to be renovated at the end of 90 years,
what present sum will provide for the erection of the building and for its
upkeep for the next 90 years, if money is worth 6% effective ?
7. A young man, just starting a four-year college course, estimates
his future earning power, in excess of living expenses, at $100 per month
for the first 3 years after graduation from college, $200 per month for the
next 7 years, and $300 per month for the next 20 years. What is the
present value of this earning power, if money is worth 4%, effective?
8. If the man in problem 7 should place hig surplus earnings in bank, what will
he have at the end of his working life if his savings earn interest at the rote (.05, m = 2),
for the first 10 years, and at the rate (.04, m = 1) for the balance of the time?
9. In purchasing a homestead from the government, a war veteran
has agreed to pay $100 at the end of 5 yearB, and monthly thereafter until
the laet payment occurs at the end of 9 years. What is the present value of
his agreement if money is worth (.045, m = 1) ?
10. At the end of 3 years, the man of problem 9 decides to pay off
his obligation to the government immediately. What should he pay if
money is worth (.045, m = 1)?
11. A savings bank accepts deposits of $1 at the beginning of each week during
the year from small depositors who creating Christmas funds. Just after the 52d
payment, what will each fund amount to if the bank accumulates the savings Lt 6%,
effective?
104 MATHEMATICS OF INVESTMENT
12. A contract provides for the payment of $1000 at the end of each
C months for the next 25 years. What is the present value of the contract
if the future liabilities are discounted at (.06, m = 2) over the last 15 years
of the life of the transaction and at (.05, = 2) over the first 10 years?
105 MATHEMATICS OF INVESTMENT

IS. In purchasing a house it wag agreed to pay $50 at the end of


each month for a certain time. The purchaser desires to change to
annual payments. What should he pay at the end of each year if
money is considered worth (.05, m = 2) ?
106 MATHEMATICS OF INVESTMENT

14. At what rate of interest compounded semi-a,nnually


will $1650 be the present value and $3500 the amount of an annuity
whose term is 14 years, if the annuity is payable weekly? Would the
result be any different if the annuity were payable annually ?
107 MATHEMATICS OF INVESTMENT

15. If money ia worth 5%, effective, what is the least sum


which you would accept now in return for contract on your part to
pay $50 at the end of each month for 20 years, first payment to
occur at the end of 10 yeare and 1 month?
CHAPTER IV
PROBLEMS ANNUITÆS
29. In every problem below, the payn:ient interval of the
annuity and the conversion period of the interest rate will be given
or, in other words, the p and m of equations I and Ill of Section 24
will always be known. For an annuity under Case 1 there remain for
consideration the five quantities (A, S, R, i, n). If three of (S, R, i, n)
are Imown, we use S = at i) to find the fourth; while, if three of (A,
R, i, n) are known, we use A = at i). If an annuity comes under Case
2, similar remarks apply to the four quantities (S, R, j, n) and to (A,
R, j, n). Problems in which A and S were unknown were treated in
Chæpter 111.
30. Determinadon of the payment. — For future
convenience it is essential to know that

1 1 1 1
— i, or (39)
(SFI at i) (ail at i) (Sil at i) (am at
From formulas 27 and 28 we obtain

-n).
and hence relation 39 ig true.
PROBLEMS IN ANNUITIES
1
Example 1. — Find the value of (81 at .05) •

67
1
SoZution.—From Table IX, — .09634229. Therefore, from
(an at .05)
1
relation 39, .09634229 — .05 .04634229. Ondoingthissub(8m
at .05)
traction mentally, we are able to read the result 04634229 directly from Table
l.x.
N0TE. —From Example 1 we gee that, because of relation 39, Table IX
1 1
gives the values of as well aa those of • It would be equally

(871 at i (aäl at i
1
convenient to have a table of the values of from which we could

(5 at i)
1
obtain those of by adding the interest rate i.
(ail at i)
Example 2. — What annuity, payable quarterly for 20 years and 6
months, could be purchased for $5000, if money is worth (.05, m =
4) ?
Solution. — present value of the annuity ig $5000. Let $$ be the quarterly
payment.

Case I n 82 5000 '


int. periods, (an at. 0125)
P = 1, i = .0125
A = $5000, R E.
110 MATHEMATICS OF INVESTMENT
1
c -5000 = 5000(.01956437) (aääl at .0125)
= $97.82. TheannuaIrentiB4c—$391.29.

Example 3. — If money is worth (.06, m = 2), find the annual rent of an


a.nnuity, payable qua,rterly for 11* years, if its amount iB $10,000.
Solution. — I.et $$ be the sum of the payments in one interest period.

Case I n 23
int. periods, P = 2, i 10000 = iil .03) - C .03 at .03),
= .03, IOOOOØ* at .03) _
S = $10,000, R = sc.
¯
.08(8nat .03) .03 (5 at .03)•
_ 10000(.02977831)
(.03081390) - $905.86.
.03
Tablo and XI were used. gnnunl rent is 2 c — $611.72.

N0TE. — Recognize that the solutions above w.ere arranged so aa to avoid


computi.ng quotients, except for the easy division by .03.
Eæample 4. — Find the annual rent if $3500 iB the present value of an annuity which iB
payable semi-annually for 8 years. Interest is at the rate 5 %, compounded quarterly.
69-
Solution. — Let $c be the annual rent.
Case 2
n = 8 yr., p 2, j 3500 = at .05, m 4) =
= .05, m = 4,
3500 = c .32801593 (Tables V and VI)
R = b, A = 8500.
2(.02515625Y

7000(.02515625)
= $536.84.
.32801593
EXERCISE xnx
PROBLEMS IN ANNUITIES
1 1
1. Compute (arg = 16.379658to verify the entry in Table IX. at .05)
Find the annual rents of the annuities below.
PREAENT
hon. PAYMENT
INTERVAL
1 TERM Va.Lua
RATE

2. .06, m 4 12 yr., 3 mo. $ 6,500


3 mo. .05, m = 2 17 yr., 6 mo.
8. 6 mo. 1 3,000
4. 15 yr.
yr. 15,000
5. 8 yr. $ 8,500
6 mo.
6. 3 mo. .05, m - 1 5 yr., 6 mo.
4,000
7. .05, m 2 15 yr. 7,000
3 mo. 17 yr. 3,750
8. 1 yr.
.06, = I
9. .05, m = 1 12 yr., 6 mo.
2,500
6 mo. 1
10. yr. .05, m = 4 10,000
11. 6 mo. .07, m = 2 9 yr.
.07, m 2 2,500

12. If money iB worth 6%, effective, find the annuity, payable


annually for 25 years, which may be purbhased for $1000.
13. If money is worth 6%, effective, find the annuity, payable
annually for 10 years, whose amount is $1.
14. If money is worth 4%, compounded annually, what annuity, payable annually
for 15 years, may be purchased for $1 ?
15. If money iB worth the effective rate i, derive formula
for the payment of the annuity, payable annually for n years, which
may be purchued for $1.
16. If money is worth the effective rate i, derive a formula
for the payment of the annuity, payable annually for n years, whose
amount is $1.
17. In order to create a fund of $2000 by the end of 10 years, what
must a man deposit at the end of each 6 montbB in a bank which credits interest
semi-annually at the rate 3% ?
112 MATHEMATICS OF INVESTMENT

18. The present liability of a debt is


$12,000. If money is worth 5.5%, compounded
semi-annually, whut should be paid Bt the end
of each year for 10 years to discharge the
liability in full?
31. Determinaüon of the term. — If the term
of an annuity is unknown, interpolation
methods furnish the solution of the problem
with suffcient accuracy for practical
purposes.
Example I. — For how long must a man deposit
$175 at the end of each 3 months in a bank in
order to accumulate L fund of $7500, if the
bunk credits interest quarterly at the rote ?
Solution. — The deposits form un annuity whose
amount is $7500. let the unknown term be k interest
periods.

Case 1 n
int. 7500 = at .016),
periods, p
1, i .015, (hi at .015)175 = 42.857.
R $175, S -
$7500.
n (am
From the column in Table VII for i = .015, we obtain
at .015) the first and third entries Lt the left..
33 42.299 By interpolation,
k 42.857 558 k 33 + — = 33.341
int. periodB.
34 43.933
1034

The term is — 8.34 years. However, since an annuity whose term iB


not an

integral number of payment intervals hag not been


defined, this result is useful only because it
113 MATHEMATICS OF INVESTMENT

permits us to make tho following statement: The $176


paymenta must continue for 8.5 yeurs to creutc n fund
of ot least $7500; when the 83d payment occura at,
tho end of 8.25 years, the fund amounts to less thun
$7500.
Dcampl,e 2. — Find the term of an annuity
whose present value iB $8500 and whose annual
rent iB $2000, pmyable quarterly. Interest iB
at the rate (.06, m = 2).
Solution. — Let tho unknown term be interest periods.

Case 1 n int.
8500 - 10000%) at .03) 1000
periods, .03),
(am at .03) _ - 8500(j2
R - $1000, 30at .03)
$8500.
8500(.02977831) = 8.437.
30

(am at .03) The first and third entries at the left Ll•e from the column
n
93 in Table
9 7.786 Vlllfori .03. 10 — — = 9.88 interest 744
8.437 periods. The term iB — = 4.94 years. Hence, for an
10 annuity whose term is 4.75 years, the present value is less
8.530
than $8500, while the present value would be greater than
$8500 if the term were 5 years.
Nom. Values of k found by interpolation in Tables VIl and VIll are in error
by less than half of the interest rate per period* In interpolating, use three
decimal places of the table entries and oompute the value of k to three decimal
places.
EnRCISE XXX

Find the terms of the annuities below.


114 MATHEMATICS OF INVESTMENT

11. For how many full years will it be necessary to deposit


$250 Bt the end of each year to accumulate a fund of at least 8500,
if the deposits 5%, compounded annually?
12. The cash value of a house iB $15,000. In buying it on the installment plan a
purchaser has agreed to pay $1000 at the end of each 6 months as long necessary. For how long
must he pay if money iB worth 6%, compounded semi-annually?

32. Determination of the interest rate. —


Example 1. — Under what nominal rate, converted quarterly, iB
$7150 the present value of an annuity whose annual rent iB $880,
payable quarterly for 12 years and 6 months ?
i
For justification of this statement see Appendix, Note 6.
See supplementary Section 33 for other problems in which the term ig unknown.
115 MATHEMATICS OF INVESTMENT

Solution. — IRt r be the unknown rate per period.

Case I n = 50
int„ periods, 7150 220(a-n ai r),
7160
R — $220, A $7150. 220 = 32.500.

at i)
The first and third entries at the left were obtained from the row in Table VIll
for n 50. Since .0200 — .0175 = .0025,
.0176 33.141
r =.0175 In'7(.0025) - — .0175 + .00093 = .01843.
32.500
.0200 31.424 The nominal rate is j = 4 r = 4(.01843) — .07372, converted
quarterly.
NON. — A value of T obtained as above usually is in error by not more than 1
of the difference of the table rates used in the interpolation. Hence, r

= .01843 probably is in error by not more than = .0001, und the


nominal rato j = .07372 is in error by not more thun .0004. We ure justified only
in saying that the rate is approximately .0737, with doubt to the ILBt digit.
Supplementary Example 2. — Determine the nominal rate in
Example I accurately to hundredths of 1%.
Solution. — From Example 1, (an at r) = 32.500, and r = .0184, approxi-
mutely. It is probable that r is between .0184 and .0186, or clse botween .0184
and .0183. Since our tables do not use the rate .0184, we compute

50 log 1.0184 =
50(.0079184) .0184, .39592.

(h at .0184) = 1 — .0184.40186 32.507. log (1.0184) (1.0184) -50 -50 = -


9.60408 .40186. — 10.
Since 32.507 is greater than 32,500, r must be greater than .0184, and prol)ably
is between .0184 and .0185. By logarithms, (an at .0185) = 32.438.

i (awl at i) From interpolation in the table Lt the left,


- .018410.
.018 32.507
4 32.500
32.438
.018
6
116 MATHEMATICS OF INVESTMENT

The nominal rate is j = 4 r = .073640, which is certainly accurate to


hundredths of 1%, and iB probably accurate to thousandths of 1%.
The author gives no justification for this statement. HO has verified iu math for numerous examples

scattered over the range of Tables V11 and vnl.


PROBLEMS IN ANNUITIES 117
Non. — We could obtain the solution of Example I with any desired
of accuracy by successive computations as in Example 2. Our accuracy would
be limited only by the extent of the logarithm tabla at our disposal.

In Example 1, the most simple formulas (Case I, with p = 1) applied


because the conversion period equaled the payment interval. In more
complicated examples, the solution may be obtained by first considering a new
problem of the simple type met in Example 1.

Ezampte 3. — Under what nominal rate, converted semi-


annually, is $7150 the present value of an annuity whose annual
rent is $880, payable quarterly for 12 years and 6 months?
Solution. — Let the unknown nominal rate bej. We could use the formulas
of Cage 1, with p = 2, in the solution, but the work would be slightly
complicated. Instead, we first solve the following new problem: 'c Determine
the nominal rate, w, converted quarterly, under which the present value of the
annuity be $7150." We choose quarterly conversions here because the
annuity is puyable quarterly. This new problem iB the Example I solved above,
so that w = .0737. The rate j, compounded semi-annually, must be equivalent
to the rate .0737, compounded quarterly, because the present value of the
unnuity is $7150 under both of these rates. Hence, the effective rates
corresponding to these two rates must be the same. l From equation 17, if i
represents the effective rate,
.0737 = (1.0184) 4 .
4

1 +ä = (1.0184) 2 = 1.03714.

Table Il was used in computing 1.03714. The desired nom-inal rate iB i


2(.03714) .07428, or approximately .0743, with doubt 8.8 to the last digit.

EXERCISE xxxr a

In the first ten problems find the nominal rates a.s closely a as is
possible by interpolation in the tables.
118 MATHEMATICS OF INVESTMENT
1
For aimilur problem see Section 10, illustrative Example 3.
a
The Miscellaneous Problems at the end of the chapter may be taken up
immediately after the completion of Exercise XXXI.
a
If the instructor desiree, tho students may be requested to accuracy to hundredths of 1%, as in Example 2 above.

ANNUAL PAYMENT INTEREST


PROB.
RENT INTARVAL PER10D A.uo TTQM

1. $1000 12 yr.
I year I year $15,700 25 yr.
2. 100 I year I year $ 1,785
8. 500 1 year 1 year 15 yr.
5,390
100 6 mo. 6 mo.
1,110
17 yr., 6 mo.
5. 3 mo. 3 mo. 2,500
6. 400 5 yr., 8 mo.
I year 1 year 53,000
7. 1000 3 mo.
26 yr.
1 year 2,750
1 200 3 mo. 9 yr.
6 mo. 2,750
8.1 200 I mo. 14,500 9 yr.
I year
9. 2400 17,500
6 mo. 3 mo. 8 yr.
10. 500
24 yr., 6 mo.
11. A man has paid $100 to a building and loan association
at the end of each 3 months for the last 10 years. If he now has $5500
to his credit, at what nominal rate; converted quarterly, does the
association compute interest ?
12. By use of the result of problem Il, find the effective rate of interest paid by the
association of problem 11.
18. It hag been agreed to pay $1100 at the end of each 6 months
for 8 years. Under what nominal rate; compounded semi-annually,
would this agreement be equivalent to a cagh payment of $14,000?
14. A fund of $12,000 has been deposited with a trust company in
order to provide an income of $400 at the end of each 3 months, for
10 years, at which time the fund will be exhausted. Atwhat effective
rote does the trust company credit interest on the fund ?
HINT. — First find the equivalent nominal rate, payable quarterly.
SUPPLEMENTARY MATERIAL
33. Dificult cases and .euct methods in finding the term.
— When the formulas of Case 2 apply to an annuity, it iB
PROBLEMS IN ANNUITIES 119
necessary to use the explicit formulas Ill in finding the term
if it is unknown.
Br,ample 1. — The amount of an annuity iB $8375, and the
annual rent iB $1700, payable semi-annually. What is the term if
money is worth (.06, 4)?
1
See illustrative Example 3. The same preliminary work should be used for both of problems 7

and 8. First determine the nominul rote. converted quarterly, under whidh 32750 is the
PROBLEMS IN ANNUITIES 120

Solution. — 2 applies. Hence, let the unknown term be years.

Cage 2
n = lc yr., p = 2, j 8375 = 1700k at .06, m = 4).
= .06, m = 4, 8375 = 1700 (1.015)" _ 1
R = $1700, A — $8375. — If

Fron•i Table V, the denorninator iB 2(.030225).


(1.015)" — 1 = = .29781.
1700
(1.015)" 1 + .29781 = 1.29781. (40)
(a) To solve equation 40 by interpolation, we use entries from the column in
Table V fori = .015. We obtain
(1.015)
n 979
1 1.28802 = 17 +— = 17.507, or = 4.377.

7 1.29781
1.30734 1932
1
8
(b) To obtain the exact value of lc from equatiori 40, take the logarithm of
both Bides of the equation, using Table Il for log 1.015.
4k log 1.015 = log 1.29781; 4k(.0064660) = .11321.
11321 11.321 log 11321 = 4.05389

4(646.60) = 2586.4• log 2568.4 3.41270


4.3771. log k — 0.64119
The solutions of problems, treated by interpolation in Section 31, may be
obtained by solving exponential equations, as in solution (b) above. In these
exact solutions it is always necessary to use the explicit algebraic expressions
for the present values and the amounts of the annuities concerned.
Example 2. — If the rate is (.06, m = 2), find the term of an annuity
whose present value iB $8500 and whose mnnual rent is $2000, payable
quarterly.
PROBLEMS IN ANNUITIES 121

Solution. — Let the unknown term be interest periods.

Case I n — 8500 =
k int. periods, 10000%) at
P — 2 t 03 .03) - 1000 (Table
R $1000, A - $8500.
I — (1.03)-k
8500 =
1000
.02977831

1 — (1.03) -k 8500(.02977831) ,25312, (By Table 1)


1000
MATHEMATICS OF NVESTMENT

- = 1 — .25312 = .74688. — 1.03 = .74688.


— — 9.87325 — 10 = — .12675. (log 1.03 from Table 11)
12675
9.8738 periods of 6 months. 1283.7
The term is —4.9369 yeare. Compare this with the result by interpolation in
Example 2 of Section 31.
EXERCISE xxm.l
1. At the end of each 6 months a mam deposits $200 in a
bank which credits interest quarterly at the rate 3%. For how many
years must the deposits continue in order to create a fund of $3000?
Use the exact method (b) of Example 1 above.
2. If money iB worth 6%, compounded monthly, for how
long must payments of $2000 be made at the end of each 6 months in
order to dischorge debt whose present liability ig $30,000? Solve by
both an interp018tion and an exa,ct method.
8. Solve illustrative Example 1 of Section 31 by the exact
method.
4. Solve problem 9 of Exercise X.XX by the exact method.
5. To create an educational fund for a daughter, a father
decides to deposit $500 at the end of each 6 monthg in bank
which credits interest annually, from the date of deposit, at the
rate 4%. When will the fund amount to at leaßt $6000? Solve
by the exact method.

YflSCELLMEOüS PROBLEMS
i. If money is worth 5%, effective, what equal payments should be made at the end of
each year for 10 yéars in purchaaing a house whose equivalent cash price is $5000?
2. If man saves $200 at the end of each month, when will he be
able to buy an automobile, worth $3000, if his• deposits
accumulate at the rate 5 compounded semi-annually?
8. A depreciation fund is being accumulated by equal deposits
at the end of each month in a bank which credits 6 % interæt
monthly on deposits from date of deposit. What is the monthly
deposit if the fund contains $7000 at the end of 5 years?
MATHEMATICS OF NVESTMENT

4. In purchaaing a, house, worth $20,000 cash, a man has agreed to pay $5000 cash and $1000
semi-annually for 9 yeare. What interest rate, compounded eemi-pn*iually, is being used in the
PROBLEMS IN ANNUITIES
(77)
6. An insurance policy, on maturing, gives the policy holder the
option of an immediate endowment of $15,000 or an annuity,
payable quarterly for 10 years. Under the rate 3.5%, effective, what
will be the quarterly payment of the annuity?
6. A fund of $50,000 has been deposited with a trust company
which credits interest quarterly on all funds at the rate 5 For how
long will thig fund furnish man payments of $1000 at the end of
each 3 months?
J 7. On the death of her husband, a widow deposited her inherited
estate of $25,000 with a trust company. If interest is credited semi-
annually on the fund at the nominal rate 4 for how long will the
widow be able to withdraw $800 at the end of each 6 months?
8. A certain loan bureau lends money to heads of families
on the following plan : In return for a $100 loan, $9 must be paid at
the end of each month for 1 year. What effective rate of interest is
being charged? HINT. — First find the nominal rate, compounded
monthly.
9. A certain homestead is worth $5000 cash. The
government sold this to an ex-soldier under the agreement that he
should pay $1000 at the end of each 6 months until the liability is
discharged. If interest is at the rate (.04, m = 2), for how long must
the payrnentB continue?
10. The annual rent of an annuity is $50, payable annually.
The present value of the onnuity is $400 and the amount is $600.
Find the effective rate of interest by use of the relation 39 of Section
30.
Il. A certLin farm has a cash value of $20,000. If money is worth (.05, m =
2), what equal payments, made at the beginning of each 6 months for 6 years,
would complete the purchase of the farm ?
12. A man borrowed $2000 under the agreement that interest
should be at the rate (.06, m = 2) during the life of the transaction.
He made no payments of either interest or principal for 4 years. At
that time, he agreed to discharge all liability in connection with the
PAYMENT OF DEBTS BY PERIODIC INSTALLMENTS
125
debt by making equal payments at the end of each 3 months for 3
years. Find the quarterly payment.
cmPTER V
THE PAYMENT OF DEBTS BY PERIODIC INSTALLWÆNTS
34. Amoråzation of a debt. — A debt, whose present value is A, is
said to be amordzed under a given rate of interest, if all liabiliåes as to
principal and interest are discharged by a sequence of periodic
payments. When the payments are equal, as is usually the case, they
form an annuity whose present value must equal A, the original
liability. Hence, most problems in the amortization of debts involve the
present value formulas for annuities. Many amortization problems have
been solved in previous chapters.
Ecanpte I. — A man borrows $15,000, with interest payable
annually at the rate 5%. The debt is to be paid, interest as due and
original principal included, by equal installments at the end of each
year for 5 years. (a) Find the annual payment. (b) Form a schedule
Bhowing the progress of repayment (or amortization) of the principal.
Solution. — Let $$ be the payment. The present value of the
Cage 1 n = payment annuity, Lt the. rate (.05, m = I), must equal $16,000. 15000
5 int. periods, p — .05).
— 1, i — .05,
R - E, A $15,000.
1
= 15000 — $3464.622.
(arl at .05)

AMORTIZATION SCEØDÜ1.m

INTERE9T Fon
YEAR
OOTEANDING PATENT RErAYMDNT
BEOLNMNG Due AT END AT END or or PRINCIPAL
OF END or YEAR

1 $16,000.000 $ 750.000 $ 3,464.622


12,285.378 614.269 3,464.622
2 9,435,025 471.751 3,464.622 $ 2,714.622
4 6,442.154 322.108 8,464.622 2,850.353
2,992.871
5 3,299.640 164.982 3,464.622 3,142.514
3,299.640
Totals $46,462.197 $2323.110 $17,323.110 $15,000.000
78

N0T1•2 1. — The schedule showg that the payments


satisfy tho creditor's dcmnnds for intorest and
likcwige return his principal in installmontg. If =
$3464.622 wag computed correctly, we know, without
forming the schedule, thut these facts must be true
because the present value of tho five payments is
$15,000. The chcck8 on the arithmetic done in the table
are that the Inst, total should bc $15,000, thc sum of
the second and the last should equal third, und tho
xeeond should be interest on the first total for one
year at 5%. Notice that tho ropuyments of principal
increase from year to year, while the intorQ8t,
payments decrcnse. Amortization schedules are very
useful in the bookkeeping of both debtor and creditor
because the exact outstanding liability at; every
interest dute iB clearly Bhown. The outstanding
principal, or Liubility at any dutc, iB sometimes
culled the book value of the debt Lt that time.
NOTE 2. — Since money is worth 5%, in Example 1, we
may assume that tho clQl)tor invest* tho $15,000 ut 5%
immediutely after borrowing it. The uccmnulution of
this fund should provide for all the annual
payments, to be mndo to t110 creditor, bocnusc their
present value is $15,000. A numericul verifie,ution of
t11iN fact is obtained in the amortization table Lbove
if we merely alter tho titlCB of tho columns, as below,
leaving the rest of the table unchanged.
PAYMENT OF DEBTS BY PERIODIC INSTALLMENTS
127
IN FUND PAYMENT TO
BEGINNING TAxBN FROM
CREDIT0R
YEAR END or FOND AT END
END OF or YEAn
Yun YEAR

1 $15,000 $750 $3464.622 $2714.622

Thug, (Lt tho end of the first year, the debtor


receivea $760 from his inveted fund and, in order to
make the payment of $3464.622 to his creditor, he takes
$2714.62 from tho principal. By the end of 5 years, the
fund reduces to zero.
Ij•.tample 2. — In Example 1, without using
the amortization schedule, determine the
principal out8tanding ut the beginning of the
third year.
Solution. —The outstanding principtd, or liability,
iB the present value of all payments remaining to bc
made. These form an annuity
whose term iB three years. Thc
outstanding principal iB
3464.62% at .05) = $9435.03.
Case 1 n This ig the third entry of the first
— 3 int. column of tho amortization Bahedule.
periods,
05, R $3464.62. 3. — debt whose present value

ig $30,000, bearing interest at the rote 4.5%, compounded semi-


annually, is to be amortized in 10 yearH by equal paymontR at the
end of each 3 months. (a) Find the quartcrly payment. (b) Find tho
principal outstanding at the end of 5 years, after the payment due
hae been made.
MATHEMATICS OF INVESTMENT
( 80.
Solution. — (a) Let $$ be the quarterly payment. The present value of the
payment annuity must equal $30,000.
.0225
Cage I n 20
int. periods,

R = 2 m, A $30,000.

30000 = 2 at .0225) at .0225).


15000 1
.0225 (an at .0225f

_ 15000(.02237484) (.06264207) = $934.403.


.0225
(b) At the end of the 5th year, or the beginning of the 6th, the outstanding
liability, L, is the present value of payrnents extending over 5 yeLrB.

Case I
n = 10 int. periods, i
= .0225, p 2, R = $1868.81.
L = 1868.810% at .0225).

L = 1868.81 .0225 .0225).


92
L - $16,661.95.
EXERCISE
I. A loan of $5000, with interest at 6%, payable semi-annually, is
to be amortized by six semi-annual payments, the first due after 6
months. (a) Find the payment, to three decimal places. (b) Form the
amortization schedule for the debt.
2. In problem I, without using the amortization table, find the
principol unpaid at the end of 1 year and 6 months, just after the
payment due hag been ma.de.
PAYMENT OF DEBTS BY PERIODIC INSTALLMENTS
129
8. A man deposits $10,000 with a trust company which credits 5%
interest annually. The fund is to provide equal payments at the end
of each year for 5 years, at the end of which time the fund iB to bc
exhausted. (a) Find the annual payment to three decimal places. (b)
Form a tablo showing the amortization of the fund.
HINT. —See Note 2, Sedon 34 ; think of the trugt company as the
debtor. 4. In problem 3, without using the table, find the amount
remaining the fund at the end of 2 years, after the payment due
ha,s been made. 6. A purchaser of house owes $7500, and interest at
6% is payable semi-annually on all amounts remaining due. He
wiBhes to discharge his debt, principal and interest included, by
twelve equal semi-annuml installments, the first due after 6 months.
Find the necessary semiannual payment.
6. A street nasessment of $500 against a certain piece of
real estate is to be amoitized, with interest at 6%, by six equal
annual payments, the first due after I year. What part of the
aggegsment will remain unpaid at the beginning of the 4th year,
after the payment due hag been made ?
7. A house is worth $25,000 and the
owner, on selling, desires the equivalent of
interest at the rate 5%, payable semi-
annually. (a) What quarterly installment,
for 8 years, in addition to a cash payment
of $5000, would satisfy the owner? (b) How
much of the principal of the debt remains
unpaid ot the end of 3 years and 6 months,
after the payment due has been made ?
8. A debt of $12,000, with interest
payable semi-annually at tho rote 5 is to be
amortized in 10 years by equal semi-annual
installments, the first due after 6 months.
Whut part of the debt will remain unpaid ut
the beginning of the 0th your, after the
payment due has been made?
MATHEMATICS OF INVESTMENT
9. In problem 8, what part of the 11th
payment iB interest and what part is
repayment of principal ?
10. A debt will be discharged, principal
and interest, nt effective, included, by
payments of $1200 ot the end of each year
for 12 years. (a) Whut is the original
principal of the debt? (b) What principal
will remain out*tanding ot thc beginning of
the 5th year? (c) What part of the 5th
payment will be interest and what port
repayment of principal ?
11. A trust fund of $100,000 wag created
to provide a regular income at the end of
each month for 20 years. If the trust
company uses the interest rate 4%, converted
semi-annually, whut is the monthly payment,
if the fund iB to be cxhausted by thc end of
20 years?
12. It was agreed to amortize a debt of
$20,000 with interest nt 5%, by 12 equal
annual payments, tho first due in onc yea.r.
The debtor failed to mako the first four
payments. What payment at the end of 5 yoars
would bring the debtor up to date on his
contract?
13. A debt of $38,000 iB to bo amortized
by poymentB of $2000 at the end of ench 3
months for 6 yearg. (a) Find the nominol
rate, compounded quarterly, at which
interest is being paid. (b) Whut iB the
effective rote of intcreat ?
14. A certain insurnncc policy on
maturing gives the option of $10,000 cash or
PAYMENT OF DEBTS BY PERIODIC INSTALLMENTS
131
$345 at tho end of each 0 months for 20
years. What rate of interest is being used
by the insurance company?
35. of a bonded debt, —In amortizing a debt which
is in the form of a bond issue, the periodic
paymonts cannot be exactly equal. If the bonds are
of $1000 denominotion, for example, the principal
repa,yments must be multiples of $1000, because any
individual bond must be retired in one installment.
82}

Example I. — Construct a schedule for the amortization, by 10


annual payments as nearly equal possible, of a $10,000 debt which is
outstanding in bonds of $100 denomination, and which bears 4%
interest payable annually. The first payment is due at the end of 1 year.
Cage I n =
10 int. periods,
.04,

R = $c, A = $10,000.
Bozqdion. — IÆt $$ be the annual payment, which would be made if the
payments were to be equal.
10000 .04).

1
= 10000
= $1232.91.
The annual payments should be as close as possible to $1232.91. Thus, at the end
of the 1st year, the interest due iB $400, leaving 1232.91 400.00 = $832.91
available for repayment of principal. Hence, retire 8 bonds, or $800 of the principal
on this date, making B total payment of $1200. At the end of the next year 1232.91
— 368.00 = $864.91 is available for r.etiring bonds. Therefore, pay 9 bonds or
$900 of the principal.

AMORTIZATYON SCHEDULE FOR A BONDn DDBT


MATHEMATICS OF INVESTMENT
PniN01PAb OUT"
AT BEGIN- PAYMEN
Doa B ONDB
YEAR T
END or YEAR END OF YEAR
END OF YEAR

1 $10,000 $400 8 $ 1,200


2 9,200 368 9 1,268
3 8,300 332 9 1,232
4 7,400 296 9 1,196
6,500 260 10 1,260
5
5,500 220 10 1,220
4,500 180 1,280
11
3,400 136 1,236
7 2,300 11 1,192
92
8 1,200 48 11 1,248
9 12
10
$58,300 $2,332 100 $12,332

EDRCISE xxnv
1. A $1,000,000 debt is outstanding in the form of $1000
bonds which pay 6% interest annually. Construct a schedule for the
retirement of the debt, principal and interest included, by five annual
paymentg QB nearly equal aa possible, the first payment due at the end
of 1 year.
2. A $1,000,000 issue of bonds, paying 5% interest annually, consiBts of 500 bonds
of $100, 200 bonds of $500, 200 of $1000, ond 130 of $5000
denomination. Construct a schedule for the amortization of the debt by
10 annual payments nearly equal as possible.
HINT. —In the a separate column for each clagg of bonds.
36. Problems in which åe periodic payment is known. — If the
present liability of a debt, the interest rate, and the size and frequency
of the amortization payments are known, the term of the payment
annuity can be found as in Section 31.
BcampZe I. —A house is valued at $10,000 cash. It is agreed to pay $1200 cash
and $1200 at the end of each 6 months 0B long as necessary to amortize the given
cash value with interest at 5%, payable semi-annuclly. (a) For how long must the
payments continue? (b) Construct an amortization schedule.
PAYMENT OF DEBTS BY PERIODIC INSTALLMENTS
133
n = k int. periods, p
=

— .025,
R - $1200, A - *8800.
Solution. — (a) After the cash payment of $1200, $8800 remains due. IRt k be
the time in interest periods necessary to amortize it
With interest at the rate (.05,

8800 = at .025) ; (ail at .025) = 7.833.


By interpolation in Table VIII, = 8.20. Hence, 8 full payments of $1200 mugt be
made in addition to the first cash payment. After the $1200 payment iB made, at
the end of 4 years, some principal is still outstanding because k is greater than 8. A
partial payment will be necæsary at the next payment date. Thæe conclusions are
verified in the schedule below.

(b) AMORTIZATION SCHEDULE


OUTSTANDING TOTAL PRINCIPAL
PAYMENT CIPAL AT BEGIN- INTEREST Dn PAYMENT PAID AT END OF
NINO or INTERVAL INTERVAL
INnRVAL END 0B INTNBVA.I,
AT END or
I.NQZVA.I,

1 88800.000 $220.000 $1200. $ 980.000


2 7820.000 195.500 1200. 1004.500
6815.500 170.388 1200. 1029.612
3
5785.888 144.647 1200. 1055.353
4
4730.536 118.263 1200. 1081.737
5 3648.798 91.220 1200. 1108.780
6 2540.018 63.500 1200. 1136.500
7 1403.518 35.088 1200. 1164.912
8 238.606 5.965 244.571 238.606
9
Totals "1,782.863 $1044.571 $9844.571 38800.000
134 MATHEMATICS OF INVESTMENT

Example 2. — Without using the amortization table, find


the principal still unpaid in Example I at the end of years, after
the payment due hag been made.
Solution. — I.Æt $M be the amount remaining due. The payment of
$M at the end of years, in Eddition to the payments already made, would
complete the payment of the debt whoee original principal was $8800.
Hence, this "Old Obligation must have the same value ag the New
Obligations " listed below.

OLD OBLIGATION New OBLIGATIONS

(a) due at the end of 2} yearE. (b)


$8800 duo at the beginning of Payments of $1200 due at the end of each
the transaction. 6 months for 2å years.
To find M, write an equation of value, under the rate (.05, m — 2), with the end
of years aa the comparison date. The sum of the values Of obligations (b) is the
amount of the annuity they form.
— M + 1200% at .025).
M — 8800(1.025)' — 1200(8snat .025) 9956.39 - 6307.59 = $3648.80, (41)
which checkg with the propor entry in the tablo of Example 1. The debtor could
close the transaction at tho end of 2} yours by paying the regular inBt811ment
plus $3648.80 or (1200 + 3648.80) = $4848.80.
NOTE 1. — Recognize that is the omount the creditor should have at
the end of 2* years if he invested $8800 at (.05, m — 2), whereos he actuany
possession Of only 1200(8 at .025) us consequence of the pay received from the
debtor. Hence, equation 41 shows that M is the difference between what the croditor
should have und what he actually hag.
NOTE 2. — By the method of Dxumplo 2 wo con find the final
instilllmont in Example I without computing the amortization tuble. let $
N be the amount remaining due just after tho lust full payment, at tho end
of 4 yeors. Then, N - — 1200(85 at .026) 10721.046 —
10483.339 - $238,607.
To close the transaction at tho end of 4} yours, the nocogsury payment is
$238.607 plus interest for 6 months at or 238.607 + 5.966 $244.67.
EDRCISE XXXV
135 MATHEMATICS OF INVESTMENT

1. (a) How long will it take to amortize a debt whose present value iB $10,000 if
payments of $2000 are made at the end of each year and if these payments include interest Bt
the rate 5%, payable annually. (b) Form an amortization schedule for this debt.
PAYMENT OF DEBTS BY PERIODIC INSTALLMENTS
136

2. @ Without using the table in problem I, find the


principal outstanding at the beginning of the 3d year. (b) Find the
size of the final payment.
3. A debt of $50,000, with interest payable quarterly at the
rate 8%, is being amortized by payments of $1500 at the end of each
3 months. (a) What iB the outstanding liability just after the 10th
payment? (b) Find the final installment.
4. A trust fund of $100,000 is invested at the rate 6%,
compounded semi-annually. Principal and interest are to provide
payments of $5000 at the end of each 6 months until the fund is
exhausted. (a) How many full payments of $5000 will be made? (b)
What •will be the size of the final partial payment ?
5. The purchaser of a farm hak agreed to pay $1000 at the
end of each 3 months for 5 years. (a) If these payments include
interest at the rate 6%, payable quarterly, what is the outstanding
principal at the bæ ginning of the transaction? (b) Find the
outstanding liability at the beginning of the 3d year. Notice that,
since the exact number of the remain.ing payments is known, part
(b) should be done like illustrative Example 2 of Section 34; it
would be clumsy to use the method of illustrative Example 2 of the
present section.
37. Sinking fund me&od. — A sin-king fund is a fund formed
in order to pay an obligation falling due at some future date. In the
following section, unless otherwise stated, it is assumed that the
sinking funds involved ore created by investing equal periodic
payments. Then, the amount in a Bin-king fund at any time is the
amount of the annuity formed by the payments, and examples
involving sinking funds can be solved by use of the formulas for the
PAYMENT OF DEBTS BY PERIODIC INSTALLMENTS
137

amount of an annuity. Thus, to create fund of $10,000 at the end of


10 years by investing $$ at the end of each 6 months for 10 years, at
the rate (.06, m = 2), must satisfy
1
10000 = at .03) ; c - 10000 $372.157.
Suppose that $Å iB borrowed under the agreement that interest
shall be paid when due and that the principal shall be paid in one
installmenl at the end of n years. If the debtor provides for the
future payment of $Å 8t the end of n years by the creation of a
sinking fund, invested under his own control, his debt is said to be
138 MATHEMATICS OF INVESTMENT

retired by the sinking fund method. Under this method, the expense
of the debt to the debtor is the sum of @) and (b) below :
(a) Interest on $A, paid peHodicaLIy to the creditor when due.
(b) Periodic deposits, to create a sinking fund of $A, to
pay die pri.ncipal when due.
NOTE 1. — Recognize that the sinking fund is a private affair of the debtor.
The rate of interest paid by the debtor on $A bears no relation to the rate of
interest at which the debtor is able to invest hig sinking fund. Usually, the
desire for absolute safety for the fund would compel the debtor to invest it at a
lower rate than he himself pays on his debt.
Ecample 1. —A debt of $10,000 is contracted under the
agreement that interest shall be paid semi-annually at the rate 6%,
and that the principal shall be paid in one installment at the end of
2} years. (a) Under the sinking fund method, what is the semi-
annual expense of the debt if the debtor invests his fund at (.04, m =
2)? (b) Form a table showing the accumulation of the fund.
Sohdion. — @ Let $$ be the semi-annual deposit to the Sinking fund, whose amount at the
end of 2å years is $10,000.
10000 = at .02) ;
Case 1 n =
5 int. periods, - 10000 at .02) — $1921.584.
(861

Interest due semi-annually on the principal of the


R = b, s — $10,000. debt is - $300. Semi-annual expense is 300 +
1921.58 — $221.58.
(b) TABLE SHOWING GRowTH OF SINKING FUND

4%
IN FUND AT PAYMENT mo
Parn.NT OF RECEIVED ON F u- FUND IN Fun END or
INERVAI., ND AT END or INTERVAL
INTÄBVAL AT END OF INTERVAL
INTERVAL

$1921.584 $ 38.432 $1921.584 $1921.584


1 3881.600 77.632 1921.584 3881.600
2 5880.816 117.616 1921.584 5880.810
3 7920.016 158.400 1921.584 7920.016
4 1921.584 10000.000
139 MATHEMATICS OF INVESTMENT

5
NOTE 2. — The book value of the debtor's indebtedness, or his net
indebtedn— at any time may be defined the difference between what he owes
and what he has in his sinking fund. Thus, at the end of 2 years, the book value
of the debt is 10000 — 7920.016 — $2079.984.
PAYMENT OF DEBTS BY PERIODIC INSTALLMENTS
140

Nom 3. — 'nie amount in the sinking fund at any time is the amount of the
payment annuity up to that date and can be found without forming the table (b).
Thug, the amount in the fund at the end of 2 years is at .02) - $7920.02.
EXERCISE xxxvl

A debt of $50,000, with interest payable semi-annually at the


rate 6%, is to be paid at the end of 3 years by the accumulation of a
sinking fund. (a) If payments to the fund are made at the end of
each 6 months and accumulate at the rate 3%, compounded semi-
annually, what is the total semi-annual expense of the debt? (b)
Form a table showing the accumulation of the fund.
2. (a) In problem 1, without using the table, determine the
amount in the sinking fund at the end of 2 years. (b) What is the
book value of the debtor's indebtedness at this time?
3. A loan of $10,000 bears 5% interest, payable semi-annually,
and 8 sin-king fund is created by payments at the end of each 6 months in
order to repay the principal at the end of 4 years. Find the expense of the debt
if the fund accumulates ot the rate (.04, m = 2).
4. A city issues $100,000 worth of bonds bearing 6% interest, payable annually,
and is compelled by law to create a sinking fund to retire the bonds at the end of 10 years. If
payments to the fund occur at the end of each year and are invested at 6%, effective, what is
the annual expense of the debt?

6. How much is in the sinking fund in problem 4 at the beginning


of the 7th year?
6. A loan of $5000 ig made under the agreement that interest shall be paid
semi-annually at the rate 5.5% on all principal remaining due and that the principal shall be
paid in full on or before the end of 6 years. (a) Find the semi-annual expense if the debt is
amortized by equal paymentg at the end of each 6 months for 6 years. (b) Find the semi-annual
to retire the debt by the sin-king fund method at the end of 6 years, if payments to the fund are
made at the end of each 6 months and are invested Lt (.04, m 2). (c) Find the semi-annual
expense under the sinking fund method if the fund earns (.06, m = 2). (d) Which method iB
most advantageous to the debtor?
PAYMENT OF DEBTS BY PERIODIC INSTALLMENTS
141

7. A sinking fund is established by payments at the end of


each 3 months in order to accumulate a fund of $300,000 at the end
of 15 years. Find the quarterly payment if interest is at the rate (.06,
m — 2).
142 MATHEMATICS OF INVESTMENT

8. To accumulate a fund of $100,000, payments of $5000


are invested at the end of each 6 •months at the rate 5%, compounded semi-annually.
(a) How many full payments must be made? (b) How much must be
paid on the last payment date?

38. Comparison of the amor&ation and the sinking fund


— To amortize a debt of $Å, in n years, with interest payable
annually at the rate i, by payments of $R at
the end of each year, we have A =

(42)
(am at i)

1 1

Since

(43)
(sm at i)
If the sinking fund method is used to retire this debt, the amount in
the fund at the end of n years is $1. If payments of $W are made
to the fund at the end of each year and
accumulate at the effective rate r, then A = W(Yl at r) or
1
(44)
(SKI at r)
The annual interest on $A at the rate i is Al, BO that the total
annual expense E under the sinking fund plan is
1
143 MATHEMATICS OF INVESTMENT

(45)
(sm at r)
When r — i in equation 45, E equals R, as given in equation 43.
Thus, the amoråzation payment $R is sumcient to pay interest on
$A at rate i, and to create a sinking fund which amounts to $A at
the end of n years, if the fund also eams interest at the rate i.
Hence, the amortization method may be considered as a special
case of the sinking fund method, where the creditor is custodian of
the sinking fund and invests it at the rate i.
When r is less than i, (Sil at r) is less than (Sil at i), so that
1 1 at r) is greater than A (sm at i)
and E is greater than R. (5
Sim.narly, when r is greater than i, the sinking fund expense is less

than the amortiza,tion expense.


MATHEMATICS OF INVESTMENT
NOTE. — The conclusions of the laat paragraph are obvious without the use
of any formulas. If the debtor is able to invest his fund at the rate r, greater than
i, his expense will be le8S than under the amortization method because, under
the latter, he is investing B sinking fund with his creditor at the smaller rate i.
N0TE. — Equation 42 is sometimeg called the amoräzadon equadon, and
equation 44 is called the sinwngfund equaåon. Table for —may be called a table
of amortization charges for a debt of $1 (A = I in equation 42), and a table of the
values of — would be a table of sinkingfund charge8 for a debt of $1.

bflSCELLMEOUS PROBLEMS
In eolving the more diffcult problems of the set below, the student should
recall that the writing of an equation of value, for a conveniently selected
comparison date, furnishes a systematic method of solution, aa in illustrative
Example 2 of Section 36.
I. At the end of 5 years, man will pay $15,000 cash for a house. (a) What
equal a,mounts should he save at the end of each year to accumulate the money
if his savings earn 6%, effective? (b) What should he save at the beginning of
each year in order to accumulate the money if the savings earn 6%, effective?
2. A loan of $5000 is made, with interest at 6%, payable Hemi-
annually. Is it better to amortize the debt in 6 years by equal semi-
annual inBtallments, or to pay interest when due and to retire the
principal in one installment at the end of 6 years by the accumulation
of a sinking fund by semi-annual payments, invested at (.04, m = 2) ?
8. A man, purchasing a farm worth $20,000 cash, agrees to pay
$5000 cash and $1500 at the end of each 6 months. (a) If the
pdyments include annual interest at the rate 5%, effective, how
ma,ny full payments of $1500 will be necessary? (b) What is the
purchaser's equity in the house at the beginning of the 3d year?
4. A county has an assessed valuation of $50,000,000. The
county borrows $500,000 at 5%, payable annually, and ig to retire the
principal at the end of 20 years through the accumulation of a sinking
fund by annual payments invested at 4%, effective. By how much, per
dollar of assessed valuation, will the annual taxæ of the county be
raised on account of the expense of the debt?
PAYMENT OF DEBTS BY PERIODIC INSTALLMENTS
145
5. A debt of $25,000, with interest payable semi-annually at
the rote
6%, is to be amortized by equal payments, at the beginning of each 6
months for 12 years. (a) Determine the payment. (b) At the beginning
of the 4th year, after the payment due has been made, what principal
remains outstanding ?
6. A debt of $12,000, with interest at 5%, compounded
quarterly, is to be amortized by equal payments at the end of each 3
months for 8 years.
• At the end of 4 years, what payment, in addition to the one due,
would cancel the remaining liability if the creditor should permit
the future payments to be discounted, under the rate (.04, m =
4) ?
7. A debt of $100,000, with interest at 5% payable annually,
will be retired at the end of 10 years by the accumulation of o
sinking fund by annual payments invested at 4%, effective.
Considering the total annual expense of the debtor as an annuity,
under what rate of interest is the present value of this annuity equal
to $100,000? The answer obtained is the rate at which the debtor
could afford to amortize his debt, instead of using the sinking fund
method described in the problem.
8. In order to accumulate a fund of $155,000, o corporation
invests $20,000 at the end of each 3 months at the rate (.08, m = 4).
(a) How many full payments of $20,000 must be made? (b) Three
months after the last full payment of $20,000 iB made, what partial
payment will complete the fund ?
9. The original liability of a debt was $30,000, and interest
iB at the rate 5%, effective. Payments of $2000 were made Lt the
end of each year for 7 years. At the end of that time it was decided to
amortize the remaining indebtedness by equal payments at the end of
each year for 8 years. Find the annual payment.
MATHEMATICS OF INVESTMENT
HINT. —Equations of value furnieh a systematic method for solving
problams of this type. Let $$ be the annual payment. Then, the voluo of the
"Old Obligation " below must equal the sum of the values of the 'C New
Obligations " on whatever comparison date is selected.
OLD OBLIGATION NEW OBLIG&TIONB

$30,000 due at the (a) $2000 due at the end of each year for 7
beginning of the years.
transaction. (b) Eight annual payments of $z, the first
due at the end of 8 years.
The end of 7 years is the most convenient comparison date.
10. At the end of 5 years, $10,000 must be paid. (a) What
equal amounts should thd debtor deposit in a savings bank at the end
of each 6 months in order to provide for the payment, if his savings
accumulate at the rate (.04, m = 2)? (b) How much must he deposit
semi-annually if his first deposit occurs immediately and the last at
the end of 5 years?
11. A debtor borrows $20,000, which is to be repaid, together
with all accumulated interest at the rate (.05, m = 4), at the end of 6
years. (a) In order to pay the debt when due, what equal deposits
must be made in a sinking fund at the end of each 6 months if the
fund accumulates at the rate (.05, m = 2)? (b) At what rate,
compounded semiannually, could the debtor just as well have
borrowed the $20,000, under the agreement that it be amortized in 6
years by equal payments at the end of each 6 months ?
12. A certain state provided for the sale of farms to war
veterans under the agreement that (a) interest should be computed at
the rate (.04, m = 2), and (b) the total liability should be discharged
by 10 equal semiannual installments, the first due at the end of 3å
years. Find the necessa,ry installment if the farm is worth $6000
cash.
13. Under what rate of interest will 25 semi-annual payments
of $500, the first due immediately, amortize a debt of $9700?
Deter±e both the nominal rate, compounded semi-annually, and the
effective
PAYMENT OF DEBTS BY PERIODIC INSTALLMENTS
147
rate.
14. A sinking fund is being accumulated by payments of
$1000 at the end of each year. For the first 10 years the fund earns
6%, effective, and, for the next 6 years, 4%, effective. What is the
size of the fund at the end of 16 years? It is advisable, first, to find
the amount at the end of: 16 years due to the payments during the
first 10 years.
15. In order to create a fund of $50,000 by the end of 20
years, what equal payments should be made at the end of each 6
months if the fund accumulates at the rate (.04, m = 2) for the first 10
years and at 6%, effective, for the last 10 years?
16. A fund of $250,000 is given to a university. The principal
and interest of this fund are to provide payments of $2000 at the
beginning of each month until the fund is exhausted. If the university
succeeds in investing the fund at 5%, compounded semi-annually,
how many full payments of $2000 will be made?
17. Under what nominal rate, compounded semi-annually, would it be
just as economical to amortize a debt in 10 years by equal payments at the end
of each 6 months, as to pay interest semi-annually at the rate
MATHEMATICS OF INVESTMENT
• 92
6%, on the principal, and to repay the principal at the end of 10
years by the accurnulation of a sinking fund by equal payments at
the end of each 6 months, invested at the rate (.04, m = 2).
18. A factory is worth $100,000 cash. At the time of purchase
$25,000 was paid. Payments of $8000, including interest, were
made at the end of each year for 6 years. The liability was
completely discharged by six more annual payments of $9000,
including interest, the first occurring at the end of the 7th year.
What effective rate of interest did the debtor pay?
HINT. —Write an equation of value at the end of 6 years. Transpose all
quantities in the equation to one side and solve by interpolation. Sec Appendix,
Note 3, Example 2.

SUPPLEMENTARY MATERIAL
39. Funds invested building and loan —A building
and loan is a coöperative enterprise whose main purpose is to provide
funds from which loans may be made to members of the association
desiring to build homes. Some members ore investors only, and do
not borrow from the association. Others are simultaneously
borrowers and investors. Shares of stock are sold to membe18
generally in units of $100. Each share is paid for by equal periodic
installments called dues, payable at the beginning of each month.
Profits of the association arise from investing the money received as
dues. Members share in the profits in proportion to the amount they
have paid on their shares of stock, and their profits are credited as
payments on their stock. When the periodic payments on a $100
share, plus the credited earnings, have reached $100, the share is said
to mature. The owner may thep withdraw its value or may allow it to
remain invested with the association.
Over moderbte periods of time, the interest rate received by an
association on its invested funds is approximately constant. The
amount to the credit of a member, who hag been making periodic
payments on 8 share, iB the amount of the annuity formed by his
payments, •with interest at the rate being earned by the association.
Example 1. — A member pays $2 at the beginning of each month on a share
in an association whose funds earn 6%, compounded monthly. What is to the
credit of the member just after the. 20th payment ?
PAYMENT OF DEBTS BY PERIODIC INSTALLMENTS 149
Solution. — The payments form an ordinary annuity of 20
pay-ment intavals, if the term is considered to begin 1
month before the first payment is made. The amount on the
20th payment date is the amount of this ordinary annuity,
or 2(8äöl at .005) = $41.96.
EzampZe 2. — A member pays $1 at the beginning of each month on
$100 share. If the association is earning at the rate (.06, m = 12),
when will the share mature ?
Solution. — Let k be the number of installments necessary to bring the amount to
the member's credit up to $100.
Case 1 n 100 at .005).
By interpolation in Table VII, lc —
int. periods, 81.8. Just after the 81st payment, at the
.00 beginning of the 81st month, the member iB
5, credited with (8äi1 at .005) = $99.558. By
R = $1, S = the beginning of the 82d month, this book
$100 value has earned (.005) (99.558) = $.498,
and the new book value ig 99.558 + .498 = $100.056.
Hence, no payment is neceasary at the beginning of the
82d month. Properly speaking, the share matured to the
value $100 at a time during the 81st month, but the
member ordinarily would be informed Of the maturity at
the beginning of the 82d month.
Example 8. — A $100 share mature just after the
83d monthly payment of $1. (a) What rate,
compounded monthly, is the association earning on
its funds? (b) What is the effective rate?
Solution. — (a) lat r be the rate per period of 1 month.
The amount of the payment annuity ig
Case 1 n $100,
83 int. periods, 100 Gil at r).
B — $1, S =
$100. By interpolation in Table VII, r =
-F
= .441%. The nominal rate is
12(.00441) = .0529, compounded monthly. (b) The
effective rate i ig obtained from
12
1 + i — (1.00441) * 1.05422. (By Tab1e 11)
Hence i = .0542, approximately.

EnRCISE xxxvrr
1. A member pays $1 at the beginning of
each month on B share in on asociation which
MATHEMATICS OF INVESTMENT
earns 5%, compounded monthly. What is to the
member's credit just after the 50th payment?
2. When will the share of stock in problem
1 mature to the value $100, and what payment, if
any, will be necesaary on the maturity date?
8. If an association earns 6%, compounded monthly, when will $100
share mature if the dueg are $2 per month per share?
MATHEMATICS OF INVESTMENT

94
4. The monthly due on B $100 share is $.50, and the share
will mature, approximately, just after the 131st monthly payment. (a)
What rate, compounded monthly, does the earn on its funds? (b)
What is the effective rate?
5. An association earns approximately 6%, compounded
quarterly, on its funds. (a) Find the date, to the nearest month, on
which a $100 share will mature if the monthly due is $1, making your
computation under the assumption that $3 is paid at the beginning of
eaph 3 months. (b) Without any computation, tell why your result, as
computed in (a), is smaller (or larger) than the actual result.
6. The monthly due on each $100 share is $1, in an
association where the shores mature just after the 80th payment.
What is the effective rate eamed on the shares ?
7. A man paid $30 per month for 80 months as dues on
thirty $100 shares in am association, and, to mature his stock at the
beginning bf the 81st month, a partial payment of $6 was necessary.
At what rate, compounded monthly, did his money increase during
the 80 months?
HINT. —If he should pay $30 at the beginning of the 81st month, the book value of his sharæ
would be $3024.
40. Reårement of loans made by building and loan associations. —
If a man borrows $A from a building and loan association, he is
usually caused, at that ti.me, to become a member of the association
by subscribing for $Å worth of stock. He must pay monthly interest
(usually in advance) on the principal of his loan and dues on his
stock. When his stock matures, with the value $A, the association
appropriates it as repayment of the principal of the loan. Thus, if a
man borrows $2000 from an association which charges borrowers 7%
MATHEMATICS OF INVESTMENT

interest, payable monthly in advance, the interest due bt the


beginning of each month is (2000) = $11.67. The borrower subscribes
for twenty $100 shares of stock on which he pays $20 as dues at the
beginning of each month if the due per share is $1. Thus, the monthly
expense of the debt is $31.67. Payments continue until the twenty
shares mature with the value $2000, Bt which time the association
takes them ag repayment of the principal of the debt, and closes the
transaction. This method of retiring a debt is essentially a sinking
fund plan, where the debtor's fund is invested in stock of the
association. The debtor iB benefited by this method because the rate
received
PAYMENT OF DEBTS BY PERIODIC INSTALLMENTS
153
on his stock investment is higher than could safely be obtained in
the outside market.
Ecample 1. — A ma,n borrows $2000 under the conditions of the
preceding paragraph. If shares in the association mature at the end
of 82 months, at what nominal rate, compounded monthly, may the
borrower consider that he is amortizing his debt?
Solution. — The debtor pays $31.67 at the beginning of eRßh month for 82
months. We wish the rate under which $2000 is the present value of this
annuity due. Let r be the unknown rate per period of 1 month. The first $31.67
is paid and the remaining 81 payments form an ordinary annuity, .under Case I,
with p = I. Hence,
2000 = 31.67 + at r),
1968.33
(aäTi at r) = 31.67 = 62.151.

By interpolation in Table VIII, r = F/O + = .681%. The norninal rateiB


12(.00681) .082, approximately, compounded monthly. The effective rate i, if
desired, can be obtained, with the aid of Table Il, from
1 + i = (1.00681)12.
EXERCISE
1. An charges borrowers 6% interest payable
monthly in advance and issues $100 shares on which the monthly
dues are $1 per share. (a) At what rate of interest, compounded
monthly, may a borrower consider his loan to be amortized, if
shares in the association mature at the end of the 84th month,
without a payment at that time? (b) What is the effective rate of
interest?
2. Which would be more profitable, to borrow from the
association of problem 1, or •to pay 5% interest monthly in advance
to some other lending source, and to repay the principal of the loan
at the end of 84 months by the accumulation of a sinking fund, at
the rate 6%, compounded monthly, if payments to the fund are made
at the beginning of each month for 84 months?
3. An association charges borrowers 7% interest, payable
monthly in advance, and issues $100 shares on which the monthly
dues are $1 per share. If the shoree mature at the end of 80 months,
without a payment at that time, at what effective rate does a
borrower amortize his debt ?
4. An association charges 6% interest payable monthly in
advance, and issues $100 shares on which the monthly due per share
is $.50. If the shares mature at the end of 130 months, without a
payment at that time, what is the effective rate paid by a borrower?
CHAPTER
DEPRECIATION, PERPETUITIES, AND CAPITALIZED COST
41. Depreciation; sin.king fund plan. — Fixed assets, such as
buildings and machinery, diminishin value through use.
Depreciadon is defined as that part of their loss in value which
cannot be remedied by current repairs. In every business enterprise,
the effects of depreciation should be foreseen and funds should be
accumulated whose object is to supply the money needed for the
replacement of assets when worn out. The deposits in these
depreciation funds are called depreciaflon charges, l and are deducted
periodically, under the heading of expense, from the cur- • rent
revenues of the business.
NOTE 1. — The replacement cost of an asset equals its cost when new
minus its salvage or scrap vdue when worn out. if a machine costs $1000, and
hag 8 scrap value of $50 when worn out at the end of 10 years, its replacement
cost is $950, the amount needed in addition to the scrap value in order to buy a
new machine worth $1000. The replacement cost iB also called the wearing
value ; it iB the value which iB lost through wear during the life of the asset.

A depreciation fund iB essentially a sinking fund whose amount


at the end of the life of the asset equals the replacement cost. Many
different methods are in use for estimating the proper depreciation
charge. Under the sinking fund method, the periodic depreciation
charges are equal and are invested at compound interest at a
PAYMENT OF DEBTS BY PERIODIC INSTALLMENTS
155
specified rate. Under this plan, (a) the depreciation charges form an
annuity whose amount equals the replacement cost, and (b) the
depreciation fund is a sinking fund to which we may apply the
methods for sinking funds used in Section 37.
Example 1. — A machine costa $1000 and it will wear out in 10
years.
When worn out, its scrap value is $50. Under the sin.king fund plan,
1
In this book, unleæ otherwise specified, we shall always eagume QIat for depreaiaåon during each
year is m8de at the end of åe year,
96
DEPRECIATION AND CAPITALIZED 156
COST

determine the depreciation charge which should be made at the end


of each year for 10 years, if the fund iB invested at 5%, effective.
Solution. — Let $$ be the annual charge. The replacement cost S $950.

Case 1 n =
10 int. periods, p =
l, i = .05,
R $4 S = $950.
950 = at .05),
1
C 950 - 375.529.
(8m at .05)

DEPRECIATION TABLE

INT. Dn ON PAYMENT TO
IN FOND YAA.R
FUND AT END FUND END
END or Ynu
OF or

O $1000.oo
1 $0 $0 75.529 924.47
2 3.777 75.529
154.835 845.16
3 7.742 75.529
238.106 761.89
4 11.905 75.529
325.540 674.46
16.277 75.529
5 417.346 682.65
20.867 75.529
6 513.742 486.26
25.687 75.529
7 614.958 386.04
30.748 75.529
721.235 278.76
8 36.062 75.529
832.826 167.17
9 41.641 75.529
949.996 50.00
75.529
10
In Figure 5 the growth Of the fund in an enterprise should be kept intact.
and the in the book value are Thus, at the end of 2 years in Example
represented graphically. I, the $154.84 in the fund should be
Non 2. — A good considered AB capital, originally in
depreciation plan iB in harmony vested in the machine, which has been
with the fundamental principle of returned through the revenues of the
economics that capital invested businesa. The book value of the
machine, 1000' — 154.84 — $845.16,
DEPRECIATION AND CAPITALIZED 157
iB the amount of capital still
invested in the mach.ine.
1000
DEPRECIATION AND CAPITALIZED 158

900
800
700
600
500
4
0
0
30

200
100

1 2 3 4 5 6 7 a 9 10
Years
FIG. 5
DEPRECIATION AND CAPITALIZED
98

The per cent of an asset at any time is


the ratio of its reznaining wearing value to
its wearing value when new.
Exampte 2. — In Example 1, find the condition
per cent at the end of 6 years.
Solution. — Oriänal wearing value iB $950. Book value
at end of 6 years is
$486.26, and the remaining wearing value is 486.26 — 50
= $436.26. The 436.26
condition per cent is — .45923 or 45.923%.

EXERCISE xxnx
1. In illustrative Example 1 above, find
the amount in the depreciaåon fund at the end
of 4 years, without using the depreciation
table.
2. A machine costs $3100 when new, it wears out in 12
years, and its final scrap value is $100. Under the sinking fund
plan, determine the depreciation charge which should be made at
the end of each 6 months if the fund accumulates at (.05, m = 2).
8. (a) In problem 2, without forming a
depreciation table, find the amount in the
depreciation fund, and the book value of the
asset, at the eud of 5} years. (b) What ig the
condition per cent at this time?
4. A building costs $100,000 and it will last
20 years, at the end of which time its salvage
value iB $5000. Under the sinking fund plan,
when the fund earns 5%, effective, determine
the size of the depreciation fund at the end
of 6 years, if deposits in the fund are made
at the end of each year.
6. A motor truck has an original value of
$2500, a probable life of 6 years, and a final
salvage value of $200. Its depreciation is to
be covered by deposits in a fund at the end of
MATHEMATICS OF NVESTMENT
each 8 months. Find the quarterly deposit if
the fund earns (,055, m — 2).
6. A manufacturing plant is composed of port (a) whose
cost is $90,000, life is 15 years, and salvage value is $6000,
and part (b) whose cost is $50,000, life is 20 years, and salvage
value is $5000. If depreciation charges are made at the end of
each year and accumulate at effective, what is the total
annual charge for the plant?
42. Staight line meü10d. — Consider am asset whose
life is n years and whose replacement cost (wearing
value) is $S. Suppose that annual depreciation charges
are placed in a fund which does not earn interest.
Then, in order to have $S at the end of n years, the
annual charge must be —. The fund increases each
cosT 99)

year by —, and hence the book value decreases


each year by
n
This method is called the s&aight line method,
because we obtain straight lines as the graphs
of the book value and of the amount in the fund
(see problem 1 below).
N0Tn. — An essential characteristic of the straight
line method iB that, it, all of the annual decreases
in book value are equal to —th of the total
n wearing
value S. It is usually stated, under thig method, that —
is written of
the book value each year for depreciation. Recall, from
the table in Example 1 of Section 41, that the annual
decreases in book value are not equal under the sinking
fund plan; they increaae as the asset grows old.
Norm. — The straight line method is the special coge
of the sinking fund method, where the rate earned on the
depreciation fund is 0%. Hereafter, any general
reference to the use of the sinking fund method should
be understood to include the straight line plan as one
possibility. In supplementary Section 48 below, another
depreciation plan iB considered which applies well to
assets whose depreciation in early life is large
DEPRECIATION AND CAPITALIZED
compared to that in old age. The student is referred to
textbooks on valuation and on accounting for many other
special methods which are in use. Usually, each of these
is particularly desirable for a certain type of aaset8.

EniCISE m,
1. A building costs $50,000 and hag a 88,1vage
value of $5000 when worn out at the end of 15 years.
(a) Under the straight line method, form a table
showing the amount in the depreciation fund and the
book value at the end of each year. (b) Draw separate
graphs of the book value and of the amount in the fund,
using the years as abscissas.
2. In problem I, find the annual depreciation
charge under the Binking fund method, where the fund
earns 4%, effective, and compare with the charge in
problem 1.
43. Composite life. — If the plant of an
enterprise consists of several parts whose
lives are of different lengths, it is useful to
have 8 definition for the average or composite
life of the plant as a whole. Let $S be the sum
of the wearing values (replacement costs) and
$D the sum of the annual depreciation charges
for all parts. Under the sinking fund method
let i be the effective rate earned on the
depreciation fund. Then, the composite life is
defined as the term of years, necessary for an
annuity of $D,
MATHEMATICS OF
NVESTMENT
100
paid annually, to have the amount $S. If lc is the composite life in
years, then
S = D(sn at 'i), (46) which can be solved for
Ic by interpolation.
Under the straight line method of depreciation, lc is the number
of times $D must be paid into a fund in order that the fund, without
earning interest, should equal $S. Hence

(47)
Non. —In equation 46 place i = 0%. Then (8 at O) k and equation 46
becomes S — kD, found above. This is an obvious consequence of the fact that
the straight line method is the sinking fund method with i = 0%.
Ezample I. — A plant consists of paa't A, with life 20 years,
original cost $55,000, and scrap value $5000; part B with life 15
years, original cost $23,000, and scrap value $3000; part C, with
life 10 years, original cost $16,000 and scrap value $1000.
Determine the composite life, (a) under the sinking fund method,
with interest at 4%, effective, and (b) under the straight line
method.
Solution. — (a) The total wearing value is 50,000 + 20,000 + 15,000
= $85,000. Under the sinking fund method, the annual charge for part A is
1
= $1679.09. Similarly, the charges for B and C are $998.82
(8m at .04) and $1249.36, respectively. The total annual charge iB $3927.27.
Int k be the compoaite life. %en, the annuity of $3927.27, paid annually for
yeaÆ, should have the amount $85,000, or

Case I n = 85000 = at .04) ;


int. periods, p = I, .04) — 21.644.
i — .04,
R — 8927.27, S —
$85,000.
DEPRECIATION AND CAPITALIZED
By interpolation in Table VII, 15.90 yea18.

(b) Under the straight I.ine method, theannual depreciation chargu for the
parts A, B, and C are, respectively, $2500, $1333.33, and $1500. The total
annual charge ig $6333.33. The composite life is
15.938 years.

5333.33
Non. — Compare the results above. Whan the rate on the sinking fund is 0%
(the straight method), the composite life differs from the life when i 4%, by
•only (15.94 — 15.90) = .04 year. Thug, under the sinking fund method,
regardless of the rate earned on the fund, the composite life may be obtained
approximately by finding the life under straight line depreciation.
COST 401
EÆRCISE Ll
1. Find the composite life for the plant with parts A, B, and C
below, under the sinking fund method (a) at 3%, effective, and DPL
at 6%, effective, and under (c) the straight line method.
PART COST SCRA* VALUE

10 $20,500 $ 500
20 35,750 750
16 19,000 1000
44. Valuation of a mine. — A mine, or any Bimilar property, is
depreciable asset which becomes valueless when all of the ore is
removed. Part of the net revenue of the mine should be used to
accumulate a depreciation, or redemption fund, which will return
the original invested capital when the mine iB exhausted. The
revenue remaining, after the depreciation charge, is the owner's net
return on his investment.
Nom. — We shall assume, in this book, that the annual revenue, or royalty,
from a mine, or similar enterprise, iB payable in one installment at the end of
the year.
Example 1. —A mine, whose life is 20 years, costß $200,000
cash. What should be the net annual revenue in order to pay 6%
MATHEMATICS OF
NVESTMENT
interest, annually, on the invested capital, and to provide an annual
deposit for a redemption fund which accumulates at 4%, effective ?
Solution. — lat $$ be the annuol deposit in the sin-king fund to provide
$200,000 at the end of 20
Cage I n — 20 int. years.
200,000 = .04); $6,716.35.
periods,
Annual interest at 6% on $200,000 is $12,000.
R — sc, S = $200,000. Annual revenue required is 12,000 + 6,716.35 -
$18,716.35.
Mining engineers furnish accurate estimates, for any given mine,
of the life, lc years, and of the annual revenue, $R. Suppose that a
purchaser pays $P for 8 mine. If the annual revenue payments of $R
are exactly suffcient to amortize the original invested principal P at
the effective rate i, then P is the present value of the annuity formed
by the revenue installments, or
P= at i). (48)
MATHEMATICS OF
NVESTMENT
1102

Recall 1 that the amortization payments are exactly suffcient to pay


interest at the rate i on P and to accumulate a sin.ldng (redemption)
fund at the rate i to repay P at the end of k years. Therefore, the
price paid under the assumption that P iB amortized iB the price we
should obtain on assuming that the investor receives the rate i on
his investment and places the surplus revenue in redemption fund
which accumulates at the rote i.
Consider determining the purchase price P if the buyer desires
the effective rate i on his investment and is able to invest his
redemption fund at the effective rate r. Let $D be the depreciation
charge deposited annually in the redemption fund, which
accumulates to the amount P at the end of lc years. Then
1
P D(snatr); D=P
at r)
Annual interest on $P at the rate i is Pi. Since revenue = (interest on
capital) +
(depreciation
charge), (49)
R = Pi + D = Pi (50)
(51)
(sm at r)
Dcample 2. — The annual revenue from a mine will be $30,000 until it
becomes exhausted at the end of 25 years. What should be paid for the mine if
8% iB to be earned on the invested capital while a redemption fund
accumulates at 5%?
Solution. — From formula 51
30000 = $297,170. (Table IX)
MATHEMATICS OF
NVESTMENT

Norn.-—ln equation 51 place r — i, and use formula 39. One obtains P — at


i), ag obtained previously for this cage in equation 48.
EXERCISE
I. A purchaser paid $800,000 for a mine which will be exhausted
at the end of 50 years. What annual revenue from the mine will be
required to pay 7% on the investnent End to provide on annual
deposit in a redemption fund which accumulates 8t 5%, effective? 1
See Section 38.
DEPRECIATION AND CAPITALIZED
cosT 103
2. The annual revenue from a mine will be
$50,000 until the ore is exhausted at the end of 50
years. A purchaser desires 7% on his investment.
What should he pay for the mine if his redemption
fund accumulates (a) at 7% ; (b) at 5% ; (c) at 4%?
3. The privileges of a certain patent last
for 10 years and the annual royalties from it will
be $75,000. If a redemption fund can be accumulated
at 5%, what should an investor pay for the patent
rights if he demands 6% on his investment ?
4. An investor in an oil property desires 10%
on his investment and assumes that he can accumulate
a redemption fund at 5%. What should he pay for an
oil field whose net annual revenue for its 10 years
of life iB estimated at $100,000?
5. A purchaser of a wooden ship estimates
that the boat will be practicnlly valueless ot the
end of 6 years. If the net earnings for each of
these years will be $50,000, what should the
purchaser pay if he desires 8% on his investment and
accumulates a depreciation fund at 4% ?
45. Perpetuities. — A perpetuity is an annuity whose
payments continue forever. Present values 1 of perpetuities are
useful in capitalization problems.
Suppose that $1000 is invested at 6%,
effective. Then, $60 interest is received at
the end of each year, forever. Thatis, at
6%, the present va,lue of a perpetuity of
$60, paid annually, is $1000. Similarly, if
$4 is invested at the rate i, payable
annually, it will yield R =
Ai interest annually,
forever. Hence, the present value $A of
perpetuity of $R paid at the end of each
year ig obtained from Ai = R; or,
(52)
i
MATHEMATICS OF
NVESTMENT
NOTE I. — If, in the paragraph above, we change
the word year to intere8t period, it is seen that,
when the interest rate per period is i, the present
value of a perpetuity of $R, paid at the end of each
interæt period, is
Example 1. — At the end of each 6 months, $50 is required to
clean a statue. If money is worth (.04, m = 2), what is the
present value of all future renovation ?
Solution. — The future renovation cost* form a perpetuity whose præent
value, by formula 52 1B
.02
The notion of the amount of a perpetuity is meaningles and useless, since

the end of term of perpetuity does not exist.

104
Consider a perpetuity of $1 paid at the end of each lc years, and
let (a*. k at i) be its present value when money iB worth the
effective rate i. In order to find a formula for E, first let us
determine the installment $$ which, if paid into a fund at the end
of each year for years, will accumulate to $1 at the end of years.
Then, $1 is the amount of the annuity of $$ per annum and
1
1 = c(SE1 at i) ; c = • Therefore, a perpetuity of $$ per
at i) annum will create a fund from which
$1 can be paid at the end of each k yea,rs, forever. Hence, the
present value of the perpetuity of $1 at the end of each years is
equal to the present value of the perpetuity of $c, per annum.
Therefore, from formula 52 with
1

(sn at i)'
DEPRECIATION AND CAPITALIZED COST

1
(G.E ati) =•
(sn at i)
The present value $A of a perpetuity of $R paid at the end of
each lc years is

A= (53)
NOTE 2. — Thug, if money is worth (.05, m = 1), the present value of a
perpetuity Of $90,000 paid at the end of each 20 years iB
90000 1
= (.03024269) = $54,436.5. (Table IX)
.06 .05
If i is not a table rate, formula 53 must be computed by inserting the explicit
formula for (8El at i).
Nom 3. — Recognize that formula 52, or formula 53, applies to B perpetuity whose first
payment comes Lt the cnd of the first payment intarval. The præent value of a perpetuity due,
or of u deferred perpetuüv, can be obtained by the methods used for the corresponding type of
annuity.

EXERCISE
1. (a) Find the present value of a perpetuity which pays $100 at the end of each 3 months,
if money is worth (.08, m = 4). What is the present value if the payments occur Dt the
beginning of each 3 months ? u. An enterprise will yield $5000 net profit at the end of each
year. At 4%, find the capitalized value of the enterprise, where the capitalized value at is the
present value of all future earning.
105
S. A bridge must be repainted each 5 years at a cost of $8000. If money is
worth 5%, find the present value of all future repainting.
4. A certain depreciable aeset must be replaced at the end of each
25 years at a cost of $50,000. At 6%, find the present value of all future
replacement,s.
MATHEMATICS OF
NVESTMENT
5. Find the present value of an annuity of $1000 paid ot
the end of each year for 75 years, if money is worth (.04, m = 1).
Compare the result with the present value of perpetuity of $1000,
paid annually.
6. To repair a certain road, $1000 will be needed at the
beginning of the 4th year and annually thereafter. Find the present
va.lue of all future repairs if money is worth 5%, effective.
46. Capitalized cost. — The capitalized cost of an asset is
defined as the first cost plus the present value of an future
replacements, which it is assumed will continue forever. Let $C be
the first cost and $R the replacement cost of a,n asset which must
be renewed at the end of each Jc years. Then, the capitalized cost
$K equals $C plus the present value of a perpetuity of $R paid at
the end of each 76 years. When money is worth the effective rate i,
we obtain, on using formula 53, i (sn at if (54)
If the replacement
cost equals the first
cost, R = C. Then, on
changing

and on placing R = C in formula 54,

(See formula 39)

(55) i (am at i)
Ecample 1. — A machine costs $3000 new and must be renewed at the end
of each 15 years. (a) Find the capitalized cost when money is worth (.05, m =
I), if the final scrap value of the machine iB $500; (b) if the scrap value iB
zero.

Solution. — (a) Use formula 54 with C $3000 and R = $2500.


DEPRECIATION AND CAPITALIZED COST
2500 1
R = 3000 + _$5317.12.
.05
IOU MATIIICMAT,ICS or INVESTMENT

(b) When the scrup value is zero, C R = $3000. From


formula 55,
3000 1
= $5780.54.
.05- (am at .05)
NOTT I. — If the renewal cost of an IROt iB $R und
its life years, the annual deprcciution churgc $D iB
given by R = D(8R at i), or,

(8E)
1 (56)
TIIOHQ future deproeiution charges form a perpetuity
Of 3D, paid annually, whoso prcscnt vuluo iB or

thefirBtcogt depreciatio
valuoof al futur
plusthopresent charges.
NON 2.— Informula 54,multiplybothsidesn i Then "
by
1 56)
Ri - Ci+R = Ci + D. (See equation

Thus, if an enterprise earns intorCEt at the rate i on


the capitalized cost K, the revonuo IG provides for
tho interest Ci ut the rate i on the invested capital
C tund likcwige for the annual deproeiution charge D.
If two assets are available for serving the
same purpose, that one should be used whoso
capitalized cost is least. If their
capitalized costs are tho same, both assets
are equally economical.
Eæampla 2. — A certain type of pavement costs $12 per square
yard, Ibid in pltvce, and must be roncwed at the same cost every
10 years. How much could a highway commission afford to pay to
improve the pavement so that it would laat 15 years, if money is
worth 4%, effective?

Solution. —Let $$ be the cost par squuro yurd of the


improved type of pavement, whoso lifo ig 15 yoarB. If
this type is just na economical the old, its
aupitulizod cost must bo tho sumo. The capitalizcd
ooetß of the two types, as given by equation 55, are
equated below :
12 1 c 1 at .04)
at .04) •
12(a at .04) 12(11.1183874) — $16.450.
.04) 8.1108958

The commiseion oould afford to pay anything 108B than — 12) or


$4.450 to improve the old pavement.
DEPRECIATION AND CAPITALIZED COST

1 107.

EXERCISEI
1. Find the capitalized cost of a plant whose original cost is
$200,000 and whose life is 25 years, if its final salvage value is $15,000.
Money is worth 4%.
2. A section of pavement coating $50,000 has a life of 25 years.
Find its capitalized cost if the renewal cost iB $50,000, and if money is worth
3%.
3. If it costs $2000 at the end of each year to maintain section of
railroad, how much would it pay to spend, immediately, to improve the section
80 that the annual maintenance would be reduced to $500? Money is worth
5B.
4. A bridge must be rebuilt every 50 years at a cost of $45,000.
Find the capitalized cost if the first cost is $75,000, and if money is worth 3%.
5. One machine costs $15,000, lasts 25 years, and has a final
salvage value of $1000. Another machine for the same purpose cost $18,000,
lasts 28 years, and has salvage value of $2000. If money is worth 5%, which
machine should be used ?
6. Would it be better to use tile costing $18 per thousand
and lasting
15 years, or to use other material costing $22 per thousand and
lasting 20 years, if money is worth 5% and if neither material has a
scrap value ?
7. A corporation is considering the use of motor trucks
worth $5000 each, whose life is 4 years and salvage value is zero.
How much would it pay to spend, per truck, to obtain other trucks
whose life would be 6 years, and final salvage value zero? Money
is worth 5%.
DEPRECIATION AND CAPITALIZED COST

8. Theinteriorofaroom can be painted at a cost of $10 and the


painting must be repeated every 2 years. If money is worth 6%, how much
could one afford to pay for papering the room if the paper would need renewal
every 3 years?
9. A certain manufacturing plant involves one part worth $100,000
new and needing replacement every 10 years at a cost of $90,000, and a
second part costing $52,000 new and needing renewal every 12 years at a cost
of $50,000. What should be the net operating revenue in order to yield 7% on
the capitalized cost?
10. A certain dam will cost $100,000 and will need renewal at a
cost of $50,000 every 10 years. If money is worth how much could one afford
to pay in addition to $100,000 to make the dam of permanent type?
1
After this exercise the student may proceed immediately to the Miscellaneous Problems at the end of the
chapter,
176 MATHEMATICS OF INVESTMENT

SUPPLEMENTARY MATERIAL
47. Diffcult cases under perpetuities. — Perpetuities are met to
which formulas 52 and 53 do not apply. A systematic means for
finding the present values of an perpetuities is furnished by infinite
geometrical progressions.
Example 1. — If money is worth (.05, m — 2), find the present
value of a perpetuity of $6 paid at the end of each 3 months.
Solution. — The praent value $A of the perpetuity is the sum of the present valua of all of the
payments as listed below :

Payment of $6
due at end of 6 mo. 9 mo. etc. to infi.nity.
Present value of to infinitely
payment etc. many terms.
etc. • . • to infinitely many terms].

The bracket contains an infinite geometrical progression whose fi.rst term a

1 — (1.025B 1 — (1.025B (1.025)}


6 6
= $482.98. (Table X)

Nom — The 52 and 53 of Section 45 can be obtained by the method of


Example 1 (see problems 3 and 5 below).

EnRCISE UV
Use geometrical progressions unless otherwise directed.
1. The annual rent of perpetuity is $1000, payable in semi-
annual installments. Find the present value when money is worth
(.06, m = 1).
2. Find the present value of the perpetuity in problem I if
money is worth (.06, m = 4).
177 MATHEMATICS OF INVESTMENT

8. Derive the formula 52 for the present value of a perpetuity of $1


paid annually, when money iB worth the effective rate i. This
present vulue iB generally denoted by the symbol a. ; that is, (a. at i)
=
Bee Fomu.la 221 Beqti04 91,
178 MATHEMATICS OF INVESTMENT
1'09)
4. (a) By use of a geometrical progression, find the present value of a
perpetuity of $100 paid at the end of each 10 years, if money is worth 5%,
effective. (b) Compare with the result obtained by use of formula 53.
5. Derive formula 53 for R at i), the present value of a
perpetuity Of $R paid at the end of each lc year8, with money worth
the effective rate i.

6. At 6%, effective, find the capitalized value of an enterprise which yields a net
revenue of $500 at the end of each month.
7. Let (apat i) represent the present value, when money iB
worth i, effective, of a perpetuity whose annual rent is $1, paid in p
instal.lments 1
per year. Prove that (apat i) = -r = at i).
8. If money is worth (.06, m = 4), find
thepresentvalueofaperpetuity of $1000, paid Semi-annually, by use of
formula 53.
9. If money is worth (.05, m = 2), find the present value of a perpetuity of $100
paid monthly, by use of the result of problem 7.
10. An irrigation system has just been completed. There will be no repair expense until the end

of two years, after which $50 will be needed 8t the end of each 6 months. If money is worth 4%,
efféctive, find the present volue of the future upkeep. Solve by any method.

48. Constant percentage method of depreciation. — Under the


constant percentage method, the book value decreases each year by a
fixed percentage of the value Bt the beginning of the year. If the life
of the asset is n years, the constant percentage r, expressed a
decimal, must be chosen go that the original cost $C is reduced to
the residual book value $R at the end of n years. The decrease in the
first year is Cr, and the value at the end of 1 yeaå• is C — Cr
r). Similarly, the book value at the end of each year iB (1 — r) times
the value at the beginning of the year. By the end of n years, the
DEPRECIATION AND CAPITALIZED COST
original value C has been multiplied n times by (1 — r), or by (1 —
n
r) , and the residual scrap value Rig C(l — ry.
Therefore, we may obtain r from
C(l — = R, or, 1 —
Each annual reduction in book value is •the depreciation charge for that year, and, if we consider
all of these reductions in book value
placed in a depreciation fund which does not eam interest, the fund
will contain the replacement cost at the end of n years.
1. — For a certain asset, the original cost is $3000, the life is 6
years, and the scrap value is $500. Find the annual percentage of de-
preciation under the constant percentage method and form a table
showing the changes in book value
Sotzdion. — From equation 57,

logå = 9.22185 - 10.


1 — r = .74183. log (1 = älog- = 9.87031 — 10.
r = .25817.
The book values in the table below were computed by 5-place logarithms.
Thus, Lt the end of 3 years, the book value iB B = 3000(1 — ; log (I — — 3 log (I
— r) - 3(9.87031 — 10) 9.61093 — 10 log 3000 = 8.47712 log B = 3.08805; B
- $1224.7.
DEPRDCIATION TABLD
Boor VALUE AT END OF DEPRECIATION IN DEPR. FUND
YEAR DURING YEAR END OF YEAN
YEAR

1 $2225.5 $774.6 $ 774.5


1651.0 574.5 1349.0
2
1224.7 426.3 1775.8
3 908.6 316.1 2091.4
4 674.0 234.6 2326.0
5 500.0 174.0 2600.0
6
Norm. — the scrap value R is relatively small, the method above gives
ridiculously high depreciation charges in the early years. The method breaks
down completely when R — O.
180 MATHEMATICS OF INVESTMENT
EXERCISE

1. (a) Find the annual percentage of depreciation under the


constant percentage method, for a machine whose original cost is
$10,000, life is 5 years, and scrap value is $1000. (b) Form o
depreciation table and draw a graph of the changes in book value.
2. In problem 1, find the annual depreciation charge under the
sinking fund plan, where the fund earns 4%, effective, and compare with the
result of problem 1,
111
3. A machine, whose life is 20 years, costs $50,000 when new and
has a scrap value of $5000 when worn out. Find the annual rote of depreciation
under the constant percenta,ge method.
4. For b certain asset, the depreciation in value during the
early years of its life is known to be very great, as compared with the
later years. Which of the three methods, straight line, sinking fund, or
constant percentage, would give a series of book values most in
harmony with the actual values during the life of the asset? Justify
your answer.

MISCELLANEOUS PAOBLEMS
Depreciation, in problems below, is under the sinking fund method.
I. An automobile costs $3500 when new, and its salvage value at the end of 6
years is $400. (a) If the depreciation fund earns 4%, by how much is the book
value decreased during the 4th year? (b) By how much is the book value
decreased during the 4th year, under the straight line method ?
2. A hotel hag been built at a cost of $1,000,000 in an oil-boom city
which will die at the end of 25 years. Assuming that the assets can be sold for
$100,000 at that time, what must be the net annual revenue during the 25 years to
earn 7% on the investment and to cover depreciation, where the depreciation
fund earns 4% ?
3. A syndicate Will build a theater in a boom city which will die at
the end of 30 years. For each $10,000 of net annual profit expected, how much
can the syndicate afford to spend on the theater if 8% is desired on the investment
DEPRECIATION AND CAPITALIZED COST
while a redemption fund to cover the initial investment is accumulated at 5%?
Assume that the theater will be valueless at the end of 30 years.
4. A machine, worth $100,000 new, will yield 12% net annual
operating profit on its original cost, if no depreciation charges are made. If the
life of the machine is 20 years, what annual profit will it yield if annual
depreciation charges are made, where the depreciation fund accumulates at 4%,
effective? Assume that the final salvage of the machine is zero.
6. The life of a mine is 30 years, and its net annual revenue iB $50,000. Find
the purchase price to yield an investor 7%, if the redemption fund accumulates at
4%.
6. A mine will yield a net annual revenue of $25,000 for 20 years. It wag
purchased for $200,000. If, at this price, the investor considers
182 MATHEMATICS OF INVESTMENT

that he obtains 10% on his investment, at what rate does he


accumulate his redemption fund ?
7. A certain railroad will cost $60,000 permileto build. To
maintain the roadbed in good condition will cost $500 per mile,
payable at the beginning of each year. At the end of each 30 years,
the traclß must be relaid at a cost of $30,000. What is the present
value of the construction and of all future maintenance aid renewals,
if money is worth 5% ?
8. Find the capitalized value at 6%, effective, of a farm whose net
annual revenue ig $3000.
9. An automobile with a *earingvalue of $1200 has a life
of 5 years. Upkeep and repairs coat the equivalent of $450 at the
183 MATHEMATICS OF INVESTMENT

end of each year. What is the annual maintenance expense if the


owner accumulates a depreciation fund by annual charges invested
at 5% ?
10. A certain piece of forest land will yield a net annual
revenue of $25,000 for 15 years, at the end of which time the cut-
over land will be sold for $15,000. (a) If money iB worth 6% to an
investor, what should he pay for the property? (b) If the investor
desires 9% on his invested capital, and assumes that he can
accumulate a redemption fund at 5% to return his original capital at
the end of 15 years, find the price he B-hould pay for the la,nd, by
use of the method which was used in deriving formula 51 for the
valuation of mine.
CHAPTER Vll

BONDS
49. Terminolog. — A bond is a written contract to pay a
definite redemption price $C on a specified redemption date and to
pay equal dividends $D periodically until after the redemption date.
The dividends are usually payable semi-annually, but may be paid
a.nnually or in any other regular fashion. The principal $F
mentioned in the face of the bond iB called the face value or pa.r
value. A bond is said to be redeemed at par if C = F (as is usually
the case), and at a premium if C is greater thon F. The interest rate
named in a, bond is called the dividend rate. The dividend $D iB
described in a bond by saying that it is the interest, semi-annual or
otherwise, on the par value F at the dividend rate.
Nom. — The following iB an extract from an ordinary bond :
The Kamas Improvement Corporation acknowledges itself to owe
and, for value received, promises to pay to bearer FIVE HUNDRED
DOLLARS on January 18t, 1926, with interest on said sum from and
after January let, 1920, a.t the rate per annum, payable semi-annually,
until the said principal sum i8 paid. Furthermore, an additional 10% of
the said principal 8hau be paid to bearer on the date of redemption.
For thig bond, F = $500, C — $550, and the semi-annual dividend D = $15
is semi-annual interest at 6% on $500. A bond is named after its face F and
dividend rate, so that the extract is from a $500, 6% bond. Corresponding to
each dividend D there 1.1BuaIIy would be attached to the bond an individual
coupon containing a written contract to pay $D on the proper date.

50. When an investor purchases bond, the interest rate i


which he receives •on his investment is computed assuming that
he will hold the bond until •it is redeemed. It is important to
recognize that the investment rate i is not the same as the dividend
BONDS
rate of the bond, except in very special cases, because •i depends on
all of the following: $P, the price paid for the bond; $C, the
redemption price ; the time to elapse before the redemption date ;
the number of per year dividends are paid, and the size of $D, the
periodic dividend.
118
In this chapter we shall solve two principal problems. First,
the determination of the price $P which should be paid for a
specified bond, if we know the investment rate demanded by
the buyer. Second, the determination of the investment rate if
we know the price which the investor had to pay.
51. Purchase price to yield a given rate. — The essential
features of a bond contract are the promises (a) to pay $C on the
redemption date and (b) to pay the annuity formed by the periodic
dividends of $D.I If an investor desires a specified investment rate,
the price $P he is willing to pay on purchasing the bond is
P = (present value of $C due on the redemption date)
-k (present value of the annuity formed by the dividends),

where present values are computed under the investor's rate.


Ezample 1. —A $1000, 6% bond, with dividends payable semi-
annually, will be redeemed at 105% at the end of 15 years. Find the price
to yield an investor (.05, m = 1).
Solution. — At 105% means at a premium of 5% over the par VLIue.
F = $1000, C = 1000 + 50 = $1050. The semi-annual dividend D =
(.03)1000 = $30. The redemption price $1050 is due at the
end of 15 years. Hence, at the rate (.05, m = I),

Div. annuity, Cage 1n


= 15 int. periods, p 2, i
= .05, R $60.

1 When a bond is sold on a dividend date, the seller takes the dividend
3D which is due. The purchuer will receive the future dividends, which
form an ordinary anmEty whose first payment is due Bt the end of one dividend and
wh0% last payment is due on the redemption date.
186 MATHEMATICS OF INVESTMENT
P — at .05).

P - 505.07 + .05) = $1135.54.

EXERCISE XLVII

1. A $1000, 5% bond, with dividends payable semi-annually, will


be redeemed at 108% at the end of 7 years. Find the price to
yield an investor 6%, compounded semi-annually.
The bonds in the table are redeemable at par. Find the
purchase prices. The life is the time to the redemption date.
(115
Drv1DßND DIVIDENDS INVEBTMDN'r
PROB. PAB VALUE LIFE
RATE PAYABLE RATE

2. $ 1,000 10 yr., 6 mo. semi-ann. (.06,


3. 100 17 yr. annually (.07, m 1)
6%
14 yr.
4. 1,000 9 yr. 7% semi-ann. (.08, m 1)
5. 500 8 yr., 6 mo.
semi-ann.
6. 2,000 quarterly (.05,
13 yr., 6 mo. 3% (.06,
7. 1,000 19 yr. semi-ann.
semi-ann. (•04, m 2)
8. 100,000
5%
semi-ann. (.06,
9.
10. A $10,000, 5% bond, whose dividends axe payable annually, will be
redeemed at par at the end of 30 years. Find the purchase prices to yield (a)
5%, effective; (b) 7%, effective; (c) 4%, effective. Compare your results. ll. A
$1000, 6% bond, whose dividends are payable semi-annually, is purchased to
yield 5%, effective. Find the price if the bond iB to be redeemed at the end of
(a) 5 years; (b) 20 years; (c) 75 years. Compare your results.
12. A $100,000, 5% bond is redeemable at 110% at the end of 15 years, and
dividends are payable annually. Find the price to yield (.06, m = 2).
If a bond is redeemable at par (C = F) and if the investor's
interest period equals the interval between successive dividends, it
is easy to compute the premium (P — F), the excess of the price P
over par value F. Let k be the number of dividend periods to elapse
before the bond matures, r the dividend rate per dividend interval,
and i the investor's rate per interest period. Then, a dividend D = Fr
BONDS
is due at the end of each interest period and the redemption price F
is due at the end of lc periods. The equations below are easily
verified.

Div. annuity, Case


1 n = int. periods,

From formula 28, — Fi (afi at i) = —


Therefore, P — F = Fr(an at i)
Premium = P — F = F(r — at O. (59)

NOT 1. —Formula 59 shows that, when r iB greater than i, P — F is


positive, or the bond is purchased at a positive premium over par value F.
When r iB legs than i, P — F is negative or the bond is purchased at a
negative premium, that is, at a discount from the par value F.

Ecample 2. — A $1000, 6% bond, with dividends payable


semiannually, is redeemable at par at the end of 20 years. (a) Find
the price to yield an investor (.05, m = 2). (b) To yield (.07, m =
2).
Solution. — (a) From formula 59 with F = $1000, the premium iB P — F =
1000(.03 — .025) = at .025) = $125.51. P 125.51
$1125.51. (b) Premium = P — F = 1000(.03 — at .035) — .085) = — $111.78. P — 111.78 — $882.22. In this

cage, we say the discount is $111.78:


188 MATHEMATICS OF INVESTMENT
N0TE 2. — Equation 59 could have been proved by direct reaaoning.
Suppose r is greater than i. Then, if an investor should pay $F for the bond, he
would desire Fi as intereet on each dividend date. Since each dividend is Fr, he
would be receiving (Fr — Fi) F (r — i) excess income at the end of each
interest period for Ic periods. Hence, he should pay, in addition to $F,
premium equal to the present value of the annuity formed by the excess income
or — at O.. Similarly, when r iB less than i, if the investor should pay $Pfor the
bond, there would be a deficiency in income of Fi — Fr F(i at the end of
each interest period. Hence, the present value of the deficiency or — r) (a at i)
should be returned to the investor aa a discount from the price F wd supposed
paid.

VALUES TO NEARØST CENT or A $100,000, 5% BOND WITH


SEMI-ANNUAL DIVIDENDS
INVEST. TIME TO REDEMPTION Darry
RATE wrrz
101 Il YEARS
m2 12
.0400 108505.60 108829.02 109146.10 109466.96
108060.01 108365.61 108665.14 108958.72
.0410 107616.62 107904.58 108186.75 108463.25
.0415 107175.43 107445.93 107710.93 107970.54
106736.43 106989.64 107237.65 107480.56
.0430 .04 106299.59 106536.71 106766.91 106993.30
105864.92 106084.11 106298.69 106508.75
40 105432.40 105634.84 105832.97 106026.89
105002.01 105187.88 105369.74 105547.69
.0450
104573.75 104743.21 104908.99 105071.10
104147.61 104300.84 104450.70 104597.26
4
1117 )
N0TE 3. — To facilitate practical work with bonds, extensive tables have
been computed ahowing the purchase prices of bonds redeemable at par. i The
table on page 116 illustrates those found in bond tables.

EnRCISE LV111
In the future use formula 59 to find P whenever F = C and thø
investor's interest period equals the dividend interval. Otherwise
use the fundamental method involving formula 58.
BONDS
J I. Find the price to yield 4%, compounded ser.ni-annually, of D $1000, 5%
bond, with dividends payable semi-annually, redeemable at par at the end of
years.
2. Verify all entries in the bond table on page 116, corresponding
to the investment yields .04 and .045.
3. A $5000 bond, paying $100 dividend semi-annually, is
redeemable at par at the end of 11 years. Find the price to yield (.06, m = 2).
4. A man W signs a note promising to pay $2000 to M
at the end of 5 years, and to pay interest semi-annually. on the
$2000 ot the rate (a) What will M receive on discounting this
note immediately at a bank which uses the interest rate 7%,
compounded semi-annually? (b) What will M receive if the bank
uses the rate 7% effective?
5. Find the price to yield 5%, effective, of a $10,000, 7% bond,
with dividends payable annually, which is redeemable at par at the end of (a)
10 years; (b) 15 years; (c) 40 years. (d) Explain in a brief sentence how and
why the price of a bond changes as the time to maturity increases, if the
investor's rate iB less than the dividend rate.
6. Find the price to yield 6%, effective, of' $10,000, 4% bond
with annual dividends, which is redeemable Lt par at the end of (a) 5 years;
(b) 10 years; (c) 80 years. (d) Explain in one brief sentence how and why the
price of a bond changes as the ti.nae to maturity iB increased, if the investor's
rate is greater than the dividend rote.
52. Changes in book value. — On a dividend date, it is
convenient to use the term book value for the price $P at which a
bond. would sell under a given investment rate i. Recall that this
price $P, at which a purchaser could buy the bond, is the sum of
the present values, under the rate i, of all payments promised in
the bond. Hence, the dividends $D together with the redemption
payment $C are suffcient to pay interest at rate i on the invested
1
Sprague's CompZgtc Bond Tabla contain the purchuge prices to the nearest cent for a bond of $1 ,000,000 par
value, corresponding to wide range of investment rates.
principal $P, and to return the principal intact. If a bond is
purchased at a premium over the redemption price $C, only $C of
the original principal $P is returned at redemption. Therefore, the
remaining principal, which equals the premium (P — C)
originally paid for the bond, iB returned in installments, or is
amortized, through the dividend payments. Thus, each dividend
$D, in addition to paying interest due •on principal, provides a
190 MATHEMATICS OF INVESTMENT
partial payment of principal. These payments reduce the invested
principal, or book value, from $P on the date of purchase to $C on
the redemption date.
Example 1. —A $1000, 6% bond pays dividends semi-annually
and will be redeemed at 110% on July 1, 1925. It is bought on
July 1, 1922, to yield (.04, m = 2). Find the price paid and form a
table showing the change in book value and the payment for
amortization of the premium on each interest date.
D=$30. —
$1144.811.

TABLE OF Boor VALUES FOR A BOND B0ÜGff11 AT A PREMIUM


DATE INT. AT 4% DUE DIVIDEND Fon AMOET1zAT10N BOOR
ON Boor OF VALUE
VALUE
July 1, 1922 $22.896 330.000 $7.104 $1144.811
Jan. 1, 1923 22.754 30.000 7.246 1137.707
July 1, 1923
22.609 30.000 7.391 1130.461
Jan. 1, 1924
July 1, 1924 22.461 30.000 7.639 1123.070
Jan. 1, 1925 22.311 30.000 7.689 1115.531
July 1, 1925 22.157 30.000 7.843 1107.842
1099.999
On Jan. 1, 1923, for example, interest due on book value iB .02(1144.81)
$22.896. Hence, the $30 dividend pays the interest due and leaves (30 —
22.896) = $7.104 for repayment, or amortizotion, of the premium; the new
book value ig 1144.811 — 7.104 = $1137.707. The check on the computation
is that the final book value should be $1100, the redemption price.
If a bond is purchased at a discount from the redemption price,
that is, if P is legs than C, the redemption payment C exceeds the
original investment P by (C — P). Hence, this excess must be the
accumulated value on the redemption date of that part of the
interest on the investment which the payments of D on the
dividend dates were insuficient to meet. Therefore, on each
dividend date, the payment D iB less than the interest due on
119
invested principal ; the interest which is not paid represents a new
investment in the bond, whose book value is thereby increased.
BONDS
This writing up of the book value on dividend dates is called
accumulati.ng the discount because the book value increases from
P on the date of purchase, to C on the redemption date, the total
increase amounting to the original discount (C — P).
Ecampte 2. — A $1000, 4% bond pays dividends semi-annua,lly and will
be redeemed at 105% on January I, 1924. It is purcha,sed on January 1,
1921, to yield (.06, m = 2). Find the price and form .a table showing the
accumulation of the discount. Solution. —C = $1050, and D = $20.
P = 1050(1.03)-' + at .03) = $987.702.
TABLE OF BOOR VALUES FOR A BOND Bougrr AT A DlßC0üNT
INT. D tm DIVIDEND FOR A coüMü1..A- FE*AL Booz
ON Boor •mor or DISOOUNT

Jan. 1, 1921 $ 987.702


July 1, 1921 $29.631 $20.000 $ 9.631 997.333
Jan. 1, 1922 29.920 20.000 9.920 1007.253
July 1, 1922 30.218 20.000 10.218 1017.471
Jan. 1, 1923 30.524 20.000 10.524 1027.995
July 1, 1923 30.840 20.000 10.840 1038.835
Jan. I, 1924 31.166 20.000 11.165 1050.000
In forming the row for July I, 1921, for example, interest due at 6%
is .08(987.702) 29.631. OfthiB, only $20iBpaid. Thebalance, 29.631 — 20
$9.631, is con.Bidered aa a new invutment, raising the book value of the
bond to 987.702 -k 9.631 = $997.333. In his bookkeeping on July 1, 1921,
the investor records the receipt of $29.631 interest although only $20 actually
came into his hands. Also, his books show anew investment of $9.631 in the
bond.

Recognize that, when a bond is purchased at a premium, the


dividend D is the sum of the interest 1 on the investment plus a
payment for amortization of the premium, or
I = D — (amortization payment). (60)
Thug, in illustrative Example 1 above on Jan. 1, 1923, the interest is
22.896 = 30 — 7.104. In accounting problems this fact ig of importance. For
instance, if a trust company purchaées the bond of Example 1 for a trust fund,
$1144.81 of the capital is invested. Suppose that the trust company considers
all of each $30 dividend as interest and expends it for the beneficiary of the
fund. Then, on July 1, 1925, the company faces
192 MATHEMATICS OF INVESTMENT
an illegal loss of $44.81 in the capitol of the fund,
because $1100 is received at redemption in place of
$1144.81 invested. The company should consider only the
entries in the 2d column of the table of Example 1 aa
income for the beneficiary.
Similarly, when a bond is bought at discount,
I = D -k (payment for accumulation of the
discount). (61)
Thus, in illustrative Example 2 above on July 1,
1921, the interest is
29.631 = 20 + 9.631.
From equations 60 and 61 we obtain, respectively, (amortization
payment) = D — I,
(payt. for accumulation of discount) = I
— D.
Let PO and PI be the book values on two
successive dividend dates, at the same yield.
Then, if the bond iB at a prernium, PI equals PO
minus the amortization payment, or PI PO
(62)
If the bond is at a discount, PI equals PO plus
the payment for the accumulation of the discount
or PI = PO + (I — D), the game as found in
equation 62. Hence, equation 62 holds true for
all bonds.
EXERCISE

1. A $1000, 8% bond pays dividends semi-


annually on February 1 and August 1, and is
redeemable at par on August I, 1925. It is
purchased on February 1, 1923, to yield (.06, m =
2). Form L table showing {the amortization of the
premium.
2. A $1000, 5% bond pays dividends on-nually on March I, and is
redeemable at 110% on March 1, 1931. It is purchased on March 1, 1925,
to yield (.07, m = 1). Form a table showing the accumulation of the
discount.
BONDS
8. By use of formula 58, find the book value of the
bond of problem 2 on March 1, 1927, to yield (.07, m =
I) and thus verify the proper entry in the table of
problem 2. Any book value in the tables of problems 1
ond 2 could be computed in thig way without forming
the tables.
4. Under the investment rate '(.04, m =
1), the book value of a $100, 5% bond on
January 1, 1921, is $113.55, and dividends ore
payable annually on January I. Find the B.mount
of the interest on the investment, and of the
payment for amortization on January 1, 1922.
121
6. A $1000, 4% bond pays dividends annually on August 15 and is redeemable at
par on August 15, 1935. An investor purchased it on August 15, 1923, to yield (.06, m
= I). (a) Without forming a table, find how much interest on invested capital should be
recorded as received, in the accounts of the investor, on August 15, 1928. (b) How
much new principal does the investor invest in the bond on August 15, 1928?
53. Price at a given yield between interest dates. — The price of a bond
on any date is the sum of the present values of all future payments
promised in the bond. Let the investment rate be (i, m = 1), and suppose
the last dividend was paid — th year ago.
At that time, the price PO was the sum of the present values, 8t the rate (i,
m = 1) of all future bond payments. To-day, the price P is the sum of the
present values of these same payments because no more dividends have as
yet been paid. Hence,
P equals PO accumulated at the rate (i, m = 1) for — years, or
1
P = PO(I + i)Ä. (63)
Thig price is on a strict compound interest yield basis. In practice, P is
defined as PO, accumulated for - years at the rate i, simple
interest; P = PO 1 -k — i
194 MATHEMATICS OF INVESTMENT
1
P = PO + Poi (64)
That is, P equals PO plus simple interest on PO from the last dividend date at the investment rate i.
r
No m 1. — flae use of equation 64 favors the seller because it gives a slightly larger
value of P thon equation 63. The difference in price iB neg* ligible except in large
transactions. Use equation 64 in •all problems in Exercise L on page 123.
Ecampte 1. —A $1000, 6% bond, with dividends payable July 1 and January 1, is
redeemable at 110% on July 1, 1925. Find the price to yield (.05, m = 2) on August 16,
1922.
Solution. — July I, 1922, the lust dividend dote. The price PO then was PO =
+ .025) $1113.77. Simple interest on $1113.77 from July 1 to Aug. 16 at

the investment rate 5%, is $6.96. The price on Aug. 16 is P = 1113.77 + 6.96

81120.73.
It is proper to consider that the dividend on a bond accrues (or is
earned) continuously during each dividend interval. Thus, d days after a
dividend date, the
(accrued dividend) = (simple int. for d days on the face F at di+idend rate). (65)
Bcample 2. —In Example 1, find the accrued dividend on August 16, 1922.
Solution. — From July I to Aug. 16 iB 45 days. Accrued dividend iB $7.50.
NOTE 2. —In using equation 65, take 360 days as 1 your, and find the approximate number of days
between dates, as in expression 9, Chapter I,

When a bond is purchased at a given yield between interest dates, part


of the price P iB a payment to the seller because of the dividend accrued
since the last dividend date. The remainder of P is the present value of
futåre dividend accruals and of the future redemption payment. This
remainder of P corresponds to what was defined as the book value of a
bond in Section 52. Hence, between dividend dates, the price is
P — (Accrued Dividend to Date) + ßook Value) ; (66) (Book
Value) = P — Dividend). (67) Equation 67 is also true on dividend dates ;
the accrued dividend iB zero because the seller appropriates the dividend
which is due, and hence the book value and the purchase price are the
same, as they were previously defined to be in Section 52.
Example 3. — For the bond of Example 1 above, find the book value on August 16, 1922, to yield
(.05, m 2).
BONDS
Solution. —On Aug. 16, P — $1120.73, from Example I. The accrued dividend to
Aug. 16 is $7n, from Example 2. Book value on Aug. 16 iB 1120.73 — 7.50 =
$1113.23, from equation 67.
NOTE 3. — The accrued dividend, although carned, is not due•until the-next
dividend date. Hence, theoretically, in equation 66 we Bhould use, inBtead of the
accrued dividend, ita value discounted to date from the next dividend date. 'Ihus, in
Example 3 we should theoretically subtract the present value at
(.05, m — 2) on Aug. 16, 1922, of $7.50 due on Jun. 1, 1923, or 7.50(1.025)-} —
$7.35. The difference (in thie cue $.15) always is small unless a lurge transaction ig
involved and, hence, it is the practice to use equation 67 aa it stands.
(123)
EDRCISE L
1. A $1000, 8% bond, with dividends payable January 16 and
July 16, is redeemable at 110% on July 16, 1928. Find the purchase price
and the book value on September 16, 1921, to yield (.04, m = 2).
Find the purchase prices and the book values of the bonds below on the specified
dates. All bonds are redeemable at par.
PAR DIV. DIVIDEND REDEMP. DATE OF INVEST.
PROB. VAL RATE DATE RATE
UM

2. $1000 5% June I 6/1/1932 5/16/1922 (.07, m = 1)


3. 100 4% Jan. 1, July .1 7/1/1940 8/13/1931 (.04, m = 2)
4. 5000 May 1 5/1/1952 9/1' /1924 (.06, m 1)
5. 1000 June 1, Dec. 1 6/1/1937 8/16/1923 (.05, m = 2)
6. 100 May 1, Nov. 1 5/1/1934 3/1 /1927 (.07, m = 2)
The book value between dividend dates may be found very easily by
interpolation between the book values at the last and at the next dividend
dates. This method is especially easy if a bond table is available.
Example 4. — A $100, 6% bond pays dividends on July I and January I, and is
redeemable at par on January 1, 1940. Find the book value and the purchase price on
September 1, 1924, to yield (.04, m = 2).
Solution. — In the table below the book values to yield
BOOK (.04, m = 2) on 7/1/1924 and 1/1/1925 were computed from
DATE VALUE equation 59. Let B be the book value on Sept. I, which is of
the way from July 1 to Jan. 1. Hence, since 122.938 —
7/1/1924 $122,938 122.396 — .542, B = 122.938 — = $122.757. From equation
9/1/1924 B 66, tho purchase price P — 122.757 + 1 $123.757.
1/1/192 3122.396
5
196 MATHEMATICS OF INVESTMENT
EnRCISE Ll
1. A $1000, 4% bond pays dividends annually on July I, and iB
redeemable at par on July I, 1937. (a) By interpolation find the book value on
November 1, 1928, to yield (.06, m = I). (b) Find the purchase price on November 1,
1928.
2. Find the book value in problem 1 by the method of illustrative example
3, Section 53, and compare with the result of problem 1.
3. A $5000, 6% bond pays dividends semi-annually on May I and
November I, and iB redeemable at par on November I, 1947. By use of interpolation
find the book value and the purchaae price to yield (.04, m = 2) on July I, 1930.
4. A $1000, 5% bond, with dividends payable March 1 and
September 1, is redeemable bt par on March I, 1935. By use of the bond
table of Section 51, find by interpolation the book value to yield (.045, m
= 2) on April 1, 1924.

SUPPLEMENTARY N0E. — The interpolation method of illustrativc Ex ample 4, page


123, gives the same book VUIUO na is obtained by the method of Example 3, which uses
equation 67. To prove this, Ict thc time to the present from the last dividend date be —
thpart of n dividend interval. let PO be the
book value on the last, and PI that on the next dividend date, P the purchasc price to-
day, D the periodic dividend, I the interest on PO for whole dividend interval at the
investment rate, und B the book vulne of the bond to-day. First use the method of
Example 3. Interest to date ou PO at the investment rate iB — and the accrued
dividend to dute iB — (D). From equation
64,
equation 67, B
P = PO = P — ic¯n — PO +

(68)
By interpolation, as in Example 4, since the present iB —th interval from the

Boor
last dividend date, B is -th part of the way from PO to Pi,
DATE VALUE or B -(PI — PO). From equation 02, PI - PO = r - D, and
hence B —po + , the sumo in equation 68.
Last div. date
Present Equation 68 shows that, when a bond is selling at n
Next div. date discount, the accumulation of the discount in — th
BONDS
interval is —th of tho total accumulLtion for the interval, for, in equation 61 it ig seen
that I — D iB the accumulation for the whole interval. Similarly, if we write equation
68 as B — PO — it is seen that the amortization of the premium on bond in —th

interval iB —th of the amortization for the whole interval.

54. Professional practices in bond transacåons. — An investor buying a


particular bond cannot usually demand a, specified yield from his
investment. He must pay whatever price is asked for that particular bond
on the financial market. On bond exchanges,
125
and in most private transactions, the purchase price of a bond is described
to a purchaser as a certain quoted price plus the accrued dividend. l That is,
the market quotation on a bond is what we havé previously called the book
value of the bond, in equation 67.
Non 1.— The quotation for a bond iB given aa a percentage of its par value. That is,
a $10,000 bond, quoted Lt 93å, has a book value of $9325.
Example I. — A $10,000, 6% bond, with dividends poyoble June 1 and December
1, is quoted at on May 1. Find the purchase price.
Solution. — Quotation = book value = $9325.00. Accrued dividend since December
1 iB $250. From equation 66, tha price is 9325 + 250 = $9575.
Norm 2. — Bond exchange methods are simplified by the quotation of book values
instead of actual purchase 2 prices. If the yield at which a bond sells remains constant,
the book value changes very slowly through the accurnulation of the
discount, or amortization of the premium, as the case may be. Hence, whon
the practice is to quote book values, the market quotations of bonds chonge very
slowly and any violent fluctuation in them iB due to a distinct change in the yields at
which the bonds ore selling. On the other hand, if the actual purchase price of a bond
were the market quotation, the quotation would increase as the dividend accrued and
then, at each dividend date, a violent decrease would occur whan the dividend was
paid. Thus, even though the yield at which a bond were selling should remain constant,
large fluctuations in its market quotation would occur.

EXERCISE
1. (a) A $1000, 5% bond, with dividends payable February 1 and August I,
is quoted at 98.75 on May 1 ; find the purchase price. (b) If the bond is purchased for
$993.30 on April 1, find the market quotation then.
198 MATHEMATICS OF INVESTMENT
2. The interest dates for the 2d Liberty Loan bonds are May 15
and November 15. Take their closing quotation on the New York Stock
Exchange from the morning newspaper and determine the purchase price
for a $10,000 bond of this issue.
3. A $1000, 6% bond whose dividend dates are January 1 and
July .1 is quoted at 103% on October 16. Find the total price paid by a
purchaser if he pays a brokerage commission of of the par value.
In. bond mm•kot pÜlance, it is called accrued intcrcet. The more proper word
dividend hae been consistently used in this book to avoid pitfclly which confront the
beginner. AE seen in Section 52, equations 60 und 01, the dividend iB not the same the
interest on the investment. The terminology accrued intercat in bond dealings must be
loarned by the student and appreciated to mean accrued dividend in the gonse of this
chapter.
a
The purchase price iB called the flat price in bond parlance, contrasted with the
price, and accrued intcrcat quotation customarily used.

55. Approimate bond yields. — On a given date the book value of a


bond is quoted on the market and the problem is met of determining the
yield obtBined by an investor on purchasing the bond and holding it to
maturity. We first consider an approximate method of solution, using mere
arithmetic.
Non. — A bond salesman, in speaking of tho yield on a bond, usually refers to an
investment rate compounded the game number of timed per year aa dividend8 are paid.
Thus, by the yield on a quarterly bond, ho means the investment rate, compounded
quarterly. We shall follow this customary usage in the future. Moreover, in computing
yields it iB usuul to neglect the accrued dividend and brokerage fee puid at tho time of
purchase in addition to the book value. A yield iB computed with reference to tho book
value of the bond.
The justification of the following rules is apparent on reading them. Let
$B be the quoted book value of a bond, t the time in years before its
maturity, and $C its redemption price. The invested principal changes
from $B at purchase to $C at redemption, so that the average book value
$Bo iB given by Bo = KB -l- C). Even though a bond pays dividends
quarterly or semi-annually, in using the rules below proceed as if the
BONDS
dividends were payable annually at the dividend rate and let $D be this
annual dividend.
Rule 1. — When the qtroted book value B is at a premium over
C. — Compute $4, the average annual amortization of the pre-
Premium
mium from A = Compute $1, the average annual
interest on the investment from I = D — A . 1 Then, the ap proximate yield
r equals the average annual interest divided by

1
the average invested capital or r = —

Rule 2. — When B is at a discount from C. — Compute $T, the average


annual accumulation of the discount from T Discount

Compute I from I = D + T. 2 Then, the approximate yield r equals the


average a.n.nual interest divided by the average invested

1
c*pital, or r =
i See equation 60.
a
See equaåon 61.
12tr
200 MATHEMATICS OF INVESTMENT
EcampZe 1. — A $1000, 5% bond pays dividends gezni-annuolly and is
redeemable at 110%. Eleven years before its maturity, the book value is quoted On the
market at 93. Estimate the yield.
Solution. — Considering its dividends annual, D = $50, C = $1100, and the book
value B $930. Using Rule 2, the overage accumulation of the discount is 1 = $15.5, and
I = 50 + 15.5 = $65.5. The average invested capital ig 4- 1100) = $1015. The
approximate yield r = .065, or 6.5%.
Example 2. — A $1000, 5 % bond pays dividends on July I and January
1 und is redeemable at par on January I, 1961. Its quoted book value on
May 1, 1922, iB 113. Estimate the yield.
Solution. — Use Rule I with B = $1130, t = 38} years, D = $50, and C = $1000. We
find Bo = $1065. To find the average amortization of the premium we take t = 39, the
nearest whole number, because the inaccuracy of our rule when t is large makes
refinements in computation useless. A =
8.3, 1 - 50 — 3.3 = $46.7, and therefore r .04, or 4.4%.
Non. The author hag experimentally verified that Rules 1 and 2 give estimated
yields within .2% of the truth if : (a) the yield is between 4% a.nd 8%, (b) the time to
maturity is less than 40 years, and (c) the difference between the dividend rate and the
yield iB less than 3%. Greater bccuracy is obtained under favorable circumstances. For
a bond whose term is more than 30 years, ag in Example 2 above, take t as the whole
number nearest to the time to muturity in years. In dl other cases use the exact time to
the nearest month.
BONDS
EXERCISE LIÜ
Estimate the yields of the following bonds, by u8e of Rules I and 2.
To BE DrvIDEND
MARRrr TIME TO
FA0E
QUOT. MATURITY
RATE PAID
1.2 $1000 par 107.24 5% Nam;-ann. 11} years
2.2
3.2
100 par 160.30
4%
semi-ann. 25 years
1000 par 84.28 6% semi-ann. 40 years
100 96.50 annually 23* years
6. 110% 5% 8 years
100 105% 115.00 annually
6. 20 yeaN
100 115% 98.75 annually
i
Inspect the table of illustrativo Example 2, Section b2. Iu that example, on
computing tbe overage semi-annual accumulation in Rule 2, we obtain i(62.30)
$10.4, a ræult very close to all of the semi-annual

2
The yields in the first three problems, determined by accurate means, are follows: (1) 4.2% ; (2) 3.4% ; (3) 4.9%. Compare
your results aa found from Rules 1 and 2 in order to form an opinion of their accuracy.
MATHEMATICS OF

nqvusTMENT
7. A $10,000, 5% bond, with dividends payable June 1 and
December 1, is redeemable at par on December 1, 1950. On May 23,
1925, it is quoted at 89. Estimate the yield.
8. A Kingdom of Belgium bond, whose dividends are semi-annual, may
be redeemed at 115% at the end of 8 years. Estimate its yield under the aasumption
that it will be redeemed then, if it is now quoted at 94.
56. Yield on a dividend date by interpolation. —When the quoted
value of a bond is given on dividend date, the yield may be determined
by interpolation. When annuity tables, but no bond tables, are available,
proceed as follows :
(a) Fmd the estimated yield r as in Section 55.
(b) Compute the book value of the bond at the rate rx nearest to
r for which the annuity tables may be used.
(c) Inspect the result of (b) and then compute the book value for
another rate r2, chosen so that the true yield is probably between ri and h.
Select r2 as near aa possible to h.
(d) Find the yield i by interpolation between the results in (b)
and (c).
Ezample I. — A $100, 6% bond, with semi-annual dividends, is
redeemable at par. The quoted book value, IOå years before moturity, is
$111.98. Find the yield.
Solution. — (a) Average annual interest I = 6 — $4.9; estimated yield r = = 4.6%.
(b) Book value years before maturity to yield (.045, m = 2) is $112.44 (by equation
59). (c) Since $112.44 is greater than $111.98, the yield is greater than .045, and is
probably between .045 and .05. The book valua at (.05, m = 2)
INVÄT. Booz iB $108.09. Let (i, n — 2) bethe yield. In the table, 112.44 -
RATE 108.09 - 4.35,' 112.44 — 111.98 = .46, and .06 — .045 = .005.
Hence, i = .045 + (.005) .0455, or the yield is approximately
.045, m = 2 112.44 4.55%, compounded semi-annually.
i, m — 111.98
2 .05, m = 108.09
2
MATHEMATICS OF

NOTE I. — A' more exact solution may be obtained aa follows: At the yield
(.0456, m = 2), found above, compute the book value P, using logarithms in equation
59 becauge the annuity tables do not apply; P — 100 + .725
1
A solution in Example I givea a result which is in error by not more than of the
difference between the table rates used in the interpolation. Wo ore,
esseninterpolating in Table VIll, and hence our result iB eubjeot• only to the error we
meet in ueing that table.
BONDS

i29
(am at .02276) = 111.998. Since $111.998 is greater than $111.980, the yield i
is greater than .0455, and is probably between .0455 and .0456. By logarithms,
the book value at (.0456, m = 2) is $111.910. From interpolation ag in Example
1, i = .0455 -l- H(.OOOI)• = .045520. The yield iB 4.5520%, compounded
semi-annually, with L possible error in the last decimal place.
Norm 2. — The method of Example I is very easy if the desired book values
con be read directly from a bond table (see problem 2 below). If the bond table
uses interest rotes differing by results obtained by interpolLtion in the table are
in error by not more than a few .001%. Extension of the occuracy of u solution
as in Note 1 is limited only by the extent of the logarithm tables Lt our
dispoaal.
NOTE 3. — If the book value B is on a day between dividend dates, the
yield may be accurately obtained by the method of Section 58 below. An
approximate resilt OR,n be found by assuming B as the book value on the
nearest dividend date and computing the corresponding yield.

EnRCISE LIV
Find the yield in. each problem as in Example I, page 128. If the
instructor so directs, extend the accuracy BB in Note 1 above.
I. A $100, 4% bond pays dividends on January I and July I and is
redeemable at par on January 1, 1932. (a) Find the yield if the
quoted value on July 1, 1919, is 89.32. (b) Find the effective rate of
interest yielded by investing in the bond.
v' 2. A $100, 5% bond pays semi•annual dividends and is
redeemable at par. By use of the bond table of Section 51, find the
yield if the quoted value Il years before maturity is 107.56.
For each bond in the table, par value is $100. Find the yields.
DIVIDEND
To BE
Boor
RAD7EMzD Tin TO MATURITY
AT VALUm
BONDS

8. 110% annually 80 years $ 78.50


par semi-ann. 15} years 110.75
6. semi-ann. 19 years 83.30
6. par
annually 12 years 24 121.00
7. par semi-ann. yeors years 88.00
8. 105% quarterly 107.00
par
9. On January I, 1923, a purchaser paid $87.22, exclusive
of brokerage, for a $100, 4% bond whose dividends are payable
July I and January I
206 MATHEMATICS OF INVESTMENT
and which iB redeemable at par on January 1, 1932. Find the yield
obtained if the investor holds the bond to maturity.
10. A $100, 5% bond pays semi-annual dividends and is
redeemable at par at the end of 9 years. If it is quoted at 83.20, find
tho effective rate of interest yielded by the investment.
57. Special types of bond issues. — On issuing a set of bonds, a
corporation, instead of desiring to redeem al-I bonds on one date,
may prefer to redeem the issue in installments. The bonds are then
said to form a serial issue. The price of the whole issue to net an
investor a specified yield is the sum of the prices he should pay for
the bonds entering in each redemption installment.
Example 1. — A $1,000,000 issue of 6% bonds wag made on
January 1, 1920, with dividends payable semi-annually, and the
issue is redeemable serially in 10 equal annual installments. Find
the price at which all bonds outstanding on January I, 1927, could
be purchased to yield an investor (.04, m 2).
Solution. — There is $300,000 outstonding. The price of the bonds for
$100,000, which are redeemable at the end of 1 year, is 1000(anat .02) +
100000 = $101,941.56; the prices of the bonds redeemable in the installments
paid at the end of 2 years and of 3 years are $103,807.73 and $105,601.43,
respectively. The total price of outstanding bonds ig $311,350.72.

An annuity bond, with face value $F, is a bond promising the


payment of on annuity. The periodic payment $S bf the annuity is
described as the installment which, if paid periodically during the
life of the bond, is suffcient to redeem the face $F in installments
and to pay interest as due at the dividend rate on all of the face $F
not yet redeemed. That is, the payments of $S amortize the face $F
with interest at the dividend rate. When F and the dividend rate
are known, S can be found by the methods of the amortization
chapter. At a given investment yield, the price of an annuity bond
iB the present value of the annuity it promises. The annuity is
always paid the same number of times per year dividends are
payable on the bond.
BONDS 207
Example 2. —A certain ten-year, $10,000 annuity bond with the
dividend rate 5% is redeemable in semi-annual installments of $S
each. (a) Find S. (b) Find the purchase price of the bond, 5 years
before maturity, to yield 6%, effective.
Solution. — (a) The payment* of $S amortize $10,000 at (.05, m 2).

Bond annuity, Case 1


10,000 = son at .025); S - $641.47.
n — 20 int. per., R $S,
A $10,000, p 1, = .025 (b) The price A at the yield (.06, m = I) is
the present value of semiannual payments of S
made for 5 years.

Case 1
n 6 int. per., p = 2, i = A = 1282.94(aß). .06) = $5484.09.
06, R - $1282.94.
EXERCISE LV I
1.. A $100,000 serial issue of 5% bonds, with dividends payable semiunnually, iB
redeemable in 5 equal annual installments. The issue made July 1, 1927. On July I, 1930,
find the price of all outstanding bonds to net the investor (.06, m = 2).
2. For the bonds purchased in problem 1, form table
showing, on each dividend dote, the dividend received, the
installment (if any) which is paid, the interest due on the book
value, cnd the final book value.
3. A house worth $12,000 cash is purchased under the
following agreement: $2000 of the principal is to be paid at
beginning of each year for Bix years; interest Bt 6% is to be paid
semi-annually on all principal outstanding. Two years later, the
written contract embodying this agreement wag sold to a banker,
who the remaining rights of the creditor ho yield 7%,
effective. What did the banker pay?
4. A 5-yeor annuity bond for $20,000, with the dividend
rate 6%, payable semi-annually, iB issued on June 1, 1921. (a) Find
the price on June 1, 1922, to yield (.03, m = 2). (b) Find the price on
September 1, 1922, to yield (.03, m 2).
6. On June 1, 1924, find the price of the bond of problem 4 to yield
(.00, m = I).
208 MATHEMATICS OF INVESTMENT
SUPPLEMENTARY MATERIAL
58. Yield of a bond between dividend dates. — If the quoted
value of a bond is given on a doy between dividend dates, the yield
may be found by interpolation by essentially the same proceduré,
with steps (a) , (b), (c) and (d), as used in Section 56 on a dividend
date. i After the completion of Exercise LV, the student may
immediatoly proceed to the oonBideration of the Miscellaneous
Problems at the end of the chapter.
209 MATHEMATICS OF INVESTMENT

Example 1. — A $100, 4% bond pays dividends annually on December I


and is redeemable at par on December I, 1931. Find the yield on Februu•y I,
1926, if the book value is quoted at 95.926.
Solution. — (a) As in Section 55, the average annual interest on the
invætment iB I — 4 + .71 = $4.71; the estimated yield iB 07-00 4.71 (b)
The nearet table rate is 5%. Tofind the book value at 5% on Feb. 1, 1926, first
compute the values at 5% on Dec. 1, 1925, and Dec. I, 1926, the last ond the
next intereat dates. The results, 94.924 and 95.671, are placed in the first row
of the table below, and from them we find by interpolLtion the book value

TABLE OF Boor VALUES


YIELD mo. 1, 1925 1, 1920 DEC. 1, 1920

.05, 394.924 $95.048 $96.671


95.926
97.421 97.485 97.805
1
1
1
on Feb. 1, 1926. Since 95.671 — 94.924 = .747, the book value on Feb. 1 at
iB 94.924 + å(.747) = 95.048. (c) Since 95.048 iB than 95.926 (the given book
value), the yield is less than .05 and is probably between .045 and .05. Prices at
.045 on Dec. 1, 1925, and on Dec. 1, 1926, were computed and from them the
book value on Feb. 1, 1926, at .045 wna obtained by interpolation. (d) The
yield i ia obtained by interpolLtion in the column of the table for Feb. 1. 97.485
— 95.048 = 2.437; 97.486 — 95.926 1.559; .05 — .045 = .005; hence i — .045
1559 .005 = .0482. The yield iB approximately 4.82%, compounded annually,
with a poæible small error in the Inat digit.
Non. — The method above iB extremely simple if the desired book vulties
can be read from a bond table. accuracy of the solution con be extended by the
method of Note I, Section 56.

EnRCISE LVI
210 MATHEMATICS OF INVESTMENT

1. By use of the bond table of Section 51, find the yield of


a $100, 5% bond, with dividends payable on September I and
March 1, and redeemable at par on September 1, 1928, if the
quoted book value December 1, 1917, is $106.78.
2. The interest dates of a $100, 4% bond are July 1 and
January 1, and it is redeemable bt par on January I, 1930. (a)
Find the yield if it is quoted at 83.25 on September 1, 1923.
(b) Find the effective rate of interest yielded by the bond.
BONDS 211

3. A man pays $87.22, exclusive of the brokerage


commission, on September 1, 1923, for a $100, 4% bond whose
dividends ave payable July I und January I, and which iB redeemable
at par on January 1, 1930. Determine the yield, if the bond is held to
maturity.
4. A $1000, 5% bond pays dividends annually on June 16
and iB redeemable at 110% on June 16, 1937. Find the yield if it is
quoted at 112.06 on November 16, 1932.
5. The 3d Liberty Loan bonds are redeemable at par on
September 15, 1928. Interest dates are September 15 and March 15.
If the
bonds were quoted at 85 on May 15, 1921, what the investment yield
?
MISCELLANEOUS PROBLEMS
In the following problems, the word interest is used in the colloquial sense in connection with
bonds in place of the word dividend previously used.

1. A certain $1000, 5% bond pays interest annually. It is


stipulated that, at the option of the debtor corporation, it may be
redeemed at par on any interest dote after the end of 10 years. The
bond certainly will be redeemed ot par by the end of 20 years. At
what purchase price would a purchaser be certain to obtain 6% or
more on hig investment?
2. What is the proper price for the bond in problem 1 to
yield 4%, or more ?
3. In return for EL loan of $50002 W giveshis creditor H the
following note:

Norfolk, June 1, 1915.


BONDS 212

For value received, I promise to pay, to H or order, $5000 at the end of


6 years and to pay interest on this sum semi-annually at the rate 6%.
Signed, W.

On December 1, 1916, sold this note to an investor desiring (.07, m = 2) on hig investment. What did H receive ?

4. What would I-I have received if he bad sold the note to


the same investor in problem 3 on February 1, 1917?
5. Two $1000 bonds are redeemable at par and pay 4% interest semiannually.
Their quoted prices on a certain date to yield (.05, m = 2) are $973 and $941.11, respectively.
Without using annuity tables, End without computation, •state which bond hag the longer term
to run and justify your answer.
213 MATHEMATICS OF INVESTMENT

6. Determine the term of the bond in problem 5 quoted at


$941.11.
7. A $100, 4% bond, redeemable at par in 20 years, pays
interest semiannually. If it is quoted at $92.10, what is the effective
rate of interest obtained by an investor ?
8. On June 1,.1921, a corporation hag its surplus invested
in bonds which are redeemable at par on June I, 1928, and which
pay interest semi-annually at the rate 5%. If the bonds are quoted at
102.74, would it pay the corporation to sell the bonds and
reinvest the proceeds in Government bonds which net 4.65%,
effective?

9. A $1,000,000 issue of 5\0/0 bonds, paying interest


annually, iB to be redeemed at 110% in twenty annual installments.
The first installment is to be paid at the end of 5 years and the lust
at the end of 24 years. Ii is desired that the annual payments
(dividends on unpaid bonds ane the redemption installment
included)' at the end of each year for the las20 years shall be equal.
Determine the payment.

10. A house worth $12,000 is purchased under the


following agreement $2,000 is to be paid cash and the balance of
the principal iB to be paid i] four equal installments due at the ends
of the 2d, 4th, 6th, and 8th years Interest at 6% is to be paid semi-
annually on all sums remaining due The note signed by the
purchaser is sold after 3 years by the originE owner of the house. If
the purchaser of the note demondB 7%, compounde semi-annually,
on his investment, what does he pay for the note ?

11. A trust fu.nd of $20,000 iB invested in bonds which


yield 5% ar nually. The trust agreement states that of the income
shall be give to the beneficiary each year and that the balance shall
14 MATHEMATICS OF INVESTMENT

be re-invested in savings bank which pays 5%, compounded


annually. The whole fun shall be turned over to the beneficiary after
10 years. If money ig wort 6%, effective, to the beneficiary, what
sum would he take now in place of h interest in the bust fund?
Assume that he will live 10 years.
12. A corporation can sell at par o $1,000,000 issue of
bonc redeemable at par in 20 years and paying interest annually. To
pay the at maturity the corporation would accumulate a sinking
fund by annu deposits invested at 4å%, effective. Would it be better
for the corpor åon to sell 8t par a $1,000,000 issue of 5% bonds, if
thege are redeemaE in such annual installments during the 20 years
that the total annual pc mentg, di«idends and redemption payments
included, will be equa
REVIEW PROBLEMS ON PART 1 215

REVEW PROBLEMS ON PART 1


1. In purchasing a farm, $5000 will be paid at the end of
each year for 10 years. (a) What iB the equivalent ca.sh price if
money is worth 5%, effective? (b) What must be paid at the end of
the 6th year to complete the purchase of the farm ?
2. A depreciation fund is being accumulated by semi-
annual deposiÉ of $250 in a bank paying interest semi-annually at
the rate 5%. What is in the fund just after the 30th payment?
8. A man wishes to donate to a university suffcient money to
provide for the erection and the mointenance, for the next 50 years,
of a building which will cost $500,000 to erect und will require
$2000 at the end of each 3 months to maintain. What should he
donate if the university is able to invest its funds at 5%,
compounded semi-annually?
4. A debt of $100,000 bears interest at 6%, payable semi-
annually. A sinking fund is being accumulated by payments at the
end of each 6 months to repay the principal in one installment at the
end of 10 years. If the sin.king fund earns 4% interest, compounded
Bemi-annually, what is the total semi-annual expense of the debt?
5. A debt of $100,000 is contracted under the agreement that interest at 6% shall
be paid semi-annually on all sums remaining due. What payment Lt the end of each 6 months
for 10 years wil.l amortize thig debt?

6. By use of geometrical progression derive the


expression for the amount of an annuity whose annuol rent iB
$2000, payable in semi-annual for 10 years, if money iB
worth 6%, compounded quarterly.
T. Find the present value of an annuity whose annual rent is
$3000, payable semi-annually for 20} years, if money iB worth
(.05, m = 4).
REVIEW PROBLEMS ON PART 1 216

8. A merchant owes $6000 due immediately. For what


sum should he make out 90-day, non-interest-bearing note, so that
his creditor may realize $6000 on it if he discounts it immediately at
a bank whose discount rate is 8%?
9. If money is worth 5%, effective, find the equal
payments which if made at the ends of the first a,nd of the third
years would discharge the liability of the following debts: (1)
$.1000 due without interest at the end of 3 years; (2) ,$2000 due,
with accumulated interest at the rate (06, m — 2), at the end of 4
years.
10. A trust fund of $100,000 is invested at 6%, effective. Payments of
$10,000 will be mode from the fund at the end of each year as long pos-
217 MATHEMATICS OF INVESTMENT

Bible. (a) Find how many full payments of $10,000 will be made.
(b) How much will be left in the fund just after the last full
payment of $10,000?
Il. Find the nominal rate of interest, compounded quarterly,
under which payments of $1000 at the end of each 3 months for 20
years will be suåcient to accumulate a fund of $200,000.
12. Find the purchase price, to yield 6%, effective, of a
$100, 5% bond with interest payable semi-annually, which is to be
redeemed at 110% at the end of 10 years.
13. Estimate the yield on bond which is quoted at 78, 10
years before it is due, if it is to be redeemed ut par and if its
dividend rote is 6%, payable annually.
14. Find the capitalized cost of machine, whose original cost
is $200,000, which must be renewed at a cost of $150,000 every
20 years. oney is worth 5%, effective.
16. Find the yield of L $100, 6% bond, with semi-annual
dividends, which is quoted at 93.70, 10} years before it iB due,
and is redeemable Lt par.
16. A $100, 5% bond, quoted at 86.33 on September 1,
1926, yields 6% if held to maturity. The last coupon dute was July
I. What is the purchase price on September 1 ?
17. A man deposited $100 in a bank at the beginning of each
3 months for 10 years. What is to his credit at the end of 10 years
if the bank pays 8%, compounded quarterly ?
18. A man deposited $50 in L fund at the end of each month
for 20 years, at which time deposits ceased. What will be in the
fund 10 years later if it accumulated for the first 20 yeors at the
rate 6%, effective, and at the rate 4%, effective, for the remainder
of the time?
19. The cash price of L farm is $5000 and'money is worth
(.06, m = 2). Whot equal payments made quarterly will have an
equivalent value, if the first payment is due Bt the end of 3 years
and 9 months, and tbe last at the end of 13} years?
218 MATHEMATICS OF INVESTMENT

20. A corporation issues $200,000 worth of 6% bonds,


redeemable at par at the end of 15 years, with interest payable semi-
annually. The corporation is compelled by the terms of the issue to
accumulate a sinking fund, to pay the bonds at maturity, by payments
at the end of each 6 months, which are invested at (.04, m = 2). The
bonds are sold by the corporation at 95 (95% of their par value).
Considering the total semiannual expense as an annuity, under what
rote of interest is the corpora,tion amorüng the loan it realizes from
the bond issue?
REVIEW PROBLEMS ON PART 1 219

21. The present liability of a debt is $100,000. It iB agreed


that payments of $5000 shall be made at the end of each 6 months
for 10 years, and that, during this time, the payments include interest
at the rate 6%, payable semi-annually. Then, commencing with first
payment at the end of 10} years, semi-annual installments of
$10,000 shall be paid as long as necessary to discharge the debt. (a)
After the end of 10 years, if the payments include interest at the rate
5%, payable semi-annually, how many full payments of $10,000
must be made? (b) What part of the payment at the end of 10} years
is interest on outstanding principal and what part is principal
repayment ?
22. From whose standpoint, that of the debtor or of the
creditor, is compound interest more desirable than simple interest?
Tell why in one sentence.
28. A 90-day note whose face value is $2000 bears interest at 6%.
It is discounted at a bank 30 days before due. What are the proceeds
if the banker's discount rate is 8%?
24. A man borrows a sum of money for 72 days from a
bank, charging 5% interest payable in advance. (a) What interest
rate is he paying? (b) What interest rate would he be paying if he
borrowed money for 1 year from this bank?
25. Estimate the yield of a bond whose redemption value is
$135, whose dividends are each $10, and ore paid annually, and
whose purchase price 6 years before due is $147.
26. (a) Find the yield 1 of L $100, 6% bond bought for $103.53 on
October
1, 1921. Coupons ore payable semi-annually on February 1 and
August 1 and the bond will be redeemed ot par on February 1, 1928.
(b) Find the effective rate of interest yielded by the bond.
27. The principal of a debt of $200,000 is to be paid after
20 years by the accumulation of a sinking fund into which 79
quarterly payments will be made, starting with the first payment in 6
REVIEW PROBLEMS ON PART 1 220

months. Find the quarterly payment if the fund grows at 6%,


compounded quarterly.
28. To amortize certain debt at 6%, effective, 40 semi-
annual payments of $587.50 must be mode. Just after the 26th
payment, what principal will be outstanding?
29. (a) Find the capitalized worth at (.06, m = 12) of an enterprise which will yield
a monthly income of $100, forever, first payment due now. (b) What is the present worth in (a)
if the first monthly payment is due at the end of 6 months?
Find the yield ae in Section 58, or, if section has not been studied, use the method of Section 55.
221 MATHEMATICS OF INVESTMENT

80. A bridge will need renewal at a cost of $100,000 every


25 years. Under 5% interest, what is the present equivalent of all
future renewals?
81. (a) By use of a geometrical progression determine a
formula for the amount of an annuity whose annual rent is $20,000,
which iB paid quarterly for 30 years, if money iB worth 7%,
compounded annually. (b) Without a geometrical progresgion find
the present value of the annuity.
32. A house is worth $50,000. In purchasing it $20,000 is paid
cash and the remainder is to be paid, principal and interest at (.05, m
= 2) included, by semi-annual installments of $2000, first payment
to be made at the end of 2 years. (a) Determine by interpolation how
many whole payments of $2000 will be necessary. (b) What liability
will be outstanding just before the last full payment of $2000?
88. A debt of $50,000 is contra,cted and interest is at the rate 5%,
compounded annually. The only payments (including interest) made
were $5000 at the end of 2 years, and six annual payrnents of $3000,
starting with one at the end of 5 years. At the end of 10 years what
additional payment would complete payment of the debt?
34. If money is worth (.05, m = 2), find the equal payments which
must be made at the ends of the 3d and 4th years in order to
discharge the following liabilities : (1) $5000 due at the end of 6
years, without interest ; (2) $4000 due at the end of 5 years with all
accumulated interest at (.06, m = 1).
86. A debt of $100,000 iB contracted and it is agreed that it shall
be paid, principal and interest included, by equal payments at the
end of each 6 months for 20 years. Interest is at the rate (4 m 2) for
the first 10 years and at (5%, m = 2) for the next 10 years. What
222 MATHEMATICS OF INVESTMENT

Bingle rate of interest over the whole 20 years would have resulted
in the same payments ?
86. A man invested $100,000 in certain enterprise. At the ends of each of the
next 10 years he was paid $4000 and, in addition, he received a payment of
$25,000 at the end of 6 years. At the end of 10 yearR he sold his investment
holdings for $80,000. 'Considering the whole period of 10 years, what was the
effective rate of interest yielded by the investment ?
HINT. —Write an equation of value; solve by interpolation as in Note 3 in
the Appendix.

37. Mr. A borrows $5000 from B to finance his college


course and gives B a note, promising to pay $5000 at the end of 10
years, together with all accumulations at 3%, compounded semi-
annually. (a) Whot will A pay
REVIEW PROBLEMS ON PART 1 223
at the end of 10 years? (b) At the end of 5 years, B sells A's
promissory note to a bank, which discounts it, considering money ag
worth (.05, m = 1). What does B realize from the sale?
38. Find the price at which a $100, 5% bond would be
quoted on the market on September 1, 1922, to yield the investor
(.06, m = 2). The bond is to be redeemed at par on August 1, 1928,
ana interest dates of the bond are August 1 and February 1.
39. An industrial commission awards $10,000 damages to
the wife of workman killed in on accident, but suggests that this
sum be paid out by a trust company in quarterly installments of
$200, the first payment due immediately. (a) If the trust company
pays (.04, m = 4) on money, for how long will payments continue?
(b) At the end of 10 years, the wife takes the balance of her fund.
What amount does she receive?
40. Determine the capitalized cost of a machine worth
$5000 new, due to wear out in 20 years, and renewable with a scrap
value of $1000. Money iB worth .05, effective.
41. Find the purchase price on December 1, 1920, of a $100, 6%
bond with annual dividends, to yield at least 5%, if the bond may at the option
of the issuing company be redeemed at 110% on any December I from 1930 to
1935, inclusive, or at par on any December 1 from 1943 to 1950. Justify your
price.
42. A father wills to his son, who is just 20 years old,
$20,000 of stock which pays dividends annually at the rate 6%. The
will directs that the earnings shall be held to his son's credit in a
bank paying 81%, effective, and that all accumulations as well as
the original property shall become the direct possession of his son
on his 30th birthday. Assuming that the market value of the stock on
the 30th birthday will be $20,000, iB the present value of the estate
for the son on his 20th birthday, assuming that money is worth and
that the son will certainly live to age 30?
43. If money is worth (.06, m 2), what equal installments
paid the ends of the 2d and 3d years will cancel the liability of the
following obligations : (c) $1000 due without interest at the end of 5
224 MATHEMATICS OF INVESTMENT
years, and (b) $2000 due with accumulated interest at the rote 4%,
compounded annually, at the end of 6 years?
44. Two years and 9 months ago X borrowed $2000 from
Y, and hag paid nothing since then. (a) If interest ig at the rate 6%,
payable gemiannually, determine the theoreticol compound amount
which X should pay to settle his debt immediately. (b) Determine
the pmount by the practical rule,
45. •At the end of each 6 months, $200,000 iB placed in a
fund which accumulates at the rate (.06, m = 2). (a) How many full
payments of $200,000 will be necessary to accumulate a fund of
$1,000,000? (b) What smaller payment will be needed to complete
the fund on the next date of deposit after the last $200,000
payment ?
46. Find the annual expense of a bond issue for $500,000
paying 5% annually, if it is to be retired at the end of 20 years by the
accumulation of a sinking fund by annual payments invested at
4%, effective.
47. In problem 46, at what effective rate of interest could the bor-
rower just as well borrow $500,000 if it is agreed to amortize the
debt by equal payments made at the ends of the next 20 years?
48. How much is necessary for the endowment of a research
fellowship paying $3000 annually, at the beginning of each year, to the fellow
and supplying a research plant, whose original COBt iB $10,000, which
requires $2000 at the beginning of each year for repairs and supplies? Money
iB worth 4%, effective.
49. A banker employs hig money in 90-day loans ot 6%
interest, payable in advance. At what effective rate i.s he investing
hig resources ?
50. Find the present value and the amount of an annuity of
$50 per year for 20 years if money is worth 4%, payable annually.
Use no tables and do entirely by arithmetic, knowing that (1.04) 20 =
2.191123.
51. $100,000 falls due at the end of 10 years. The debtor put $8000
into asin.kingfund at the end of each of the first 3 years. He then
REVIEW PROBLEMS ON PART 1 225
decided to make equal annual deposits in his sinking fund for the
remainder of the time in order to accumulate the necessary
$100,000. If the fund earns (.04, m = 1), what was the annual
deposit ?
52. A corporation is to retire, by payments at the end of each of
the next 10 years, a debt of $105,000 bearing 5% interest, payable
annually. The tenth annual payment, including interest, iB to be
$15,000. The other nine are to be equal in amount and are to include
interest. Determine the size of these nine payment.
58. Compute the purchase price to yield (.05, m = 4) of L $1000, 6% bond
redeernable at 110% in years, if it pays interest semi-annually.
54. Compute the present value of an annuity whose annual rent is $3000,
payable quarterly for 6 years, if interest is at the rate 5.2%, effective.
55. The maximum sum insured under the War Risk Insurance Act
pays $57.50 at the beginning of each month for 20 years certain
after death or disability. What would be the equivalent cash sum
payable at death, or disability, at interest?
56. A company issues $100,000 worth of 4%, 20-year
bonds, which it wishes to pay at maturity by the accumulation of a
sinking fund into which equal deposits will be made at the end of
each year. The fund will earn 5% during the first five years, for the
next 5 years, and 4% for the last 10 years. Determine the annual
deposit.
57. The amount of a certain annuity, whose term is 7 years,
is $3595 and the present value of the annuity iB $2600. (a)
Determine the effective rate of interest. (b) Determine the nominal
rate, if it is compounded quarterly.
58. How long will it take to pay for a house worth $20,000
if interest is at 5%, effective, and if payments of $4000, including
interest, are made at the beginning of each year? Find the last annual
payment which will be made, assuming that the debtor never pays
more thun $4000 at one time.
59. A sum of $1000 is due at the end of two years. (a) Discount it to
the present time under the simple intcrest rate 6%. (b) Discount it under the
226 MATHEMATICS OF INVESTMENT
simple discount rate 6%. (c) Discount it under the compound interest rate (.06,
m = 1).
60. A concern issues $200,000 worth of serial bonds,
paying 5% interest annually. It is provided that $30,000 shall be
used at the end of each year to retire bonds at par and to pay interest.
How long will it take to retire the issue? Disregard the denomination
of the bonds.
61. Find the value of a mine which will net $18,000 per year for 30
years if the invegtment yield is to be 6% and if the redemption fund is to be
accumulated at 31%, compounded annually.
62. A man expects to go into business when he has saved
$5000. He now hag $2000 and can invest his savings at (5%, m = I).
How much must he save at the end of each year to obtain the
necessary amount by the end of 5 years ?
63. Find by interpolation the composite life on a 4% basis of a plant
consisting Of : Part (A), with life 10 years, cost new, $13,000, scrap value,
$2000; Part (B), with life 15 years, cost new, $20,000, and scrap value, $3000.
64. How much could a telephone company afford to pay per $10 unit
cost in improving the material in its poles in order to increase the length of life
from 15 to 25 years? The poles have no scrap value when worn out, and money
is worth (.05, m = 1).
65. What are the net proceeds if 9Q-day note for $1000, bearing 6%
interest, is diedounted at 8%?
66. X requests a 60-day loan of $1000 from a bank
charging 6% interest in advance. How much money does the bank
give him and what interest rate is X paying on the loan ?
67. A woman has funds on deposit in a bank paying (.04, m = 2).
Should she reinvest in bonds yielding .0415, effective ?
68. How long will it take for a fund of $3500 to grow to $4750 if
invested at the rate 6%, compounded quarterly?
69. The sums $200, $500, and $1000 are due without interest in 1, 2,
and 3 years respectively. When would the payment of $1700 equitably
dißcharge these debts if money is worth (.06, m = 1) ?
70. A father has 3 children aged 4, 7, and 9. He wishes to present
each one with $1000 at age 21. In order to do BO he decides to deposit equal
sums in a bank at the end of each year for 10 years. If it iB assumed that the
REVIEW PROBLEMS ON PART 1 227
children will certainly live and that the bank pays (5%, m = I), how much must
the father deposit annually?
71. Which is worth more, if money is worth 6%, effective:
(a) an income of 12 annual payments of $500, first payment to be
made at the end of 2 years, or (b) 120 monthly payments of $50,
first payment due at the end of 3 years and 1 month?
72. A $100, 5% bond pays interest quarterly and is redeemable at
110% at the end of 10 years. Find its price to yield 6%, effective.
78. Find the present value and the amount of an annuity of $3000
payable at the end of each 3 years for 21 years if interest is at the
rate (.05, m = 2).
74. Find the nominal rate, converted quarterly, under which
money will treble in 20 years.
76. (a) What effective rate iB yielded by purchasing at par a $100,
4% bond, redeemable at par, which pays interest quarterly? (b)
What rate, compounded semi-annually, does the investment yield ?
78. @ In order to retire L $10,000 debt at the end of 8 years
sinking fund will be accumulated by equal semi-annual deposits, tbe
first due immediately and the last at the• end of 7} years. Find the
semi-nnnuul payment if the fund is invested at the rate (.04, m 2).
(b) Find the size of the payments, under the same rate, if the first is
mode immediately and the last at the end of 8 years.
77. X lends $600 to B, who promises to repay it at the end
of 6 years with all accumulated interest at (.06, m = 2). At the end of
3 years, B desires to pay in full. If X is now able to invest funds at
only 4%, effecåve, what should the debtor pay ?
228 MATHEMATICS OF INVESTMENT
78. Find the nominal rate, converted quarterly, which
yields the effective rate .0635.
79. (a) A house costs $23,000 cash. If interest is at the rate
(.05, m = I), what equal payments made at the beginning of each 6
months for 6} years will amortize the debt? (b) What liability is
outstanding at the beginning of the 3d year before the payment due
is made ?
80. How much Inust man provide to purchase and maintain forever
an ambulance costing $6000 new, renewable every 4 yeurs at a cost of $4500
and requiring annual upkeep of $1500 payable at the beginning of eaeh year?
Money is worth 4%, effective.
81. A corporation wax loaned $200,000 and, in return,
made annual payments of $12,000 for 8 years in addition to making
a final payment of $200,000 at the end of 9 years. What rate of
interest did the corporation pay?
82. A loan of $100,000 is to be amortized by equal
payments at the end of each year for 20 years. During the first 10
years the payments are to include interest at 5%, effective, and,
during the laet 10 years, interest at 6%, effective. Determine the
annual payment.
88. $10,000 is invested at effective. Principal and interest ace to
yield a fixed income at the end of each 6 months for 10 years, at the
end of which time the principal is to be exhausted. Determine the
semi-annual income.
84. A house worth $10,000 cash is purchased by B. A cash
payment of $2000 is made and it is agreed in the contract to pay
$500 of principal at the end of each 6 months until the principal is
repaid and, in addition, to pay interest at the rate 6% semi-annually
on all unpaid principal. Just after the payments are made at the end
of two years, an investor buys the contract to yield 7%, compounded
semi-annually, on the investment. What does the investor pay?
86. A form worth $15,000 cash is purchased by B, who contracts
to pay $2000 at the beginning of each 6 months, these payments
including semi-annual interest at 6%, until the liability is
discharged. At the end of 4 years, just after the payments due are
REVIEW PROBLEMS ON PART 1
made, the contract signed by B is sold to an investor to yield him
(.07, m = 2) on the investment. What does he pay?
86. A state, in making farm loans to ex-soldiers, gant,s
them the following terms: (a) interest- shall be computed at the rate
(.04, m = 2) throughout the life of the loan; (b) no interest shall be
paid, but it shall accumulate a liability, during the first 4 years; (c)
the total indebted-
ness shall be discharged by equal monthly payments, the first
due at the end of 4 years and 1 month and the last at the end of
10 years. Determine the monthly payment on a loan of $2000.

87. (a) A boy aged 15 years will receive


the accumulations at 5%, effective, of an
estate now worth $30,000, when he reaches the
age 21. What is the present value of his
inheritance Lt effective, assuming that he
will certainly live to age 21? (b) Suppose
that the boy is to receive, annually, the
income at 5% from the estate and to receive
the principal at age 21. Find the present
value of the inheritance Lt effective.
88. The quotation of certain $100, 5%
bond to-clay (an interest date) ig 88.37
and it yields 7% to on investor. Find the
purchase price and market quotation 2 months
later at the same yield.
89. A note signed by Y promises to pay
$1000 at the end of 90 days with interest at
5%. (a) What would the holder X obtain on
selling the note 30 dayg later to banker
whose discount rate is 6%? (b) What would he
obtain if the note were discounted under the
simple interest rate 6%?
90. A certain man invests $1500 at the
rate (.04, m = I) on each of his birthdays,
starting at age 35 and ending at age 65. (a)
230 MATHEMATICS OF INVESTMENT
At age 65, what does he have on hand? (b)
Suppose that at age 65 he decides to save no
more and to spend all of his savings by
taking from them am equal amount at the end
of each month for 15 years, and suppose that
he will certainly live that long. What can he
take per month if the savings remain invested
at (4%, m = I)? (c) If he desires to have
$5000 left at the end of the 15 years, what
wi-u be his monthly allowance ?
91. A depreciation fund ig being formed by
semi-annual deposits, to replace an article worth
$10,000 new, when it becomes worn out after 6 years.
(a) If money is worth (5%, m = 1), what is the semi-
annual charge if the scrap value of the article is
$1000? (b) How much is in the depreciation fund just
after the third deposit? (c) Find the condition per
cent of the article at the end of 3 years.
92. A debt of $50,000 is being amortized with interest
at (.06, m = 2) by 24 equal semi-annual payments, the first
payment cash. Find the poyment and determine how much principal
is outstanding just after the 12th payment.
98. Find the present value of a perpetuity of $1000, payable
semiannually, if interest iB at the rate 6%, effective.
94. A man borrowed $10,000, which he
agreed to amortize with interest at the rate
5%, payable annually, by equal payments, at
the end of each
145
year for 12 years. Immediately after borrowing the money he
invested it at 7%, payable semi-annually. In balancing his books at
the end of 12 years, what is his accumulated profit on the
transaction?
95. A loan agency offers loans to salaried workers under
the following plan. In return for a $100 loan, payments of $8.70
mu.Lst be made at the end of each month for 1 year. Determine the
REVIEW PROBLEMS ON PART 1
nominal rate, compounded quarterly, under which the transaction is
executed.
96. A corporation can raise money by selling 6% bonds, with
semiannual dividends, at 95% of par value. To provide for their redemption at
par Lt the end of 15 years, L sinking fund would be accumulated by investing
equal semi-annual deposits at (.04, m = 2). The corporation also can raise
money by issuing, Lt par, 15-yeur, 7% a,nnuity bonds redeemable in semi-
annual installments. (a) Which method would entail the least semi-annual
expense in raising $100,000 by a bond issue? (b) If money can be invested at
(.04, m = 2) by the corporation, what would be the equivulent profit, in values
at the end of 15 years, from choosing the best method ?
97. A corporation will issue $1,000,000 worth of 5% bonds, paying
interest semi-annually and redeemable at par in the following amounts :
$200,000 at the end of 5 years; $300,000 at the end of 10 years; $500,000 at the
end of 16 years. A banking syndicate bids $945,000 for the issue. Under what
interest rate is the corporation borrowing on the proceeds of the bond issue ?
98. An investor paid $300,000 for a mine and spent
$30,000 additional at the beginning of each year for the first 3 years
for running expenses. Equal annual operating profits were received
beginning at the end of the 3d year and ceaaing with n profit at the
end of 25 years, when the mine become exhausted. The investor
reinvested all revenue from the mine at 5%, effective. What wag the
net operating profit for the last 23 years if, Lt the end of 25 years, he
has as much as if he had received, and reinvested at 5%, effective,
8% interest unnually on all capital invested in the m.ine and
likewise had received back his capital intact at the end of 25 years ?
99. A man who borrowed $100,000 under the rate 6%,
payable semiannually, is to discharge all principal and interest
obligations by equal payments at the end of each quarter for 8
years. At the end of 2 years, his creditor agrees to permit him Co
discharge his future obligations by 4 equal semi-annual payments,
the firgt due immediately. (a) What will be the semi-annual payment
if the creditor, in computing it, uses the rate
MATHEMATICS OF
NVESTMENT
5%, compounded semi-annually? (b) What will be the semi-annual
payment if the rate (.07, m = 2) is used in the computation ?
MATHEMATICS OF
NVESTMENT
100. A contract for decd is the name assigned to the
following type of agreement in real estate transoctions : In
purchasing a piece of property worth $2000 cash B agrees to pay
MATHEMATICS OF
NVESTMENT
$500 cash and to pay $25 at the end of each month, these payments
to include interest at the rate 6%, payable monthly, until the
MATHEMATICS OF
NVESTMENT
property iB paid for. The owner A ogrees on his part to deliver the
deed for the property to B when payment is completed.
MATHEMATICS OF
NVESTMENT
Six months after the contract above wag made, A sellB it to an
investor, who obtains the rate 7%, compounded monthly, on his
MATHEMATICS OF
NVESTMENT
investment. What does he pay, if A hag already received the $25
due on the contract on this date ?
PART
Il—LIFE INSURANCE
CHAPTER Vill
ANNUITIES

59. Probability. —The mathematical definition of probability


makes precise the meaning customarily assigned to the words
chance or probability a.s used, for example, in regard to the
winning of a game. Thus, if a bag contains 7 black and 3 white balls
and if a ball is drawn at random, the chance of a white ball being
obtained is because, out of 10 balls in the bag, 3 are white.
Defiifion. — If an event E can happen in h ways and fail in u
ways, au of which are equally likely, the probability p of the event
happening is h
(1)
and the probability q the event failing is
u
(2)
NOTE I. —In the ball problem above, the event B the drawing of a white
ball; h — 3, h u — 10, p The probability of failure q = 6. The denominator (u
4- IL) in the formulae should be remembered aa the total number of ways in
which E can happen or fail.
From formulas 1 and 2, it is seen that p and g are both less than 1.
Moreover,
PART
or the sum of the probabilities of failure and of success iB 1. If an
event is certain to happen, u = 0 and p = - = 1
EXERCISE LVII
1. An urn contains 10 white and 33 black balls. What is
the probability that a ball drawn at random will be white?
2. A deck of 52 cards contains 4 aces. On drawing a card
at random from a deck, what is the probability tbat it will be an ace?
( 1472
MATHEMATICS OF NVESTMENT

148
8. Out of a class of 50 containing 20 girls and 30 boys, one
member is chosen by lot. What is the probability that a girl
will be picked?
4. A cubical die with six faces, numbered from I to 6, is tossed.
What is the probability that it will fall with the number 4 up?
5. A coin is tossed. What is the probability that it will fall head up?

6. If the probability of a man living for at least 10 years iB .8, find


the probability of him dying within 10 years.
7. If the probability of winning a game is i, what is the
probability of losing?
Norrn 2. — It is important to recognize that when we say, in problem 7,
above, "the probability of winning is we mean : (a) if very targe number of
games are played, it is to be expected that approximately Of them
will be won, and (b) if the number of games played becomes larger and lurger
without bound, it is to be expected that the quotient, of the number of them
which won divided by the total played, will approach as B limiting value. We
do not imply, for instance, that out of 45 gamæ played exactly of 45, or 27
games be won. We must recognize that, if only few games are played, it may
happen that more, or equally well less, than of the total will be won.
8. As a coöperative class exercise, toss a coin 400 times
and record at each trial whether or not the coin falls head up. How
many were heads out of (a) the first 10 trials; (b) 50 tria,ls; (c) 400
trials? Compare in each case the number of heads with of the number
of trials 80 as to appreciate Note 2, above.
NOE 3. — The aasumption in the definition of probability that all waya of
happening or failing are equaUy likely, iB a very important quulificotion. For
example, we might reaaon follows : A man selected at random will either live
one day or else he will die before to-morrow. Hcnce, therc ure only two
posibilitieg to consider, and the probability of dying before to-morrow is
MATHEMATICS OF NVESTMENT

This ridiculous conclusion would neglect the fact thut he is more likely to live
than to die, and hence our definition of probability Bhould not be applied.

60. Mortality Table. — Table XIII wag formed from the


accumulated experience of many American life insurance
companies. This table should be considered as showing the observed
deaths among a group 1 of 100,000 people of the same age, all of
whom
The actual construction of mortality table is 8 very diffcult matt,or and
ca.nnot be congidered here. It is, of course, impossible to obtain for observation
100,000 children of same age, 10 years, and to keep record of the deathB until
till have died. However, data obtained by insurance companies, or census
records of births and deaths, can be used to create 8 table equivalent to a deuth
record of L repreeentative goup of 100,000 people of tho same age, all of whom
alive at ago 10.
LIFE ANNUITIES 149

were alive at age 10. In the mortality table,


represents the number of the group still
alive at age c, and da the number of the
group dying between ages c and c + 1. Thus, =
89,032, and = 718 89032 — 88314). In
general, dz = iz — 1+1. out of alive at age
c, remain alive at age c + n, and hence —
die between ages c and c + n. Thus, — or 2154
die between ages 25 and 28.
When the exact probability of the happening
of an event is unknown, the probability may
sometimes be determined by observation and
statistical analysis. Suppose that an event
has been observed to happen h times out of m
trials in the past. Then, as an approximation
to the probability of. occurrence we may take
This estimated value of p becomes
increasingly reliable

aa the number of observed cases increases.


The statistical method is used in determining
all probabilities in regard to the death or
survival of an individual selected at
random ; our observed data is the tabulated
record given in the mortality table.
EcampZe 1. — A man is alive at age 25. (a)
Find the probability that he will live at
least 13 years. (b) Find the probability that
he will die in the year after he is 42.
Solution. — (a) We observe — 89,032 men alive at
age 25. Of these, 79,611 ( = h) remain alive at age
88. The probability of living to age 38 is
79611 (b) Out of 89,032 dive at age 26, 785
76567 — 75782 - du)
= 89032•
is da —
— 785 die in their 43d year. The
probability of dying
89032
EXERCISE
In the first nine problems find the probability :
1. That a boy aged 10 will live to gaduate
from college at age 22.
2. That a man aged 33 will live to receive

an inheritance payable at age 45.


3. That boy aged 15 will reach age 80.
4. (a) That a man aged 56 will die
within 5 years. (b) That he will die during
the 5th year.
6. That a man aged 24 will live to age 26.
6. That a man aged 28 will die in his
88th year.
MATHEMATICS OF NVESTMENT

150
7. That a man aged 28 will die in the year
after he is 38.
8. That a man aged 40 will live Lt least 12
years.
9. That man aged 35 will live at least 20
years. 10. If a man iB olive at age 22, between what ages is
he most likely to die and what is his probability of dying in
that yea.r ?
Norm. — The problems in probability solved in the future in the
theory of life annuities and of life ineuronce, so fur it is presented in
thia book, will be like those of' Exercise LVIII. None of tho well
known theorems on probability are needed in solving such problems;
the mere definition of probubility iB guffcient. Hence, no further
theorems on probability are discussed in this text. The student is
referred for their consideration to books on college

algebra.

that the number dying between ages 25 and 28 is — 128 =


2E + d26 -k d27, the sum of those dying in their 25th, 26th,
or 27th years. Similarly, those dying between ages and c +
n are

— = dz + dz+l + • • • + (3) From Table XIII,


= 0, and hence 107, 198, etc., are zero becausc all are dead
before reaching age 96. The group of alive at age a are
those who die in the future years, so that
= da + dc-I-I + • • • d95.
It is convenient to use " (x) " to abbreviate a man aged c.
Le• apa represent the probability that (x) will live at least n
MATHEMATICS OF NVESTMENT

years, or that .(x) will stm be alive at age c + n. Since remain aliv at
age (c + n), out of alive at age c,

(5
When n = 1, we omit then 1 on npsand write p, for the prob ability
that (x) will live 1 year ;

(6
The values of pa are tabulated in Table XIII. Let q, represen the
probabüity that (x) will die before age (c + 1). Since of the group
of die in the first year,
150 MATHEMATICS OF INVESTMENT
7. That o, man agcd 28 will dic in the year after he is 38.
8. Thut [L nun aged will livc at leust, 12 years.
9. That, man agcd 35 will live at least 20 years.
10. If a man is {dive ut ago 22, between what ages iB he most
likely to
die and whut is his probability of dying in that your?
Nomn. — Tho problemR in probability solvcd in tho futurc in tho theory of
life unnuitios und of life insurance, NO fur us it prcHented in this book, will
bo like those of' IdxcrciBe, LV.III. None of the well known theorctng on
probabilit,y are needed in solving such the moro definition of
sufficient. Hence, no further theorems on probubilit,y tu•c diRcusxed in this
text. The student is referred for their conxidorution to books on college
algebra.

61. Formulas used with Table xm. — In Table XIII, we verify


that the munbcr dying between ages 25 and 28 is — 128 = ++
(127, the sum of those dying in their 25th, 26th, or 27th years.
Similarly, those dying between ages and -k n are

From Table Xlll , t — - 0, and hence 707, tog, etc., aro zero
because all urc dead before re:whing uge 90. "l'he group of nlivc at
age c arc those who die in tho future years, so that
(4)
It is convenient to use 'c (x) " to Lbbreviute, a man aged c. Let
npz represent tho probability that (x) will live at, lonst, n years, or,
thut .(x) will still bc olive, at age + m,. Since remuin olive at oge (x
+ n), out of ulivo Ot ago c,

(5)
When n = 1, we omit then = 1 on n'l)ø und writo pa for the
probability that (x) will live 1 year ;

(6)
LIFE ANNUITIES

Tho volues of pr ure tubulated in Tol)le XIII. Lot represent, the


probability thot, (x) will die, before age @ + 1). Since of the
group of dio in tho first your,
(7)
151
The values of qz are tabulated in Table XIII. Let nlqz represent the
probability that will die in the year after reaching age (c + n) ,
between the ages (c + n) and (c -k n + 1). Of the original group of
alive at age c, dz+n die in the year after reaching age (c + n).
Hence

(8)

Let Inqz represent the probability that will die before reaching
age + n. Of the group of alive at age c, (Zz — Zc-yn) will die
before reaching age + n, and hence the probability of dying is

(9)
Example I. — State in words the probabilities denoted by the following
symbols and find their values by the formulas Lbove :
(a) i7P36; (b) 151Q22; (c)
Solution. — (a) iB the probability that a man aged 25 will be alive at age
4.2; — = 89032•76567 (b) is the probability that o man aged 22 will die in the

year after he reaches age 37; — is the probability that a


man aged 22 will die before he iB 15 years older @efore reaching age 22 + 15
= 37); hoqn — 137.

EXERCISE LIX
1. Find the probability that a man aged 25 will live at
least (a) 30 years; (b) 40 years; (c) 70 years.
152 MATHEMATICS OF INVESTMENT
2. Find the probability that a man aged 30 will die in the
year after reaching age 40.
8. From formula 7 find the probability of mon aged 23 dying
within I year, and verify the entry in Table XIII.
4. From formula 6 find the probability of a man aged 37 being
alive a} age 38, and verify the table entry.
6. Find the probability that a man aged 33 will die in the year
after reaching age 55.
6. State in words the probabilitiea repre3ented by the following
symbols and express them aa quotient* by the formulas above: IdPa;
i71gu; 117Q25; 10P9.a; pqa; 181qu.
7. From formulas 5 and 9 provéthat I —Inq„.
NOTØ. —If we coneider the event of (x) living for at least n years, the
failure of the event means that (x) dies within n years. Hence, the result of
problem 7 should be true because, from Section 59 the sum of the
probLbilities of the BUCCæg and of the failure of an event is I, and p = 1 —
q.
8. What is the probability of a man aged 26 dying some
time after he reaches age 45?
9. Verify formula 4 for c = 90.
10. Verify formula 3 for = 53 and n = 5.
62. Mathematical expectation; present value of an expecta
— If a man gambles in a game where the stake is $100, and where
his probability of winning is .6, hig chances are worth .6 (100) =
$60. Such a statement is made precise in the following
Definiåon. — If p is the probability of a person receiving a sum
$S, the of the person is pS.
If the sum $S is due at the end of n years, the mathematical
expectation at the end of n years is pS. If money is worth the
effective rate i, the present value $A of expectation iB given by
A = psa + i)-n (10)
Norm. — In t,he future, the arithmetical work in all examples will be
performed by 5-place logarithms.
LIFE ANNUITIES

Example 1. — If money is worth 3å%, find the present value of


the expectation of a man aged 25 who iB promised payment of
$5000 at the end of 12 years if he is still alive.
Solution. — The probability p of recei'hng the payment iB the probability
of the man living to age 37, or p = From equation 10, the present value of the
expectation is
(Formula 5)
126

= 32986.4. (Tables VI and Xlll) No rrn. — In Example I it


would be BDid that the payment of $5000 Lt the end of 12 years ig
conti.ngent (or dependent) on the survival of the man. For brevity, in using
formula 10, we shall speak of the proaent vatue of a contingent payment
instead of, more completely, the present value of the eæpectætion Of thi8
paymen$.
EXERCISE LX
1. In playing a game for a stake of. $50, what is the
mathemåtical expectation of a player whose probability of winning
ig .3 ?
153
Norm —Suppose that a professional gambler should operate the game of
problem I and charge each player the value of his mathematical expectation aa
a fee for entering the game. Then, if a very large number of playera enter the
game, the gambler may expect to win, or lose, approximately nothing. This
follows from the facts pointed out in Not,e 2, Section 59, because, if a large
number play, approximately .3 of them may be expected to win the stake, and
the money won would, in this case, be approximately equal to the total fees
collected by the gambler. If, however, the gambler should admit only a few
player8 to the game, he might happen to win, or equally well lose, a large
sum, because out of a few games he hae no right to expect that exactly 3 of
them will be won. The principle involved in this note is fundamental in the
theory of insurance, and fin& immediate applicotion in problem 4, below. In
any financial operation which is esaenti811y similar to that of the profæiona.l
gambler above, the safety of the operator depends on his obtaining a large
number of players for his game.
154 MATHEMATICS OF INVESTMENT
2. At the end of 10 years a man will receive $10,000 if
he is olive. At 5% interest, find the present worth of his
expectation if his probability of living is .8.

8. A young man, aged 20, on entering college iB promised $1000 at the end
of 4 years, if he graduates with honors. At 5% interest, find the present value
of his expectation.

4. Out of buildings of a certain type, assume that the


equivalent of 2500 total losses, payable at the end of the year, will
be suffered through fire in the course of one year. (a) If an owner
insures hig building for $20,000 for one year, what is the present
value of his expectation, at 3% interest? (b) What is the least price
that a fire insurance company could be expected to charge for
insuring building?
5. A certain estate will be turned over to the heir on his
23d birthday. If the estate will then be worth $50,000, what is the
present worth of the inheritance if money is worth and if the heir is
now 14 years old ?
6. A boy aged 15 haa been willed an estate worth
$10,000 now. The will directs that the estate shall be allowed to
accumulate at the rate (.04, m — 2) until the heir is 21. If money is
worth to the boy, find the present value of his expectation.

63. Present value of a pure endowment. — Lf $1 is to be paid


to (x) when he reaches age (c + n), we shall say he has (or iB
promised) an n-year pure endowment of $1. Let nEz be the
present value of this endowment when money is worth the
effective rate i. The probability p of the endowment being paid
equals the prob-
155 MATHEMATICS OF INVESTMENT

ability of (x) living to age c -l- n, or p = np,. Hence, from formula


10, with S = 1,

(Formula 5)
In the future we shall use v as an abbreviation
for the discount factor (1 + Thus, v = (1 +
v2 = (1 + etc., vn = (1 + Hence,
nEx =(11)

The present value $A of an n-year pure endowment of $R to a man


aged is given by
Rvnz
A = R(nEx) - (12)
Non I. — Remember the subscript (c + n) on in formula 12 as the age 8t
which (x) receives the endowment.
Ecample 1. — A man aged 35 is promised a $3000 payment at age
39.
Find the present value of this promise if money is worth 6%, effective.

Solution. —The mnn aged 35 4-year pure endowment of $3000.


From formula 12, ite present value iB

_ = $2290.8. (TablæV1,X111)
Non 2. — Formula 11 may be derived by the following method. The present
value nEø iB the sum which, if contributed now by a man aged z, will make
possible the payment of $1 to him at the end of n years, if money cun be invuted
at the rate i, effective. Suppose that men of age make equal contributions to
common fund with the object of providing all survivors of the goup with $1
payments at the end of n years. Since men will survive, the nece•ary paymenta
at the end of n years total $13+n. The pregent value of this amount at the rate i
ig ZZ+B(I + ZZ+nvn, which is the Bum needed in the common fund. Hence, the
share which each of the people must contribute ig

the same aa obtained in formula Il.


EXERCISE LXI
156 MATHEMATICS OF INVESTMENT

1. A man oged 31 iB promiBed a gift of $10,000 when he


reaches age 41. Find the present value of the promise ot interest.
2. State in words what is represented by $2000 G7E38)
and find its volue at 5% interest,
LIFE ANNUITIES
S. A will specifies that the estate shall be turned over to the heir, now
aged 23, when he reaches 30 years of age. If the estate will then amount to
$150,000 find the present value of the inheritance ot 4% interest.
4. A man aged 25 has $1000 cash. What pt.u•e endowment, payable at the
end of 20 years, could he ptu•chase from an insurance company which will compute
the endowment ot a 4% rate? Make use of equation 12, to determine the unknown
quantity R.
5. If money is worth 3å%, what endowment payable at oge 45 could a man
aged 30 purchase for $7500?
64. Whole life annuity. — A whole life annuity is an annuity whose periodic
payments continue as long as a certain individual (or individuals) survives. We
shall deal only with the case where one individual is concerned. In speaking of a
life annuity we sh all always mean B whole life annuity unless otherwise
specified.
NOTE 1. — The periodic payments of all annuities will be supposed equal and will
be due at the ends of the payment intervals unless otherwise stated. When no rate of
interest is specified, it will be understood ag the rate i, effective.
Example I. — If money iB worth find the present value of a life annuity
of $1000 payable annually to man aged 92.
Solution. — He is prorn.iged payment, or endowment, of $1000 at age 93, another
at 94, and third at 95, which he will receive if he iB alive when they are due. No
payment is possible after he is of age 95 because he is certainly dead at age 96. The
present value of his expectation from the annuity is the gum of the present values Of
the equivalent three endowments, due in 1, 2, and 3 years. Trom formula 12, the
pregent values of these endowments are
10001En, IQ002Dn, and 10008Eu. Hence, by use of formula 12 and tables VI and XIII, we obtain

) - $456 65.
Let az be the present value of a ife annuity of $1 payable at the end of
each year to a man now aged $. This annuity is equivalent to pure
endowments of $1 payable at ages (c -F 1), (c + 2), • to age 95. The
MATHEMATICS OF NVESTMENT
present values of these endowments are tabulated below, and equals their
sum.

156.
AGE AT $1 ENDOW- TIME FROM Now ENDOW- 1B PRESENT VALn OF
PAYABLE PAYABLE Ermowxzrr

I yr.
2 yr.

95 95 — x
95—x

On adding the last column we obtain

ax + (13)
The present value $å of a life annuity of $R paid at the end of each
year is given by = Raz.
N0TE 2. —In contraet to the annuities certain considered
in Part I, life annuities are called contingent annuities,
because their paymenta are contingent (or dependent) on
the survival of (x). The life annuity interpreted aa being
paid to (x). Recognize that a, iB the progent value of
paymenta made at the end of each year during the life of
(x), regardees of who receives the payment8.
EERCISE LXII
I. By the process of Example I above, find the present
value of a life annuity of $2000 payable Bt the end of
each year to a man now aged 91, money is worth 5%.
2. If money is.worth 6%, find the present value of a
whole life pension of $1000 •payable at the end of each
yea,r to 8 mau now aged 92. Use formula 13.

8. (a) By use of formula 13, write the explicit


expression which would be. computed in finding the
LIFE ANNUITIES 159
present value of a life annuity of $500 pa,id at the
end of each year to a man aged 65, if money is worth
6%. (b) How many multiplications would be necessary in
computing the nu merator ?

4. By the method used in deriving the formula


for ax, find the present value of a Iffe annuity of
$1000 payable at the end of each 3 years to a man aged
35, if money is worth (.04, m = 1). Do not compute the
expression obtained.
65. Commutation symbols. — Auxiliary symbols (such as Dk and NE
below), called commutation symbols, are used in life an nuity and
insurance
formulas. From
formula 11 for Z,
we obtain

Let DE be an abbreviation
(14)
Thus, =
= Dz+n, and
(15)
The present value $A of an n-year pure endowment of $R is

A = R(nEc) = RDZ+n (16)


Ecample 1. — Compute D25 if money is worth 3å%.
Solution. — Dn — = — 37674.
Nom. —It iB very customary for insurance Companies to use as the rate in annuity
computations. The values of DID, Dlr, to DOE at are tabulated in Table XIV, and the
result of Example 1 above is seen to check the proper table entry. Formulas 15 and 16
may be uaed, in connection with Table XIV, only when the rate is Tables of the values
of Dk at other rates 1 are found in collections of actuarial tables. In problems in this
book, when the rate is not 3å%, formulas Il and 12 must be used.
To simplify formula 13 for az, .multiply numerator and denominator by tP. We obtain

1 See Tablea of AppZied Ma$hema*ica, by Glover.


MATHEMATICS OF NVESTMENT

v"lz
Since =
(17)
Introduce Nk as an abbreviation for the gum of all D'S from Db to DDS :
Nk = Dk + Dk+l + . . . + DBS. (18) Thus, — - + D 91 + D92 + D93 + Du +
D96. Since the numerator in formula 17 iB N
ax = (19)
LIFE ANNUITIES 161
The present value $Å of a life annuity of $R per year to (x) is

A = Raz = (20)
Norm. —The values of Nk axe tabulated in Table XIV ior the rate 3å%. For this
rate, formulas 19 and 20 may be used in connection With Table XIV. For 811 other
rates the previous formula 13 must be used. The values of Nk for B few other interest
rates are found in actuarial tables.
Example 2. — If money is worth what life annuity, payable at the end of each year,
can man aged 50 purchase for $10,000?
Solution. — lat R be the payment of the unnuity. From formula 20,

_ IOOOODEO = 10000(12498.6) = $738.83.

169165

EXERCISE

1. Compute the value of Dag for i = .035 and verify the enå ry in
Table NV.
2. By use of formu18 18 and Table XIV for the DB, find the
volue of

8. Find the present value of a life annuity of $1000 at the end of each year for a man
aged 24, at 3å%.
4. (a) Find the present value of a pure endowment of $3500 at the end of
12 years for man aged 33, at (b) Find the present value of the endowment at the rate
4%.
5. A man aged 65 is promised pension of $2000 8t the end of each year as long as he
lives. (c) If money is worth 3å%, find the present value of his pension. (b) What is the present value if
$2000 is to be paid at the beginning of each year?
6. An estate is worth $100,000 and is invested at 5%, effective. The annual
income iB willed to 8 woman, aged 30, for the rest of her life. Find the present value
of her inheritance if money is worth 3å%.
162 MATHEMATICS OF INVESTMENT
7. A man aged 45 agreed to pay $75 insurance premium at the
end of each year long as he lives. At intereat, what is the present value of
his premiums from the standpoint of the insurance company?
8. A man aged 26 has cgreed to pay $50 insurance premiums 8t the end of each year for
the rest of his life. At what is the present vüle of his premiums ?
9. A man aged 60 gives $10,000 to an insurance
company in return for an annuity contract promising him
payments at the end of each year as long ae he lives. If
money is worth to the company, what annual payment does he
receive ?
10. From the formulas previously developed,
prove that az = vpz(l +
Non. — Recognize that this formula would make the
computation of a table of the values of az very simple.
First, we should• compute '196, which iB zero; then, ax —
vpu(l + 095) gives the value of ago etc., for an, am, • •
• , down to azo.

66. Temporary and deferred life annuiåes. — A


temporary life annuity of $R per year for n years
to (x) furnishes payments of $R at the end of 1
year, 2 yeafs, etc., to the end of n years, if
(x) continues to live. The payments cease at the
end of n years, even though (x) remains alive.
Let represent the present value of a temporary
life annuity of $1 paid annually for n years to
(x). This annuity promises n pure endowments
whose present values are tabulated below.
Now uyrn.l
AGE AT WEIOH $1 ENDOWMENT 1B 2BEBENT oy
ENDOWMENT IS PAYABLE ENDOWMENT
PAYABLE

1 yr.
2 yr. v21Z+2
LIFE ANNUITIES 163

n yr.
The sum of the present values is an-I, or
2

Vlø+l
agil = (21)
Formula 21 applies for all interest rates. To
obtain a formula in terms of N and D (which, with
our tables, will be useful only
164 MATHEMATICS OF INVESTMENT

when the rate is multiply numerator and denominator in equation 21


by v". + vz+21z+2 + • . . + vz+nlz+n aril =
vzlz

= + DZ+n.
From formula 18,
= Dz+l +

NZ+n+l =
Dz+n,
Hence,
fore
- Nz+n+l =
+ +
(22)
If the annual payment of the temporary annuity is $R, the present
value $Å is given by
A = R(axäl) = R(NX+I — Nx+n+l) (23)

The definition of a deferred Me annuity is similar to that for- a


deferred annuity certain (see Section 26, Part I). A life annuity of $1
per year, whose term is deferred 10 years, to a man aged 30,
promises the first $1 payment at the end of (10 4- 1) or 11 years,
and $1 annually thereafter. Let nlaz be the present value of a life
annuity of $1 per year, whose term is deferred n years, to a man
aged c. The first payment of the deferred annuity is due at the end of
(n + 1) years. It is clear that a whole life annuity of $1 per year to a
man aged c pays him $1 at the end of each year for the first n years,
and also at the end of each year after that, provided that he lives.
The payments during the first n years form a temporary life annuity
whose present value is azil. The payments after the nth. year are
those of the deferred annuity, whose present value we are
representing by Blog. Hence, the present value az of the whole life

+
165 MATHEMATICS OF INVESTMENT

annuity is the sum of the Other two present values or az = nlaz (24)
(25)
LIFE ANNUITIES 166

On using formulas 19 and 22 in formula 25,

(26)

(27)

The present value $A of B life annuity of $R per year, deferred n


years, for a man aged c, is

(28)

EDRCISE INV
I. At 31%, find the present value of a life annuity of $1000 per annum,
deferred 20 years, to o man aged 23.
2. At find the present value of a life annuity of $2000
paid annuaJly for 25 years to 8 man aged 45.
3. If money is worth 5%, find the present value of a life
annuity of $1000 paid annually for 3 years to a maun aged 27.
4. A man aged 25 will pay 20 annual premiums of $50 each on a
life insurance policy, if the man remains alive. If the first premium is cash, find
their present value, at 3å%.
5. A man aged 50 gives an insurance company $10,000 in
return for a contract to pay bim a fixed income at the end of ea.ch
year for 20 years, if he lives. If money is worth to the company,
what is the an;nual income ? Use formula 23.
LIFE ANNUITIES 167

6. A man aged 40 pays an insurance company $20,000 in


return for a contract to PLY him a life annüity whose first annual
payment will be made when his age is 65. Find the annual payment,
if money is worth to the insurance company, by use of formula
28.
T. A corporation has promised to pay an employee, now aged 48,
8 pension of $1000 at the end of each year, starting with 8 payment
on his 61st birthday. At 3å%, what is the present value of this
obligation?
NOTE. — Any pension system instituted by a company constitutes a
definite prent obligation whose value can be determined by finding, ng in the
problem above, the present value of the pension promised to each employee.
8. A man aged 43 estimates his future earnings at $5000 at the
end of each year for the next 25 years. At find the capitalized
(present) value of his earning power.
168 MATHEMATICS OF INVESTMENT

67. An.ntffes due.—A life annuity due is one whose payments


occur at the beginnings of the payment intervals, so that the first
payment is cash. Let az be the present value of a life annuity due of
$1 paid annually to (x). A cash payment of $1 is due and the
remaining payments of $1 at the end of each year form an ordinary
life annuity whose present value is az. Hence, the present value of
the annuity due is given by
(29)
From formula 19,

ax = (31) Dx
The present value $.4 of a M.fe annuity due of $R paid annually is
A = R(ax) = (32)
Let a;äl be the present value of a temporary life annuity due,
whose term is n years; paying $1 annually, to a man aged c. The
first $1 is paid cash and the remaining payments form an ordinary
temporary life annuity whose term is (n — 1) years. Hence,
(33)
From

formula 22,
169 MATHEMATICS OF INVESTMENT

Hence,
Since Dz +
(34)

The present value $Å of a temporary annuity due of $R payable


annually for n years to (x) is
A = R(axäl) R(Nx — Nx+n). (35)

Ecampte 1. — In a certain insurance policy, the present value of the benefits


promised to the policyholder is $3500. If the policyholder is of age 27, what
equal premiums should he pay to the insurance company at the beginning of
each year for 10 years, in payment of the policy, if money iB worth to the
company ?
LIFE ANNUITIES 170

Solution. — 18t $R be the annual premium. The premiums form a


temrary life annuity due whose present value equals $3500. From
formula , with A = $3500, = 27, and n — 10,

R(N27 — N") . 3500 Dr,


NW — Nfl

Pure endowment : = R(nEx) = R DX+n. (16)


x
Whole life annuity: Nx+l . (20)
Temporary life annuity : = R(axä) RNx+1 — N*+n-+l. (23)

Deferred life annuity : = R(nlax) = RNx*-n+1.


(28)

3
5
0
0(34601) = $421.21. (Table
XIV) 287510
NOTE 1. — In discussing premiums on life insurance policies,
formulas 32 d 35 are of greut use. Formulas 16, 28, 32, and 35 of
this chapter are the we shall use most frequently in the future.

Summary of present value formulas

Whole life annuity due • (32) Temp. l.ife annuity due :(35)

MISCELLANEOUS PROBLEMS
LIFE ANNUITIES 171

L. A mam aged 40 pays $10,000 to an insurance company in return for ontract to pay him o
fixed annual income for life, starting with a paynt on his 60th birthday. Find the annual
income if money iB worth 6 to the company.

2. At age 65 a man considerg whether he should (a) pay his total


ings of $20,000 to an insurance company for B life annuity
whose first lual payment would occur in 1 year, or (b) invest his
savings at 6%, bctive. Find the difference in hig annual income
under the two methods, uming that money iB worth 31 0/0 to the
insurance company.
l. In problem 2, what be received by the heirs of the man at his
Ith if he adopts plan (a)? What will they receive under plan (b) ?
172 MATHEMATICS OF INVESTMENT

4. A certain insurance policy taken out by a man aged 28 calls for


premiums of $200 ot the beginning of each year as long as he lives. Find the
present value of these premiums at 31%.
6. A certain insurance policy matures when the policyholder is
of age 35 and gives him $2000 cash or the option of equal payments
at the beginning of each year for 10 years as long as he lives. If
money iB worth find the annual payment under the optional
plan.
6. A boy of 16 has been left an estate of $100,000, which iB
invested at 5%, effective. . If he lives, he will receive the income annually for
the next 10 years a,nd the principal of the estate when he reaches age 26. If
money is worth 3å%, find the present value of hig inheritance.
7. Derive formula 13 for by the mutual benefit fund
reasoning used in Note 2, Section 63. Thus, at the end of 1 year,
$1C+i will be needed for payments; at the end of 2 years, etc., $106
at the end of (95 — c) years. Discount all of these payments and
divide by
8. Derive formula, 21 for by the mutual fund method of
reasoning.
9. A man aged 22 agrees to pay $50 as the premium on an
insurance policy at the beginning of each year for 10 years if he
lives. Find the present value of his premiums at interest.
10. The present value of the benefits promised in L certain
insurance policy iB $8000. If the policyholder iB aged 30, what equEd
premiums should he agree to pay ot the beginning of each year for 15 years,
provided be lives, if money is worth to tho insurance company?
173 MATHEMATICS OF INVESTMENT

11. A man is to receive a life annuity of $2000 per year,


the first payment occurring on his 55th birthday. If he postpones
the annuity so that the first annual payment will occur on his 65th
birthday, what will be the annual income, if the new annuity has the
some present value the former one, under interest ?
12. A certain professor at age 65 enters upon a pension of the
Carnegie Foundation which will pay $2000 at the end of each year for life. In
order to have, at age 65, an amount equal to the present value at of the pension
he is to receive, what equal sums would tho professor have had to have
invested annually at 6%, aasuming that his first invesånent would have
occurred at age 41 und hig laet at oge 65?
CHAPTER IX
LIFE INSURANCE

68. Terminology. — Insurance is an indemnity or protection


against loss. The business of insuring people against any variety of
disaster is on a scientific basis only when a large number of
individuals are insured under one organization, BO that individual
losses may be distributed over the whole group according to some
scientific •principle of mutuality. That is, ea,ch of the insured should
pay in proportion to what he is promised as an insurance benefit. In
this chapter we shall discuss the principles and most simple aspects
of the scientific type of life insurance furnished by old line, or legal
reserve companies.
When an individual is insured by a company, he and the company
sign a written contract, called a policy. The individual is called a
policyholder, or the insured. In the contract the compony promises to
pay certain sums of money, called benefits, if certain events occur.
The person to whom the benefits are to be paid is called the
beneficiary. The insured agrees to pay certain sums called gross or
omce premiums in return for the contracted benefits. The policy date
is the day the contract was entered into. The successive years after
this date are called
policy years.
The fundamental problem of a company iB to determine the
premiums which should be charged L policyholder in return for speci
fied benefits. Every insurance company adopts certain mortality
table and an assumed rate of earnings on invested funds as the basis
for its computations. We shall use the American Experience Table
and 3å%, os is the custom among many companies. The net
premiums for a policy are those whose present value is equal to the
present value of the policy benefits under the following
aasumptions : (a) the benefits from the policy will be paid at the ends
of the years in which they fan due; (b) the company's funds earn
interest at ecactly the specified rate (3å% in our case) ;
105
176 MATHEMATICS OF INVESTMENT

(c) the deaths among the policyholders will occur at


ecactly the rate given by the mortality table (Table
XIII in our case). Under these assumptions, if a
company were run without profit or administrative
expense, it could afford to issue policies in return for
these net premiums. The actual gross premiums for a
policy are the net premiums plus certain amounts
which provide for the administrative expense of the
company and for added expense due to violations of
the theoretical conditions (a), (b), and (c) assumed
above. In computing gross premiums, insurance
companies use their own individual methods. Our
discussion is concerned entirely with net premiums
and related questions.
Non. — In the future, if the interest rate in a problem is not
given, it iB understood to be 3å%.
69. Net single premium; whole life — If
a policyholder agrees to pay all premium obligations
in one installment, it is payable immediately on the
policy date and is called the single premium for the
policy. The net single premium is the present volue
on the policy date of all benefits of the policy.
A whole life insurance of $R on the life of (x) is an
agreement by the company to pay $R to the
beneficiary at the end of the year in which (x) dies. A
policy containing this contract iB called a whole life
policy.
Example 1. — Find the net single prernium for a wholo life
policy for $1000 for a man aged 91.
Solution. — Suppose that the company issues whole life
policies for $1000 to 101, or 462 men of age 91. During the
first year, dot = 246 men will die; $246,000 in death claims
will be payable to beneficiaries at the and of I your. The
present value of this payment ia 246,000
v. The other entries below mre eaaily verified.
177 MATHEMATICS OF INVESTMENT

YAAR Dun or ENI) PliEB10NT or


P0L10Y YEAR
YEAR

1 $246,000 246,000 v
= 246
137,000 137,000
2 - 137 dn
58,000 58,000 v'
3 = 58 d94
18,000 18,000
4 - 18 d9b
3,000 3,000
3
5
LITE INSURANCE 178

Hence, on the policy date, the insurance company


should obtain through the net single premiums from the
men, a fund equal to the sum of the last column. This
sum, divided by 462, is the Bhare or net Bingle
premium paid by each of the men. By use of Table VI,
we find that each pays
246000 + 137000 + 58000 VS + 18000 V' + 3000
= $943.98.
462

Let $14 c be the net single premium for a


whole life insurance of $1 on the life of (x).
To obtain by the method of Example 1 above,
suppose that a company issues whole life
policies for $1 insurance to each of men of
age c. During the first policy year, dz will
die ; $dz in death benefits is payable to
beneficiaries at the end of 1 year. The
present value of these benefits at the rate i
is dz (1 -l- = vdz. The other entries below
are ea,sily verified.
BENDFITS END
POLICY DEATHS PRNENT VA-LUZ 0B

t.ßdZ+1
1 v8dZ+2
2
3

96 —c
In the last row, notice that when the group
reaches age 95, the policies have been in
force (95 — c) years, or the (96 — z)th year
is just entered on. Hence, $d9b is due at the
end of (96 — c) years. From the net single
LITE INSURANCE 179

premiums paid on the policy date, the company


must obtain a fund equal to the sum of the
values in the last column. Thc share of each
of the men, or his net single premium A z, ig

(36)
On multiplying numerator and denominator
above by vz,
180 MATHEMATICS OF INVESTMENT

Introduce a new symbol Ck = vk+ldk. Thus, C53 = v54d63, CDS = v 94d93,


etc., v*+idz = Cc, vz+2dZ+1 = Cz+l, and v9dd05 = C95. Hence
Introduce new symbol

Mk = Ck + CHI + . . . + C95• (37)


Thus, M92 + + + CD6 ; MC = cc + +
Hence(38)

The net Bingle premium for whole life policy of $R for (x) ig
RMx

(39)
Non. —The values of ME for the rate 3å%, — (1.035) -r, are given in
Table x.1V.
EXERCISE LXV
Use formulas 38 and 39 unless otherwise directed.
1. Compute the values of C94 and of CBD at 3}3 to; verify the
entries for and for in Table xlv.
V 2. By the method of illustrative Example 1, page 166, find
the net single premium for a whole life insurance for $1000 for
a man aged 93, if interest at the rate 5%.
8. Find the net single premium for a whole life insurance of $1000 on
thel.ife of a man, (a) aged 90; (b) aged 50; (c) aged 30; (d) aged 10.
4. How much whole life insurance can a mon aged 50
purchase from a company for $1600 cash ?
6. How much whole life insurance oun a man aged 35 purchase from a company for
$2000 cash ?
70. Term insurance. — An 0-year term insurance for $R on the life of (x) promises the
payment of $R at the end of the year in which (x) dies, only on condition that hig death
occurs within n years. Thus, a 5-year term insurance gives no benefit unlesa (x) dieé
within 5 years. I-at A 1 7 represent the present value of an Il-year term insurance for $1 on
the life of (x). To obtain the value of aasume that the company issues n-year term in-
LITE INSURANCE 181
surance policies for $1 to each of men aged c.
The present values of the benefits which will
be paid are tabulated below ; the policy has
no force after n years.
BENEFITS DOE AT PRESENT VALUE
Por,rcy YEAR Dnarzs DÜRINO END OF YEAR or

vdz
1
2

The net single premium paid by each man is the


sum of the 188t column, divided by la, or
Vdz + V2dz+1 + • • • + vndæ+n—l. (40)

On multiplying numerator und denominator above


by and on using the symbol = vk+tdk,

Al C" + Cz+l -l- (41)


Since = cz + Ca+L + • • + Cc+n—l + Cz+n -k • • • + C96' and MZ+n
= it is seen that the numerator in equation 41
is Mz — M • hence

xnl ¯ D* (42)

Tho net single premium $A for n-year term


insurance of $R on the lifo of (x) is

(43)
The not single premium for a I-year term
insurance for (x) is called the natural
premium ut age c. The natural premium for $1
insurance iB obtained from oquation 42, with n
= 1 :
182 MATHEMATICS OF INVESTMENT

Natural Premium — A (44)


where — Mz+l = Cz, because of formula 37.
EDRCISE LXVI
Use formulas 42 and 43 unless otherwise specified.
1. By use of the method used in deriving
formula 40, find the expresBion for the net
single premium for a 3-yeur term insurance for
$1000 on the life of a man aged 25 and compute
its value at 5% interest.
2. Find the net single premium for a 10-year
insurance for $2000 on the life of a ma,lf
aged 31.
3. Find the natural premium for $1
insurance at agc 22; at age 90.
4. (a) Find the net single premium for a
whole lifc insurance of $1000 at age 50. (b)
Find the net Bingle premium nt age 50 for a
10-year term insurance for $1000.
5. How much term insurance for 10 years can
be purchased for $2000 cash by a man aged 35?
6. I-low much term insurance for 10 years can
be purchased for $2000 cash by o man aged 55?
71. Endowment insurance. — An n-year
endowment insurance of $R on the life of a man
aged furnishes
(a) a paym,ent of $R at the end of the year in which (x) dies,
'if he diC8 within n years, and (b) a purc endowment of $R to
(x) at the end of n years if (x) is alive at that time.
Thus, a 20-year endowment insurance of *1000
pays $1000 at death, if it occurs within 20
years; or, if (x) ig alive at the end of 20
years, he receives the endowment of $1000. Let
represent the net single premium (or present
value) of on n-year endowment insurance of $1
LITE INSURANCE 183
on tho life of (x). Tho present value is the
sum of the present values of (a) the n-yeur
term ingurance for $1 on the life of (x), and
of (b' the n-ycuu• pure endowment of $1 to
(x). Hence, on using formulas 16 and 42,

(45)
If the endowment insurance ig for $R, tho net singlo premium $A iB
given by
R Mg-Mx n+Dx n
EXERCISE LXVII
I. (a) Compute the net single premium for L $1000, 20-year endowment
insuraneo on the life of a man aged 23. (b) Compute the present value of n purc
endowrnent of $1000 payable to the man at age 43. (c) Find the not single
premium for a 20-yeai term insurance for $1000 on tho life of the mun aged 23,
by using (a) and (b).
'2. Find tho net singlo premium for a 10-year endowment insurance for $5000
on the life of a mun aged 30.
3. (u) Find the net singlc premium for a 10-year endowment insurance for
$3000 on the life of man aged 26. (b) Find the net single premium för 10-year
tcrm insurance for $3000 on his life. (c) From the results of (a) uncl (b), find tho
present vuluo of a 10-year pure endowment of $3000 for the man.
J 4. How much 20-ycar endowment insurance can a man aged 33 purchaso for
$3000 cash ?
6. How much 10-yeur endowment insurance can a man aged 45 purchose for
$2500 cash ?
72. Amual premiums. — If the net premiums for a policy are
payable annually, instead of in one installment (the net single
premium), they must satisfy the condition that the
(pr. val. of annual premiums) = (net single premium), (47)
because the' net single premium is the present value of the policy
benefits. When paid annually, the premiums for a policy are always
equal and are payable •at the beginnings of the years, as long as the
policyholder lives.
184 MATHEMATICS OF INVESTMENT
E',cumpZe 1. — (u) Find the net Bingle premium for a 10-year
term insurance for $10,000 on the life of a man aged 46. (b) Find the
equivalent annual premium which tho man might agree to pay for 10
years, if he lives. Solution, — (a) Ih•om formula 43, tho not Bingle
premium is
10000(M46 - Moa) - 31119.30. (Table XIV)
Thut is, tho preeont value of the insurance benefits is $1119.80. (b) lat P be the unnuui promium.
Tho 10 premiums form 10-year life annuity due puid by u man aged 46. Thcir present value is -n)
and it must equal $1119.30. Hence,
1119.30 (Formula 35)
(Table xrv)
185 MATHEMATICS OF INVESTMENT
The annual payments of $137.33 have a value equivalent to $1119.30 paid

No-E 1. — The solution of (a) wag not necessary in order to solve (b) above.
Thus, we may write, immediately, from equation 47,

p — N68 = $137.33.

In insurance practice the most simple forms of insurance policies


are those tabulated below. Their names, policy benefits, and manner
of premium payment should be memorized. All prcmiumg are
payable in advance, at the beginning of the year. The numbers
assigned are merely for later convenience in this book.

To determine the net annual premiums for these policies, we use


the fundamental equation 47, amd the method 1 of Note 1 above.
Consider an ordinary life policy for $1 for a man aged c. Let Pa be
the net annual premium. The premiums paid by the man aged c form
a whole life annuity due whose present value is Pa(az). The net
single premium for the policy ig A z. Hence,
from equation 47,

Pø(az) Am; Pa (Formulas 32, 38)

(48)
The is advised to solve problem below before reading the rest of Vhe eec#on.
LITE INSURANCE 186

Let nPz be the net annual premium for n-payment life poLicy for
$1 for a man aged c. The premium payments by (x) form an n-year
life annuity due whose present value is Hence, from equation 47,
nPc(aEl) = (Formulas 35, 38)
A

(49)

It is left as an exercise (problem 3, below) for the student to prove


that the net annual premium for an n-year term insurance policy for
$1 for (x) is given by

(50)
Let Pfil be the net annual premium for an n-year endowment policy for $1 for
(x). The premiums paid by the man aged form on n-yea,r life annuity due whose
present value is PEI(afiD. The net single premium for the policy is Aäl. From
equation 47, PEI(azöD = Azm;

p . (Formulas 35, 45)

. (51)

If the policies I to IV are for $R instead of $1, the annual


premiums are found from formulas 48 to 51 by multiplying by R.

LXVLI
LITE INSURANCE 187

1. By the method of Note 1, page 172, find the net annual premium fo a 5-
payment life policy for $1000 for a man aged 45.
2. A man aged 29 hag agreed to pay 15 annual premiums of $100 for
a certain policy. Find the net Bingle premium for the policy.

3, EstabliEh formula 50 for the net annual premium for an n-year


term insurance policy for $1 for a man aged c.
4. The net Bingle premium for a certain policy for a man aged 26 iB
(a) the net m.auaa premium it he to pay premiums
188 MATHEMATICS OF INVESTMENT

annually for life? (b) What is. the net annual


premium if he agrees to pay annually for 12 years ?
6. For a man aged 25, find the net annual
premium (a) for an ordinary life policy for $2000 ;
(b) for a 20-payment life policy for $2000.
6. Find the net annual premium for 10-year
term policy for $1000 for man (o) aged 32; (b) aged
42; (c) aged 62.
7. Find the net annual premium for a 20-year
endowment policy for $1000 for a person of your
own Lge.

8. (a) Find the net annual premium for a 5-year

9. A whole life insurance policy for $1000


taken at age 30 states that the annual premiums were
computed us if : (a) term insuranco of $1000 were
given for the first year, and (b) an ordinary whole life
policy were then written when the man •reaches age
31. Find the net premium (a) for the first year, and
(b) for the subsequent years.
NOTE. — Such a policy is very common and is said to be
written on the I-year term plan. The advantages from un
insurance compuny's standpoint are apparent after reading the
next chapter.
10. A certain endowment policy for $1000
taken ot age 23 provides that the net premium for the
1st yeur is that for I-yenr term insurwnce and that the
net premiums for the remaining 19 years are those
for u 19year endowment policy for $1000, taken Lt
189 MATHEMATICS OF INVESTMENT

age 24. Find the net annual premium (a) for the first year; (b) for
subsequent yenrs. This policy iB another example of the I-year term
plan.
Il. How much insurance on the 20-pnyment life plan can u man aged 32
purchase for n net annual premium of $75? It is udviBabIc, filSt, to find the
equivalent net single premium.
12. How la,rge 20-yeur endowment policy can L man agod 23
purchase for net annunl premiums of $100?
13. (a) For a boy aged 16, find the not annual prelnium for
IL endowment policy, which matures nt age 85. (b) Find the not
nunual premium for an ordinary life policy for $1000 tulcen at uge
16. (c) Explain the small difference between the reNuIts.
14. A man aged 30 takes out L policy which provideB him
with $10,000 insurance for the first 10 years, $8000 for tho next 1()
years, und $5000 for the remainder of his life. He iB to pay
prcmiumg annually for 10 years. Find the net annual premium.
190 MATHEMATICS OF INVESTMENT
15. A certain policy on muturing at age 55 offers the option
of pure cndowmcnt of $2000, or an equivalent amount of paid up
whole life insurancc, that is, as much insurancc as the $2000,
considered as a net single premium, will buy. Find the alnount of
puid up insurance.
No•rn ON Gnoss PnnM1üMB. — Premiums previously discussed were net
premiums, or values Of the bcnefits to be paid under the policy. In conducting
un insurance company there is expense duc to the salaries paid to administrative
ofliciuls, tho commissions paid to agents for obtaining new policyholders, the
expense of the medical ex:unination of policyholders, bookkeepiug oxpcnse, otc,
To provide for these it0111s and for unforeseen contingoncies, it is ncccsgary for
the company to add to the net premium an amount called the loading. The not
premium plu8 tho loading is the gross or actual promitun paid by the
policyholder. Sometimog tho loading iB detormined a certain percentage of tho
not premium plus constant charge independent of the nature of the policy.
S01netimes the loading moy be determined simply percentugo of the net,
promium, the percentage either being independent, of tho policyholder's uge, or
varying with it. Each company uses its own method for loading, but the resulting
gross premiums of all large, well-munugcd companies ure essentially the eume.

SUPPLEMENTARY MATERIAL
73. Net single premiums as present values of expectations. — A
whole life policy on life agcd promises only one payment, due at the,
end of the year in which (x) dies. However, we may think of the
policy as promising payments at the end of each year up to the man's
96th birthday, the payment at each date being contingent on his death
during the preceding year. Then, the method used in doriving
formulas for life annuities may be used to obtain the present value, or
net single premium, for the policy.
Consider Obtaining the net singlo premium A, for whole lifo
insuranc.c of $1 on tho life of (x). At the end of 1 year, $1 will be
paid if (x) dies during the preceding year. The probability of (x)
dying in this year is from formula 10 with S 1, the present value of
LITE INSURANCE 191

the expectation of this payment iB -2 (1 -k or The other present


values below aro verified similarly.

PAYMENT or
PROBABILITY PRESENT
Sl OF PAYMENT
MAN DEES AGOS V&LÜE
WILL BE MADE BRING MADE OF PAYMÄNT
AT END OF

I yr. a.nd (z + I)

2 yr. (z + I) and (z + 2)

96 — $ yr. 96 und 96
voe—ZdB5

The expression obtained for A z on adding the


present values in the last column is the same
as previously obtained in formula 36.
Non. — From the present point of view, an insurance
company may be likened to o gambler who plays against
all of the beneficiaries of the policies. The net
single premiums are the present values of the
expectations of the beneficiories. So many players are
involved as opponents of the company that the
probabilities of winning and of losing a.s given by
the mortality table will be practically certain to
operate. Hence, the Will neither loge nor win in the
long run.
EXERCISE INX
1. By the method which was used above to obtain Az, find
tho expresBion for the present value of a 10-year term insurance
policy for $1000 on a life aged 29.
2. By the method above, derive the
formula 40 for A
192 MATHEMATICS OF INVESTMENT
74. Policies of irregular Vpe. — Equation 47
enables us to find the premiums for any policy
for which the present value of the benefits is
known.
Example 1. — A policy written for mun aged
32 promises tho following benefits : (a) Term
insurance for $5000 for 28 years; (b) a life
annuity of $1000 paid annually, first puyment
due at age 60. It iB agreed that premiums
shall be paid annually for 28 years. Find P,
tho net annual premium.
Soh%n. — The præent value of benefit (a) iB ;
benefit
(b) iB B life annuity, term deferred 27 years, whose
present value ig 1000G71aa).
LITE INSURANCE 193

The annual premiums form a 28-year temporury annuity due, whose present
value iB Hence, from equation 47, P(aom) = 5000(Aå2Ä) + IOOO(mlau), p No ¯
+ 1000&. (Formulaa 35, 42, 28)

MOO)

1000
= $234.43. (Table XIV)

EXERCISE LXX
Find the periodic premium payment for each policy described. The
age of policyholder is the age Lt the time the policy was written.
OF METE0D or
PROB PAYING
BENEFITS OF POLICY POLICYBOLD*
.
R PREMIUMS

(a) 10-year term insurunce for 10 annual


$100().
1. (b) A pure endowment of $2000 at
27 premium
the end of 10 yean. s
(a) Term insurance of $10,000 for 20 years.
20 annual
2. (b) Life annuity of $1000 paid annually, first 46 premiums
payment at age 66.
Life annuity of $1000 paid annually, first 25 annual
3. payment at age 65. 30 premiums

10 annual
4. 45 premiums
LITE INSURANCE 194

Life annuity of $1000 paid annually, first


PLYment Lt age 70.
(a) Term insurance of $10,000 for
first 10
yearg. 20 annual
5. 45 premiums
(b) Term insurance for $6000 for
next 20 years. (c) Life annuity of $2000, paid
annually with first payment ut age 66.
6. In what way does the policy in problem 1 differ from a 10-year endowment
policy?
NOTE. — The policy of problem 4 iB culled an annuüv policy and iB a
familiar form for thoso wishing protcction in their old age. Thig same feature of
protection iB preaent in the policy of problem 2.
CHAPTER X
POLICY RESERVES
75. Policy reserve. — At age 30, the natural premium for $1000
insurance, that is, the net single premium for I-year term insuronce
for $1000, is found to be $8.14. This is the sum which each of men of
age 30 should pay in order to provide benefits of $1000 in the case of
all of the group who will die within 1 year. The $8.14 premium is the
actual expense of an insurance company in insuring a man aged 30
for $1000 for 1 year. The expense of insurance for 1 year increases
continually during life, after an early age, being $17.94 per $1000
insurance at age 55 and $139.58 at age 80.
Consider a man aged 30 who takes out a $1000, ordinary life
policy. Throughout life he pays a net level premium (that is, a
constant premium) of $17.19, ag obtained from formula 48, and is
insured for $1000 all during life. The expense of the company in
insuring bim during the year is the natural premium, $8.14. Hence, in
the first year the man pays (17.19 — 8.14) — $9.05 more than the
expense. The insurance company may be considered to place this
unused $9.05 in a reserve fund which will accumulate at interest for
future needs. Up to ngo 54, cach annual premium of $17.19 is more
than the expense of insurnncc and the company places the excess
over expcnge in the rcscrvo fund. At age 55 the $17.19 premium is
less thun the insurance expense, which is $17.94. The deficiency,
(17.94 — 17.19) = $.75, is taken from the reserve fund. From then on
until the end of life, the expense of insurance is met more and moro
largely from tho rcserve fund. Thug, at age 80, the expcnso iB
$139.58 (tho natural premium, ag given above) so that (139.58 —
17.19) = $122,39 comes from the reserve.
For every insurance policy (except o I-year term policy) where a
net level premium P is paid, the annual expense of insurance
178
during the early policy years is less than the premium P. Hence, the
insurance company should place the unused parts of the premiums in
196 MATHEMATICS INVESTMENT
a reserve fund and accumulate it at interest to answer the future needs
of the policy. When the expense of insurance, in later years, becomes
greater than the level premium, the deficiency is made up by
contributions from the reserve fund. The reserve funds should be
regarded as a possession of the policyholders, merely held and
invested by the insurance company.
The reserve on a policy at tho end of any policy year, before the
next premium duc is paid, is called the terminal reserve for that year.
In this chapter we consider the determination of the terminal reserve
for a given year.
Example 1. — Form a table showing the terminol reserves for the first 6 years
for a 5-poyment life policy for $1000 written Lt age 40.
Solution. — From formuhL 49, the not unnun.l premium iB $89.4574.
Assume thut the compuny issues the surne policy to cach of = 78,106 men. The
following table shows the disposition of the funds received aa premiums.
Pam FüND
AT START orOF FOND
Ynan DEATH Sunvrvoa
POLICY
END or
F,ND OF

1 $ 84
171
2
262
3 357
4 450
33,817,598
5 400
(5 34,173,224

For cxnmplo, at tho beginning of tho fiZ'8t your, (78,106) (89.4574) iB received
in prcmiumx. At, the end of tho your, intercat nt .3å% added to the premium
fund gives $7,231,71(). During the year, = 705 deaths occurred 80 thut $705,000
iB puyuble to bonefiuiuries, leuving Thero ure 77,341 Bttrvivorg; tho total fund
divided by 77,341, gives $84 tho sharo, or rogervo, per policy. At the beginning
of the 2d your, 77,341 men pny premiums, etc. After tho 5th your, no more
premiums will be, rcceivcd, go that
POLICY RESERVES 197
OF
EnRCISE LXXI
1. Asume that an insurance company issues $1000 ordinary life
policies to each of men of age 92. Compute table showing the
disposition of funds received aa premiums and the total ræerve per
policy at the end of each year.
76. Remaining benefits of a policy; computation of the reserve. —
At any time after a policy iB written, the remaining benefits of a
policy are the promised payments of the policy as they affect the
policyholder at hig attained age.
Ecample I. — A certain insurance policy, written for man aged 32,
promises him (a) temporary life insurance for $1000 for 25 years; (b)
a pure endowment of $1000 payable at the end of 25 years; (c) a life
annuity of $1000 payable annually, first payment at age 60. (1)
Describe the remaining benefits, 8 years later. (2) Find the present
value of the remaining benefits, 8 years later.
Solution. — (1) Eight years later, the attained age of the man is 40 years. The
policy promises (a) term insurance for $1000 for 17 years to a mm aged 40 ;
(b) a pure endowment of $1000 payable at the end of 17 years to man now aged
40 ; (c) a deferred life annuity, for a man aged 40, of $1000 paid annually. Since
the first payment of the anhuity iB due at age 60, which is 20
years later, the annuity iB deferred 19 years.
(2) The present value of the remaining benefits at age 40 iB the sum of the
present values or net single premiums for the three individual benefits or

which can be computed by use of the proper formulEE

Consider the conditions in regard to a policy, written for a


man aged c, n years after the policy date. The attained age of
the policyholder is $ + n, and the reserve fund for the policy
contains a certain amount $ V, the terminal reserve at the
198 MATHEMATICS INVESTMENT
end of n years. The company ié liable for the remaining
benefits of the policy, and the policyholder is • liable for the
futtire premiums. Since all future benefits must be paid from
the reserve and from the future oremiums, the following
equation is satisfied :
'ingle premium for Pr. val. at age x + n Terminal
. val. of) remaining = • of net premiums + reserve at (52) at age X + n due future age x

+n
To find the terminal reserve on policy, first find the net annual
premium and then use equation 52.
Ezample 2. — Find the terminal reserve at the end of 6 years on a
20year endowrnent policy for $1000 written at age 24.
Solutton• . — @ The net annual premium is 1000 (P24iö1) $39.085, from
formula 51. (b) The remaining benefits at the attained age of 30 years are a
pure endowment of $1000 payable at the end of 14 years to mon now aged 30
and term insurance for $1000 for 14 years on a life aged 30 ; in other words, the
benefits form a 14-year endowment insurance for $1000 for a man aged 30.
The remaining premiums form a 14-year temporary annuity due. Let V be the
reserve at the end of 6 years. From equation 52,
1000(Aafi1) 39.085(agoØ) + V'
IOOO(Mao — Ma + _ 39.085(N30 — N„). (Formulas 46, 35)
DSO Dao

IOOO(M80 — + Du) — 39.085(Nao —


6634000
$217.9. (Table XIV)
The method used in Example 2 may be applied in the case of any
standard policy to obtain a general formula for the reserve at the
end of a given number of years. For example, consider an ordinary
life policy for $1 written for a man aged c. Let n v, represent the
terminal reserve at the end of n years. The net annual premium is
Pa = The remaining benefit at the at-
POLICY RESERVES 199
tained age (c n) iB whole life insurance for $1 for a man aged (c + n).
The remaining premilims form a whole life annuity due of Pz payable
annually by a man now aged @ -k n). From equation 52,

(53)
Non I. — For the cdvantage of the insurance actuary, who has occaeion to
compute the reserves on numerous policies, it iB advisable to devolop general
formulag and convenient numerical methods for the computotion of rcgerveg.
In the case of student meeting the subject for the first time, it iB more
important to appreciate thoroughly the truth of equation 52. Bueh appreciation
is attained only by direct application of the equation. The problems of Exercise
LXXII below should be solved by direct application of equation 52, aa was done
in illustrative Example 2 above.
200 MATHEMATICS INVESTMENT

or
EXERCISE
1. If the net single premium for the remoining
benefits of a policy is $745, and if the present
value of the future premiums is $530, what iB the
reserve ?
2. At an attained age of 42, the net single
premium for the remaining benefits of a policy
is $750. There ore Bix annual premiums of $50
remaining to be paid, the first due
immediately. Find the policy reserve.
S. At the attained age of 44, the reserve
on L certain policy is $500. Annual premiums
of $25, the first due immediately, must be
paid for the remainder of life. Find the
present value of the remaining policy
benefits.
4. A $1000, 10-payment life policy is
written Lt age 34. (a) Find the reserve on the
policy at the end of 6 years. (b) Find the
reserve at the end of 10 years.
6. A $1000, 5-payment life policy is
written at age 40. (a) Take the premium as
computed in illustrative Example 1, Section
75; compute the reserve at the end of 3 years
and compare with the result given in the table
of that example. (b) Find the terminal reserve
ot the end of 5 years and compare with the
table.
201 MATHEMATICS INVESTMENT

6. A $2000, 20-year endowment policy is


written otage 33. (a) Find the terminal
reserve at the end of 15 years. (b) What is
the terminol reserve ut the end of 20 years,
before the endowment iB paid ?
7. In the case of a I-year term policy,
why iS the reserve zero at the end of the
year?
8. An ordinary life policy for $5000 is
written at age 25. Find the terminal reserve
at the end of 15 years.
9. Derive a formula for the terminal
reserve at the end of n yeors for an n-poyment
life policy written at oge c. (b) Derive o
formula for the reserve at the end of m years,
where m is greater than n.
10. Find the reserve at the end of 5
years for 10-year term policy for $10,000
written at age 35.
11. (a) Find the reserve at the end of 5
years for an ordinary life policy for $10,000
written ot oge 35. (b) Compare your answer
with that in problem 10 and give a brief
explanation of the differonce.
12. A man aged 25 pays the net Bingle
premium for a 10-year term ingrance for $1000.
What iB the policy reserve, 5 years later?
13. A man aged 30 pays the net single
premium for a whole life insurance for $1000.
Ten years later, what is the policy reserve?
i
After the completion of Exercise LXXII, the
studont mny proceed immediately to the Miscellaneoug
Problems at tho eng of tho chapter.
POLICY RESERVES 202

14. Derive a general formula, a.s in equation 53, for the reserve at
the end of m years for an n-year endowment policy for $1 written at
age c.
Norm 2. — The method for computing reserves, furnished by equation 52, is
called the prospective method because the future history of the policy is the
basis for the equation. Retrospective methods also are used.
N0TE 3. — Insurance companies are subject to legal regulation. It is usually
specified by state law that, at periodic times, an insurance company must show
net assets equa.l to the sum of the regervea for all of its outstanding policies.
The law specifies a standard mortality table and interest rate to be used. A
company is insolvent if it cannot show net assets equal to the necessary reserve.
It is likewise recognized by law that a company's ræerve belongs to its
policyholders as a whole. Hence, the reserve on apolicy is the basis for its cash
surrender value, the amount which the company must pay to L policyholder if
he decides to withdraw from the company and surrender his policy. The cash
surrender value equals the reserve, minus a surrender charge. The surrender
charge in most states is specified by law and may be considered as a charge by
the insurance company on uccount of the expense entailed in finding new
policyholder to •take the place Of the one withdrawing. This charge is leätimate
because the theoretical reaerve wag computed by the company on the
assumption that it had 80 many policyholders that the laws of averages, dealt
with in using the mortality table, would hold. Hence, the number of
policyholders must be maintained and any one withdrawing should pay for the
expense of obtaining a new policyholder in his place.
Non 4. — It should be recognized that the discussion in the preceding three
chapters iB merely an introduction to the subject of life annuities and of life
insurance. We have not considered joint life, or survivorship annuiües and
insurance. Moreover, the subject of reserves requirea a thorough treatment,
beyond what we have given, from both the theoretical and the computational
standpoint. The surplus of a company, its manner of declaring dividends to
policyholders, and many other practical questions connected with the accounting
and business methods of insurance companies have not even been mentioned.
The student who wishes to pursue the subject further ig referred to the Tect
POLICY RESERVES 203

Book of tho Inditute of Actuaries, and to the courses of study described by the
Educational Committeeg of the Actuarial Society of America and of the
American Listitute of Actuaries.

SUPPLEMENTARY EXERCISE LXXIL


Students working the problems below should have previously completed Supplementary
Section 74 of Chapter IX.
1. A policy written for a person aged 38 promises whole
life insurance for $10,000, and a life annuity of $1000 payable
annually, with the
204 MATHEMATICS OF INVESTMENT

firet payment at age 65. Premiums are payable annually for 20 years.
(o) Find the reserve at the end of 10 years. (b) Find the reserve at the
end of 20 years.
2. A policy written at age 27 promises $1000 terin
insurance for 20 years and a pure endowment of $5000 at the end of
20 yeurs. Premiums are payable annually for 10 years. Find the
reserve at the end of 6 years.
8. A policy written at age 15 promises 20-year endowment
inxurance for $1000, and the premiums are payable annually for 10
years. (a) Determine the reserve at tho end of 10 years. (b) Determine
the reserve at the end of 9 years.

4. A certain pure annuity policy written at age 40 promises life


annuity of $1000 with the first poyment at age 61. The premiums ore
payable annually for 21 years. Find the reserve (a) at the ond of 5
years ; (b) at the end of 20 years.
N0TE, —When a corporation or association promises pension to n person, its
act is equivalent to writing a pure unnuity policy for the person involved. Hence,
a pension asaociation should be conNiderod solvcnt only when its reservo fund
iB equal to the sum of the reservcs on cach of its pension contruct*. judged by
thig standard, there are an unfortunately Inrgc number of insolvent pension
associations in operation. Thcir insolvency does not boom no appurent until Lfter
they have been operating long enough so thut tho theoretical reserve (which they
do not possess) becomes necessary in ordor to meet liobilitics fulling due.
6. A group of workers of the same ago entered a ponsion
association which promised $500 annual payments for life, starting
with payments at age 61. At ago 55, 10,000 worker* rennin alivo.
They are required to pay $50 at the beginning of each year up to and
including thoir 00th birthdays. How much should the a8HociLtion
have on hand OH a reserve before the $50 payment* due at ago 55
havo been nude?
205 MATHEMATICS OF INVESTMENT

MISCELLANEOUS PROBLEMS ON INSURANCE i


1. Write a sample of each of thc following types of ingurnnco
policies, stating the age of tho policyholder, tho bencfitH ho will
rcceivo, and how he iB required to pay premiums: (a) 20-payment
life; (b) endowment•; (c) ordinary whole life; (d) 10-year term.
1
Insurance oompbniea mentioned in those problomg uro ugumed operate under zeeumptions (a), (b), and (a) of
Section 68.
POLICY RESERVES 206

2. (a) A man aged 47 desires to set aside a suffcient sum


which he can invest at 5%, effective, to pay him an annual income of
$1000 for 10 years, Bta,rting with a payment on his 61st birthday.
Find the amount Bet aside, assuming that he certainly live to age 70.
(b) At age 47 what would he have to pay to an insurance company
for a contract to. pay him $1000 at the end of each year for life, with
the first payment at age 61, with the understanding that the company
would compute the charge in accordance with the principleg of
scientific life insurance, at 3å%?
3. A woman offers $3000 to a benevolent organization on
condition that the organization pay her 5% interest thereon at the end
of each year for life. If the organization can purchase the required
annuity for her from an insurance company, which uses the rate
31%, will it pay to accept her offer if she is 55 years old ?
4. According to 8 will, trust fund of $200,000 will go to a charity at
the death of a girl who is now a,ged 19, and she is to receive the income at 4%
for the remainder of her life. On L basis, find the present value of (a) her
inheritance and of (b) the bequest to the charity.
6. A ma,n borrows $200,000, on which he pays 5% interest
annually. The principal is due at the end of 8 yean. To protect his
creditor he is compelled to take out an 8-year term insurance policy
for $200,000. Assume that the man will certainly live to the end of 8
years, and find the present value at 6%, effective, of all of his
payments on account of the debt, assuming that he pays merely the
net premiums for his insurance as computed by a company which
uses the rate 3B. His age is 40 years.
6. A man aged 35 pays the net single premium on a whole life
insurance for $1000. What is the policy reserve 10 years later?
7. A man aged 30 took out a 10-payment life policy. At the end of
10 years he desires to convert it into a 20-yea,r endowment insurance
POLICY RESERVES 207

as of that date. How much paid up endowment insurance will he


obtain if the company permits OIL of his reserve to be used for that
purpose? Notice that his reserve is the net single premium for the
new insurance.
8. (a) Find the net amnunl premium at age 43 for on ordinary life
policy for $2000. (b) Suppose that the man is alive at the end of 25 years. Find
the reserve on his policy and compare it with the sum he would have on hand if
he hod invested all of his annual premiums at 5%, effective.
9. A man aged 42 borrows $100,000 and agrees to pay
intærest annually. He agrees to provide for the payment of the
principal at his death, or at the end of 10 years if he lives, by taking
out a 10-year endowment policy for $100,000, with the creditor
beneficiary. The debtor
208 MATHEMATICS OF INVESTMENT
considers his future payments, assuming (1) that he will pay merely
the net premiums at for his policy, (2) that he will certainly
live to the end of 10 years, and (3) that he is able to invest hig money
at 7%, effective. He asks if it would pay to borrow $100,000
elsewhere at 6%, payable annually, with the agreement that the
principal may be repaid at the end of 10 years through the
accumulation of a sinking fund. @ Which method is beat? (b) In
terms of present values, how much could the debtor save by selecting
the best method ?
10. Compare the net single premiums for whole life insurance for
$1000 @ at ages 25 and 26; (b) at ages 75 and 76. (c) For which pair is the
change in cost greatest?
PART 111—ÅUXILIARY SUBJECTS
CHAPTER Xl
LOGARITHMS
77. Definition of logarithms. — Logarithms are exponents. The
logarithm of a number N with respect to a base a, where a is > 0, 1,
iG the exponent of the power to which a must bc raised to obtain N.
That is, by definition, if
(1)
then, the logarithm of N with respect to the base a iB c; or, in
abbreviated form,
logaN = x. (2)
Thus, since 49 = 72; then logr 49 = 2; since 1000 = 10 3, then log10
1000 = 3. Also, if log6 N = 2, then, from equation 1, N = 52; if
logo N = 4, then N = 64 —— 1296.
In the future, whenever we talk of the logarithm of a number we
shall be referring to a positive number N. This is necessary
because, in the definition of a logarithm, the base a is positive, and
hence only positive numbers N have logarithms as long as the c in
equation 1 is a real number.
EXERCISE LXÜV
1. Since 32 9, what is logs 9?
2. Since 54 = 625, what iB log, 625?
3. 2
Since 100 — 10 , what iB 10<10 100?

4. Since 23 8, what iB log28?


5. Since 100 = 1, what is logtol? Since 170 — 1, what iB
logi7 1? Since every number to the power zero iB 1, whot iB the
logarithm of 1 with respect to every baae; that is, since a o = 1,
what is loga 1 ?
e. What is Ipgo 36? 8. What is 16? 10. 25?
7. What iB 10,000? 9. What is 7? 11. What is loga a?
187
210 MATHEMATICS OF INVESTMENT

.zi2. If log, N = 2, find N.


19. Find log10 1.
13. If = 4, find N.
20. Find log10 4.
14. If
= 5, find N.
21. Find 10.
15. If = å, find N.
22. If = 31.62, find 10%31.62.
28. If 10•w9 = 5, find logro 5.
16. If log2N å, find N. 24. If = 303, find logt0303.
17. If 6.5, find N. 25. If - 3, and 3.

18. Find loge 81.


Express in another way the fuct that :
26. log1086.5 = 1.9370. 29. 4730 = 3.6749.
27. loglO 684 = 2.8351. 30. 343 = 73 .
28. 6.6 .8195. 81. Vä = 1.732.
32. If N - i, find log4N. (4) -1.
1 1 1
33. find log2 N. HINT.— 17
34. If N = .1, find loglO N. HINT.—.I =
35. Find log10 .001. Find .OOOOV: Find .0000001.

78. Properties of logarith.ms. — Logarithms have properties


which make them valuable tools for simplifying arithmetical
computation.
Property I. —iThe logarithm the product of two numbers M and N is equat to the sum of the
togarithms of M and N :
log, MN = logaM + N. (3)
Proof. — lat log' u = and log. N — V. Then, since logo M = c,
then M — ax, (Def. of logarithms) and since logaN = V, then N a v.
(Def. Of logarithms) Hence,(Law of exponents)
Since . MN—aø+v, then logo MN (Def. of logarithms)
211 MATHEMATICS OF INVESTMENT
Hence, logaMÄT = loga M + logo. N. (Subst. logo M ; V — loga N)
Property Il. — The logarithm of the quotient of two numbers, M
divided by N, is equal to the logarithm of the numerator minus the
logarithm of the denominator :
loga— — logaM — logaN. (4)
Proof. — I.Æt loga M and logo N V. • Then,
Bince loga M = z, then M — aø, (Def. of logarithmg) and since loga N =
V, then N av. (Def. of logarithms)
LOGARITHMS 212
Hence, — = az—n. (law of exponents)

Since — az—v, then logn (Def. of logarithms)

Hence, log = logo M — loga N. (Subst. z = logo M; y


logo N)
Property m. — The logarithm of a number N,
raised to a power .7c, is lc times the
logarithm of N:
logaNk = kiogaN. (5)
Proof. —Let loga N = z; then N = ax, by the
definiti01i of logarithms.
Hence,(Law of exponents)
Since - a, then - E. (Def. of
logarithms)
k
Hence, logo N = logo N. (Sub8t. c = loga N)
Non. — In the future we shall deal entirely with
logarithms to the baso 10. Hence; for convenience,
instead of writing log10 N we shall write merely log
N, • understanding that the bage always iB 10.
Logarithms fo the base 10 Bre called Common
Logarithms; the name Briggs' logarithms iB also used,
in honor of an Englishman named Henry Briggs (1556—
1630), who computed the first table of Common
Logarithms.
Example I. — Given that: log 2 = .3010, log 5 .6990,
log 17 =
1.2305, find the logarithms of each of the following
numbers: 34, 85, Vit, 25.
Solution. —log 34 log 2(17) — log 2 -Flog 17 — .3010
+ 1.2305= 1.5315. log85 = log 5(17) log5 +
log .6990 + 1.2305 2.2295. (Prop. 1) log* - log 17 —
log5 - 1.2306 — .6990 = .5315. crop. 11) log - log
17} log 17 = .61625. (Prop. Illwith/c = i)

18
LOGARITHMS 213
log25 = log 52 = 210g5 - 2(.6990) - 1.3980. (Prop.
111 with k 2)
EXERCISE LXXV
In the problems below find the logarithms of the given numbers,
given
that : log 2
= .3010 log
7 .8451 log 17
- 1.2805
1. 6 2. 9 8.
log 3 = .4771 log Il
= 1.0414 log 23
1.8617
46 4. 51 10.
16. 50
22. 100
log 6990 log 18
= 1.1139 log 29
1.4624
6. 6.
11. 49 12. 16
1.7. 55 18.
154
23, 1000 10,000
214 MATHEMATICS OF INVESTMENT

79. Common logarithms. — If one number N = is larger than


another number M = then c must be larger than y. Since = log N and
y = log M, it follows that, if N is larger than M, then log N is larger
than log M. Thus, since 9 is larger thon 7, log 9 must be larger than
log 7.
The table below gives the logarithms of certain powers of 10.
SINCE : TKEN :

10000 = 104 log 10000 log


= 103 1000 = 3 log
102 100 log 10 log I
= 101 log .01 - —2
. = = IO¯i log .001
.01 = = 104
.001 — 10—8

Consider the number 7, or any other number between I and 10.


25. 230 26. 2300 27. 23,000 28. 230,000 29. .1 80. .01
81. .001 32. .0001 88. .5 = % 34. .05 35. .005 36. .0005
Since 7 is greater than 1 and less than 10, log 7 is greater than log 1,
which is 0, and is less than log 10, which is 1. That is, since 7 is
between 1 and 10, log 7 lies between 0 and 1. Hence, log 7 = O + (a
proper fraction). Fpm a' table of logarithms, 88 described later, log
7 .84510, approximately, so that tho fraction mentioned above
is .84510. Similarly, since 750 is between 100 and 1000; log 750
lies between 2 and 3; therefore, log 750 = 2 -k (a proper
fraction) ; since 6473 is between 1000 and 10,000, log 5473 = 3 +
(a proper fraction). In the saane manncr, since .15 lies between .1
and 1, log .15 lies between — 1 and 0, and henco 1log .15 = — 1 +
(a proper• fraction). In general, the logarithm of every positive
number can be ecpressed as an integer, either positive or negative,
plus a positive proper fraction.

1 Any number between — 1 gnd O oen be expressed ae — 1 + (a proper fraction). Thu, .57 I
+ .43; — .88 — 1 + .12, etc.
215 MATHEMATICS OF INVESTMENT
The integral part of a logarithm iB called its characterisåc. When
a number N iB greater thon 1, the characteristic of log N is positive;
when N is less than 1, the characteristic of log N is negative.
LOGARITHMS 216

The fractional part of a logarithm iB called its mantissa.


Thus, given that log 700 — 2.84510, the characteristic of log 700 is 2, and
the mantissa is .84510 ; given that log .27 = — 1 + .43136, the of log .27 is — I
and the mantissa is .43136.
80. Properties of the mantissa and the characterisfic. —Given
that log 3.8137 .58134, then, by use of Properties I and Il of Section
78, and from the logarithms of powers of 10 given in Section 792
we prove the following results :
log 3813.7 = log 1000(3.8137) = log 1000+10g 3.8137=3+.58134 —3.58134.
log 381.37 log 100(3.8137) -log 100
log 38.137 = log 10(3.8137) — log 10 *log
3.8137-1+.58134=1.58134. log 3.8137 = 0.58134.
3.8137
log .38137 = log 3.8137— log 10 — .58134— — 1+.58134.
10
3.8137 log .038137 = log= log 3.8137-log 100
= .58134—2
100
3.8137 log .0038137 = log = log 3.8137— log
1000 - .58134_ 3 _ 3+.58134. 1000
Nom. — The characteristics of the logarithms above could have been
obtained in Section 79. Thus, since 3813.7 lies between 1000 and 10,000, log
3813.7 lies between 3 and 4; log 3813.7 = 3 -f- (a proper fraction).
Therefore, the characteristic of log 3813.7 is 3, aa found above.

From inspection above, we see that .58134 iB the mantissa of all


of the logarithms. This result, which obviously would hold for any
succession of digits as well aa it does for the digits 3, 8, I, 3, 7, may
be summarized as follows :
LOGARITHMS 217

Rule 1. — The mantissa of the logariå.m of a number N depends


only on the succession of digits in N. If two numbers have the same
succession of digits, that is, if they differ only in the position of the
decimal point, their logarithms have the same mantissa.
The logarithms above' also illustrate facts about the
characteriBtic.
Rule 2. — The characteristic of the logarithm of a number greater than 1 is positive and is 1

less than the number of digits in the number to the left of the decimal point.
N0TE. —Thus, in accordance with Rule 2, 3 is the characteristic of log 3818.7; 2 is the characteristic of log 381.37,
etc. Rule 2 is jugtified in general
218 MATHEMATICS OF INVESTMENT
by recognizing that, if a number N has (k -k I) digits to the left of the decimal point,
then N iB between 10 k and 10+1 ; hence log N is between Jc and (Ic + 1) and log N =
+ (a proper fraction). That is, lc is the characteristic of log N.
Rule 3. —If a number N is less than 1, the characteristic of log N is a
negative integer; if the first significant figure of N appears in kth decimal
place, then the of log N is — k.
Thug, the first significant figure of .38137 iB 3 and appears in the first decimal
place, and, in accordance with Rule 3, the characteristic of log .38137 iB — I. The
first significant figure of .038137 appears in the 2d decimal place, while the
characteristic of log .038137 is — 2, etc.
NOTE. — It may appur strange to the student that we write, for example,
log .0038137 — — 3 + .58134, instead of performing the subtraction. For every
number N which is less than I, log N is a negative number; thus, log .0038137 - _ —
3 + .58134 = — 2.41866. Written in this way, the mantissa .58134 and the
characteristic — 3 are lost sight of. We write the logarithm in the form — 3 + .58134
to keep the characteristic and the mantissa in a prominent position.

Since the mantissa depends merely on the succession of digits in the


number, it is customary -to speak of a mantissa as corresponding to B
given succession of digits without thinking of any decimal point being
associated with the digits. Thus, above, we would say that the
mantissa for the digits 38137 is .58134.
Example I. —Given that the mantissa for the digits 5843 is .76664,
find log 5843; log 584.3; log log 5.843; log .0005843.
Solu&n. — The characteristic of log 5843 is 8 ; hence, log 5843 = 3.76664.
Similarly, log 584.3=2.76664; log log 5.843 - 0.76664;
log .0005843 = — 4 + .76664.

EnRCISE LXXVI
1. Given that .75101 is the mantissa for the digits 56365, find log
6636.5; log 56365; log 563.65; log log .0056365; log .56365.

2. Given that .93046 is the mantissa for 85204, find log 85.204 ; , log
log 8.5204; log•85204; log .085204; log .0000085204,

3. Given that .39863 is the mantissa for 2504, find log 2504 ; • log
2.504 ; log 25,040; log .2504; log .00000000002504.
4. Given that log 273 . 7 = 2.43727, find log 2.737; log 27.37; log
27,370; log .02737; log .002737. Make use of Rule 1.
LOGARITHMS 219
5. Given that log 68,025 = 4.83267, find log 68.025; log 6.8025;
log .68025; log 6802.5; log .00068025.
6. What is the mantissa of log I ; of log 10; of log 10,000; of log . I ; of
log .00001 ?

81. Tables of mantissas. — The mantissa for a given succession of


digits can be computed by the methods of advanced mathematics. The
computed mantissas are then gathered in tables of Logarithms which,
more correctly, should be called tables of mantissas. Except in special
cases, mantissas are infinite decimal fractions. Thus the mantissa for
10705 .is .02958667163045713486 to 20 decimal places. In a 5-place
table of logarithms, this mantissa would be recorded co rrrect to 5 decimal
places, giving .02959. In an 8place table, it would be recorded
as .02958667, correct to 8 decimal places.
NOTE. — Table I in this book is a 5-place table of logarithms. A decimal point is
understood in front of each tabulated mantissa. To find the mantissa for N — 3553, for
example, go to the sixth page of Table I. Find the digits 355 in column headed N ; the
mantissa for 3553 iB entered in the corræponding row under the column headed 3. The
entry iB 060," but the first two digits OPt%e mantissa are understood to be 55 " the
game as for the first entry in the row. Thus, the mantissa for 8553 iB .55060. From
Table I the student should now verify thBt :
FOR DIGIN Bazow MANTISSA 1B

3630 .55991
3947 .59627
4589 .66172
9831 .96993
9332 .96997
93331 .97002
Example 1. — Find log 38570; log .008432.
Solution. — By inspection, the characteristic of log 88570 iB 4; the mantica ag
found in Table I is .58625. Hence, log 88570 = 4.58625, The characterigtic of
log .008432 is — 3; log ,008432 = — 3 + ,92593.
1
In Table I, for 9333, we and the entry "*002." The LBteri8k on the • '002" means
that tho first two digits gro to be changed from 96, as at the beginning of the t row, to
97.
In order to obtain greater convenience in computation, it is customary
to write negative characteristics in a different manner than heretofore.
Thus, in log .008432 = — 3 + .92593, change the — 3 to (7 — 10). Then
220 MATHEMATICS OF INVESTMENT
log .008432 = — 3 + .92593 = 7 — 10 + .92593 = 7.92593 — 10.
Recognize clearly that log
.008432 _ 3 + .92593 - — — 2.07407. We verify that 7.92593 — 10 =
— 2.07407. The two ways introduced for writing log .008432 are merely
two different ways of writing the negative number — 2.07407, which is
the actual logarithm involved. Similarly, log = 1 .92593 - 9.92593
— 10; log .000'000'000'008432 12 + .92593 = 8 — 20 + .92593 =
8.92593 — 20, etc.
NOTE. — The change from the new form to thc old or vice versa is oaay. Thus,
given that log .05383 = 8.73102 — 10, we see that the churacteristic iB (8 — 10) or
— 2; given that log .005849 = — 3 + .76708, then log .006849
= 7.76708 — 10.
EXERCISE LXXVII
1. What are the characteristics ofthefollowing logarithms:
9.8542—10 ; 7.7325 — 10; 6.5839 — 10; 4.3786 — 10?
2. Write the following logarithms in the other form: —
3 .5678; — 5 + 7654; — 7+ .8724; — 1 + .9675.
8. Write the following logarithms as pure negative numbers: — 3 +
.5674; — 1 + .7235; 9.7536 — 10; 7.2539 — 10.
4. By use of Table I verify the logarithms given below :
Loa N LOG N

3.54593 35.88 1.55486


3515.
8.26293 — 1.170 0.00819
.01832
10 .0008141 0.91008 —
889,900
5.94934 27,770 10
.6761
9.83001 — .00004788 4.44358
621.
10 5.08015 —
8
2.79365 10
Nom. — When the characteristic of log N is 0, log N iB equal to its muntissa.
Thus, log 1.578 = 0.19811. Henco, B tablo of muntixsns ig a table of the actual
logarithms Of all numbers between I end 10.
82. Logarithms of numbers with ave significant figures. — If n
number N hag five significant digits, log N cannot be road directly from
the table•. We must use the process of interpolation as described in the
following examples.
Example 1. — Find log 25.637.
LOGARITHMS 221
Solution. — The characteristic is 1. To find the mantissa, recognize that 25.687
is between 25.630 and 25.640; the mnmtissas for 2563 and for 2564 were read
from Table I and the logarithms of 25.630 and 25.640 are given in the table below.
Since 25.637 iB .7 of the way from 25.630 toward 25.640, we
asume 1 that log 25.637 is .7 of the way from 1.40875 toward
1.40892. The total way, or difference, iå .40892 — .40876
= .00017; .7 of the way is .7(.00017) = .000119. We reduce
25.680 1.40875 this to .00012, the nearest number of five decimul places.
25.637 Hence, log 25.637 = 1.40875 + .00012 = 1.40887.
25.640 1.40892
Norm. —At first, the student should do all interpolation in
detail as in Example 1 above. Afterward, he should Lim to
gain speed by doing the arithmetic mentally. The small tables in the column in Table
I hooded PP,
n.n abbreviation for proportional parts, are given to reduce the arithmetical work.
Example 2. — Find log .0017797.
Solution. —The characteristic is — 3 or (7 — 10). The digits 17797 form L
number between 17790 and 17800. The tabular difference between the
corresponding -mantissas is (.25042 — .25018) = .00024, or 24 units in the 5th
decimaJ place. Since 17797 iB .7 of the way from 17790 to
NOMDE MANTISS
A 17800, we wish .7(24). By multiplication, .7(24) = 16.8. This
R
should be found wUhout multiplication from the small table
17790 .25018 headed 24 under tho column PP. From this table we read .1(24)
17797 - 2.4, .2(24) = 4.8, etc., .7(24)
17800 25042 10.8. Hence, the mantissa for 17797 is .25018 -k .17 = 25035, and
log .0017797 7.25035 — 10.
Nom — The following situation is sometimes met in interpolating. Suppose
that .5(15) — 7.5 is the part of the tabular difference which-we must odd. Wo may,
with equal justification, call 7.5 cither 7 or 8. AB a definite rule in this book,
whonever such un ambiguity is met, we agree to choose tho even number. Henco,
we choose 8 above. Similarly, in using .7(16), or 10.5, we should call it 10, because
we have a choice between 10 md 11.
t
Thig assumption iB justified bythe first paragraph of Sectlon 70. Since 25.037 is
botwoon 25.030 and 25.040, log 25.637%UBt be between log 25.630 nnd log
25.040. In interpolating in Example 1, we 'merely go one step farther thtvn thig
admitted fLCt when we assume that tho change in the logarithm is proportional to
the change in the number. This assumption, although not exactly true, iB
gumciently accurate for prueticul purpoaes.
222 MATHEMATICS OF INVESTMENT
EXERCISE LXXVIII 1.
Verify the following logarithms :
log 256.32 2.40878 log 8956.1 = 3.95211
log 13.798 — 1.13982 log 931.42 = 2.96915
log .073663 = 8.86666 — 10 log 33.581 1.52609
log .59834 9.77695 — 10 log .00047178 = 6.67374 — 10
log 1.1676 = 0.06725 log 676.93 = 2.83054
a. Find the logarithms of the following numbers :
18.156 .31463 .061931 151.11
5321.7 83196 48.568 6319.1
67.689 113.42 384.22 9.3393
.031562 .92156 .62793 .000031579 .009567
5.6319 1.1678 83.462

83. To find the number when the logarithm.is given.


I—Find N if log N = 7.67062 - 10.
Solution. — Since the characteristic is (7 — 10) = 3, the first Bißiificont figure
of N will appear in the 3d decimal place; N .00 To find the digita of N, we must
obtain the number whose mantissa is *67062. We search for thig mantissa, or
those nearest to it, in Table I ; we find .67062 aa the mantissa of 4684. Hence, N
= .004684.
Ecampte 2. — Find N if log N = 5.41152.
Solution. —We wish the 5-figure number whose muntisga ig .41162. On
inspecting Table I we find the tabular mantisgas .41145 and .41162 between
which .41152 lies. The total way, or tabular difference, between .41146
and .41162 is .00017, or 17 units in the 5th decimal place.
Nuunmn The partial difference .41152 — .41145 — .00007, or 7 in
the 5th decimal place. Honce, .41152 is + of the way
25790 .41145 from .41145 to .41162. We then assume that tho number z,
whoso mantissa ig .41152, is of the way from 26790 to
.41152
25800 25800. The totul way, or differencp, iB 10 units in tho 5th
.41162
place; +(10) 4.1; the nelt•egt I-mit is 4. Hence, .41162 is the
mantissa of 25790 4 26794. Binoe tho Characteristic of log N is 5, N = 257,940.
LOGARITHMS 223
r
No m. — The arithmetic in Example 2 above iB simplified by uso of the table
headed 17 under the column of proportional purtg. In Example 2 wo desire '1
0), which we can easily obtain if we know + correct to tho nearest tenth. the
table headed 17, we read .4(17) - 6.8, or .4; .5(17) -8.5, .5. Since 7 iE
between 6.8 and 8.5, is between .4 and .5, but is cest to .4. Thug, "10) 4, to the
nearest unit. With practice, this result should be obtained almost
instantaneously. Thug, Zook under the tabtg headed 17 for the number neare8t
to 7; we find 6.8; at the left it 18 8hown that thia i.8 .4 of 17; hence +(10) = 4.

EXERCISE LXXIX

1. Find the numbers corresponding to the given logarithms and


verify the answers given :
log N = 3.21388; N = 1636.4. log N - 3.75097; N = 5636. log N = 8.40415 — 10;
N .02536. log N = 0.46839; N 2.9403. log N - 2.15931; N 144.31. log N 3.33590; N
2167.2. log N - 9.52163 — 10; N - .33238. log N - 8.65267 — 10; N .044944. log N
0.89651 ; N = 7.8797. log N 0.35217; N - 2.2499.
2. Find the numbers corresponding to the following logarithms :
log N = 5.21631 log N - 8.19008 log N - 9.64397 — 10

log N - 1.39875 log N = 7.55642 — 10 log N 2.57938 — 10


log N — 8.95321 — 10 log N = 0.89577 log N = 1.77871

log N = 4.32111 — 10 log N = 1.21352 log N - 7.77853


log N = 2.15678 log N - 8.45673 — 10 log N - 3.15698
84. Computation of products and of quotients.
EcampZe 1. — Compute P = 787.97 X .0033238 X 14.431.
Solution. — From Property I of Section 78, log P is the gum of the logarithms of the factors. From Table I,

log 787.97 - 2.89651 log .0033238 7.52163


— 10 log 14.431 — 1.15931

From Table 1, P - 37.796.


4.8031 x 269.97 x 1.6364
224 MATHEMATICS OF INVESTMENT
Ecampze 2. — Compute Q =
78797 x 253.6
Sohdion. — From Property Il of Section 78, log Q equals the logarithm of the
numerator minus the logarithm of the denominator. Both numerator and
denominator are produota whose logarithms are determined by Property I.
log 4.8031 - 0.68152, log 78797 — 4.89651 log 269.97 —

2.43131 log 253.6 = 2.40415 log


1.6364 0.21389 (add)
225 MATHEMATICS OF

INITEsTMDNT
We recognize that the result on subtrocting will be negativo. To obtain log Q in
standard form, we add and nl eo subtract 10 from the log numerator. log Numer.
8.32672 - 13.32672 — 10
7.30066

NOTE. — Before computing any expreasion by logarithms, u computing form


should be made. Thus, the first operation in solving Example 2 above wag to
write down the following form :
log 4.8031 — log 78797 log 269.97 = log 253.0 =

log 1.6364
(add) log Numer. —
log Denom. =

A systematic form prevents errors and makes it easy to repeat the work if it iB
desired to check the computation.

EXERCISE LXXX Compute


by logarithms :
226 MATHEMATICS OF

1. 563.7 x 8.2156 x .00565. 2. 4.321 x 21.98 x .99315.


3. 675.31 4. 56.854
13.215 2356.7'

5. .008315 6. 783.12 x 11.325


.0003156 8932

7. 85 x 73 x 139.58. e. 9.825 x 531.75.


3215.7 x .4503 .8319 x .5685

.42173 x .21567. 5.3172 x .4256.

9. 10.
.3852 x .956 18.11 x 31.581

85. Computation of powers and of roots.


Example 1. — Find (.3156)4.

Solution. —From Property Ill Of Section 78 witli 4, log (.3156)' - 4 log .3156 -
4(9.49914 — 10) - 37.99050 -40 - 7.99656 — 10. From Table 1, (.3166)'
- .009921.

log "56.31
8
LOGARITHMS 227

3. —Find 1Y.iää3i; 1%.08351.


Solution. — Since = we obtain from Property III,
log = .08351 =If we divide 8.92174 thiB as it BtundB, 6 we obtain
1.48696 — a most inconvenient form. Hence, we add and, at the same time,
subtract 50 from log .08351 in order that, after the diviEion by 6, the Will be in
the stundaxd form for logarithms with negative characteristicg. Hence, log
#.08351 =8.92174 — 10 50 + 8.92174 — 10 — 50 58.92174 — 60
6 6 6 log - 9.82029 -
10; hence, from Tabiel, = .66113.
From Property III, log = log .08351 =8.92174 — 10 3 log _ 28.92174 — 30 =
9.64058 — 10; hence, #.08351 - .43710.
EDRCISE LXXXI
Compute by logarithms :
2
1. (175) . 2. (56.73)8. 8. (.013821)4.
4. 0531.2. 5. +'.079677. 6.
7. (353.3 x 1.6888) 2. 8. #199.62. 9. (1.05)7.
10. (1.06) 4. 12. (1.06)29.

18.15. (1.03)-o 1
85.75 (45.6)2 (1.03)”

16. | 56.35 x 4.3157

N
21.36 x
HINT. — For this problem, the computing form ig :
log 56.35 log 521.9
log 4,3157
(add)
(add) log Denom. =
(Búbtract) log fract.
Regult =
535 X 881,5 X (1.08) B 18. (189.5)',
g.
LOGARITHMS 228

475 x 938
19. 20.
(158.2) 2 x 257.8 21. .03156 x 75.81
22.

1893.2 x 35830 221.38 x (.3561) 2


28. (1.09 2'. 24, (1.035F
LOGARITHMS 229
INvEsTMßNT
86. Problems in compatation. — It is very important to realize
that the properties I, Il, nnd Ill of logorithms may bo used in
eomputing products, quotients, and powers, but that they may not
used in computing diferences or sums except in the auxiliary
nanner illustrated below.
VSjö+ (.567) (35.3).
Ecample 1. — Compute Q = 532 —
Solution. — By logarithms, ve perform ouch of the three computations
below. log •./äöö = log896 - 2.95231 = 1.47616; A/Söö 29.934.
2
log.567 975358 — 10

log prod. — L 1.0135 — 10; = 20.015.


log = 210g 15.31 Q.18498) = 2.36996; hence, (15.31) 2 - 234.40.

Therefore, using the Lbove, Q = 29.934 632 — + 234.40 20.015


297.oo•49.949

log 49.940 = 1.69853 = 11.69853 — 10


10 297.60 = 2.47363 - 2.47363
(gubtract) log 9.22490 — 10; henec,Q .16784.
Non. — A computation done with a 5-ploce table of logarithms will givo
reults which are accurate 4 significant figures, but the 5th figure alwayB will be
open to question. mantissa in tho tuble, and ouch of those we by interpolation
iB subject to an error of part of I unit in thc 5th Emal place, even though all
of our interpolation is dono correctly. During a long computation, these
a.ocunulated errors in the logarithms, together with ile allowable error due to
011* final interpolation, cause on unavoidable error i' the 5th significant figure
our final result. Therefore, if a number With xore than 5 significant figures,
such 2,986,533, is mat, in L computation with L5-place table, we should this
number to tho neorost number luving 5 significant figures, b&ore finding itg
logarithm. To retLi11 inoro than 'significant digits is fictiäouxaocurocy, since
our final rcmiltB will bo accurato only 4 digits. For the same reagon, in
230 MATHEMATICS OF
looking up the number corresponding t' a given logarithm, the inteQolation
should not bo carried beyond the nearest tm.it in the 5th significant, place.
Non. — Logarithmio comrmtation of producta, quotients, and powers must deal entirely
with positive nunbers, according to the statemøntg of Section 77. Eence, if negative numbes
aje involved, we first compute the expression by if all numbers vere positive, and
then by inspection determino tie proper sign to be aggigna to the result. Thug, to compute
75.3) X ( 8.392) X 32.15) we find 75.3 X 8.392 X 32.15 20316; then, we note that a negative
Bign mast be attached, giving — 20816 ag the result.
EXERCISE LXXXII Compute
by logarithms :
(35.6)2 + 89.53
1. 2.
- 2.513 5.923
3. (1.03)E — 1. 4.
.04

(1.07) 6 — 1 251 +
5. 6. 63.95 x 41.27
.07 787
7. 395 x 856. 3852
8.
5321
9. (log 856). log 3852
That is, compute 10. • That is, compute
log 5321
3.72599
3.58569
11. log 88 — 2 12. 3 log (1.04)
654 log 2
13. log 6.532 14. log 8.957
log 1.04 log 1.06

15. 16.
17. (1.05)}. 18.
19. 20. log .85 -l- 3
(.235)2
LOGARITHMS 231
Rule, — To solve a simple exponential equation, take the
logarithm of both sides of the equation and solve the resulting
equation.
Ecample 1. — Solve the equation 132=+2 = (356)5z.
Solution, — Toko tho logarithm of both Bides Of the equation, maki.ng use
of Property I. Then (2 + 2) log 13 log 356 + log 5, or
INVESTMENT
(2$ + = 2.55145 + $(.69897). (Table 1)
2.22788 c + 2.22788 = 2.55145 + .69897 c.
1.52891 = .32357 ;
.32357 - .21164. log .32357 9.50997 — 10
1.52891 log 1.5289 = 0.18438

(subtract) logc = 9.32659 — 10

The exponential equations met in applications to the


mathematicg of investment are of the form
(6)
where A and B are constants, and where v is a function of the
unImown quantity.
Bcampze 2. —Solve (1.07)" = 4.57.
Solution. — Taking the logarithm of both Bides, we obtain

2 n log 1.07 = log 4.57; n = log 4.57 .65992 _ .65992


2 log 1.07 2(.02938) .05876'
log .65G92 9.81949 — 10
log .05876 = 8.76908 — 10

(subtract) logn = 1.05041; n —11.231. EXERCISE

LXXXIII
232 MATHEMATICS OF
Solve the following equations :
1. (1.05)" = 6.325. 2. = 95.
s. 12+1 = 38. 4. (1.025) 1n = 8.8261.
5. 53z = 569. 6• 2n — 31.
7. 27(2s). 8. = 282.
9. (1.035)ß — 1 2.75.
.035
HINT. — Clear the equation of fractions and reduce to the form of equation 6, obtaining
(1.035) n — 1.06875.

10. (1.045)-n = .753.


HINT. —The equaåon becomes — n log 1.045 = 9.87679 — 10 = — .12321.
11. (1.03)-n = .8821. 12. =
1638.

18. 52.67, u. 750 - 8500,


SUPPLEMENTARY MATER-UL
88. Logarithms to bases different from 10. — To avoid confusion
we shall explicitly denote the bases for all logarithms met in this
section. From Sectidn 77, = logaN satisfies the equation a z = N. By
solving this exponential equation, we can find c when N and a are
given. Thus, taking the logarithm to the base 10 of both sides of a z =
N, we obtain
c logioa = logzoN; = log10 N log10 a or =
loga N = logio N 1 log10N. (7)
logloa log10 a
Norm. — Equation 7 enables us to find the logarithm of any number with
respect to a given base a, provided that wc have a table of logarithms to the
buse 10. The quantity log10 a is called the modulus of the system of logarithms
to the bese 10 with respect to the system to the base a.

The natural system of logarithms is that system where the base is


the number e = 2.718281828. . The number e is very important
mothematical constant and logarithms to the base e are useful in
advanced mathematics, From an 8-place table, we find
LOGARITHMS 233
= 0.43429448; log .43429448 = 9.63778431 —10.
Emmple 1. — Find log, 35.
Sohdion. — Let c log. 35. Then, 35 ; taking the logarithm of both sides to thø
base 10, logia = log10 35 ;
_ 35 _ 1.54407 log 1.5441 — 10.18868 — 10 logloe ¯ 0.43429•
log .43429 — 9.68778 — 10
3.5555. (subtract) log — 0.55090

EÆRCISE LXXXIV
1. Find logo 75; loga 10; log0830; loga 657.
2. Find the natural logarithm of 4368.
3. Find 108853; logs 10; 100896; loglE33.
4. If a and b arc any two positive numbers, prove that 109 N logg N • 109 a.
HINT. —Let logo N and V log N. Then N a' = by. Take the logarithm with
respect to the b Qf both *ides of the equation a",
CHAPTER Xll
PROGRESSIONS
89. Arithmetical progressions. — A progression is a sequence of
numbers formed according to some law. An arithmetical
progression is a progression in which each term ig obtained from
the next preceding term by the addition of a fixed constant called
the common difference. Thus, 3, 6, 9, 12, , etc., is an arithmetical
progression in which the common difference is 3. Similariy, 3, 2,
9, , etc., is an arithmetical progression in which the common
difference iB
Let a represent the first term of an arithmetical progression, d the
common difference, and n the number of terms in the progression.
Then, in the progression,
a = 1st term, a + 3d = 4th term, a 4- d = 2d term, • etc. •
(8) a + 2d = 3d term, a + (n — I)d = nth term.
If we let represent the last, or the nth, term, we have proved that
(9)
If we start with the last term, the next to the last term is formed by
subtracting d, the second from the last by subtracting 2 d, etc. That
is, in going backward, we meet an arithmetical progression with the
common difference ( — d). Thus,
= last term, — 3 d 3d from last term,
— d = 1st from laet term, • • • etc, (10)

— 2 d = 2d from laat term, a = — (n — = (n


— l)st from last.
lat s represent the sum of the terms of the progression. Then, we obtain equation 11 below
by using the terms given in equations 8, and equation 12 by using equations 10.
PROGRESSIONS 235
+ (n — l)d].
(11)
— (n — I)d]. (12)
204
On adding equations 11 and 12, we obtain
. . . + (a + 1). (13)
There are n terms in equation 13, one corresponding to each term of
the progression. Hence, 2 s = + l), or
(14)

If any three of the quantities (a, d, n, Z, s) are given, the


equations9 and 14 enable ug to find the other two. We call (a, d, n, t,
s) the elements of the progression.
EcampZe 1. — In an arithmetical progression with the first term
3, the 6th term is 28. Find the common difference and the
intermediato

Solution. — We have a = 8, n = 6, and = 28. Hence, from equation 9, 28 —


3 +5 d; 5 d = 25; d 5. The terms of the progresaion are 3, 8, 13, 18, 23, 28.
EXERCISE LXXXV
1. Find the last term and the sum of the prograsion 3, 5, 7, 9, . . . to twelve
terms.
2. Find the sum of the progression 5, 4, 3, 2, . . . , to eighteen •terms.
8. Find the last term and the sum of the progression
1000(.05), 950(.05), 900(.05), . . . etc., to twenty terrng.
4. If 10 is the first term and 33 is the 20th term of an
arithmetical progression, find the common difference and the gum
of the progression.
5. If 15 is the 4th term and 32 iB the 10th term of an
arithmetical progression, find the intermediate terms.
90. Geometrical progressions. — A geometrical progression is a, progression in which each
term is formed by multiplying the preceding term by a fixed constant r. The number r is called
the common ratio of the progression because the ratio of any term to the preceding term is
equal to r. Thus, 4, 12, 36, 108, • • • etc., is a geometrical progression with the common ratio r
= 3, The sequence
(1.05), (1.05)2, (1.05)8, (1.05)', • • • etc., is a geometrical
progression with the ratio r (1.05).

206 MATHEMATICS OF INVESTMENT


IÆt a represent the first term, r the common
ratio, and n the number of terms in a
geometrical progression. Then,
a = 1st term, arb = 5th term,
ar = 2d term, etc., ar2 = 3d
term, arn-a = (n — l)st term,
ar8 = 4th term, arn-l = nth
term.

If we let represent the last, or nth, term,


we have proved that
(15)
Let s represent the sum of the terms of the
progression. Then s a + + + + +
arn¯i , (16) ar + ar2 + + . . + +
17) On subtracting equation 17 from
equation 16, all terms will cancel except a
from equation 16 and — arn from equation 17.
Thus, s(l— r) = a — arn.
Hence, a — arn a(l — rn) (18)

Since = arn—l , then rl = arn ; on


substituting this in the first fraction
of equation 18,
PROGRESSIONS 237
(19)
Bzample 1. —Find the sum of 1 + 4- + ... etc. .. . to six
terms.
Solution.— Use formula 18 with a = 1, r i, and n =
6.
8 = [I — (i)6) = = 364.
243
Eccmple 2. — Find an expression for the sum of

1 + (1.05) .6 + (1.05) + (1.05)1.5 + . etc. .


+
Solution. — The terms form a geometrical progregsion for which
a I, r = and — From formula 19,
(1.05) 24 — 1
(1.05) • — 15 (1.05) B
EXERCISE LXXXVI

1. Find the laet term ond the sum of 25,


5, 1, i, k, etc. to seven terms.
2. Find the last term and the sum of 2,
4, 8, . . . etc. to eighteen terms.
8. Find the ratio, the number of terms, and the sum for the
progresBion 3, 9, 27, , , , 729.
4. Find the sum of 2, I, i, • • • etc. to eight terms.
5. Find the sum of 1 . . .
Find expressions for the following sums :
6. (1.05) + (1.05) 2 + (1.05) 8 + • • • etc. • • • + (1.05B.
T. (1.04)2 + (1.04)' + (1.04) d + • • • etc. • + (1.04)".
8. (1.06) 45 + (1.06) -24 + (1.06) -23 + • etc. • + (1.06) -
1.
9. + (1.03) + 4
• • • etc. • • • + (1.03) .17

10. (1.02) + (1.02) 2 + (1.02)8 + • etc. • • • + (1.02) 60.


91. Infinite geome&ica.l progressions. — Consider the following
hypothetical example. A certain jar contains two quarts of water.
One quart is poured out ; then, of the remainder, or quart, iB
poured out; then, of the remainder, or quart, is poured out, etc.,
without ceasing. The amounts poured out are , , • • etc. • to
'infinitely many terms.
The sum of the amounts poured out up to and including the nth
pouring is 1
1 _ + • etc. •
2n—1

Since the aanount originally in the jar was 2 quarts, sn can never
exceed 2. Also, it iB clear intuitionally that, as n increases without
bound, sn must approach the value 2 because the amount of water
left in the jar approaches 0 as the process continues. We can prove
this fact mathematically ; from formula 19,

sn = (20)
1 continually decreases and
Ag n grows large without bound, 2n—l

approaches zero. Thus, from equation 20 we prove that, as n


increases without bound, sn approaches the limit 2, as was seen
intuitionally above. Hence, we may agree, by definition, to call this
value 2 the sum of the infinite geometrid progression, or to say 2 =
1 + + + • • etc. • • • to infihitely many terms.
This example shows that a sensible definition, in accordance with
our intuitions, may be given for the sum of an infinite geometrical
progression,
208 MATHEMATICS OF INVESTMENT

In general, consider any infinite


geometrical progression for which the ratio r
is numerically less than 1, that is, for
which r lies between — 1 and + 1. The terms
of the progression are a, ar, ar2, arg , • • •
etc. • • • to infinitely many terms.
Let sn represent the gum of the first n terms
of the progression :

The siatement as n approaches infinity will


be used as an abbreviation for the statement
as n increases without bound.
The sum S of an infinite geometrical
progression is defined as the limiting value,
if any exists, approached by sn as n
approaches infinity.
From formula 18,

(21)
As n approaches infinity, it is evident that
r" approaches zero because r iB numerically
less than 1. Hence, from equation 21
a
it is seen that, as n approaches in.finity,
sn approaches as a limiting value, because
the other term in equation 21 approaches
zero. Since, by definition, this limiting
value of sn is the value we assign to the sum
S = . • etc. • • to
infinitely many terms, we have proved that a
(22)
Ecampte I. — Find the sum of the progression
4 A
(1.04) + (1.04) + + • • • etc . • • • to infinitely
many terms.
Solution. — The ratio of the infinite geometrical
progresåion is r (1.04) , a = (1.04) -a. From formula
22, the sum is
(1.04) *

1 — (1.04) -2
Example 2. — Express the infinite repeating
decimal .08333 • • • as a fraction.
Solution. — We verify that .08333 • • • equals .08
plus
.003 + .0003 + .00003 + • • • etc. •
• • to infinitely many terme.
'1 For rigorous proof of this intuitional fact the
student ig referred to the theory Of l.imiu
presented, for example, in booke on the Calculus.
PROGRESSIONS 241

These terms form an infinite geometrical progression with a = .003, andr = . I.

Their sum is 003 - .003 Hence,

.08333 • = .08 + .003 8 3 25 1


.9 100 900 äöö
Non. — By the method of Ekumple 2 above, any infinite repeating decimal
can be shown to represent fraction whose numerator and denominator are
integers.

Find the sums of the following progressions :

1. 2 + 1 + + . • • to infinitely many terms.


PROGRESSIONS 242

2. 5 + 1 + + + • . • to infinitely many terms.


1
+ . . • to infftely many terms.
(1.05) (1.05) 2 (1.05)3
4. + (1.04) -2 + (1.04) -3 + • • • to infinitely many terms..
6. -k + + • • • to infinitely many terms.
6. -k + • • • to infinitely many
terms.
Express the following infinite decimals as fractions :
7. .333333 • • .. 8. .66666 • 10.
11. .5636363 12. .24222222 •
APPENDIX

Note 1
Proof of Rule 1, Section 15, Part I. — Consider the equation

The solution of this equation for n is the time required for money to
double itself if r is the rate per period. On taking the logarithms,
with respect to the base e = 2.71828 . of both sides of the equation,
we obtain log 2 log (1 + r)' where log means loge." From textbooks
on the Calculus, we
find that
and from Hence

On' neglecting obtain


as an approximate

Note 2
Proof of Rule 1, Section 17, Part I. — Consider three obligations
whose maturity values' are Sl, Sa, and Sa, which aro due,

211
APPENDIX

respectively, at the endg of m, na, and na yours. We shall prove Rule


1 for this special cage. Thc reasoning and the details of proof are the
same for the cage of any numbcr of obligations. Let i be the
effective rote of interest, and let n be the equated time. BY
definition, n satisfies the equation
246 MATHEMATICS OF INVESTMENT
By use of the binomial theorem, we obtain the following infinite
series :

Since i is small, we make only a slight error if we use only the first
two terms of each infinite series as an approximate value for the
corresponding power of (1 -l- i). On using these approximations in
equation 1, we obtain
(1 + S2 + Sa) — + S2(1 — + —
On expanding both sides and on solving for n, we obtain
n ISI + n as 2 + ngSa

which establishes Rule 1 for the present case.

Note 3
Solution of equations by interpolation. — The method of
interpolution which the student hag used in connection with logarithm
and compound interest tables con be uRed in solving equations whose
solution would otherwise present very great diffculties.
Dæample 1. — Solve for n in the oquution
5 (1.06) 7.5 + (1)
Solution. — On rewriting the equation und on u8ing the abbreviation F(n) for
the left member, we obtain

F(n) - 7.5 - .45n - O.


APPENDIX
We desire a value n Ic such that Fm 0. If we find a value n nt euch that Fox) iB
negative, and another vduo n na euch that iB positive, then it will follow that
there iB u value n h, between nt and n., such that F(E) 0. Thut is, there muat be a
solution n k between und na. From a rough inspection of Table V we guess that
the solution ia greater than n 19. With the aid of Table V we compute F(O) for n
19, 20, and 21. F(19) 922; F'(N) — — .464; F(2i) + .048. Hence, there ig a
solution n of the equation between n 20 n 21, We and lc by interpola-
213
tion in the table below where we use the fact that Fm 0. The total difference in the
tabular entries ig .048 — .464) = .512. The purtial difference iB O — .464)
= .464. Hence, since O is = .91 of the way from —.464 to
+.048, we assume that the solution is .91 of the way from 20
to 21, or that = 20 + .91 — 20.91. Of coui•se, thig is only
approximate solution of the equation, but such a one iB
20 n — .46
extremely useful in practical applications. An inspection of k 21 4
the equation shows thut there cannot be any other solution
because increases much more rapidly than .45 n, and hence .048
F(n) will be positive for al-I values of n greater than 21.
Example 2. — A man invests $6000 in the stock of a corporation.
He receives $400 dividend at the end of each year for 10 years. At the
end of that time he sells his holdings for $5000. Considering the
whole 10-year period, at what effective rate may the man consider his
investment to have been mode ?
Solution. — Let r bc the effective rote. With the end of 10 years a comparison
date, we write the following equation of value :
6000(1 + r) 10 5000 +
- 6000(1 + r) 10 — 5000 — (1) We shall solve equation

I by interpolation.
If the $1000108B in capital hud been uniformly distributed over the 10 years,
the 1088 per year would have been $100. Hence, under this false (but
approximately true) condition, the net tunnual income would have been BOO.
Tho average invested capital would have been å(6000 + 5000) = $5500. Hence,
siuce .055, we guess 1 .056 am approximation to the solution of the equation.
When r = .055, F(.056) + 98.73. Since this ig positive, the solution must be legs
than .055. We find F(.05) —257.80. Hence, the solution r = of equation I, for
which O iB between r = .056 and r = .05, We interpolate in the table below. 98.7
— 257.8) 356.5; o — 257.8) 257.8; .056 — .05 - .005. Eence,

.05 —
257.8
.05
5 98.
7
248 MATHEMATICS OF INVESTMENT
257.8
- .05 + .0036, or approxi356.5 mutely, = .054. Tho
solution could be obtained accurately to hundredths (or to thousandths, or less) of
1%, if desired, by the method uscd in Example 2, Section 32, Part I.
Note 4
Abridged multiplication. — Consider forming the product
(11.132157) X (893.214). We decide in advance that wo desire the
result accurately
i
Notioo the similarity botwoen this reasoning and that employed in Soction 55 of Port 1.
249 MATHEMATICS OF INVESTMENT

to the nearest digit in the second decimal place. The ordinary


multiplication would proceed as at the left below, while the
abridged method proceeds as at the right.
ORDINARY METE0D ABRIDGED METH0D

11.132157 xxxxx Multiply


893.214 11.132157 by
893.214 800
8905.7256 90
44528628 1001.8935
11132157 3
33.3963
22264314 2.2264
.2
33396471 .01
.1113
100189413 .004
89057256 9943.3975 Add
Result = 9943.40
9943.398481598
In multiplying by the abridged method we proceed as follows :
Since we desire the result to be accurate in the 2d decimal place, we carry
two extra places, or four decimal places, for safety. To multiply by 893.214
we multiply in succeuion by 800, 90, 3, .2, .01, and .004 and then add the
results (this is the same as is done in the ordinary method of multiplication,
except that the multiplications are performed in the reverse order). We first
multiply by 800, that is, we multiply by 8 and then move the decimul point.
All digits of 11.132157 are used in this operation in order to obtain four
significant decimal places in the result. To obtain four decimal places when
multiplying by 90 we need one less digit of 11.132157 ; we put X over the cc
7" to indicate that we multiply 11.13216 at this time. Wo put X over thé 5"
and then multiply 11.1321 by 3; we put X ovcr the last 1 and then multiply
11.132 by .2; etc. The advantages of this method are obvious, Less labor is
involved, the decimal point is accurately located, Dund fewer mistakæ will
occur in the final Addition.

Note 5
250 MATHEMATICS OF INVESTMENT

Accuracy of the interpolation method in solving for the time in the


compound interest equation. — Consider the equation
(1)
where and r are known. To determine the value of n by interpolation,
we first find from our interest table (Table V if > 1, Table VI if A <
1) two integers m and na, na — nx = 1, such that the corre-
APPENDIX

215
na
sponding values = (I -k r) % ond (1 + r) include A between them.
That is, A < .42. Then, ag obtained by interpolation, the solution of
equation 1 is

The exact solution of equation 1 is obtained by taking the logarithm


of both sides; log A = n log (1 + r), where log means loge.
log A • (2)
log (1 + T)
From equation 2 we obtain
APPENDIX

Hence since dn/dÅ


n is an increasing
A. Moreover,
negative, the
function of A,
horizontal, will
downward,as
torted for illustration.
seen graphically
ference between
equation 2,
by the line EF
This is
error
where H is the
the tangent
tersects the
Sinco CD
= 1,
(3)
APPENDIX

(act.8)
FIG. 6
Since

rAi,
Hence, on inserting the infinite series for log (1 -i- r), as obtained from any textbook on the
Calculus,

DEI
254 MATHEMATICS OF INVESTMENT

r
DTI < 1

If r $ .10, as iB the case in the tables of


this book, DH 1

which is appro}dmately r, if computed to only


two decimal places. Hence, if we are
computing results to only two decimal places,
a solution of equation 1, obtained by
interpolation, is in error by at most

Note 6
Accuracy of the interpolation method in
solving for the time in the annuity
equations. — Consider the equation

(8ä1 at r) = S, (1) where S


and r are known. From equation 1, on
inserting the explicit algebraic expression
for (sm at r), we obtain

If we solve equation 1 for n by interpolation


in Table VIl, our solution iE the same as.we
should obtain in solving the equivalent
equation
(2) (where
A = Sr + 1) for n, by interpolation in Table
V. For, the solution of equation 1 by
interpolation would be

while that for equation 2 would be


255 MATHEMATICS OF INVESTMENT


1'
— Slr
— 1
which is the same 88 the result for equation 1. Hence, it
follows from Note 5 of the Appendix that the error in the
solution of equation 1 obtained by interpolation in Table VIl
is at most of the interest rate r. Similarly, it follows that,
if we should solve for n in the equation (Fl at r) = A, by
interpolating in Table VIll, the error of the result would be
at most r.
256 INDEX
Numbers rcfer to

INDEX
Numbere refer to pagea
Abridge of a
d debt,
multipli 78; of
cation, the
213 premi
Accrued um
dividen on
d, on a bond,
bond, 118
126 Amortiz
Accumula ation
tion equatio
factor, 16 n, 89
Accumu Amortiz
lation of ation
discount plan,
, on B 78;
bond, bonde
119 d debt
Accumu retire
lation d by
problem a, 81;
, 15 comp
Americ arison
an of
Exper sinkin
ience g
Table fund
of metho
Mort d
Llity, with
148 the,
Amortiz 88;
ation, final
paym certai
ent n, 39
under Annual
the, premiu
84 m ; see
net
Amortiz
annual
ution premiu
sched m
ule, Annual
for a rent
debt, of on
78 ; annuit
for y, 39;
the deter
premi minot
um ion of
on a the,
bond, 68
'118 Annuiti
Amount es
, at certai
comp n, 39;
ound formu
intere lua
st, 16; for,
at 43,
simpl 47,
e 50;
intere interp
st, 1; olatio
in a n
sinkin metho
g ds
fund, for,
87; of 70,
an 72;
annuit summ
y
258 INDEX
Numbers rcfer to

ary of ion of
formu thc
lae annua
for, l rent
50 of an,
Annuity 68 ;
; gee intero
annuit st rate
y borne
certai by
n, and on,
life 71 ;
annuit term
y of an,
Annuity 70;
bond, due,
130 56;
Annuity paym
corta. ent
in, intorv
39; al of
Amo an, 39
unt of ;
on, prese
39; nt
annun value
,l rent of an,
of on, 39;
•39; term
conti of an,
nuous 39
, 62; Annuity
defcrr duo,
ed, cort,oin,
59; 56; life,
deter 162
minat
Annuity a
policy, t
177 e
Approxi ,
mate
bond 8
yield, 4
126
Arithme A
tical v
progress e
ion, 204 r
A.æot, a
scrap g
value i
of an, n
96; g
weari
ng a
value n
of an,
96; a
condi c
tion c
per o
cent u
of an, n
98 t
,
A
v 3
e 4
r Bank
a discount, 9
g Base of
e system of
logarithms,
187
d
260 INDEX
Numbers rcfer to

Benefici 126;
ary, 165 book
Benefit value
of a of a,
policy, 117,
165 122;
Binomi chang
al es in
theorem book
, 63 value
Bond, of a,
113 ; 117;
accu divide
mulat nd on
ion of a,
disco 113;
unt face
on a, value
119; of a,
accru 113;
ed flat
divide price
nd on of B,
a, 125;
126; purch
amort ase
izatio price
n of of c,
premi 114,
um 121;
on a, quote
118; d
appro price
ximat of o,
e 125;
yield redem
on g, ption
price
of a, logarith
113; ms, 189
the Building
yield and loan
of a, asociation
126; s, 92;
yield dues of,
on a, 92;
by interest
interp rateg
olatio earned
n, by, 93;
128, loane
131 made by,
Bond 94; profits
table, in, 92;
116
Book shares in,
value, 92; ti.me
of a for stock
debt, to maturo
86; of in, 92
a Capitali
depre zed
ciable cost,
asset, 105
97 Coah
Book valuesurrend
of a
bond, on er an
value,
interest date,
183
117; between
interest dates,
Churaot
122 origtic
of a
Briggs' logarith
system m, 190
of
262 INDEX
Numbers rcfer to

Common st, 14;


21
7
logarithms, accu
189
mulal
Commutat
ion
ion
symbols, probl
157 em
Compar under,
ison 15;
date, amou
för nt
comp under,
aring 14;
value contin
s, 26; uous
for conve
writin rsion
g an under,
equati 35 ;
on of conve
value, raion
27 perio
Compos d
ite life, under,
99 14;
disco
Compo
unt
und
probl
amount,
em
14; for a
under,
fraction
16 ;
al
effect
period,
ive
20
rote
Compo under,
und 18;
intere nomi
nal Deferre
rate d
under, annuity,
18 certain,
Conclud 59
ing
paym Deferre
ent, d life
under annuity,
amort 159
izatio Depreci
n ation,
96;
84 const
Conditi ant
on per perce
cent, 98 ntage
Conting metho
ent d for,
annuity, 109 ;
39 valuat
ion of
Contingen
t payment, minin
present value g
of a, 152 prope
Continuou rty
s annuity, under
62 ,
Continu 101 ;
ously sinkin
convert g
ed fund
interest, plan
35 for,
Conversio 96;
n period, Btrmi
14 ght
line
264 INDEX
Numbers rcfer to

metho buildi
d for, ng
98 and
Discount, loan
bankin aæoci
g use btion,
of, 9; 92
proble
m of,
under
compo Effectiv
und e rate of
interest interest,
, 15; 18
bond
purcha
Endow
aed at ment
b, 119; insura
rate of, nce,
7; 170;
simple, also
7 see
Discount pure
factor, 16 endo
Discounting of wmen
t
notes, Equated
under date, 33
simple Equated
discoun
t, 10;
time,
under 33;
compo equatio
und n for,
interest, 33,
24 212
Dividend, pages
on a,
Equatio
bond, 113
Dues of n of
a value,
27; n, 41,
comp 43,
ariso 46;
n date use
for, of,
27 under
Exact perpet
Simplo uities,
interest, 108
2 Glover's
Expone tables,
ntial 157
equatiGraphic
on, al
201 ; repres
use entati
of, in on, of
annuit accu
y mulat
probl ion
ems, under
75 intere
st, 23;
Force of of
deferr
ed
Geomet
annuit
rical
y, 59;
progr
of on
ession
unnui
s,
ty
205;
due,
use
57 ;
of,
of
under
depre
annuit
ciatio
ies
n, 97
certai
266 INDEX
Numbers rcfer to

Gross premi
premium, ums
165 for,
165;
Infinite net
geom single
etricu premi
l um
progr for,
ession 166;
s, 207 policy
; use of,
of, 165;
under term,
perpet 168;
uities, whole
108 life,
Insuran 160
ce, Insuran
165 ; ce
endo policy
wmen , 105;
t, 170 benefi
; ciary
gross of an,
premi 165;
um benefi
for, ts of
165, an,
175; 165;
net policy
unuua date
l of an,
premi 165;
um endo
for, wmen
171 ; t,
net 172;
n- of,
paym 18;
cnt exact,
life, 2;
172; force
n- of,
ycur 36;
term, graph
172; ical
ordin repres
ary entati
life, on of
172; accu
reserv mulat
e on ion
an, under
178; , 23;
whole in
life, udvan
160 co, 9;
Insurance nomi
premium; nal
sea rate
premium of,
Interest, 18;
1 ; ordin
comp ary,
ound, 2;
14; rate
convo of, 1;
rted simpl
contin e, I
uousl Interest
y, period,
35 ; 14
effect
Interpol
ive
ation,
rate
annuit
268 INDEX
Numbers rcfer to

y bond
probl by,
ems 128,
solve 131
d by,
70,
72,
213;
book
valuo
of
bond
betwe
en
intere
st
dates
by,
123;
comp
ound
intere
st
probl
ems
solve
d by,
29;
usc
of, in
logari
thmic
comp
utatio
n,
194;
yield
of
INDEX
Numbers refer to

Investment yield of a bond, 113, 126; npayment life policy, 173; for an
by approximate method, 126; by nyear term policy, 173 ; for ordinary
interpolation, 128, 131 life policy, 172
I.ægal reserve insurance company, 165 Net premiums, 165
Level premium, 178 Net single premium, 106, 175; for
Life annuity, 155; deferred, 159; due, endowment insurance, 170; for
162; present value of a, 155, 159, irregular benefits, 176; for term
insuranco, 168; for whole life
163; temporaa•y, 159; whole, 155 insuruncc, 166
Life insurance; Bee insurance
Nominal rato of intcregt, 18
Louding, 175
Notes; geo dücounting of nota
Ingarithm-s, 187; base of a system of,
Offce premium, 165
187; Briggs' system of, 189; change
of baae of, 203 ; . characteristics Old Iino ingurance company, 165
of, 190; common, 189; montigeas 219
of, 190; Napierian, 203 ; natural,
203; properties of, 188; use of tables pagøe
of, 194, 196 Ordinary life policy, 172
Mathematical expectation, 152; net Ordinary Aimple interest, 2
single premium as a, 175; of a
contingcnt payment, 152 Par value of a bond, 113
Manti— 190 Payment of a debt, amortization
Mining property, valuation of, 101 process for, 78 ; amortization
Modulus, of a system of logarithms, 203 schedule for, 78; building and loan
Mortality, American Experience Table næociation arrangement for, 94 ;
of, 148 comparison of amortization and
sinking fund methods for, 88;
n-poyment endowment policy, 172
sinking fund method for, 85
n-payment life policy, 172 n-year
term policy, 172 Pensiong, present value of, 184
Perpetuities, 103; infinite geometrical
Natural premium, 169 progressions applied to, 108 ;
Net annual premium, 171 ; for an present values of, 103, 104; use of,
endowment policy, 173 ; for an in cupitnli.zation problems, 105
irregular policy, 176; for an Policy ; see in8urance policy
INDEX
Numbers refer to

Policy date, 165


Policyholder, 165
Policy year, 165
Premium, annual, 171 ; gross, 165;
level, 178; natural, 169; net, 165; net
annual, 171; net single, 166
Premium on a bond, formula for the,
115; amortization schedule for the,
118
Preaent value, of a • contingent
payment, 152; of a pure endowment,
153; of a life annuity, 155, 159, 162;
of an annuity certain, 39; of life
insurance, 165; under compound
interest, 15; under simple discount, 7
; under simple interest, 2
Principal, I ; amortization of, 78
Probabilitiea of life, 150
Probability; 147
20R; arithmetical, 204;
geometrical, 205 ; infinite
geometrical, 207
Proportional parts, 195
Prospective method of valuation, 188
Pure endowment, 153; preaent value of
a, 153, 157

Rate of digcountJ 7
Rate Of interest, I ; borne by an
annuity, 71; effective, 18; nominal,
18; paid by a borrower of a build-
271 INDEX
Numbers rcfer to

ing and loan asociation, 95 ;


yielded by a bond, 126, 128, 131
Redemption fund, for a mine, 101
Reserve, terminal, 178; table
showing growth of B, 179;
formula for the, 180

Scrap value, 96
Serial bonds, 130
Simple discount, 6
Simple interest, I; exact, 2;
ordinary, 2; dx per cent rule for,
3
Sinking fund, 85; amount in a, 87;
table showing growth of a, 86
Sinking fund equation, 89
Sinking fund plan, for depreciation,
96; for retiring a debt, 85
pages
Temporary life unnuity, 159
Tenn, of an annuity certain, 39; d+
termination of the, 70
Tcrminal reserve, 178; for an ordinary
life policy, 181; prospective method
for obtaining the, 183
Term insurance, 168
Time, to double money, 30, 211
Valuation, of L mine, 101; of an
insurance. reserve, 180
Value, cash gurrcndcr, 183; of an
obligation, 24
Values, compurison of, 26
Wearing value, 96
Whole life annuity, 155
Whole life insurance policy, 166
272 INDEX
Numbers rcfer to

Six per cent rule, 3Yield of a bond, by approximate


Straight line method for depreciation,method, 126; by intcrpolation,
98128, 131
TABLE 1
COMMON LOGARITHMS OF NUMBERS
TO FIVE DECIMAL PLACES
Pages 2 to 19

TABLE 11
COMMON LOGARITHMS OF NUMBERS
FROM 1.00000 to 1.10000
TO SEVEN DECIMAL PLACES
Pages 20 to 21
ο 1 2 8 8
087 130 ι 73 217 260 303 340 380
οοο 043 776 *242
432 475 518 δβ1 647 *ΙθΩ
0 οι 604 *072 689 732 620 42
903 945 988 *115 817
4 326 368 410 452 494 536 *157 *036 4.4 1.3 4.2
703 578 4-40 2 8.8
745 787 828 870 912 053 *802
119 202 243 284 325 005 857 8. 8.4
572
531 612 653 735 776 816 898 3 13.2
938 979 *100 *141 *181 *222 *202 12. 12,6
842 883 423 463 503 543 583 023 4 17.0
703 17.2 10.8 5 22. 21,5 21.0
743 782 822 862 002 941 981 454 ο 20. 26.8 20.2 7 30.8 80.
139 179 218 258 297 336 370 415 403 ι 20.4
571 650 β8θ 727 883 8 35.2 84.4 3.3.6
30.0 38.7 87,8
532 961 423 461 *115 760 805 652
610
922 *164 *102 844
308 729 385 805 843 881 576 408
108 183 221 258 638 *200
690 767 558 505 θ5β 004 781 M 30
070 488 145 633 018
200 333 371 *032 4.1 4.0 8.0
446 856 521 930 987 *004 707 744 518
208 835 372 070 2 8.2 8.0 7.8
819 225 882
803 664 700 737 *078 *115 3 12.8 12.0 11.7
188 591 262 408 446 482 *151
4 10.4 ιο.ο 15.6
555 628 773 800 20.5 19.5
218 24.8 23.4
990 *135 *171 *207 *248
θ54 386 θ 7
600 65 28.7 28,0 27.8
*307 8 32.8 32.0 31.2
814 743
422 458 493 620 505 9 30.0 36.1
072 850 056
279
707 447 778 814 849 884 020 340
991 877 ηβ1 795 *182 *167 *202 *237 *272
342 726 140 482 517 552 587 621 38 87 88
412 *025
691 072 760 483 830 864 899 034 908 361 2 3.0
037 415 106 823 175 200 243 278 312 3
380 755 449 517 651 685 σιο 058 4 14.
18.0
721 093 789 857 890 024 058 002 6 4
21.0
20.2
059 428 120 198 227 201 204 327 7 28.8
82.4
394 461 528 561 504 628 601 8
72712 760 798 826 860 898 928 050 352 ο
090 123 ι 56 18θ 222
7
483 254 287 678
385 418 460 516 648 581 613 *001
710 748 775 808 840 872 937 322
038 066 008 180 162 194 226 258 640
854 886 418 450 481 513 545 577 2
672 704 736 767 790 830 862 803 925 *270
7.2
988 883 864 895 *114 *145 *176 *208 *230 682 10.8
801 644 675 706 426 467 489 620 551 801
618 737 768 709 820 ΐ2 33
*168 *ΙΩΒ a 3.3
473 603 0.0
770 806 13.2
10.6
*077
28:1
320 376 702 30:a 29:7
025 678
922 953 983 927 *137 289
229 259 290 227 861 381 412 442 260
534 594 524 685
715 746 561
82 81 30
886 866 897 820 967 087
"έ.ΓΤ.Γ¯Ο'.-ο-
187 167 197 114 256 286 316 846 6.4 6.2 0.0
435 466 406 554 584 618 643 0.0 0.3
732 761 791 850 870 038 12.8
Ιθ.ο
026 058 085 148 173 202 231
19,2 22.
319 348 377 435 464 493 622 26. 24:8 24:0
609 688 667 725 754 782 811 28.8 27.9 27.0
8 5 7 8
1
2
ο 1 2 8 ο 7 8 9
609 638 687 696 725 754 782 811 840 860
898
18 29
4
926 984 *018 2.0
*070 6.8
752 213 241 270 298 327 355
384 *127
03 498 554 583 667 412 .7
611 639 14.6
3 780 808 837 949 724
812 061 080 145 893 921 229 077
117 173 257 23.2
201
590 340 368 424
451 479
507
535
285 20.1
866 818 645 673 700 783 28
14 893 921 048 070 728 756 *058 811 838
Ι 2.7
0 167 194 222 240 270 303 330 358 385
2 6.6
412 430 408 493 520 548 576 002 020 656
8 8.4
883 710 763 808 4 11.2
952 978 737 817 871
*032 790
844 *105 025 14.0
219 245 272 *086
299 825 405 481 192 16.8
362 378 458 7 10.0
484 511 537 584
748 775 801 827 854 017 643 θ32 058 722 8 22.4
011 037 003 08θ 115 880 οοο Ιθ4 220 9 25.2
141 24
272 208 324 360 107 453 479
376
401 427
531 557 583 008 634 712 737 763
780 814 840 866 89 ι 017 043 068 904 *019 27 28
045 070 096 121 147 172 108 223 240 274 2.7 2.0
300 350 370 5.4
325 452 477 502 528
603 020 401 426 10.8
563 578 704 720 754 770 5 6
805 855 880 005 030 13.5 Ι8Ϊ2
830 080 ο 16.2 20.8
05 105 130 155 18Ω 204 254 270
080 220 18.
5 353 378 403 428 452 502 627 23.4
829 477 21.
304 601 625 074 748 773
570 724 24.3
551 846 871 895 920 944 993 *018
822 980
797 001 115 189 164 188 237 261
212
04 310 334 358 382 406 431 479 603
466
2 551 575 624 048 072 720 744
285 702 810 840 864 888 912 035 950 083
627 031 055 070 102 120 Ι50 174 108 221
708 200 203 310 340 364 387 411 435 458
00 520 558 570 οοο 647 670 004
7 741 7M 788 811 834 023 881 028
*138
245 075 θ08 *021 *008 858 *114 *101
482 370
207 231 277 300 323 340 303
717
απο 402 485 508 531 554 577 830 823
051 '715 701 784 807 852
18 808 602 044 738 12 *035
4 021 067
410
646
876
103 120 140 171 104
330 375 308 421 217 240 202 285 307 o 2Ι.σ 20.7
353 443 533
5δβ 601 623 ΜΟ 460 488 511
578 758
780 803 826 847 718 735
870 802 014
003 020 048 070 092 937 081
116 137 181 203
226 248 270 202 314 336 858 380 40,3
425
447 401 513 635 557 579 601 β23
007 088 710 732 754 776 708 820 842
885 007 929 051 073 *081
103 125 146 168 Ιθδ 211 233 255 278 208
8
o 1 2 '1 8

8
8 9
ο 1 2 8 7
208
0 103 125 148 168 190 211 283 256 278

820 341 408 471 492 3


449 514
δ3δ 557 678 884 621 428 085 707 5υ
750 728 78
771 792 814 835 643 878 899 020
042
866 υ
968 984 *006 *027 *091 *112 *133 *154
1 175 197 218 239 260 302 323 346 306
887 408 471 281 513 534 555
429 450 402 578
081 21
597 618 889 723 744 765 786
800 702 2.1
806 827 848 869 931 052 073 004
2 015 085 077 Ωθ8 118 130 100 181 201
Ι 0.5
222 243 263 284 305 325 340 308 387 408 12.0
14.7
428 449 4θθ 490 510 631 552 572 503 013 10.8
634 654 675 716 736 750 777 707 818 18.0
838 858 879 940 08Θ *001 *021
102 122 148 183
041 062 082 304 183 203 224
825 366
244 284 284 δοσ δ2β 345 386 406 425
486 • σου
646 708 726 748 768 580 808 820 2 2.0
886 885 104 925 945 163 *025 4.0
064 084 786 203
301 124 143 361 085 400 223 6.
242 282 282 321 341 183 420 0
380 8.
0
10.0
12.0
14.0
10.0
18.
0
439 34
636 47θ 498 518 637 667 ο7
577
830 674 718 733 753
772
702 8
908 028 947 080 81 Ι
21 850 869 88θ ΩΟ7
122 141 180 *006
8 044 083 102 ισο
411 238 257 278 2θδ 815 384 363 372 ΙΩ9
507 620 564 302 19
603 449 468 488 545 583
755 1.0
793 622 ΜΙ 879 098 717 736 774 2
3.8
813 870 889 908 Ω4β
832 851 927 Ωθ5
5.7
17 *003 *040 *078 *007 *186
7.0
3 192 211 229 248 207 280 305 324 342 0.0
ΙΙ.4
361 630 13.3
549 611 717 17:
738 430 698 903 ι
380 418 474 403
922 624 455 080 884 *088 3
668 588 791 810 842 •
7 10 880 273
754 773 .977
7 829 *070 467
291 940 ι 82 181 218 2.36 254 030
310 144 346 385 401 420
47δ 438 822.
828 389
658 493 580 548 685 603 621
840 876 511 712 781 767 785 803
1
8 02 85β 876 894 912 931 949 007
1 039 057 075 093 112 130 • 148 160
8
1.8
202 220 238 3.0
382 399 417 6.
578 4
739 775 7.
757 2
917 984' 952
9 094 129
ΙΙΙ
270 287 805
445 463 480
820 637 855 258
794 811 829 435 4
814- 274 202 .3Ι0 348 384
453 471050 489 828 7
792 525 643
832 828 βφ8 8
• 703 721
970 810 5 848 863 881 8ΩΩ
148 987 182 109 217 *058 262
322 358 375 803 235 428
498 840 633 550 688 410 802
β72 707 724 742 759 777
846 883 881 898 015 933 950
17
10.
2
ΙΙ.ο
13.
0
Ιδ.
3
6
78ο

7 8
ο 1 2 δ.
4
о 1 2 8 5 6 8
9 794 811 829 840 863 881 898 015 933 950

967 985 *002 *019 *10B *128 18


0 140 157 175 192 *054 *071 8088
200 226 243 261 278 206 1.
312 329 346 364 381 308 415 449 406 8
483 600 618 535 662 586 320 037 57
654 071 688 705 722 739 756 773 790 807 7.2
824 841 858 875 892 909 926 943 060 970 0.0
ш.8
993 *027 044 *0B1 *078 Ю9Б *111 *128 *146 12.
102 179 190 212 229 240 263 280 296 313 3
330 347 заз 980 307 430 447 404 481 14.
497 514 631 647 504 581 507 014 031 047 4
10.
064 607 714 731 747 704 780 707 814 2
830 681 833 890 003 970 з45о7
880 913 929 946
906 847 020 *0В2 $127 *144
*078 8
2 18 103 210 226 из 259 275 292 808
0 177 357 374 300 406 423 439 466 472
325 341 621 537 653 570 686 602 010 035
488 604 17
В07
684 700 710 732 749 706 781 707
661 830 840 802 878 804 011 027 043 950 2 з 4 6 1.7
813 991 *008 *024 *040 *060 *072 *088 *104 *120 о 7 зл
975 5.
162 100 185 201 217 233 249 265 281 8 1
3 1ЗВ 9 0.
207 313 320 345 301 377 зод 400 426 8
467 473 480 505 521 537 663 500 584 000 8.
032 648 064 080 690 712 727 743 769 15
775
701 807 823 838 864 870 10.
0
0.33 880 002 017
949 005 081 090 *012 028 21.
4 00 *044 *050 *075
107 122 138 154. 170 185 11.
6
1 201 217 232
08.
248 204 270 295 311 326 342 ЗВ 373 380
420 430 451 407 483 408 514 620 645 213.
404 0
570 602 607 023 038 ово 686 700 4.
600 15.8
710 731 747 702 778 703 809 824 840 855 36.
871 017 032 048 4
880 002 поз 979 094 *010
6 025 071 080 8.
040 050 102 117
133 148 шз 2
179 225 240 0 з
104 200 265 271
280 301 317
9.
332 347 302 378 30.3 408 423 454 4B0
439 3
484 поо 515 530 545 501 676 воо 021
037 062 вст 082 097 712, „728 743 768 773
15
788 803 818 8.34*00 804 804,
030 054 опо 084 0 *016 870 *046 024
0 090 106 120 135 160 106 180 106 210 225
240 266 270 285 зоо 315 3,30 345 350 374
10.
410
6
508
12.
710
0
389 804 8о
404 *012 434 470 404 067 523 13.
638
702 150 583 013
77
• 0,12 806 072 5
087 740 7Ll 700 820
731 053
R35 850 306 гюп
023 038 $100 907
082 007 *0БП *070
7 120 173 188 202 217 232 240 201
270 200 310 334 303 .378 302 407
422 430 406 480 404 600 624 638 563
667 582 оп 025 640 009 683 698
712 727 750 770 784 790 813 828' 342

8 7 8
о 1 2 в
6
N o 2 3 8 7 8 9
1
47 712 727 741 756 770 784 7θθ 813 828
800 857 871 885 οοο 914 929 058 972
οι 48 001 015 029 044 058 078 087 101 110 130
02 144 173 187 202 210 230 244 259 273
03 302 316 344 359 378 387 401 416
287 47C
04 444 458 487 601 515 530 644 558
572 586 β2θ
601 648 657 071 686 700
714 758
728 742 807 770 786 700 813 827 841
07 856 809
08 883 *038 011 026 040 968 982
*010 *052 *094
186 178 *080 *108 *122
150 234
810 192 200 220 248 202
11 276 200 804 318 332 346 360 374 388 402 14
12 415 429 443 457 471 486 688 513 527 641
18 554 δθ8 682 610 624 β7θ 1.
14 603 707 721 734 748 762 776 700 803 817 3 4
15 881 845 859 872 886 900 914 927 θ41 4 2.
082 *010 *024 *037 8 5
188 202 220
17 50 120 133 147 215
101 174 325 338 365 7.0
18 243 270 284 352 8.
303 297 311 401 501
4
0.8
11.
2
19 379 406 420 433 447 474 488 637 8 13
515 542 583 610 623 4 1.
21 651 664 705 718 732 745 769 772 3
22 786 799 678 820 840 853 800 880 803 007 2.6 2
23 020 984 813 οαι 974 987
*028 *041 8.
24 51 055 068 947 005 108 121 135 148 0
175
25 188 202 081 228 242 255 268 282 6.2
206 308 0.6
26 322 385 216 362 375 388 402 415 7.8
27 465 468 848 405 508 621 534 548 501 574 0.
28 587 601 481 027 640 064 607 680 βθ3 7Ρ6 1
29 720 733 746 759 772 786 700 812 826 888 10.
851 878 891 004 017 930 943 057 970 4
31 *000 *035*048 *075 *088 11.
983 140 108 170 206 218 231 7
32 127
33
52 114
257 270 153 207 310 192 336 849 802
284 323
34 401 427 440 470 492
375 888 δΩ5
35 580 414 45,3
808 621
504 517 543 οοο 582
724
737 760
634 647 711 863
87 780 673 815 827
870
763 770 017 802 043 050 082 866
88 892 038 110 004 *007
046 071 084
89 N 020 058 007 237 122 136
173 212
148 181 180 224 260 203
41 276 288 301 314 326 339 352 364 377
42 403 415 428 441 463 503 479 491 504 517
43 542 555 567 580 618 081 643
529 719
ββ8 681 732 744 767 700
820 832 845 857 870 882
45 782 794 807 970 806
908 920 933 945 958 983 *008 *020
47 095 108 120 133
54 038 045 058 070 083
220 145
48 168 170 183 1θδ 208 233 245 268 270
49 346 357 370 382
283 295 807 820 332
4β9 304
850 407 419 482 444 450 481 494 518

N ο 1 2 8 4 7 8

7
N ο 1 2 3 5 7 8 9
850 419 432 444 4θθ 481 404 518
51 548 δβ8580 503 13
555 θθ1 605 617 β30 642
52 531 667 704 716 765
070 814 728 741 753 1.
58 790 827 830 888
777 802 851 864 876 3
M 913 937 049 962
οοο 025 οοο 072 084 974 2.
55 0.35 108 121 133 0
55 023 047 182
157 104 206 218 230 242 255 3.
145
57 270 303 315 328 340 376 0
267. 201 352 364
58 400 425 437 440 461 407 5.
388 413 47,3 485
60 522 546 658 570 582 018 2
534
380 630 842 (354 678 σοι 703 715 727 730 7.
1.12 8
751 783 775 787 811 859
63 8,17 0.
871 883 895 907 700 031 823 835 070
M 1
*003 '015 *027 *050 948 *080 *008
05 *074 10.
110 122 134 140 170 206 217
158 4
220 241 253 205 280 182 104 324 336
277 11.
07 348 360 372 384 407 301 812 443 455
08 7
407 419
478 490 502 514 526 549 561 573 2
870 585 597 608 020 032 644 538 ββ7 679 βθ1 84
703 714 726 738 750 701 773 785 707 808 a7
71
72 820 832 844 855 867 870 801 002 014 026
73 937
57 064 3 1
74 072 906
2
75
171 049 078 080 101 113 124 136 148 Ιδθ
76 287 183 104 200 217 220 241 252 264 276
403 910 334 345 857 308 380 392
77 290 322
510 420 438 440 401 473 484 400 507
78 415
79 034 530542 553 505 578 688 οοο 011 828 0.0
74 Ω 680
380 657 οοο 602 703 715 720 738 10.
81 701 772 784 705 807 818 830 841 852
078 8
82 875 887 898 910 921 033 944 056 907
58 002 8
83 οοο *001 *013 *035 *047 *058 *070 *081
200
84 320 104 115 127 138 203 161 172 184 105
85 433 218 220 240 252 377 • 286 297 309
80 331 343 3M 305 400 274 399 410 422
87 444 450 407 478 602 888 512 524 585
88 771 657 500 580 78ο
704 715 501 025 740 047
80 005 070 081 βΩ2 614 737 760
810 827 720 801
390 50 100 782 704 805 872
028 030 850
073
02 218 838 001
017 ΟΜ
93 320 804 050 3
430 *017 *028
M 118 120 140 151 162 173 105 207
550
οοο
07 770 2
08 43
870
400 088
80 007 9
200

220 240 262 273 205


340 351
251 373 384 284 406 10
302 483 404 395 417
460 461 617 528 Ι.ο
472
501 572 594 006 027 638 649
082 583 704 715 βΙβ 737 759
671
701 603 813 824 726 840 748 888
780
802 885 857
800 001 023 934 977 9.
900 *010 012 *032 *043 046 *086 0
5
108 110 141 162 103 173 184 105
217 228 239 240 260 271 282 203 804 8ο
N ο 2 8 8
1

9
ο 1 2 3 5 7 8
N
228 230 240 200 271 282
οι 80 206 217 412
02 314 825 330 347 370 300 521)
358 380 ΙΕ27 l.i¯
08 423 433 444 456 487 408 401
477
04 631 641 652 503 674 505
684 017 735
638 070 081 703 713
002 72-1
746 750 707 778 788 810 821
07 831
853 8133 874 886 805 θ00 017
08
050 070 981 001 *002 *013 *023 *034
61 οσο 077 087 008 100 110 130 140 151
11 172 183 10,1 204 216 225 230 2-17 257 3
7,1
12 27θ 280 300 310 321 331 342 362 303 471)
13 384 305 405 410 420 437 58,1
532 5-12 448 458 ΙΙΙ")
14 490 500 511 521 503
605 010 027 037 57,1
16 ΙΙΙ}Ο
1571)
700 711 721 731 742 752 7113
815 820 847 857 773
17 805 830 808 78-1
020 030 051 Ωβ2 878 21 Ι
18 900 072
024 315
62 034 045 055 070 Ι}07
120 128 140 150 ι 70 100 418
118 138 180 2111
21 232 252 203 278 204 304
221 02-1
22 335 340 358 380 377 387 307 408
23 825 720
428 450 480
24 δ3Ι 542 502 572 583 400
552 003 511
644 005 075 086 δΙ)3
26. 700 013
27 737 747 757 707 778 788 ΗΩ8 710 194
839 840 870 880 800 708 010
28 850 8ΙΗ
20. 941 051 061 072 082 002 *012
*022
68 043 053 063 073 083 004 104 114
31 144 Ιδδ 105 175 185 105 205 215
32 246 256 260 276 286 200 300 317 327 7311
38 847 357 367 377 387 307 407 417 428
448 458 488 478 488 408 608 1128 θ:ΙΩ
34 518 *038
548 558 δβ8 570 580 000 υιο
35
36 640 050 070 680 137
7110 700 82θ
37 740 750 770 780 800 810 237
860 871)
88 θ4θ 850 880 008 d\i.i
• οοο 079
39 050 088 11.28
64 048 008 078 008 108 118 227 15,31
147 068 088 11>7 21>7
317
167 177 729
41 246 157 187 306 310
260 276
42 260 280
345 414
43 365 306 875 385 305 404 513
444
44 454 473 483 403 503 01 Ι 710 ΙΙΗ
10
46 542 562 562 572 582 080 000 700 81 u 21δ
46 640 βοο 670 ΟΒΟ
787 707 807
788 748 758 885 805 408
47 768 777 004 ΙΩΒ
836 848 860 875 082 002 206
49 943 θδ3 003
938 072 οοο
070 080
040 060 170 292 898
031 167 070 186
128 137 147 264 107 273 28J 889
225 234 244 360 203 309 870
321 381 841 860

N ο 1 2 5 8

11
N
ο 1 2 8 8 9
321 331 341 350 360 369 379 389 398 408

437 447 456 475 485 1


418 427 652 502 504
533 643 571 581 0
514 523 β2θ 048 βδ8 βΙΙ7
51 677 886 1.
52 725 734 744 753 763 772 782 792 0
715
53 801 820 880 830 849 858 808 877 887 2.
811
896 916 925 085 944 954 063 973 982 0
54
092 *020 *030 *030 *077 3.
55
087 οοο ιου 115 124 134 143 ι 53 102 172 0
181 101 200 210 219 220 238 247 257 200 4.
57
58 270 285 205 304 314 323 332 342 351 301 0
0.
870 0
οι 464 380 380 308 408 417 427 7.
558 474 488 402 521 530 446 455 0
62 502 511
507 577 586 005 014 024 630 640 8.
ω 052 500
745 661 671 080 689 οοο 708 717 0
727 730 9.
889 755 764 773 783 792 801 811 829
848 857 807 870 885 804 904 820 0
07 932 018 022 2
88 07 041 050 900 969 978 987 907 34
025 034 043 052 002 071 080 089 108
117 127 136 145 164 173 182 101 201 ο78
470 164 θ
71 210 210 228 237 247 256 205 274 284 293
72
78 802 311 330 339 348 357 376
394 403 321 422 431 440 867 408385
74 488 495 413 514 523 532 541 459 500477
75 604 638 550 560
578 587 605 814 624 βδΙ
76 042 742
οοο 070 500 097 700 715 7u
77 70Ι 770 088 788 797 800 815 733 834 752
78 852 779 825 025 843
79 801 879 888 897 900
943 870 Ω1β *015
952 970 979 088 907 *024
480 68 034 070
043 052 061 070 088 097 115
81 124
82 133 142 151 160 ΙΟθ 178 187 205
215
83 305 242 287
224 200 2β0 298
84 395 233 332 251 350 278 377 386
314 323 449 808
85 422 841 407 476
485 404 413 538 458
80 511 431 520
574 404 028
87 602 001 520 019 647 640 055
073 502 010 708 717 637 735 744
88 753
.89 842 702 681 780 Ι]θ9
707 800 720 824 833
490 931 851 771 809 780 880 984 815 913 022
12
09 800 058 878 004
020 040 040 075 9θ8 *011
108 028 037 065 ΟΜ 073 082 090 099 5.
10 4
7 101 170
17 0.
285 249 207
07 0 3
338 355
08 873 268 .
401 425 448 2
340 531
δΟΟ 648 513
601 434 018 2
086 52
8
135 688 705 3
723 117 152 2 188 0.
223 144 775 703
810 205 120 241 009 270 8
811 232 802 880
807 204 214 329 364
302 320 09
381 487 417 7 452
400 300 408 504 539
574 78
567 478 683 502 4 027
002
644 740 071 079 87 714
732 053 831 758 767 1 801
810 827 3 845 854 95 888
900 014 028 932 940 8 075 2
N o 8 4 9
1

13
N ο 1 ο 7 8 9
2
500 023 040 049 058 075
01 897 914 032
02 984 *001 *018 *027 *030
θθ2 *010 114 122
03 70 070 079 088 101 200 131
1-10
1-18
157 165 174 183 200 4.6
M 278 217 234
312 6.4
06 243 252 260 200 304 280 205
303 321 11.3
329 338 346 440 372 381 308 7.2
07 355 380
468 407 484 402 8.1
08 424 475
501 518 520 β2Ι 644 562 500 578
09 509 501 055
072 603 012 700 629 6.38 ΙΊΙ3
510 723 731 7-10
757 680 680 791 714 740
842 760
12 774 783 800 808 817 825 8.34
808 803
13 927 851 8δθ
885
052 876 078 ΩΟ2
14 71 012 935 944
υυ3 010
020 029 037 001 064
Ιδ 040 071 079 *003
181 106 113 122 130 147 088 8
180 155
17 205 189 198 200 223 231 2411 2-18
214 172
290 307 315 324
18 849 273 282
290
332
19 433 867 374 891 300 408 410
620 517 441 450 458 883 476 483 402 600
425
61>8
21 οοο 525 533 642 4ββ 559 507 584
560 δΙ)2
22 βορ 617 625 (3,34 642 650 οπο 1507 (175
23 884 709 759
767 692 700 792 726 7-12
717 734 750 025
860 776 784 875 800 825
26 800 817 834 *008
858 807 802 008
26 933 958 883 017
72 016 941 950 975 983 001 173
27 041 ΩΩ9
024 032 057 οσο 074
28 123 148 082 255
181 107 115 132 140 150 337 7
29 200
580 263 189 198 214 222 230 230 247 410
81 272 280 08 2Ωβ 804 313 321 321)
32 364 362 870 378 887 305 403 411
38 436 444 452 400 460 477 485 403
691 518 620 534 542 032 558 507 575 501
673 607 6113 024 713 (548
35 β81 722 738 583
86 754 089 697 705 730
795 803 810 740
886 702 779 787 870 811
87 770 884 827
843 800 892 981
38 852 868 057
997 925 941 973 *002
80 983 *088
73 078 *006 1-13 *071)
159 086 *014 *022 *080 110 127 *054
223
094 102 111 199 207 135 151
41 239 167 304
14
42 176 183 191 215 384 231
43 u7 2δδ 2β3 272 280¯ 288 4114
320 328 352 380- 200 644
46 408 336 344 482 440 388 370 024
480
46 488 416 424 612 520 448 450 703
560 496 604 312
47 668 δθ2 οοο 528 630 783
40
719 048 570 584 072 β7θ 608 010 802 032
759 087 711
799 727 735 748 761 941 791
878 807 815 823 830 838 707 *02υ
967 886 010 854 870
894 902 018 840
049
74 080 973 981 980 997 020 933
044 *013 *028
062 060 068 070
084 092 099 107
N ο 1 2 8
7 8

15
N ο 1 2 ο 7 8 9
550 74 014 052 οοο ου8 07υ 084 092 οοο 107
61 123 131 130 155 102 170 178 180
52 115 233 8
202 210 218 24 Ι 240 267 343
58 194 147 312
280 288 320 327 335 421 1.
273 225
54 369 874 390 808 400 σοο 0
851 437 307 453 304
468 .17β 484 492 678 8.
429 515 445 531 382 547 564 502 570 2
57 623 050
507 539 024 048 7.33 4.
68 οοο 632 640
580 071 001 087 017 702 710 718 720 811 0
59
741 740 757 704 772 780 788 803 4.
8
819 827 834 842 860 858 805 873 881
5.
62 896 904 012 027 935 043 880
8
03 974 081 θ8θ 020 *005 *020 *028 958 900
6.
οου 007 106 *036 *043
64 76 Οδ1
074 082 089 4
113 120
65 128 130 143 159 ισο 174 182 7.
206 213 220 151 23U 243 251 250 180 107
2
282 289 297 228 312 320 328 335 343' 274 2
08 305
358 300 374 389 397 404 412 420 851 3
09
435 442 450 881
1168 405 473 481 488 490 427
570 611 519 626 534 612 540 557 565 572. 504
7
71
587 595 003 αιο (318 υ26 (133 041 648 580
72 717 8
004 071 079 080 004 702 700 793 724 732
78 9
740 747 755 762 770 778 786 808 800 808
74 815 823 831 838 848 853 870 884
75 801
891 899 900 014 921 020 944 952 050
70 0.37
907 974 082 089 907 *006 *Ο12 *020 *027 *036
77 70 042 060 067 072 080 087 005 103 110
78
79 118 126 133 140 148 166 103 170 178 186
580 193 200 208 215 223 230 238 245 253 2θ0
208 275 283 290 208 806 813 320 328 335
81
843 350 858 305 373 380 388 395 403 410
83 418 470
433 440 448 455 402
84 492 425 507 515 522 530 637 645 477 485
85 567 682 680 597 004 012 019 652 034
641 049 071 078 603 820 708
87 716 723 730 738 7,15 763 780 708 701 782
88 790 797 805 812 819 827 834 842 775 850
89
804 871 870 880 893 001 008 910 849 030
590
938 045 953 900 907 075 082 989 923 *004
02 77 012 026 034 041 048 056 008 070 078
03 085 003 100 107 115 122 129 187 1,14 151
160 100 173 181 188 195 203 210 217 226
M
232 240 247 254 202 200 270 283 201 208
16
90 305 313 327 342 349 357
820 336 304 371
97 379 380 401 415 422 490 44-4
98 452 393 474 408 488 495
459 481 603 510 517
526 532 630 640 601 588 590
654
597 612 010 634 641 648
070 077 086 692 027 706 714 721 728 735
743 750 757 764 772 779 78β 793 801 808
815 822 880 837 844 851 850 860 873 880
5 8 7 8
Ν ο 1 2

17
N ο 1 4 8 9
600 77 816 822 830 837 844 851 859 866 873 880
02 887 θβ7 902 θ1θ
θΟθ 924 938 *017 θδ2 8
039
981 •
088 031
03 78 032 974 053 001 008 097 0.
075 082 089
M 111 046 Ι 40 8
104 125 132 147 154 101 168
05 183 118 197 211 240 1.
178 204 219 226 233
06 254 283 0
247 262 260 278 290 297 806 312
07 356 2.
819 826 338 840 347 362 360 376 388 4
08 890 398 405 412 419 420 440 447 455
497 504 3.
462 460 476 488 512 510 520 2
010
533 640 547 564 561 576 583 500 597 4.
11
12 033 054 0
604 611 618 640 047 608 4.8
13 704 711 718 725 ουι
θ7δ 082 760 β25 774 732 739 0.4
781 789 790
15 746 753 7β7 803 810 7.2
831 845 852 850 800 2
817 824 902 838 916 873 880 84
923 930 037
17 888 972 986 944 *021 α7
979
18 968 8
19 030 043 060 057 064 071 078 092
029 10β 113 120 127 134 141 148 085 102
1βθ 176 188 Ι 90 Ιθ7 204 211 218 225 232
21
289 240 253 260 267 274 281 288 295 302
22
23 809 316 323 330 337 344 368 365
24 879 380 303 400 407 414 851 428 435 372
25 449 456 468 470 477 484 421 498 506 511
20 518 525 532 539 546 653 567 574 581
27 688 596 602 609 016 623 887 044 050
057 680 2
0.
28 671 678 685 706 713 720
8 7
29 727 734 741 748 754 761 768 775 782 780
796 803 831 851 858 1.
810 817 824 837 844
8βδ 872 4
31 879 886 893 900 913 020 927
2.
82 934 948 955 962 975 082 980 990
8
83
80 003 010 017 024 030 087 044 061 068 3.0
34 072 079 085 092 Οθθ 100 113 120 127
134
4.9
168 175 182 6.6
35 140 147 154 161 188 195
202 0.
30
209 216 223 229 236 243 250 257 8
271
37 277 284 2θΙ 298 305 312 318 325 332
4
38 346 358 859 378 880 387 393 400 407
39 402 408 7
414 421 428 434 441 448 455 476
482 489 496 602 509 530 630 643 8
584
41 557 564 570 577 598 004 011 9
βδ2
42 818 625 632 688 645 623 672 070
18
48 686 706 720
713 747
754 767 774 787 726 738 740 814
780 781
45 821 835 841 855 794 801 808 882
828 848
46 889 θ22 862 868 875
895 902 909 916 949
47 81 023 963 969 976 057 929 930 943 *017
983
48 037 048 084 070 077 084
090 080 050
49
168 097 104 ΙΙΙ 117 124 131 187 144 151
224 ι 84 171 178 184 191 198 204 211 218
291 231 288 245 251 258 266 271 278 285
298 305 811 818 825 831 888 346 351
ο 2 8 8
N 1

19
N ο 8
1 3 5 9
81 291 298 305 311 318 325 381 388 351
52 358 365 378 385 898 405 411 418
53 426 431 371
4-45 451 391 465 471 478 486 7
401 498 505 518 458 531 544 561
54 1.
571 578 625 604
55 558 584 598 611 β17 4
624 631 637 644 051 591 664 671 677 684 2.
βρο βθ7 704 710 βδ7 730 737 750
57 717 743 1
723
58 757 763 770 776 783 7Ωβ 808 809 818 2.
60 823 820 838 842 849 790
862 8θθ 875 882 8
889 8θ5 002 008 016 921 028 936 041 948 8.
01 054 008 074 987 *014 5
02 82 020 040 053 073 079
027 033 105
63 080 125 132 138 145 6.
151
002 οοο
171 112 119 101 204 210 3
04 107
158 178 184 2
06 217 223 230 236 249 256 283 200 278
282 302 243 8
341
07 280 205 315 321 328 400
847 387 308 406
4
08 354 360 380 387 393
465
373
413 41θ 432 452 458 630 471
870
428 430 445
478 484 407 517 523 δθ5 δ3β 8
401 504 510
71 543 502 675 682 688
72 607 614 020 027 038 ΜΟ 040 653
73 672 670 685 750 608 706 711 718 724 730
74 737 743 760 821 703 769 776 782 789 7θδ
75 802 886 827 834 840 847 853 860
70 800 872 870 060 802 898 006 011 018 924
77 030 937 043 *014 *020 063 *033 975 982 088
78 905 *001 *008 078 085 *027 *062
007 104 110
70 83 050 120 072 142 149 091 161 168 174 117
680 128 193 136 208 213 155 225 239 238 181
81 187 257 200 270 276 210 28θ 2θθ 302 246
251 264
82 283 308
83 316 321 327 334 340 347 353 369 366 372
378 385 891 808 404 410 417 428 42θ 438 8 3.
85 442 4-48 455 461 467 474 480 487 493 499 0
81) 8.
512 518 525 531 537 544 550 78θ 6
87 632
575 582 688 δθ4 807 613 620 620 4.
β30 058 683 β8θ
88 045 070 677 2
οοο
80 702 708 716 721 727 784 740 746 758 4.
750 700
778 800 818 8
6.
4
92 822 705 771 784 707 803
93 886 828 835 847 853 800 866 872 879
04 948 801 807 004 010 016 028 029 936 042
84 011 οοο 973 079 986 Ωθ2 098 *004
073 017 023 9β7 036 042 048 066 061 067
97 088 020 008 105 111 117 123 130
196 080 002
08 ΙΩ8 142 148 181 167 173 180 180 102
201 205 211 156 223 230 236 242 248 265
273 217 280 202 298 305 311 817
328 287
280
380 830 336 348 354 361 367 873 370
448 302 398 842 410 417 423 429 435 442
454 460 473 470 485 491 407 604
δΙθ 522 528 536 ΜΙ 647 563 569
N 610 2 8
1 7 8
o

N ο 2 3 8 8
1
700 84510 516 522 528 535 541 547 653 550 506

01 684 507 600 615 621 028 7


572 578
02
634 640 646 500 058 603 671 677 683 089
2 1.4
03
702 708 052 720 726 733 730 745 741
3 2.1
04 714
757 763 770 782 788 794 800 807 813 4 2.8
05 819 825 831 770 844 850 856 802 868 874 6 3.
880 887 893 905 911 917 924 930 930 6
07 907 4.2
08 942 048 973 970 985
7 4.
85 003 016 022 028 034 040 040 052 058
09 071 077 083 089 9 8 5.0 ο 0.3
101 107 114 120
710 126 132 138 144 150 156 103 160 175 181
11
12 187 193 211 217
109 230 236 242
13 248 254 205 272 278 224
260 291 297 303
14 809 315 321. 833 339 286
327 345 352 358 804
370 394 400
376 382 388 412 418 426
431 455 461 4013
17 407 603 500 467 473 479 485
518 522 -i 1.2
18 528 534 1.
10 552 558 664 570 576 582 2
612 018 625 681 637 648 588 504 600 8 3.
8
720 βθ7 703 700 715 721
678 679 685 αρι 727 5
21 0
733 730 745 751 757 763 7β9 775 781 788 7
22 3.
23 794 800 806 812 818 824 830 886 842 848 8 6
4.2
21
24 860 866 872 878 884 002 908
25 920 926 932 944 890 890 962 908
26
864 980 986 θθ2 098 '004 *022 *028
27 914 040 046 052 058 064 070 076 082 088
28 124 130 136 141
974 100 106 112 118 147
29
80 034 150 185 171 177 183 180 105 201 207
730 ΟΜ 219 225 281 287 243 249 255 201 267
81
82 153 279 285 291 297 303 308 314 320 320
218 338 844 350 356 862 868 374 380 386
83
278
84 398 404 415 421 433 445
882 457 427 493 430
85 463 410 475 481 504
36 392 516 487 552 558 564
522 528 534 640
451 546
87 581 587 βΙΙ 017 623
510 β3δ
38 641 652 658 670 676 682
39 570 894 705 711 717 720 735 741
700 723
688 758 764 770 776 782 788 704 800
41 747 812 817 823 820 835 841 847 853 859
42 806 870 876 882 888 804 οοο 011 017
864 935 941 947 068 970 076
44 923
982 904 7 *023 *035
45
088 990 *005 *020 003
48 87 062 Οδ8 064 070 075 081
47 099 040 111 116 122 128 134 140 087 151
106 146 210
48 157 169 175 181 180 192 198
49 216 183 227 233 239 245 251 266 204 268
221 262 320
274
280
286 291 297 303 809 315
320
332 844 355 3β1 807 878 384
340 379
390 402 408 413 419 425 481 500
448 464 480 471 477 483 489 4θδ
508 512 518 523 529 585 ΜΙ 647 552

8 δ 8 7 8 9
N ο 1

N ο 1 .2 3 7 8
750 87 512 518 528 529 535 541 547 552 558
61 564 587 503 714 602 β1θ
52 022 570 651 720 σιο 574
581 703 772
53 679 028 576 708 008 731
760 829 777
β8δ 633 β07 726
737 743 740 0.
705 800 806 812 818 835 783 780 8
54 841
852 868 864 869 875 823 887 802 904 1.
881 898 2
010 016 021 027 033 044 050 901
57 007 073 978 084 000 038 *001 *007 056 *018 1.
58 88 030 036 ΜΙ 047 053 058 004 070 070 8
59 081 087 003 008 104 110 110 121 127 133 2.
760 150 150 184 190 1
144 101 107 173 178
61 188 207 213 241 247 3.
201 218 224 280 235
02 195 204 270 208 304 0
258 275 281 287 202
63 252 3.
315 321 320 332 338 343 355 360 8
300 372 377 383 406 412 417
65 389 305 4.2
866 420 434 440 467 463 408 474 4.
423 513 510 8
67 491 407 502 508 525 530
480 542 547 553 650 564 570 576 581 687 2
68
536 598 βιο 615 021 627 032 638 043 3
770 503 β6δ β72 077 083 680 604 700 4
71 640 722 67
72 705 711 717 770 728 734 730 750
762 767 784 700 705 745 812
73 773 885 807
818 824 840 840 862 801 808
829 801 803
74 857
75 874 880 885 047 897 002 008 010 925
980 936 953 958 064 913
76 041 075 081
986 092
. *000 *014 *020
097 *087
77
78 042 048 053 050 064 070 070 081 087 092
79 008 104 100 115 120 120 131 137 143 148
780 154 150 165 170 176 182 187 103 108 204
81 209 215 221 220 232 237 243 248 254 260
82 265 271 282 287 293 304 310 315
321 270 208
83 320 337 343 348 360 365 371
370 332 354 5
84 382 803 308 404 416 421 420
887 400
85 432 487 448 454 450 470 476 481
80 487 492
443
604 500 516 465 520 531 537 3.
542 648
498
550 664 670 520 581 586 502 o7 0
87 558 575
88 507 0-03 014 β20 626 036 642 847 3.
80 053 658 600 οοο 875 080 691 697 702 6
708 713 710 724 730 735 741 740 752 767
οι 768 768 774 770 785 700 700 801 807 812
02 818 840 000 851 856
03 873 823 820 834 804 055 011 802 887
027 878 883 880 *ΟΟΟ 910 022
04
05 082 033 038 044 084 *020 971 077
οο 088 903 098 113 119 075 *020 *031
0.37 124
97 042 048 053 ι ά78 129 080 086
001 227 170
98 097 102 108 184 135 140
23
800 146 151 157 162 189 106
200 206 211 217 22 233 238 244 249
255 200 200 271 270 282 287 203 298 304
314 320 325 331 836 342 347 352 868
Ν 300 2 3
1 7 8
ο
N ο 1 2 8 5 8
800 ρο 809 314 320 325 331 336 842 347 852 358
01 374 380 390 401 407 412 6
02 863 428 484 385 445 450 455 461 466
520 0.6
08 417 477 482 488 430 499 504 515
1.2
04 472 531 536 542 493 553 558 563 569 574
528 585 590 596 547 607 612 617 623 628
580 650 655 871 677 082
07 4.8
684 893 698 703 714 720 725 730 786
09 6.4
887 747 752 757 763 768 773 779 784 780
810 2
741 800 806 811 816 822 827 832 838 843
11 795 854 850 865 870 875 881 886 801 807
12 849 78o
907 018 024 920 θ4δ 060
13 902 913 034 940
001 972 977 982 988 203 098
14 030 *004
91 οοο014 020 025 030 041 052
15 057
Ιβ 062068 073 078 084 089 094 100 105
116121 126 132 137 142 148 183 158 110
17
189174 180 185 100 196 201 206 212 217
18
222 228 233 238 248 249 254 259 265 270
820 275 281 286 291 297 302 307 312 818 823
21 328 334 330 344 360 355 365 371 376
22 381 387 302 307 403 408 413 418 424 420
23
434 461 471 477 48
u 25 445 450 455
487 514 519 524 520 535
26 492 498 503 508
540 506 572 577 682 587
27 545 551 550 561 8
593 819 624 030 040 4.
28 598 614 4
645 661 672 677 682 687 693 0
29
698 703 700 714 719 724 730 785 740 745 7 4.
751 756 781 772 777 782 787 793 798 8 6
31
808 808 814 819 824 829 834 840 845 850
32
855 861 8ββ 871 876 882 887 892 897 903
33
908 918 918 924 029 934 244 050
34
35 92 012 965 971 976 981 086 091 297 *007
86 065 018 023 028 033 038 044 049
ιω 059
158
87 117 070 076 080 085 091 101
210
111
38 148 153
221 122 127 132 137 143
200 205 262 163
174 179 184 180 195 215
ΜΟ 273 231 286 262 267 314 207
241 247
41 324 278 366
288 288 293 298 804 30θ 310
42 876 830 418
335 840 845 350 865 361 371
48 428 381 407 412
387 802 397 402 423
44 480 483 450
45 531 438 449 454 484 474
46 583
485 490 495 500 505 511 516 621 526
834 542 547 552 . 562 572 678
48 686 588 598 598 603 657 614 624 029
49 737 639 670
850 θθ1 646 650 655 609 876 681
788 742 716 722
747 701 706 773 727 732
840 762 758 711 788 778 788
891 798 799 763 819 824
942 845 850 804 809 829 834
855 860 814 870 875 881 886
947 952 908 911 8βδ 921 927 982 937
916 θ78
957 962 988
907

o 2 4 5 7 8 9
N 1

26
27
N ο 1 2 3 6 7 8
850 92 942 947 052 057 962 007 973 078 983 988
51 *013 '-018 *024 *034
993 040 *003 *008
52 004 Οθθ 076 080 085 090
98 044 100 105
53 110 115 120 125 131 136 141
54 140 151 156 161 171 170 181 186 Ι 02
55 107 202 207 212 217 222 227 282 237 242
247 252 258 203 268 278 278 283 288 293
57 303 308 313 318 323 328 334 330 844
298
58 354 369 384 369 374 379 384 380 804
340
59 404 400 414 420 425 430 435 440 445
860 450 455 460 405 470 476 480 1185 400 405
82 500 620 531
δΟ5 515 526 586
561 556 510 571 581
63 σοι 5ββ 576
βΙΙ 621 031
687
65 851 058 661 671 682 697
737
702 707 712 717 722
727 782 742 747
787
87 752 757 782 707 772
777 782 702 797
68 837
802 807 812 817 822
827 832 842 847
09 887
862 857 862 807 872
877 882 802 897
870 937
002 907 012 917 022
027 032 047
71 087
952 057 902 907 972
077 982 992
242 907
72 94 002 007 012 017 022
027 032 037 042 047
73 052 057 062 067 072
077 082 086 091 146
74 101 ιιι 116 121
126 131 136
75 141 106
151 156 161 1ββ 171 17β 181 186
76 211 191 245
201 206 216 221 226 231 236 240 205
77 250 255 280 205 270 275 280 286
78 290 345
300 305 310 315 320 325 380 335
70 840 394
349 354 364 419 374 370 384 438
880 390 401 389 443
458 433
81 408 424 420 498
448 453 473 478 483 488
82 408 503
83 507 512 517 522 527 632 537 542
547 552
557 662 567 671 57β 581 586 591
84 β01 011 621 626 630 635 640
85
86 650 670675 880 685 689
748 704 709 714
748 753 768 710724 729 734 788
87 758
702 708778 778 788 787
88 707 802 812
841 807 817822 827 832 838
89 846 851 881
800 858 871 876 880 885
800 895 οοο 910
030 905 910 924 929 934
01 914 049 050
088 908 973 978 983
02
93 085 003 908 *002 *007 *012 7 *022 ι075 *032

28
M 134 041 143 100 168 061 114 071 124 080 6
95 182 000 102 148 202 109 163 119 129
78
231 130 240 107 260 158 211 188 0.
221 177.
97 6
279 187 280 246 2θθ 207 260 216 270. 228
08 1.
328 236 337 294 347 255 808 818 274 2
376 284 386 342 305 30.3 357 313 8βθ 323 1.
οοο
424 332 434 300 444 352 405 301 415 371 8
381 439 400 453 410 463 419 2.
458
429 448 468 4
3.
0
2
3
4

1
2
1.
0
1.
5
2.
0
1-
1
4.
5
8
4

7
8
9

29
2 7 8
N ο 1 9

ο 8 8 7 8
N
1 2
453 458
οοο 434 444 448 408
οι 424 477
02 472 525
03 521 574 482 487 492 497 511
501 δου 516
04 622 530 535 540 545 550 564 550
05 617 670 578 583 588 593 508 602 807 012 0.6
713 718 626 631 636 θ50 655 οοο 1.
07 674 679 684 680 604 608 703 708 0
08 761
813 722 727 732 737, 742 746 761 756 1.
09 809
861 770 776 780 785 700 804 5
856 789 704
818 823 828 832 847 852 2.
004 837 842
ΙΙ 866 871 875 880 805 899 0
952 885 800
2.
12 914 918 928 028 083 038 042 047
047 6
13 971 076 080 086 οοο 005
005 057 θβ1 3.
*028 *033 *038 *042
ι 42 *014 *019 *023 0
15 076 080 085 οοο
100 052 057 071
ορο 104 109 114 123 128 133 137
17 237 118
• 18 147 152 150 101 171 175 180 185
19 284 194 204 200 213 218 223 227 232
382
020 242 246 251 256 261 266 270 276 280
879 318 817
21 289 204 208 303 808 322 327
22 336 341 846 850 355 360 305 300 374
23 884 888 393 308 402 407 412 417 421
24 481 440 445 454 459
426
25 478 483 487 492 497 601 500 408
473 611
26 525 630 534 544 648 558 516
520 568 502
27 572 577 581 586 591 595 οοο
567 οοο
28 019 624 628 633 688 042 647 652
29 670 675 680 686 080 604 703
708 718 717 722 727 731 736 741 745 750
31 755 759 764 700 774 778 783 788 7θ2 707
32 802 806 811 810 820 826 880 834 830 844
33 848 853 858 882 867 872 870 881 880 800

30
34 928 032 037
οοο 951 909 014 018 023 274
36 942 070 084
988 θθ7 *007 *011 070 *021 *080
37
07 035 099 044 049 053 058 072 077
38 067
30 081 086 ορο 005 100 104 083 114 118 123
1.28 182 187 142 146 151 109 ιαο 165 189
155
174 179 183 188 192 197 206 211 216
42 220 225 230 234 239 243 202 253 267 262
43 267 27 Ι 270 280 285 2θ0 204 200 304 808
318 317 322 327 381 386 340 3,54
45 359 382 887 345 350 400
384 368 373 377 428
405 410 433 301 390 447
47 451 414 410 424 474 403
470 437 442
460 465 470
49 497 520 525 488 488 630
602 511 510
643 506 671 620 534
548 557 562 612 017
589 652 575 680
594 603 607 βδ8
698 621 626 676
885 704 ββ3
844 722
681 685 749 708 667 672 768
727 731 738 740 745 754 713 717
772 777 782 786 791 800 759 763
5 804 809 818
N ο 1 2 8
7 8

31
N ο 2 3 6 7 8
1 4
ΟδΟ 97 772 777 782 780 701 795
51 818 823 827 832 841 800 804 809 813
52 864 868 873 877836 886 845 850 856 850 5
οοο 014 018 923882 032 801 896 οοο 906 2.
54 055 059 968028 078 937 941 950 2
55 08 οοο οοο 014073 028 982 987 991 996
050 037
040 019 068 028 032 4.
57 οοο
001 141 100 105 064 114 073 078 082 087
58 6
137 186 146 150 100 160
118 128 127 132 4
50
182 101 105 155 204
164 108 173 177
οσο 232 230
227 200 260 209 214 218 223 7
61
62 050 246 254 250 8
272 277 281 280 263 268 0
63 295
318 322 327 331 200 290 304 308 313
340
05 363 807 372 378 336 890 340 354 358
385
408 412 417 421 881 394 403
430 435
453 457 462 426 480
08 475 484 489 403
498 502 507 471 526
520 629 534 638
543 547 552 001
588 661 665 570 574 579 583
71 502 597 614 623 628
72 632 037 041 050 664 608 673
73 677 682 086 605 700 704 700
74 722 713 717
720 731 735
75 707 740
780 740 753 758 762
771 776 780
70 811 784
884 793 708 802 807
810 825
856
829 838 843 847 851
77 860 866 800 874 878
78 900
918 023 883 887 892 896
905 900 014 927 032 936 041
70 θβ3 907
089 068 972 981 985
980 904 098
οο *007 *012 *021
*016 065 *025 *029
81 034 038 043 047 052 056 074
82 078 083 087 002 100 105 100
83 118
123 127 131 136 140 145 149 154
84 107 171 176 180 185 180 103 108 158 162
85 211 216 220 224 220 233 238 242 202 207
80 256 200 264 273 277 282 286 247 251
87 800 804 308 313 317 322 326 330 291 295
88 844 348 352 857 361 360 370 874 885 330
80 388 302 80β 401 405 410 414 419 428 388
432 441 446 440 464 458 407
427
01 470 480 484 480 403 408 502 550 511 471
32
02 520 524 528 588 537 642 565 515
03 δβ4 508 572 577 581 585 594 599
β21 625 620 (334 603
05 638 642 647
051 612 οια 708 660 673 677 082
600 704 752 712 717 721 726 730 734
07
730 743 747 795 700 765 769 774 778
08
782 787 701 830 800 804 808 813 817 822
1000 820 830 836 883 843 848 852 856 861 865
870 874 878 026 887 801 806 904 009
013 017 922 070 930 935 980 944 948 052
957 974 078 983 087 091
00000 004 οοο 018 017 022 026 080 085 080
5 7 8
o 1 2 8

33
N ο 3 5 6 8
100 οοο 1 2
1303 2171 2605 3473
0 0000 0434 0869 1737 3039 8Ω07
100 434 477 564 051 094 781
1 1 5 2 0 3 737 0
100 867 911 520 097 607 *084 *127 7 *214
2 7 1 8 7 6 4 7 *1710 3 824.*
100 001 344 054 480 *041 517 560 603 647 *257C
3 8009 2 4 8 1 4 7 0 2
100 738 777 387 863 474 949 903 *036 *070 *122
4 7 0 5 5 1 9 2 4 6 8
100 002 209 820 295 906 382 426 408 511 554
5 1661 8 2 7 7 1 3 5 6 8
100 598 641 262 727 888 813 856 000 943 086
6 0 1 5 5 0 8 0 1 2 3
100 003 072 684 158 770 245 288 331 374 417
7 0295 6 3 8 6 1 2 3 4 4
100 460 503 115 580 2019 676 719 762 805 818
8 5 6 7 8 832 0 0 0 1 1
100 891 034 546 *020 8 *106 *140 *102 *236 *278
9 2 2 7 3 *008 3 3 4 4 4
1010 ΟΜ 9772 450 3 536 570 622 006 708
101 3214 4074 4 4033 3 3 3 2 2
1 751 794 8371 880 9229 065 *008 *061 *0Ω47 *137
1012, 2 1 2663 0 3521 9 8 7 5237 6
101 180 223 βθ52 309 7809 395 487 480 052 ΩΩ5Ι
3 5 4 2 3
123 2002 0 9 8
4233
609 652 738
101 6 6372 823 8ββ 8805
8510
4 3 0 8 θ 337
4 551 *084 8082
088 7 *278
101 080 6 166 7 252 204
0 765 2
5 8 979 4 4017 1 0 6024
101 466 508 2 504 722 6 *088 7051
0184 *107 *102 *Ι.31β
6 0 8 4 7 0
406 4 6570
898 936 *021 7704 *150 8 615
1017 4 5344 0832
7 5 9 1 010 α
1018 833 οσιο
007 8 04074084
1019 363 1 440 3872 677
3210 8332
7 250 0 8130 1 4724
747 4 7007 *2575
1021 7θ0 875 *003 8981 *2151
8 4 685 7 7 6815
1022
1023 008 216 3 4208 6391 *1050
1742 8 727 8560 *0027 5282
1024 600 9 4850 9500
642
1025 2
34
1026 7 0080 *3732
1027 025 *3310 7051
1028 7 7520
1029 450
1080 9
875
1031
6
1032
1033 010
8000 110 158
1034 0683 8
723 3
1035 4934 536 2384 2809 3234
9 578 1069 6633
1036 9181 9 7068 7483
011 4 6208 *087
1037 1474 3424 *1803 *1727
*045 8
1038 7662 8848 4272 5544 5θ67
570 4
1089 189 8086 8510 5120 θ78
4 «)2Ο
1040 7 2320 2743 Ο
998 8933 9367 4
1041 1 6127 6650 6978 3500 4013 4-430
1042 012 *085 *077 *119 7306 7818 8241
1048 4154 4 6 8 *1621 *2043 *2405 *2887
1044 837 4576 4998 5420 5842 0264 0685 7107
1045 2 87Μ 9215 9637 *0059 *0480 *0001 *1323
018 8008 5534 *174-4 *2185
1047 2587 7218 9742 5055 0376
1048 β7θ7 1424 3945 *0162 *0583
1049 014
5625 8144 4305 4785
1050 1003
9823 *2340 8δθ4 8084
520 4017 0531 *276 *8178
5 0 7.300
8206 *0718 σο50
940
2392 3429 4271 4692 5113 4001 *1556
3 3850
6573 7639 8480 8001 0321 0080 *1137 5737
015 8059
0751 1844 2β8δ 3105 8525 5310 0010
8608 2264
6885 7305 7725 0408
778 6045 6465
*0243 *1082 *1501 *1920
8 4855 6274 5θθ3 6112
016 4436
1974 8625 0044 9462 9881 *0300
2810 8229 3647 4065 4483
017 6991 7400 7827 8245 8063
0333 1168 1586 2003 2421 2838
4507 4924 5342 57δθ 6178 7010 3256 3673 4000
8677 9094 0511 9927 *0344 8593 *1177 7427 7844 8200
018 8259 8670 4002 4508 *0761 5841 *1504 *2010 *2427
2848 7421 7837 8253 8660 4925 5757 6178
1578 1994 2410 2825 9084 *0747
7005
βδθ2 7807 0916 *0332 4002
010 5732 6147 6977 3240 4071 4486
1163 *0711 *1126 7892 *1055 8222 8637 0052
0882
35
5317 4027 4442 4856 *2809 *2784 *3198
9487 8169 8583 89θ7 0513 0927 7341
020 2807 2720 3134 *0652 *1060 *1479
3613 *1540
7755 5270 6684
021 9411 0824 *0238
1893 3547 3061 4374
5 6 4787 5201 5614
1 2
8

36
9
N ο 1 2 6 7 8
1050 021 1803 2307 2720 3134 3547 3Ωβ1 4374 4787 5201
1061 022 0157 0864 7287 7680 8093 8500 8019 9332 5β14
1052 4284 0570 0θ83 1390 1808 2221 2034 3040 3469 9745
1053 8406 5109 5521 5933 0345 6758 7170 7682 3871
1064 028 2625 8818 9230 9642 *0054 *04βσ *0878 *1280 701 7994
1055 2036 3348 3759 4171 4582 4994 5406 5817 *2113
024 0750 7050 7402 7873 8284 9517 9928 0228
1057 4857 1101 1572 1982 2303 2804 3214 3625 4030 *0339
1058 8960 6267 6678 6088 6498 *1010 7310 7720 8139
025 9870 9780 *0190 *0800 6107 *1410 *1829 *2230 8549
1000 7164 *2649
8468 3878 4288 4007 5920 0335
026 1245 7503 7972 8382 8701 9200 0009 *0018 *0427 *0830
1002
6333 1654 2003 2472 2881 3289 8098 4107 4515 4024
1003
9416 5741 0967 7375 7788 8192 8600 9008
1064
027 3490 0824 *0233 *0641 *1049 *1457 *1805 *2278 *2680 *3088
1065
7572 3904 4312 4719 5127 5535 5942 0850 0757 71 βδ
1007 028 1044 7979 8387 8794 9201 *0016 *0423 *0830 *1237
1008 5718 2051 2458 2865 32-72 4086 4492• 4890
9777 7745
0119 0520 0032 7339
*1808 8152 8558 8904 0371
1070 029 3838 '*0183 *0600 *09θσ *1402 *2214 *2620 *3020 *3432
1071 5867
7896 4244 4049 6055 5461 0272 6078 7084
7,180
Ι 072 030 1948
8300 8700 0111 9510 9922 *0327 *0732 *1138 *1643
1073 5997 2358 2758 , 3168 3508 3973 4378 4783 5188 5502
1074 031 0043 6402 6807 7211 8020 9638
7010 8425 8830 0234
1075 4085
0447 0851 1256 1060 2084 2468 2872 3277 3681
1076 8123
4489 4803 5290 5700 0104 6508 0912 7315 7719
1077 032 2157 8520 8930 9338 9737 *0140 *0047 *1360 *1754
1078 6188
2560 2003 8367 3770 4173 4570 4979 5382 5785
1079 033 0214
0590 09θ3 7890 77θΩ 8201 8604 9007 0812
1080 4238 1010 1422 1824 2220 2620 3031 3433 8836
0017
1081 8257
4640 5042 5840 6248 6050 7052 7453 7855
1082 034
2273 8050 9000
9402
9804 *0265 *1068 *1470 *1871
1083 *0607 5081 6482
β28δ 2074 3075 3878 4270 5884
1084 3477 4080 9091 9491
0680 7087 7888 8289 9892
1085 035 0293 7487 8690
4297 0003 1004 1405 1805 2290 3000 3407 3807
1086 2090 7098 7498
8298 4608 0098 5408 5898 0208 78θ8
1087 *0007 *1097 *1490
8008 0098 0498 9898 *0297 *1898
1088 030 2206
1089 0289 2005 3004 3494 8803
4293 4002 5091 5491 6890
1090 037 0279 0088 7087 7480 7885
8284 8083 0082 9181 9880
0078 1070 1874 2272 2071 3070 3468 3867
1001 4205 1475
1092 8248 4003 5062 6,100 5868 0257 0666 7058 7451 7849
1008 038 2220 8640 0044 0442 9830 Τ2ϊ7- *0035 *1033 *1131 *1829
2024 3022 8419 3817 4012 6407 6804
37
1006 6202 7701 4214
080 0173 1701 8188
100 4141 0590 7893 5727 2168 8586 8982 9379 9776
7 8100 0570 1304 0124 2564 2951 3348 8745
109 4538 0067 5331 3050 *0080 6520 0917 7313 7709
8 040
2080 8502 8808 0204 7005 4045 *0482 *0878 *1274 *1070
1100 0023 2402 2858 3254 *1557 8001 4441 4887 6232 5028
9977 ΜΙΟ 6814 7210 *Ιθδ2 8396 8701 9187 9682
041 *0372 *0707 *1102 5900 *2347 *2742 *3137 *3632
3027 4322 4716 5111 6295 ωρο 7084 7479
N ο 2 8 4
1 8 7 8 9

38
ТАВТ.,Е Ш—ТЫе Number of Each Day 0f the Уеат

1 32 00 91 121 152 182 213 244 274 306 836 10


1
2 2 33 61 92 122 153 183 214 246 275 800 330
2 з 34 62 93 123 154 184 215 240 270 307 337 12
8 4 36 63 94 124 155 185 210 247 277 308 338
5 86 04 96 126 160 180 217 248 278 309 339
в 6 37 90 120 157 187 218 240 279 310 340
7 7 во 188 250 280 311 341
38 97 127 158 219
8 67
9 8 39
68
98 128 169 189 220 261 281 312 342
9 40 90 129 160 190 221 252 282 313
69
10 41 100 130 161 191 222 253 283 314
70 345
131 162 192 254 284 316
11 42 71
101
132 108 193
223
265 285 .ЯШ
340
43 102 224 347
12 72 317
46 103 134
1B4 104 226 256 280 348
18 73 106 318
40 104 136 195 220 257 287 340
14 74 136 310
106 196 227 258 288
16 47 75 136 350
107 197 228 260 289 320
48 76 187 361
16 107 198 229 200 290 321
49 77 138 362
17 108 170 199 230 201 2f)l 322
50 78 139 353
18 109 171 200 231 202 202 323
61 79 140 354
19 110 172 201 232 203 203 324
и 2-5 62 141
20 80 111 173 202 326 365
53 142 из 264 204
26 21 54
81 112
143
174 203 234 206 206 320 363
28 22 82 113 175 204 236 327 367
65 144 200 290
114 176 236 328 358
28 и 23 83 146 206 267 207 360
60 115 320
31 24 84 177 206 237 268 298
26 57 116 140 зоо
85 178 207 209 200 330
58 да 117 147 239 301
20 170 208 270 зоо 331
69 87 118 148 240 362
27 180 209 271 801 332
149 241 303
28 88 119 181 210 272 302 333
120 150 242 304
29 89 211 273 зов 334
161 243 366
до 212
31

5
7
8
10
1111
12
1313
14

39
1
5
1310
1717
1818
1919
2
0
2121
2
2
2323
2
5

2727
2
9

Non. — In [вар уевн. after February 28, add 1 Во the tabulBted number.
TABLE IV — Ordinary and Exact Interest at 1% on $10,000
Exact In•øregt to 865 Dayg Ordinary for to 360 Days
For days below add to For days below add to interest
Interest column
aye interest column Days Interest

1 $ .2730726 $20 $40 220 $80 1 $ .2777778 $20 $40 $60 289
2 .5479452 74 147 221 203 2 .5656556 73 145 217 200
8 .8219178 75 148 222 294 3 .8333833 74 140 218 291
4 1.0058904 76 14B 223 295 4 75 147 219 202
5 1.8698630 77 150 224 200 70 148 220 293
1.6438856 78 151 225 297 5
o
7 1.3888880 77 140 221 294
7 1.9178082 70 162 220 298 1.0606607 78 150 222 295
8o
8 2.1917808 80 163 227 290 10 79 151 223 296
9 2.4057534 81 154 228 300 11 80 162 224 297
10 2.7397200 82 155 229 301 12 81 153 225 298
3.0180080 150 230 802 2.2222222 82 154 226 299
11 88 18 2.5000000
12 3.2876712 84 157 231 803 83 165 227 300
8.5610438 158 232 804 15 2.7777778 84 150 228 301
13 86 3.0555556
3.8350104 169 233 305 10 85 157 229 302
14 3.3333333
4.1095890 87 100 284 300 17 80 158 230 308
10 88 18
4.3835610 101 285 307 87 159 231 304
17 89 19 88
4.0575842 102 236 808 100 232 305
18 90 20 8.8888889 89
4.9316008 103 237 809 161 233 300
19 91 21 4.1060607 go
6.2054795 104 238 310 102 234 307
20 92 22 4.444444 91
5.4794621 106 239 311 163 235 308
21 98 23 4.7222222 92
5.7684247 100 240 312 105 230 809
22 94 24 5.0000000 93
0.0278973 167 241 813 100 237 810
23 25 5.2777778 94
0.8018099 90 108 242 314 167 238 811
24 26 6.5655656 95
0.5753425 97 100 243 815 108 289 312
25 27 5.8833333 90
6.8498151 98 170 244 316 109 240 313
20 28 97
7.1232877 99 171 245 317 170 241 314
27 29 98
7.8972003 100 172 246 318 171 242 316
28 2-17 30 99
7.6712829 101 173 319 6.3888889 100 172 243 810
29 248 320 31
7.9452065 102 174 6.6660007 101 173 244 817
30 249 321 82
8.2191781 103 175 6.944444 174 245 318
81 250 322 33 102
8.4931607 104 170 7.2222222 103 176 240 810
82 251 823 34
8.7671238 105 177 7.6000000 104 170 247 320
34 9.0410959 106 178 252 824 86 248 321
7.7777778 105 177
36 9.3150085 107 170 253 825 30 249 322
8.0556656 100 178
30 0.5800411 108 180 254 320 87 250 323
8.3333383 107 179
37 9.8630137 100 181 255 327 38 251 324
89 108 180
88 10.1869863 110 182 250 828 262 325
40 109 181
39 10.4100580 111 1B'd 257 320 253 320
41 8.8888880 110 182
40 10.0849315 112 184 258 330 254 327
42 9.1d6ßOd7 111 183
42 10.9589041 113 186 259 331 255 328
9.4444444 112 184
43 11.2328707 114 180 260 332 44 256 320
9.7222222 113 185
44 11.5068493 115 187 201 333 46 257 330
10.0000000 114 186
46 11.7808210 110 188 262 334 40 258 381
10.2777778 115 187
40 12.0547046 117 189 • 203 335 47 259 332
10.5555550 110 188
47 12.8287071 118 190 204 330 48 260 383
10.8333338 117 180
48 12.0027807 119 191 205 337 49 261 384
118 100
49 12.8707123 120 192 207 838 50 282 335
119 101
50 13.1500849 121 198 208 339 51 263 330
192

41
61 18.4240576 122 194 269 840 62 11.388880 120 108 204 337
62 13.6980301 123 195 270 341 53 11.0660607 121 194 206 338
53 13.9726027 124 IOU 271 342 54 122 196 266 839
54 14.2405753 125 197 272 343 56 123 197 267 340
66 14.620547B 126 108 278 344 57 12.2222222 124 198 208 841
50 14.7946206 127 199 274 345 58 12.5000000 125 190 269 342
67 15.0084032 128 200 275 340 59 12.7777778 126 200 270 343
68 15.3424058 129 201 270 347 00 13.0666550 127 201 271 345
59 15.0164384 130 202 277 348 61 13.3333333 128 202 272 840
00 15.8904111 131 203 278 349 02 129 203 273 347
01 16.1043830 192 204 270 850 03 130 204 274 348
02 10.4388502 188 205 280 361 131 276
an 64 18.8888880 205 349
03 10.7123288 134 200 281 05 132 300 270 860
363 14.1006607
04 10.0803014 13b 207 282 00 133 207 277 351
354 14.
05 17.2002740 186 208 283 07 134 208 278 862
355 14.7222222
67 17.5342480 137 200 284 08 136 209 279 353
210 360 15.0000000
68 17.8082192 138 280 09 137 210 280 354
211 367 15.2777776
18.0821018 139 280 70 138 211 281 366
70 212 358 15.5565550
18.8501644 140 287 71 139 212 282 350
71 213 369 15.8333333
18.6301370 141 288 72 140 213 283 357
72 214 300
18.0041090 142 289 214 284 358
73 215 301
19.1780822 143 200 215 285 350
10.4520548 144 210 291 302 10.388888B 144 286
863 216 300
19.7200274 145 217 292 16.6660007 287
20.0000000 140 218 364 288
219 806
17.2222222
17.5000000
17.7777778
18.0555566
18.3338833

18.8888889
10.1060067

19.7222222
20.0000000
for I Interest 1
V— COMPOüND AMOUNT 1
OF

1.0041 6667 1.0050 0000 1.0058 3333 1.0075 0000 1.0100 0000
1.0083 5069 ı.oıoo 2500 1.0117 0069 1.0150 5025 1.0201 0000
1.0125 5210 1.01ö0 7613 1.0170 0228 1.0220 0017 1.0303 0100
1.0167 7112 1,0201 5050 1.0236 3830 1.0303 3919 1.0400 0401
1.0210 0767 1.0252 6126 1.020ö 0894 1.0880 6073 1.0510 1005
1.0252 6187 1.0303 7761 1.0355 1440 1.04ö8 5224 1.0015 2015
1.0295 3379 1.0366 2940 1.0415 5400 1.0630 0613 1.0721 3635
1.0338 2352 1.0407 0704 1.0470 3004 1.001ö 9885 1.0828 5671
1.038! 3111 1.0460 1058 1.0637 4182 1,0695 8084 1.0936 8627
1.0424 5006 1.0511 4013 1.0698 8805 1.0775 8265 1.1040 2213
1.0408 0023 1.0603 9683 ı.oooo 7133 1.0856 0441 1.1156 0835
1.0611 0190 1.0610 7781 1.0722 9008 1.0038 0000 1.1208 2503
1,0556 4174 1.0060 8020 1-0785 4511 1.1020 1045 1.1380 9328
1,0599 3983 1.0723 2113 1.0848 3002 1.1102 7553 1.1494 7421
1.0043 6026 1.0770 8274 1.0911 0483 1.1186 0259 1.1600 0890
1.0887 9100 1.0830 7116 1.0975 2000 1.1260 0211 1.1726 7864
1.0732 4430 1.0884 8061 1.1030 3222 1.1364 4465 1,1848 0443
1.0777 1021 1.0030 2804 1.1103 7182 1.1439 6039 1.1961 4748
1.0822 0070 1.0093 0858 1.1168 4890 1.1625 4000 1.2081 0895
1.0887 1589 1.1048 0658 1.1233 0395 1.1611 8414 1.2201 0004
1.1104 2000 1.1299 1690 1.1098 9302 1.2323 9194
1.0012 4387
1.1169 7210 1.1306 0808 1.1780 8722 1.2447 1680
1.0057 9072
1.1215 6202 1.1431 8771 1.1875 0723 1.2571 6302
1.1003 5852
1.1271 5078 1.1408 0002 1.1984 1353 1.2607 3405
1.1049 4134
1.1327 9658 1.1565 1322 1.2053 8683 1.2824 3200
1.1005 4520
1.1384 6066 1.1632 5955 1.2144 2703 1.2062 5031
1,1141 6830
1.1441 5186 1.1700 4523 1.2235 8523 1.3082 0888
1.1188 1073
1.1498 7261 1.1708 7049 1.2327 1175 1.3212 0097
1.1234 7244
1.1560 2107 1.1837 3557 1.2419 6700 1.3345 0388
1.1281 5358
1.1014 0008 1.1000 4000 1.2612 7176 1.3478 4802
1.1328 5422
1,1072 0708 1.1075 8010 1,2006 6030 1.3618 2740
1.1875 7444
1.1730 4312 1.2045 7202 1.2701 1122 1.3749 4008
1.1423 1434
1.1780 0833 1.2116 9869 1.2796 3700 1.8880 0009
1.1470 7398
1.1848 0288 1.2180 0034 1.2802 3434 1.4026 7000
1.1618 5346
1.1007 2080 1.2257 7523 1.2989 0860 1.4160 0270
1.1666 6284
1.1014 7223 1.1908 8052 1,2320 2550 1.3086 4537
V— COMPOüND AMOUNT 1
87 1.1083 1170 1.2020 8303 1.2401 1706 1.3184 6021
88 1.1711 7133 1,2080 7726 1.2473 6107 1.3283 4808
89 1.1700 5121 1.2147 2083 1.2540 2789 1.3383 1128
1.4307 0878
1.1800 6142 1.2207 0424 1,2019 4056 1.3483 4801
1.4450 7847
41 1.1858 7206 1,2003 0701 1.3584 0128 1.4596 2724
42 1.1908 1319 1.2707 1220 1,3080 4960 1.4741 2251
48 1.1067 7491 1.2841 5900 1,3780 1450 1.4888 0373
1.2288 9821
44 1.2007 6731 1.2010 5002 1.3802 5642
1.2330 3270 1.5037 5237
45 1.2057 0046 1.209! 8525 1.3906 7584
1.2391 9780 1.6187 8980
48 1.2107 8440 1.2453 0385 1.3007 8383 1.4101 7341 1.6330 7779
47 1.2158 2940 1.2510 2082 1.3143 8662 1.4207 4071 1,5403 1767
48 1.2208 9530 1,3220 5388 1.4314 0633 1,6048 1075
1.2578 7892
50 1.2260 8242 1.2641 8882 1.3207 8680 1.4421 4087
1.6804 5885
1.2310 9068 1.2704 8916 1.3875 2283 1.4529 5003
1.5902 0344
1.2768 4181 1.0122 2808
1.2832 2581 1.0283 4834
1.6440 3182
26
COMPOUND 1
TABLE V — ΑΜΟΤΝΤ 0F

8 1%
COMPOUND 1
51 1.2302 2002 1.2800 4194 1.346,3 2504 1.4038 5411 1.8010 7814
52 1.2413 7114 1.2060 9016 1.3531 7277 1.4748 3301 .Ι.0770 8802
53 1.2406 4352 1.3025 7060 1.3010 0628 1.4858 9420 1.00"
1.2517 3745 1.3000 8346 1.3090 0583 1,4070 8847 0581
55
1.2509 5302 1.3150 2887 1.3709 9170 1,5082 0020 1.7114 1047
56
1.2021 0033 1.3222 0702 1.3850 2415 1.51Ω5 7825 1.7285 2457
1.2074 4946 1.3288 1805 1.3931 0340 1.5300 7509 1.7458 0082
οο 1.2727 3050 1.3354 0214 1.4012 2000 1.6424 5740 1.7032 6792
1.2780 3354 1.3421 3040 1.4094 0374 1.δΜ.0 2683 1.7800
01 1.2833 5868 1.3488 5016 1,4170 2526 1.6056 8103 1.7087
02 1.8655 0440 1.4258 0474
1.2887 οσοι 1.5774 2303 0961)
1.2940 7501 1.3023 7238 1.4342 1240 1.5802 6431 1.8100 9070
65 1.3001 &124 1.4425 7870
1.2094 0700 1.001Ι 7372 1.8348 0307
1.3048 8204 1.3700 3010 1.4500 9374 1.6131 8252 1.8532
07
1.3103 1905 1.3829 1031 1.4504 5787 1.6252 8130 12.30
88
οο 1.3157 7872 1.3898 2480 1.4079 7198 1.0374 7100 1.8717 4443
70 1.3212 0113 1.3007 7399 1.4765 3464 1.6497 6203 1.8904 0187
71 1.3267 8038 1.4037 5785 1.4851 4700 1.0021 2617 1.0093
72 1.3322 9458 1.4107 7664 1.4988 1102 1.0745 0111 ΟΜΟ
78 1.3378 4589 1.4178 3053 1.5025 2492 1.6871 5056 1,0284 0015
74 1.3484 2010 1.4249 1008 1.5112 8905 1.00θ8 0418 1.9477 4476
1.8490 1774 1.4320 4428 1.5201 0560 1.7125 5271 1.9072 2220
1.3546 3805 1.4392 0450 1.5280 7270 1.7253 0086 1.0808 9442
77 0052 1.5378 0170 1.7383 3733 2.οασ7 0337
1.3002 8298
78 1.4538 3252 1,5468 6283 1.7513 7480
1.3650 5082 2.0208 3100
79
1.3710 4220 1.4000 0069 1.5558 8020 1,7046 1017 2.0470 9931
80
1.3773 6748 1.4082 0519 1.5649 0220 1.7777 4400 2.0075 7031
81 1.3830 0645 1.4755 4622 1.5740 0115 1.7010 7708 2.0882 4001
82 1.3888 5035 1.4820 2395 1.5832 7884 1,8046 1015 2.1001 2847
88 1.3940 4627 1.4908 3857 1.5925 0010 1.8180 4808 2.1302 1075
85 1.4004 5729 1.4977 9026 1.6017 0874 1.8310 7031 2.1615 2196
1.4062 0253 1.5052 7921 1.6111 4257 1.8454 1001 2.1730 3717
86
1.4121 5209 1.5128 1.0205 4000 1.8502 δ7Ν 2.1047 0754
87
1.4180 3005 1.5208 1.6290 0405 1.8782 01 2.2107 1522
88
1.4280 1.δ27θ 7148 1.08θδ 0285 1,8872 5098 2.2388
80
90 1.4298 7764 1.5850 1134 1.6490 0012 1.9014 0586 8237
1.4358 8540 1.6482 8040 1.6586 8567 1,0160 6590 2. 2(512 7119
92 1.4418 1811 1.5610 0585 1.βθ83 0134 3339 2.2838
1.4-478 2568 1.5587 6087 1.6780 9344 1.044δ 0805 8.390
95 1.4638 5829 1.δ6βδ 1.6878 8282 1,9690 0240 2.8007
2274
οο 1.4δθ0 1003 1.5743 8746 1,0077 2830 1.9737 8585 2.3297 8007
1.4659 1.5822 5930 1.7070 3172 1,0885 8005
98 0902 1.7175 9290 2.3530
1.5901 7069 2.0085 0346
COMPOUND 1
100 1.4721 1,5081 21M 1.7270 1210 2.018E 2θ74 8787
0735 1.6001 1215 1.7376 8908 2.0380 6871 2.3700
1.4782 1.6141 4271 1.7478 2646 2.048θ 2123 1875
4113 1.6222 1342 1.7580 2211 2.0642 8814 2.4003
1.4844 1.6308 24- 1,7082 7724 2,0707 7030 8404
0047 49 1.778δ 0219 2.0Ωδ3 0858 8870
1.4θ0δ 8647 1.6384 7011 1.7889 6731 2.1110 8384 2.4480
1.4967 1.64ββ 6849 3207
9024 2.4731
1.5030 1000
8289 2.4978
1.5092 5019
0558 2.5228
1.5156 2800
8420 2.5480
5008
2.5736
3755
2.50Ω2 7293
2.(12δ2
2.β6Ιδ 1831
2.6780
3340
2.7048
1883
26
ΤΑΒΙ,Έ V— COMPOOTD AMOUNT 1
0F

1%

540 0183 1.7994 0295 2.1260 1097 2.7318 β1θ7


β3Ι 7034 1.8098 0947 2.1428 8885 2.7δθ1 8059
714 9223 1.8204 6722 2.1589 2.7807 7239
798 1.8310 7056 2.1761 3242 2.8146 4012
82 4894 1.8417 6783 2.1914 4501 2.8427 8062
β0 0018 1.8526 0142 2.2078 8175 2.8712 1438
51 7363 1.8033 0768 2.2244 4087 2.8θθ9 2658
36 1.8741 7097 2.2411 2417 2.9289 2679
22 8800 1.8861 0907 2.2670 3200 2.9582 1505
08 7934 1.8061 0014 2.2748 6710 2.9877 θ72Ο
θδ 3373 1.9071 607.0 2.2919 2860 3,0170 7517
82 3140 1.9182 0100 2.3091 1807 3.0478 5192
0θ 7256 1.9204 8194 2.3204 3&45 3.0783 3044
57 5742 1.0407 3725 2.3438 8472 3.1091 1376
45 8621 1.0520 5822 2.8614 6380 8.1402 0489
34 5914 1.9034 4522 2.3791 7484 8.1710 0098
28 7844 1.9748 9805 2.8θ70 1866 3.2033 2300
13 3832 1.9804 1800 2,4140 9029 3.2353 5828
08 4501 1.0θ80 0634 2.4831 0878 8.2677 0080
θ3 9073 2.0090 0188 2.4δΙ3 6708 8.3003 8689
84 0372 2.0213 8440 2.4097 4220 8.8333 0070
78 3010 2.0331 7681 2.4882 6532 8.8607 2407
08 2437 2.0460 2.60βθ 2731 8.4008 0192
00 6840 2.0δ0θ 0538 2.5267 2927 3,4343 0584
53 8878 2.008θ 6434 2.5446 7224 3.4087 3980
40 0648 2.0810 3330 2.5637 5728 3.5034 2710
40 3880 2.0θ31 7266 2.682θ 3.6384 6147
34 2.1063 8284 2.0023 5786 8.6738 4008
29 2029 2.1176 2.β2Ι8 7553 3.6095 8464
24 4002 2.1300 1728 2.6416 8.6466 8030
20 0318 2.1424 4238 2.0613 5115 8.0821 3719
10 1314 • 2.1549 2.8813 1128 9.7180 6856
12 7121 2.1675 1044 2.7014 2112 3.7501 4816
Οθ 7757 2.1801 6425 2.7216 8177 9.7037 0908
07 3245 2.1928 7182 2.7420 8.8310 4073
05 8012 2.2056 0357 2.7020 0000 3.80θ0 0319
03 8880 2.2185 2004 2.7833 8005 3.9080 0282

49
ΤΑΒΙ,Έ V— COMPOOTD AMOUNT 1

02 9074 2.2314 7137 2.8042 6640 8.9477 4046


02 4210 2.2444 8828 2.8282 8731 3.9872 2βθδ
02 4340 2.2575 8118 2.8484 7097 4.0270 0022
02 9402 2.2707 6030 2.8078 2554 4.0073 7021
03 2.2830 0640 2.8803 4.1080.48θ1
05 4808 2.2073 1071 2.9110 0424 4.1491 2435
07 5082 2.3107 2074 2.9328 3077 4.1900 1550
10 0457 2.3241 2.θΜ8 3305 4.2826 2175
13 2.9877 5778 2,9709 9430 4.2748 4097
Ι6 0014 2.8613 9470 2.0θ93 2175 4.3176 9544
Ω 7447 2.3061 1117 3.0218 1067 4.3607 7130
25 3484 2.3789 0705 3.044-4 8020 4.4043 7910
80 4752 2.3927 8401 3,0673 1389 4.4484 22θ0

50
COMPOUND 1
1*70 1*70 1*70
2%

2 1.0112 6000 1.012δ 1.01δ0 0000 1.0175 0000 1.0200 0000


1.022β 2656 1.0251 5β25 1.0302 2600 1.035,3 0625 1.0404 0000
ε 1.084Ι 8111 1.037θ 7070 1.045U 7838 1,0534 2411 1,0612 ouo
5 Ι.Ο4δ7 6509 1.0509 *534 1.0βΙ8 0855 1.0718 6903 1.082,1 3210
a 1.0δ76 2994 1.0640 8215 1.0772 8400 ι.οοου 8080
7 1.0θ94 2716 1.0773 8318 1.0934 4320 1.1097 0235 1.1201 0242
1.12ΩΙ 2215
8 1.0814 5821 1,0908 5047 1.1098 4-
1.1488 8178
8507
1.0938 2482 1,1044 8610 4Ω1 1.17ΙΒ 5938
9 1.10δθ 2789 1.1264 9250 1.1080 8721 Ι.1Ω5Θ 0257
1.1182 9218
10 1.1183 6958 1.1433 89θ8 1.18M 1.218D 9142
1.1322 7083
1.1605 4088 1.243.3 7431
1.130θ 51U 1.2102 5077
12 1.1464 2422 1.2082 4170
1.1436 74- 1.1779 4894 1.2314 8031
18 1.1607 6452 1.20.30
4-4 1.1060 1817 1.2629 8950
1.1752 8895 1.3104 7870
14 1.1565 4078 1.1899 6475 1.2185 5244 1.2710 1082
1.3458 0834
15 1.1βθ5 5186 1.2317 5573 1.2972 2780
1.2048 2θ18 1.8727 8571
18 1.1827 0932 1.2502 3207 1.3109 2935
1.2108 8955 1.4002 4142
17 1.1980 1480 1.2080 8556 1.8430 2811
1.2351 8817 1.4282 4025
18 1.2004 8907 1.2880 2083 8111
1.2505 7739 1.4508 1117
1.2280 7660 1.3073 4004 1.3004 4540
19 1.2662
1.4147
1.4860 4740
20 1.2368 3611 1.2820 8728 1.3209 5076
78201 1.5168 0834
1.2607 5052 1.3408 5601 Ι.Μ59 7007
21 1.2980
1.2648 2146 1.3142 8848 1.8670 5783 1.4305 3681 1.57θ8 0020
22 1.2720 5071 1.8875 0370 1.4847 2871 1.6084 3725
28 1.8307 1709
1.2934 4003 1.3478 5105 1.4083 7715 1.4903 1.6400
26 1.8079 9123 1.4296 0281 1.5164 4270
1.8641 9294 1.0734 1811
1.8227 0618 1.4509 4635 1.6422 8054
28 1.8812 4695 1.7088 8048
27 1.8876 8657 1.4727 0053 ι.5β0θ 8209 1.7410 2421
1.8985 1092
1.8526 3442 1.5974 5739 1.7758 4409
29 1.4159 9280 1.4948 0018
1.3678 5156 1.5172 2218 1.6264 1200 1.8118 0168
80 1.4388 9221
1.8882 3989 8051 1.8538 5702
1.4δΙθ 1886 1,8475 8882
81 1.8088 0184 1.5630 8022 1.6828 0013
1.4697 5853 1.8845 4050
32 1.4145 3786 1,7122 4913 1.9222 3140
1.4881 8051 1.5805 2842
88 1.4304 5140 1.7422 1340 1.οαοιι 7003
1.5087 3214 1.0103 2432
1.4485 4898 1.6344 7018 1.7727 0223 1.θθ08 8055
1.5255
1.4828 1760 1.64-46 8587 1.6580 9887 1.8037 2452
1.8362 8970 2.03θ8 8734
1.4792 7480 1.5089 4882 1.6888 8182 2.0800 8600
88 1618 1.δ8Μ θ8Ι2 1.70θ1 8954 1.8074 0727 2.1222 Ω87θ
89 1.5127 4519 1.6032 8678 1.7347 7ββ3 1.0000 8680 2.11547 4477
40 1.5297 8357 1.6283 2787 1.7607 9828 1.9333 3841 2.2080
7841 1.Μ8β Ιθ4θ 1.7872 1025 1,9071 7184
41 2,0015 0784
2.2522
1.5643 7687 1.8641 8471 1.8140 1841 2.2972 4447
COMPOUND 1
12 1.5819 7811 1.6849 0677 1.8412 2888 2.080C 2630 2.3431 8936
48 7834 1.7060 2885 1.8888 4712 2.0722 0024 2,3000 5314
1.6177 7079 1.7273 6421 1.8θβ8 7982 2.1085 8000 2.4378 5421
1.8869 7071 1.7489 4614 1.9253 3302 2.1454 3010
2,4800 1120
1.6M3 7588 Ι.9Μ2 1301 2.1829 7622
1.7708 0797 2.6303 4351
1.6729 8710 1.7929 4808 1.0836 2021 2.2211 7728 2.5870 7039
48
1.8918 0821 1.8153 5485 2.0182 7910 2.2000 4789 2.8.388 1170
40
1.7108 4105 1.8380 4879 2.0434 7820 2.2996 9872 8803
1.7300 8801 1.8610 2237 2.0741 3046 2.8398 4170
1.7496 2.1052 4242 2.3807 8898
ΤΑΒΙ,Ε V ΑΜΟΤΝΤ
ΤΑΒΙ,Έ V— COMPOOTD AMOUNT 1
0F
1*70
2%

42 8615 2.1808 2100 2.924 2.7454 1979


78 3872 2.1088 7337 5274 2.8003 2819
16 8670 2.2014 0647 2.4648 4506 2.8503 3476
58 3279 2.2344 2757 2.5070 80116 2.9134 0144
02 8070 2.2679 4398 2.5618 7012 2.9717 3067
50 3420 2.3010 6314 2.59β5 2786 8.0311
00 2.3364 9259 2.6419 6708 3.0017 88δθ
5" 2.8716 3098 2.6882 0151 3.1530 2436
5 2.4071 1308 2.7352 4503 3.21ββ ρσ8δ
2.4432 1978 2.7831 1182 3.2810 3070
11 8070
71 8135 2.4708 6807 2.8318 1628 8.34θβ 5140
85 2111 2.5170 2.8813 7306 3.4135 8443
01 0018 2.6548 2208 2.0817 9709 3.4818 5612
71 9250 2.5031 4442 2.9831 0854 8.5514 9324
45 3241 2.6320 4168 3,0348 0785 3.6225 2311
22 1407 2.6715 2221 3.0884 2574 8.βθ49 7357
2.7115 9504 3.1424 7310 3.7088 7304
02 4174
2.7622 0898 3.1074 0647 3.8442 6050
86
2.7θ3δ 5300 3.2534 2213 3.θ211 3561
73 5251
2.8864 5629 8.8103 5702 3.θθθδ 5822
64
58 θθθ7 2.8770 8814 3.3682 8827 4.0795 4930
2.9211 5706 3.4272 3381 4.1811 4038
57 2372 2.Ωβ4θ 7533
59 2027 3.4872 0900 4.2443 0318
8.00M 4996 4.8292 6045
04 9427 3.064δ 9171 3.5482 8607
74 6045 3.6103 8020 4.4168 3646
87 0358 3.1004 10δθ 3.θ73δ 10θ8 4.6041 5216
8.1469 1674
05 2860 8.7377 9742 4.5042 3521
8.1941 2060
26 8011 8.8032 0888 4.6861 1021
8 2420 3230
51 0336 812006 8279 3.8697 6603 4.7798 4281
81 3327 3.9874 8502 4.8754 8916
8.3400 2273 4.00β8 9192
14 8404 4.072θ 4794
3.3001 2307
52 5350 3.4400 7492 4.0785 0378 δ.0724
04 4417 . 8.4θ2δ 8054 4.1478 4260 5.1788
40 6222 8,5449 7838 4.2204 2984 6504
91 1300 4.2942 8737 5.2778 3214
4β 01θΙ 3.6081 4,3694 87*0 5.8828
8.6521 2δ3δ 7878
10δ 3,7069 0728 4.4450 0265

53
V— COMPOüND AMOUNT 1
4 8.7625 1084 4.5237 0584 5.49Οδ
69 3.8189 4851 4,6028 7070 8636
2 3.8762 8278 4.6834 2003 4708
37 5267 8080 5.7128
8.9343 7022
10 4048 3.0θ88 9187 4.8487 5402
88 1260 4.0532 0275 4.9336 2863 5.8266
70 4776 4.1140 9214 δ.ΟΙ9Ο 0110
867 8703 5.1078 6.9431
4.1758 0352 5.1072 0324 3313
5 4.2884 4057
49 5786 4.3020 1718 5.2881 5429 6.0619
46 4483 4.8ΒβΙ 4744 .δ.380β 9670
M8 2780 4.4820 4565 6.4748 6.1832
9δδ 1824 6019 8670
367 5.5706 0042
6 6923 8,4330
δ,ββ81 δδΗ 3843
784
0
206 β.β92θ
0 3318
8.8267
684 θ184
7 6.2633
2708
7.1026
9423
7.2446
4612

ΤΑΠΕ

η 3%

1.0225 0000 1.0250 1.0275 0000 1.0300 όοοο 1.0350


1 1.0455 0625 0000 1.0657 Ι.οαοο 0000 0000
2 1.0690 8014 1.050-6 2500 1.0M7 8055 1.0027 2700 1.0712
3 1.0930 8332 1.07β8 1.1140 2120 1.1255 0881 2500
δ 1.1170 7760 1.1038 1.1452 7384 1.1502 7407 1,1087
6 1289 1.1040 5230 1788
1.1428 2544 1.1707 6830
1.1314 1.2208 7387 1.1475
8 1.1685 8001 1.2091 2040
0821 1.2607 7008 2300
9 1.1048 3114 1.2423 8056
1.1500 1.8047 7318 1.187U 8631
10 1.2217 1484 1.2706 4602
1.2402 0348 9942 1.3116 5108 1.3439 1038 1,2202
11 1.2773 1050
1.1886
1.3477 2144 1.8842 3387 5533
12 8575 1.4257 6080 1.2722
1.8060 4990 1.8847 8378

54
ΤΑΒΙ,Έ V— COMPOOTD AMOUNT 1
13 1.8354 8011 1.2184 1.4228 0633 1.4686 3871 7926
14 1.8654 8343 0290 1.4019 9413 1.5125 8972 1.8188
1.8962 0680 1.2488 1.5021 9890 1,6670 6742 0004
16 1.4276 2146 0207 1.5435 0944 1.6047 0644 1.3028
17 1.4697 4294 1.2800 1.5859 5505 1.6528 4703 0735
18 1.4925 8454 1.6295 6973 1.7024 3300 1.4105
19 8716 Ι.52βΙ 1.8120 18748 8200 1.7635 0006 9870
20 7037 Ι 8448 1.7204 2843 1.8061 1123 1.4590
1.5β0δ 8882 0072
21 0920 ι•.878δ 1.7677 4021 - 1.8602 0457
22 1.81β3 5307 1.01β1 0341 1.5110
1.5956 2086 1104 0860
23 1.4129 1.8ββ3 0278 1.0735 8051
u 1.6315 2212 1.9176 2610 2.0327 9411 1.5439
1.8882 3137 7882
1.4482 1.9703 6082 2.0037 7793 1.8186
20 Ι.7Οδ7 ββ58
9817 0452
1.7441 4632 2.024E 4575 2.1505 0127
1.0768
29 1.4845 2.0802 2075 2.2212 8001
1.7838 8062 4883
80 0562 2.1874 2082 2.2879 2708
1.8235 1588 1.7389
81 1.8645 4499 1.52Ιβ 1826 2.1962 0600 2.3δβδ 6551
82 1.5506 2.2566 0173 2.4272 6247 8604
1.9064 9725 1.7046
83 1.9493 6872 2.3186 5828 2.5000 8035
M 1.5986 7565
1.9082 6479 2.3824 2138 2.5750 8276
85 1.6386 1.8574
2,0881 0303 2.4479 3797 2.6523 3524
1644 8020
86 2.0889 6034 2.5152 5620 2.731θ ΟΝΟ
1.9225
87 2.1808 4945 1.8796 2.6844 2581 2.8138 6245
0132
88 2.1787 9356 8185 2.6554 0752 2.8982 7833 1.08θ7
1.7215 2.7285 2370 2.08δ2 2608 8886
2.2278 1842 7140
2.2779 4229 2.8035 5810 3.0747 8348
1.7Μθ 1068 2.0504
42 2.3291 9699 2.8806 5505 8.1670 2008
1.8087 8147
43 2.3810 0290 2.9508 7899 3.2620 8779
2595 2.1315
Μ85Ι 8897 3.0412 7052 3.3598 0803 1158
1.8δ3θ 4410
2.4809 8072 3.1240 8.4β0β 2.2061 14-
2.δ4β0 0528 1.9002 8,2108 4036 3.5Μδ 1677 48
9.6032 9040 9270 8,2901 8847 8.6714 5227 2.2833
2.6018 "44 1.9478 8.8898 6478 8.7815 9584 2849
50 0002 2.3632
2.7217 5639 3.4830 3.8θδ0 4372
1.9964 4408
2.7829 3,5788 7098 4.0118 9508
9502
2.8466 1831 8.8772 8088 4.1322 5188 2.4450
2.0464
2.00θβ 3201 3.7784 1535 4.2682 1944 5856
0739
2.9751 3.8823 2177 4.3839 0602 2.53Ιδ 6711
2.0θ7δ β758
ΟβδΟ 2.6201
2.1600 0677 7106
3.090 2.2037 5βθ4 2.7118
4640 2.2588 5086 7708
2.8153 2213 2.8087
2.3732 9370

55
V— COMPOüND AMOUNT 1
2.4325 3582 2.0050
2.4938 4870 8148
2.5556 8242 8.00β7
2.6195 7448 0750
2.6850 6884 3.1110
4286
2.7521 9043
3.2208
2.8209 9520
6033
2.891δ 2008
3.8885
2.9838 0808
9045
3.037θ 0328
3.4502
3.1138
ββ11
5086
3.5710 25-
8.1916
43
9713 3.βθΟΟ 1132
3.2714 3.8253
8956 7171
8.3532 3,0502
7880 5972
3.4371
0872 4.0θ78
8381
4.2412
4.3807
0202
4.5483
4160
4.7023
5855
4.8609
4110
5,0372
8404
6.2135
8898
5.3960
6459
5.5849
2680

56
ΤΑΒΙ,Έ V— COMPOOTD AMOUNT 1
0F
*70

230 3.0890 4.5154 2320 5.7803 0030


4 8502 4.6508 8500 5.0827 1327
111 4.ΩΙ)87 8547 4.7004 1247 0.1921 0824
5 4.2115 0208 4.9341 2485 8.4088 3202
013 οοια 4.3273 6.0821 4869 θ.0331
030 1838 0.8063 0108
5.2340 1305
1 4.4403 7.1065 8062
5.3016 5144
887 1064 7.3M2 8215
5.5634 0008
3 4.5685 6.7200 0301 7.611" 8203
850 0343 5.8010 0310 7.8780 0000
0 4.0042 2075 8.0083 5120 8.1538 2408
860 4.8233 2107 0.2604 0173 8.4302 0703
7 4.9659 0230
δ.Οθ22 513β 6.4879 1370 8.7345 8020
877 0.6310 5120 9.0402
2 5.2322 8827 6.8290 8273 0.3507 0008
024 5.3761 7020
0 5.5240 2105 7.0348 8222 9.6841 8620
997 5.6759 3182 7.2450 2808 10.0231 3168
5 5.8320 1074 7.4033 0054 10.373Ω 412Ω
7.8872 0574 10.7370 2024
007 6.0924 11.1128 2626
7.9178 2101
0 0020
0.1671 0130 8,1553 5657 11.5017 7414
226
0.3205 1400 8.4000 1727 11.0043 3024
0
0.5004 8.6520 1778 12.8200 8801
380
θ.β702 5076 8.0115 7832 12.7522 2250
3
0,1780 2507 13.1985 5038
565 4- 0.8020 3632
7.0510 0.4542 9344 13.0604
770 7,2465 8791 9.7370 2224 ΩΩΜ
β 7.4448 4158 14.1380 1713
024 7.04θδ 7472 10.0300 14.6334 6873
10.3300 0171 15.1450 4013
1 7.8509 3802 10.0408 005β 15.6757 3754
290 8.0700 8032
Ω 8.2981 7809 10.0601 1727 10.2243
007 8.5203 7801 11.2880 2070 88,95
8 8.7008 5402 11.6275 8842 10.7922 4195
047 • ΙΙ.Ω7β4 1607 17.3790 7041
9,0017 7751
0 12.3357 0866 17.9882
9.2493 2030
321 0,5030 8288 12.7057 7981 θΩ38
6 13.0800 5320 18.6178 5881

57
ΤΑΒΙ,Έ V— COMPOOTD AMOUNT 1
729 0.7050 3414 13,4705 0180 ΙΩ.2βΟ4 8387
3 10.043%) 1580
10.033δ 13.8830 4805
172 20.041Ω 5285
7258 14.3004 6711 21.3044 2120
6
βδ1 14.7204 8112 22.1121 7605
6
107 15.1713 22,8801 0210
10.3004
3 Ιδ.β2β5 0652 23.6871 1508
0683 10,0063 0172
722 24.5101 8473
10.5Ω3Ο 10.5781 6077 25.3742 304')
3 10.884.3 1406 20.2023 2850
315 11.1836
1 3331 27.1815 1000
17,0765 0650 28.1328 0201
048 11.4011 8322
17.5877 7070 2Ω.1Ι7Π 1311
3
11.8071 9076 18.1154 0388 30.1360 2007
021
12.1318 18.0588 31.1θΙ4 0798
4
37 2470 8861 10.2180 3198
005 12.4055
2 1644
12.8083
898
1711
1
13,1005
745
4584
9
039 13.6224
θ 0085
580 13.8Ω43 2862
0 14,2784
00 2255
4 2417
15.0724
008 2234
4
ΩΩ 1065
841
2
037
8
288
3
505
4
ao 0718
384
880
8

58
ΤΑΒΙ,Έ V— COMPOOTD AMOUNT 1
Ιβ 0385
020

702

444
0
255

137

59
COMPOUND AMOUNT 1
V— 0F
(1 + δη
η
6%

60
COMPOUND AMOUNT 1
1.04ΟΟ 0000 1.0450 1.0500 Ι.0δ50 0000 1.0600
1.0818 0000 0000 0000 1.1130 2500 0000
1.1248 6400 1.0920 1.1025 1.1742 1.1238
1.1008 5856 2600 0000 4138 0000
1.2166 5290 1.1411 1.Ιδ7β 1.2388 1.1910
1.2653 1902 6613 2500 2405 1600
1.3169 8178 1.1925 1.2155 1.3000 1.2824
1.3685 1860 0625 6001 7606
1.4233 1181 1.2461 1.2762 1.8788 1.8382
1.4802 4428 8104 8150 4281 2558
1.6394 5406 1.3022 1.3400 1.4546 1,4185
1.0010 3222 6012 1.4071 7916 1911
7851 1.3608 0042 1.5846 1,5036
1.7316 7645 6183 1.4774 8651 3026
1.01θΟ
1.8009 4351 1.4221 6544 1,5988
0427
1.8729 8125 0061 1.5613 1.7081 4807
1.9479 0060 1.4860 2822 4446 1.6804
2.02δ8 1852 0514 1.6288 7896
1.5529 9463 1.8020
2.1008 4918 1.7908
0240
2.1911 2314 1.8228 1.7103 4770
1.0012
2.2787 6807 5306 8986 0749 1.8082
2,3699 1879 1.6958 1.7058 2.0057 9856
2,4647 1554 8143 5683 7300 2.0121
2,5683 0416 1.7721 1.88δθ 2.1180 0647
2.6658 3633 1.8519 4914 9146 2.1329
4402 1.070θ 2.2824 2826
2.7724 θθ78 1.9852 3100 7640 2.2809
2.8833 6868 8244 2.078θ 0306
2.9987 0332 2818 2.9552
2.0228 2.8θβδ
3.1186 δ14δ 0270
7015 2,1828 5819
3.2438 9751 2.4848
2.1133 7450 0215 2.5403
3.3781 3341 7681 2.2020 δ1β8
8.5080 5876 2.02Ι4
2.2084 1832 6627 2,6027
3.6483 8110 7877 2.40θβ 7279
8.7948 1634 2.7650
2.3078 1923 4601 2.8648
8.9460 8800 6031 2.5209 8θΙδ
2.9177
4.1089 3265 2.4117 5020 5740 3.0255
4.2880 1402 2.6582 9050
9771 3.0782
4.988 2.5202 3415
8.2071
1346 4116 2.7850 8547
3.2476
4.β1β3 2.β33β 6250 , 8703 8.3095
4.8010 2063 5201 2.θ2δ2 3.4261 8.0035
2.7521 6072 5157 8742
4.9930 6145
2.8760 8.0715 3.6145 3.8197
5.1927 8391
1383 2376 8990 4,0489
5.4004
8.00Κ 3.2250 3.8188 3464
9527
61
ΤΑΒΙ,Έ V— COMPOOTD AMOUNT 1
ΟΡ

η
ΤΑΒΙ,Έ V— COMPOOTD AMOUNT 1
51 7.3909 5068 9.4391 0490 12.0407 15.8417 10.5258
52 7.β8θ6 8871 9.8838 0403 0978 6007 6363
53 7.9040 5220 10.3077 3863 12.6428 16.1855 20.β9β8
54 8.8138 1436 10.7715 8077 0820 0037 8534
55 8.6463 0692 11.2δβ3 0817 13.2749 17.07δ7 21.9388
56 8.9022 2160 11.7028 4204 4868 7252 9840
57 9.8519 1040 12.2021 13.9886 18,0149 23.2550
58 9.7259 12.8453 1758 14.6350 4001 2037
50 8088 13.4238 6087 3002 10.00δ7 24 0503
10.1150 14.0274 0703 15.8674 0171 2159
2035 ιο.δΙ0β 1240 20.0510 26.1203
61 14.8586 4129
2741
62 15.3182 8014 Ιβ.13β7 8309 7860 4089
03 10.0404 Ιβ.007β 0276 16,0425 21.1538 27.βθ71 0134
M 1250 10.7279 4487 7224 8703 29.8580
85 11.3780 17.4807 0280 17.7897 22.3173 2742
M 2000 18.2073 8400 0085 5176 31.1204
11,8331 10.0803 6408 18.6791 23.6448 6307
68 5010 8589 0611 32.9878
10.9483 8641
12.3004 20.8430 0276 19.6131 24.8307 0085
7617 21.7841 8558 4519 7045 34.9809 5230
71
12.7087 20.δθ88 0245 26.2050 87.0649 6944
72 22.7644 2168
3622 21,0234 6782 39 2888 0761
78 28.7888 2008
13.3106 24.8693 1759 0257 27.6472 4110481
75
8403 25.077D 8688 22.7046 8550 44.1449 7105
77 13.8431 27.1460 0629 0720 20.1678 40.7938 6094
78 1201 23.830θ 8820 40.6012 0014
28.3686 1112
70 14.8068 0066 80.7721 52.5773 0755
20.6461 0802
80 3649 25,0318 1004 56.7320 0900
30.9702 3266.
14.9727 26.2834 9037 82.4645 50.076θ 3018
81 32.3782 9802
15.5716 27.5976 6488 34.2501 3880
82 88.8300 θβ43 a2.0204 8509
1835 28.0775 4813 30.1838 0β43 00.3777 1616
83 35.3524 6077
84 16.1044 30.4264 2554 38,1212 70.8608 7806
36.0488
85 8308 31.9477 4681 0074 74.6820 0074
38.6067 0006
16.8422 38.6451 3415 40.2170 8008 70.0589 2079
86 40.3430 1926
6241 36.2223 0086 42.42θθ 1023 83.8008 3808
87 42.1684 5513
17.51δθ 36.9836 1040 44.7635 6163 88.8283 5020
88 δ2θΟ 44.055δ 8501
38.8326 8692 47.2255 94.1680 5757
89 18.2165 46.08Μ 6761
40.7748 2022 99.8076 4102
0102 48.1008 0087 40.8220 105.7969 9848
91 18.0452 60.2747 4101 42.8130 3023 6818
92 6460 52.5371 ΟΝΟ 44.053β δ2.δβ32 112.1437
98 8804 2015 118.8723 7828
10.7080 54,0012 7508
47.2018 7244 δδ.4542 12β.0047 2097
67.3718 3241
ΤΑΒΙ,Έ V— COMPOOTD AMOUNT 1
5 6486 50.053δ 6487 4107 0859 133.δβ50 0423
M 20.4911 8744 62.βδΙ4 7629 62.03θ5 .13Ι 58.Μ41 141.5789
7 21.3108 3404 86.4707 9108 2 Μ.β414 8479 150.078β 3876
8 22.1032 6834 08.41β0 7730 8878 βΙ.7219 159,0780 5708
23.0407 9007 71.4067 4128 67.3735 1495 108.6227 4060
0 6322 66.1166
23.9717 0108 74.7130 4964 002422 4138 178.7401 0403
2027
24.9306 6207 78.07δΙ 8687 03.2δ48 5344 180.4645 1123
68.βθ80 3439
26.9278 8018 81.588δ 1803 200.8823 8100
σσ.4170 7112 72.4764 2628
20.0060 0476 212.8823 2482
28.043β 0494 60,7370 2467 76.4028 2973
73.2248 2001 226.βδβ2
808880 7486
20.1053 4014 70.8880 6105 239.1θ4δ 8017
85.1048 1845
30.3310 6310 80.7803 0605 80.7865 8347 268.5462 5498
31.5452 4183 94.7237 268.7890 8028
84.7a08 8330
32.8070 5120 284.8846 7200
89,0052 2747 ορ.983δ 0004
34.1193 3334 801.9776 4642
93.4654 8884 105.4209 4698
35.4841 08.1282 0328 111.2285 9407 320.0θθ8
Οββ8 103.034β 117.3461 0674 330,8020 8351
30.0034 7004 7645 123.8002 οσοι
38.37θβ 0078
108.1884 130.8002 1724
3θ.θ147 M17
1027 187.7927 2410
41.5113 8504 145,8718
113.δ9δ7
43.1718 4138 3078 2402
44,8087 1508 119.2755 163.3667
4θ.θθ4θ 1732 4684 ισι.80Ιθ
6,363 1791
125.2302
48.5624 5018 9319 170.7010 2840
50.604θ 4818 181.5012 180.08θδ 7980
5785 189.9016 0857
200,4442 0443
211.4686 8687
88
ΤΑΒΙ,Ε AMOUNT 1
V— 0F
ΤΑΒΙ,Ε AMOUNT 1
Ι 0050 0000 1.0700 0000 1.0750 0000 1.08Ω0 0000 1.0850
1 111342 2500 1.1440 0000 1.1556 2500 1.1004 0000 0000
2 1.2079 4963 1.2250 4300 1.2422 0688 1.2597 1200 1.1772
8 1.294 . 1.3107 1.3354 1.8004 8806 2500
1.3700 1.4025 5173 1.4350 2033 1.460,3 2808 1.2772 8013
_8θ60 1.5ά]7 3035 1.5433 0153 1.5808 7432 1.3858
7 1.4501 4230 1.6057 8148 1.6590 4014 1.7138 2427 6870
8 1.5539 8655 1.7181 8818 1.7834 7783 1.85ΟΩ 3021 1.5030
Ι.ΟΩ9Ο Ι.α.3Ι4 6751
ιο 1.0540 9507 1.8384 5921 Ι.θΙ72 3866 0463 2.1580
1.7625 7039 1.Ωθ7Ι 5136 2.0010 3150 1.7701
11 2500 4225
1.8771 3747 2.215C 0803
18 2.1048 5195 2. 3.31a 3000 Ι.Ω20β
Ι.θ9Ο1 5140 2.2521 2.3817 7000 0434 2.0838
14 2.5181 7012
2.1290 9024 2.4098 4500 2.5βΜ 1307 5571
2.7100 2373
Ιδ 2.2674 8750 2.5785 3415 2.7524 4405 2.2800 8344
2.Ω37Ι 0302
16 2.4148 7418 2.7590 3154 2.9588 7785
3.1721 2,4531
17 2.5718 4101 3.1807 0315
2.θδ2Ι 0375 8.425Ω 4264 6703
18 2.7390 1067 3.4103 δ2β4
3.1588 1521 8.7000 1805 2.6610
19 2.9170 4637 3.6758 0409
3.37θθ 8228 3.00θ0 1050 802,3
3.10ββ 6488 3.θΙβ5 2754 8.9614 8040 4.3157 0106 2.8879
21 8.308δ 8691 4.2478 5110
3.8696 8446 5714 3,1384
23 8.5236 4506 4.5664 3993 0357
4.1405 6287 5.0338 3372
25 8.7526 8199 4.9089 2203 5.4365 4041 3.3007
0174
3.9966 0682 5.2770 9215 5.8714 0365 4288
28 4.7405 2986
27 4.2563 8573 5.072.3 5.6728 7400 6.8411 8074 3.8887
4.5330 5081 6.0083 3061 6.8484 7520 2102
28 5.974 32M
20 4.8276 9911 6.6557 1508 7.9983 5321 4.0022
5.8078 6292 0231
30 5.1414 7.0478 θ371 7.0880 6147
6.2138 6783 4.3424
5.4756 9702 7.5759 4824 8.6271 0039
81 6.6488 8836
8.1441 4436 6401
5.8316 1738 7.1142 5705 0.3172 7490
32 8,7549 5510 4.7115
6.2106 7245 7.6122 5504 10.002θ 5080
88 0325
6.6143 9.4115 7683 10.8076 0θ44
35 7.04-42 9θ9θ 8.1451 1290 10.1174 4500 5.1120
11.7870 8800 4β12
80 7.5021 7946 8.7152 7080 10.8762 5347 12.07βΩ 4064
87 7.9898 2118 9.3258 3975 ΙΙ.β019 7248 5.5485
13.0θ0Ι 3361
88 8.5001 5950 9,0781 1364 12.5688 7042 7005
14.7863 442θ
89 9.0β22 6487 10.6765 8148 2850
13.5115 3570 15.068Ι 7184
40 9.6518 0148 11.423θ 4210 14.5240 0088 0.5205
17.2450 2558 6002
10.2786 8603 12.2230 1814 15.6142 8844
41 18.6252 7563 7.0&45 7360
10.9467 4787 13.07W 7141 10.7853 3858
42 20.1152 Ω7β8 7.6887
11.6582 8595 13.9948 2041 18.0442 3897
43 21.7245 2150 6280
12.4160 7463 14.07" 19.3975 5689
45 23.4624 8322 8.8401
18.2231 1938 5784 20.8523 7306 25.3304 8187
40 3716
14.082θ 2214 16.022β 22.4103 27.3666 4042
47 0.04Ω0 4881
14.9979 9258 17.1442 5078 24.0θ75 2431 29.5559 7160
ΤΑΒΙ,Ε AMOUNT 1
48 15.9728 6209 18.3443 5475 25.9048 3803 31.9204 4930 0,8182
50 17.0110 9818 19.6284 5959 27.8477 0153 34.4740 8534 1790
18.1168 1951 21.0024 5176 20.9362 7915 87.2320 1217 10,0527
19.2944 1278 22.4728 2388 82.1815 40.2105 7314 11.5582
20.5485 4961 24.0457 0702 34.5951 1259 43.4274 1809 12.6407
21.8842 0633 25.7289 0651 37.1807 4803 46.9016 1251 0303
.28.3Μ6 7868 27.6299 29θ7 13.6000
29.4570 2506 0279
14.7032
2018
10.0181
0300
17.3700
4241
18.85β0
1201
20.4507
4053
22.1988
2824
24.0857
2805
26.1330
1568
28.3543
2190
30.7644
8027
33.3704
1060
8β,21ββ
0702
30.2960
8371
42.0351
6683
40.2591
5402
50,1911
8302
64.4574
3366
50.086,3 ΙδδΙ
νι— PRESENT VA.LTE
ΤΑΒΙ,Ε
0.9058 5062 0.0950 2488 0.9942 0050 0.092δ 6583 0.0900 9001
1 ο.θΩ17 1840 0.9000 7450 0.08" 3403 0.085Ι 0708 0.9802 9005
8 0.9870 0346 0.9851 4876 Ο.Ω827 0.9778 ο.0705 9015
4 0.0886 0551 0,0802 4762 0220 3333 ο.οαοο 80,34
δ 0.0704 2457 0.0753 7067 0,0770 Ο.Ο7Οδ 0.9514 0500
0302 ο.θ7Ι3 6417
8 0.0758 6067 0.0705 1808 3088 0.9033 0.0420 4624
0.9718 1843 0.9050 8003 2θ20 0.03'27
7 ο.θβδ7 0301
0.9072 8308 0.0008 8520 1805
8 0.θβ32 ο.Ωθ0Ι 0301 0.0501 5802 Ο.Ω234
9 0.9501 0408 0.0545 3480
0.0592 7240 Ο.θδ13 4704 0.9400 4022 8822
10 0.048θ 0007 0.0419 7640 0.01,13
0.9552 0211 1480 0.9434 0634 3082 0.9052
0.0δ13 2824 Ο.θ34θ 6318
12 0.0410 0534 0.0380 2364 0.0280 0316 8005
0.0478 8082 0.0372 1924
13 0.9484 4978 0.032ΰ 8347 0.9210 0494 ο.8θβ3 2372
14 0.9325 5040 0,0271 7405 0.8874 4923
0.9395 3505 0.027θ 1888 0,0142 3815
15 0.9217 9780 0.8786 6280
0,0350 8656 0.9074 3241
10 0.9233 0037 0.0104 5183 Ο.θ0Οθ 7738 0.8609 6297
0.9817 5425 0.9187 0684 0.8013 4047
17 0.9111 3088 0.8930 7254
0.9278 8805 0.0141 3016
18 0,0058 6272 0.8873 1706 0.8528 2120
0.9240 8789 0.00Ωδ 8822
19 0.9005 0023 0.8807 1231 0.8443 7749
0.9202 0871 0.9050 0290
0.8963 7620 0.8741 5014 0.8360 1731
0.9103 8544 Ο.θ0Οδ 0.8001 8346 0.8277 3992
21 0.8070 4878
0.9125 8301 0010 0.8000 0.8195 4447
22 7071 0.8850 2084 0,8011 8985
28 0.9087
0.8916 2180 0.8708 8810 0,8547 7001 0.8114 8017
0,0060 2642
25 0.8871 8567 0.8747 8526 0.8484 1589 0.8033 9821
0.00Ι2 7012
0.8827 7181 0.8807 1103 0.8421 0014 0.79M 4170
28 . 0.8975 3041 0.8646 0803 0.8368 3140 0.7875 6013
27 0.8938 0622 0.8783 7001
0.8600 5380 0.82θβ 0933 0.7707 0844
28 0.8000 θ748 0.8740 0986
29 0.8008 0155 ο.8δ4β 6782 0.8234 3368 0.7720 4790
0.8864 0413
80 0.8053 M88 0.8497 1118 0.8173 0380 0.7044 0392
0.8827 2610
0.8010 2973 0.8447 8827 0.8112 0.7608 3557
81 0.8700 0384 0.8308 8306 0.7493 4215
0.8507 4000 0.8061 8080
82 0.8764 1577 0.8350 1304 0.7901 8690 0.7410 2202
83 0.8717 8334 0.8824 8368
0,8482 4237 0.8301 7038 0.7032 8762 0.7345 7715
0.8081 0590 0,8253 5581
0.8440 2220 0.7878 3202 0.7273 0411
0.8646 6364 0.8205 6015
80 0.8308 2314 0,7814 7158 0.7201 0307
0.8600 7024 0.8158 1020 0.7120 7334
0.835β 4492 0.7750 5418
0.8574 0372 0.8110 7807 0.7050 1420
30 0.8314 8748 0.7608 8008
0.8538 4β03 0.8003 7511 Ο.ΟΩ80 2495
Ω.8608 0310 0.8273 5073 0.7641 4806
41 0.8232 3455 0.80Ιβ 0864 0.7584 6001 ο.0Ω20
0.8407 7487
42 0.8101 3880 0.7070 4008 0.7528 1440 0490
0.8432 6128 0.7024 2000 0.13851 5337
0.8397 6227 0.7472 1032 0.6788
0,8302 7778 0.7878 3092 0.7418 4700 0.071β 5814
0.8328 0775 0.81M 686,1 0.7832 6189 0.7301 2701 0,0650 0311
0.8298 5211 0.8110 0850 0,7787 1930 Ω.730θ 4718 0.0584 1802
0.8009 7883 0.7742 0317 0.7252 0800 0.6518 9002
0.8260 1082
0.8020 5884 0.7607 1318 0.7108 0952 O.MM
0.8224 8380
48 0.7989 6402 0.7144 5114 0.β8θΟ
0.8100 7100 0.7052 4023
49 0.8Ι5β 7237 0.7608 1110 0.7001 8264 5402
0.794Ω
0,8122 8784 8θΩ7 0.7603 9884 0.7038 5374 0.0327 2704
0.7620 1210 0.6080 1414 o.a•au 6801
0.7010 3300
0.7476 6080 ο.β034 1853 0.0202 0041
,0.7870 0841
0.6882 o. (]141
0.7831 8250
0.7702 8007 11121
0.3080 3882
35
ΤΑΒΙ..Ε νι —PRESENT VA.LUE
ΟΡ 1

η 1%
ΤΑΒΙ,Ε
51 0.8089 1735 0.7764 0.7433 0.6831 0.6020 1864
52 0.8055 6084 0902 1480 2819 0.6θβ0 5800
58 0.8022 1827 0.7715 0.7390 0.6780 0.6001
ο.7θ88 8966 5127 0304 4280 0.6848 1336
0.7955 7407 0.7677 0.7347 0.6720 0.5785 2808
57 0.7922 7353 1270 1809 0.0070 0.5728 0008
68 0.7889 8608 0.7638 0.7304 8551 0.6071 2879
59 0.7857 1228 9324 5700 0.6030 0.5016 1305
00 0.7824 5207 0.7600 0.7262 1201 0.6650 6411
0.7792 0638 9277 2080 0.6580 0.δδΟ4
61
82 0.7759 7210 0.7563 0.7220 7733 4962
83 0.7727 5236 1122 0008 0.6531 0.6440
84 0.7695 4591 0.7626 0.7178 7849 0858
85 0-7063 5278 4847 2179 0.6483 0.5342 0097
0.7831 7289 0.7488 0.7136 1012 0.5289 7120
M
0.76W 0620 0445 5878 0.6434 0.5287 3302
67
0.7568 52βδ 0.7450 0.7005 8095
88 0.5185 4844
0.7687 1218 7906 1091 0.6886
89 0.5134 1429
0.7505 8474 0.7418 0.7054 0070
70 0.5083 3000
0.7474 7028 7220 0506 0.033θ 4511
71 0.6032 0801
72 0.7878 0.7013 1405 0.6202 2δθ2 0.4083 1480
0.7443 6874 ο.βθ72
78 8878 0.8245 4186
0,7412 8008 4678 0.4933 8105
0.7840 0.6108
0.7882 0428 0.6032 0.4884
1371 0.6152 7807
0.7361 4114 0310 0,4830
0.7320 0.7303 0.6106 0784 0.4788 7078
0.6801 0.8061 6170
6190 0.4741 2949
0.7200 5304 8286 ο.β01β 3040
0.7267
0.7260 2792 0.6851 8694 ο.6θ71 6070 0.4004 8614
80 2826
0.7230 1588 0.7281 0.6812 1221 0.6027 1533 0.4047 8726
31 0.7200 1629 0.6772 0151 0.4001 8541
82 1209 0.5883 0306
0.7170 2768 0.6733 8373 2912
88 0.71θδ 1512 ο.ββθ4 2873 0.5830 2363
0.7140 5246 0.4511 1704
84 0.7159 ο.6βδδ 4638 0.5705 7681
85 0.7110 8959 3544 0.5752 8234 0.4400 5142
0.7081 3901 0.7123 8664 0.5709 0.4422 2013
88 0,7052 0067 0.0578 • 0.4878
87 7357 ο.δββ7 2052
0.7022 7453 0.7088 0.8540 8389 0.4335 1647
88 0.5025 1009
0.6993 8052 2048 0.6502 4082 0.6583 2326 0.4202 2324
89 O.MM
0.6964 6881 0.7053 Ο.5Μ1 0.4240 7350
0.6935 0291 0.6427 2054 6701 0.4207 0585
01 0.5500 4170
6874 ο.β880 0308 0.4165 9085
92 Ο.θ9Οβ 0.7017 0.6362 87M
98 9394 0.5459 4710 0.4124 7510
9086 0.8318 0289 0.6418 8207 0.4083 0110
0.6878 ο.βθ83 0.6279 3991
2493 0243 0.6378 4911 0.4043 4771
0.6048 0.8242 9817 0.5338 4527 0.4003 4427
97 0,68-49 2829 0.6208 7755 7123 0.8903 8040
98 7088 0.6821 Ο.βθ13 0.6170 7793 0,3024
2888 0.5250 2678
99 0.3879 0.6184 0.3885 7020
ο.β7θ2 9827 0.5220 1180
100 8177 0.60θθ 4120
Ο.θ7Μ 7960 0.5181 2575 0.3847 2207
ο.β84δ 0923 0.6064 0384 0.5142 6878 0.3809 1883
ο.β78β 7263
0.6811 0.8028 0.5104 4043 0.3771 4241
0.6708 7731 0371 8700 ο.δ0θβ 4083 0.3734 0832
0.6680 9359 0.6777 0,5993 0.5028 0.8697 1121
0.6663 2141 1518 9056 ο.4θ9Ι 2507
0.6625 6074 0.6743 ο.6θ5θ ο.4θΜ 1009
0.6598 1158 4842 1439 04917 2217
0.6709 0.5924
8847 6888 0.4880 8171
0.5890 2242 0.4844 2850
0.6676
5022 0.68δβ 0638 0,4808 2233
ο.θΜ8 0,5822 1016 0.4772 4801
2858 2846 0.6788 3388 0.4788 9088
0.6577 Ο.δ7δ4 7688
8470 0.5721 8020
0.6644 0.6688 2108
6248 2220 ο.Μ22
0.6512 0.5589 8172
0644
0.6479
0.8447
4290
0.6416
3522
0.6888
4850
0,6351
β7ββ
0.6320
0763
0.6288
6831
0.8257
8484
0,3223
2158
ο.θΙθδ 2891
Ο.βΙΜ 4170
0.6133
ΤΑΒΙ,Ε
7483
0.6108
2821
0.6072
8878
ΤΑΒΙ.,Ε 0F 1

1%

0.6570 0.6042 0.5557 0.4701 ο.3ββ0 5071


7872 0545 3901 0410 0.8024 2644
0,0548 0.6012 5015 Ο.5δ2δ ο.4βββ 0.3588
4727 0.5082 6781 Ιβ8θ 6412 8806
0.0δ1β ο.δθ52 0.5403 0.4031 0.3552
3214 9130 1257 9019 8521
0.5023 0.5481 0.4597 0.8517
0,6489 2971
2827 2083 4213 6753
0.6402 0.6803 0.5429 0.4563 1978
0.3482
3662 8279 5957 0.4529 2281 0.8448
0.8435 0.5864 0.5308 0.4405 3632
5415 5064 1067 6117 0.3414
0.8408 0.5835 0.58ββ 0.4402 2210
8380 3288 8004 0404 0.3380
0.6382 0.5806 0.5335 0.4428 8302 4108
2453 2θ73 0750 0.4395 8012 0.8348
0.8355 7080 0.6777 0.5804 0.4863 1377 9474
0.β32θ 8905 4102 7813 0.4330 8577 0.3213
0.5748 0.5273 0.4298 4196
0.6308 8093
1276 0.5720 0.5243 0.4208 4124 0.3280
0.0270 9734 0.5801 3801 0.4234 ββ15 0.8248 6141
0.0260 9279 0085 0.5212 0.4203 1379 0.3210
0.8224 0.5803 9711 0.4171 84θ1 3500
0.8199 2921 0.5182 0.4140 7931 0.8184
1606 0.5β35 7885 0.4109 0083 0.3152 9768
0.6007 0.5152 0.4079 3780 0.3121 7582
0.0173 4379
0811 0812 0.8000 8497
ο.β147 8220 0,4049 0055
0.0122 3123 Ο.δδ7Ο 0.5122
0.4018 8840 0.3060 2473
o.aooo 0080 1862 7082
0.3988 9409 0.3029
0.6071 0102 0.5551 0.5098 0.3952 2525 9478
4280 0886
0,0040 4188 0.5528 0.8920 7792 0.2999 9488
0.0021 3279 •ο.5003 0.2970 2459
8000 5512 0.3900 5262
Ο.δΟθΟ 0.6490 0.8871 4891 0.2040
3481 0.6034 8375
3273 1851 0.8842
0.5271 0.2911
4620 0.5408 0,5004 0.3814 0630
0.3786 6711 7208
0.5946 9824 0893
0842 0,2882 8914
0.6441 0.4975 0.8767 4899
7736 0029 0.3720 δΙ8δ 0.2864
ο.δθ22 0091
0.5897 0.6414 0.3701 7553 8479
0.4047 0.2820
0.5872 9058 7001 0.8074 1988
1047 0870
0.6848 0.5387 0.8646 8476
0.4018 0.2798
7612 0.3619 βθ97
0F 1
VI—PRESENT
ΤΑΒΙ.,Ε

0,6824 3286 0.5300 4140 0.3592 7641 0.2770


0.5800 1613 0.5834 2850 0,4889 0.8666 0090 4019
ο.δ77β 0042 0.6807 7463 8896 0,3539 4830 0.2742 9722
0.5762 1270 0.5281 8896 0.4801 0.3613 1147 0.2715
0,5728 2δθ3 0.62δδ 0648 5807 0.3480 9625 8141 0.2688
0.5704 0,6228 9197 0.4883 0.3461 .004 0248
8863 o.2M2 8018
0.5680 8205 0.5202 9052 9
0.2636 9424
0.5657 2486 0.6177 0201 0.4806 0.3435 2406
0.2609 8439
0.5883 7746 0.5151 2687 3058 0,3409 ββ8Ι
ο.ωιο 3979 0.4777 0.3384 2800 0.2584 0030
0.5587 1182 0.6125 0.2558 4197
0.5100 1349 4869 0.8350 0928
0.4749 0.2683 0888
0.5508 0351 0.5074 7811 0.8334 0871
7302 0.2508 0087
0.6540 8483 0.6049 0,9309 2676
0.4722 0.2483 1770
0.6517 8572 0.5024 8016 0.3284 6329
Ο.Μθ4 ο.4θ0θ 8946 1841 0.3260 1815 0.2458 5911
0.6472 0.4974 5220 0.4604 0.2434 2486
7978 0.2410 1471
0.5449 4548 0.4049 7731
0.4667 0.2380 2843
0.5426 8429 0.4025 1474
6703 0.2382 βδ77
3249 0.4900 6442
0.6381 0.4870 2628 0,4840 6007 0,2330 2050
0.63δθ 5688 0.4852 0028 0.4618 5881 0.2316 1040
0,4580 8816 0.2293 1723
0,4827 8β3δ 0.4δβ0 2308 0.2270 4070
0.4808 8448 0.2247 9877
0.4770 9448 0.4688 7832
0.4750 1637 0.4607 4895
0.4732 W12 0.4481 8488
0.4455 3687
0.4420 6108
0.4408 8308
0.4378 2908
0.4352 8080
0,4327
0.4302 5δβ0
0,4277 0033
0.4252 7953
0,4228 1312
0.4203 0102
0.4179 2813
87
0F
1
TABLE VI— PRESENT VALUE
0F
1
1*70 1*70

0.9888 7516 0.9878 0.9852 2107 0.9828 0008 0.9803


1 0.9778 6432 0,9706 6176 0.9058 0777 9218
8 7407 0.9754 0.9568 1090 0.9,1θ2 8528 0.0βΙ1 0878
9537 0106 0.9421 8423 0.9320 5851 0.9423
5 0.9562 3770 0.9634 0.9282 6033 0.9109 1254 2233
6 Ο.θ4δ5 1833 0.9238
7 0.9145 4219 0.9011 4254
9970 0.95Ιδ 2428 0.8850 4378 4543
8 0.9010 2070
o 0.9350 8005 0.9397 7706 0,8877 1112 0.8704 1157 0.0057
10 0.9246 7743 0.9281 7488 0.874δ 9224 0.8564 4135 8081
0.9143 9054 0.0107 Ι ο.8θ1β 6723 0.8407 2800 0.8879
11 0.9042 1808 593 7138
12 0.8489 3323 0.8202 0880
0.8941 5881 0.9053 9845 0.8368 8742 0.8120 5788 0.8705
18 u
0.8842 1142 0.8942 2069 0.8240 2702 0.7980 0128 6018
10 0.8743 7470 0.8831 8093 0.8118 4928 0.7848 6490 0.863-4 0037
17 0.7998 0.7708 7450 0.8307
0.8046 4742 0.8722 7740 6527
18 0.8550 2835 0.8015 0800 0.7880 3104 0.757β 1031 0.8203
10 0.8455 1629 0.8508 7269 0.7783 8520 0.744δ 8605 4830
20 0.8361 1005 0.8403 0809 0.7β4θ 1159 0.7317
0.8299 9318 0.7536 0747 7θθΟ 0.8042
21 0.8288 0846
0.7424 7042 0.7Ιθ1 0401 0304
22 0.8176 1034 0.8197 4035
0.7008 2468 0.7884
23 0.8085 1455 0.80θθ 2002 0.7314 9795 9318
25 0.7θθδ Ιθθδ 0.720β 8703 ο.0Ω4β 0.7730
0.7906 2542 0,7100 3708 0789 0.8827
26 2028 3253
27 0.7818 2983 0.78θ7 5800 0.0θθ5 4302 0.7578
0.8892 0688 ο.β7ορ 7817
28 0.7731 3210 0.7800 0856 0.0δθ4 3800 7502
29 0.7645 3112 2052 0.6480 9032 0.7430
0.7703 7881
80 0.7580 2583 8574 1473
0.7608 0796 0.03βθ 4070
81 0.7476 1518 0.7514 7453 0.6590 0926 0.7284
0.6259 0479
82 0.7392 9800 0.7421 9707 5887 0.6162 4581
83 0.7310 7348 0.7330 0.8397 8243 2820 0.7141
u 0.7229 4040 0.6303 0781 0.6040 0250
85 0.7230 8434
0.7148 9780 0.6209 9292 4607 ο.δΗ2 0.7001
0.7150 4626 4764
30 0.70β9 4407 0.6118 1508 5087
0.7002 1868
37 0.6990 8002 0.8027 7407 0,5840 2710 0.6804
0.0974 9978
ο.βθ13 0287 ο.δθ38 8608 0.5739 8247 3076
0.8888 8867
ο.β83θ 1223 0.5850 8974 0.5641 1053 0.0720
0.6760 0715 0.6803 8387 7133
0.8710 8407 0.6784 4309 0.5644 0830
ο.ββ84 8867 ο.θβ3β 8707 0.5679 298 0.6448 7811 0.05θ7
0.6610 4986 ο.θ564 9429 0.5595 3126 7582
0.6355 0183
• 0.6536 9578 0.6474 0177 0.5512 6232 0.6408
0.5202 9172
0.6464 2352 3904
ο.θ3θ4 0910 0.6431 Ιδδ9 0.6172 4002
0.6392 3218 6502
0F
1
0.8821 2080 0.6315 1622 0.6360 8926 0.5083 4400 0.6217
0.6250 8865 ο.β237 1873 0.6271 8158 0.4996 0008 2140
0.8181 3454 0.6160 1860 0.6193 ο.β0θ5
0. β084 0.4910 0884
0.8112 5789 0.6117 1494 3087
1334 0.4825 6348
0.6044 5774 0.6041 5285 0.4742 6386 0.6975
0.0009 020β 0.4θβ7 0212 7928
ο.δθ34 8362 0.4893 6170 0.6868
o. 6881 0.4-821 2976 6204
0.5977 9324
48 0.578θ 2008 0.4750 0468 0.4502 1170 0.5743
0.5910 8355
40 0.5717 7290 0.4424 6860 7455
0.5845 0784
50 0.6780 0528 0.6647 1907 0.4348 5848 0.5631
0.5715 7506 0.5577 4219 0.4273 7934 1231
0.5608 5649 0.4200 2888 0.6520
0.6440 5579 7089
0.5373 3905 0.6412 4507
0.6300 8330
0.6202 2873
0.5100 2817
0.5000 2701
0.4902 2316
0.4806 1093
0.4711 8719
0.4010 4822
0.4528 9042
0.4440 1021
0.4363 0413
0.4207 0876
0.4184 0074
0.4101 0080
0.4021 5379
0.3942 0880
0.8865 8701
0.3789 6844
0,3715 2788

88
0F
1
ΤΑΒΙΕ VI—PRESENT
VALTE ιμι = (1 +

η 1*70
2%

51 0.5652 1637 0.5807 0624 0.4679 8491 0.4128 0475 0.3642 4302
52 0.5680 2843 0.5241 5332 0.4010 0887 0.4057 0492 0.3571 0100
α.5δ27 10,14 0,5176 8220 0.4542 6505 0,3987 2719 0.3600 9902
53 0.6405 0102
M 0,6112 θ115 0.4476 4192 0.3918 0.3432 3433
0.6404 8120 0.5040 7892 0.4409 2800 (3947 0.330δ 0425
55
0.5344 0843 0.3851 2970 0.32θθ 0613
58 0.4087 4401 0.4344 1182
0.5285 2250 0.3785 0585 0.3234 3738
67 0.4925 8727 0.4270
0.6228 4282 0.3710 0502 0.3170 9547
58 0.4805 0594 0.4210
0.51θ8 2860 0.3665 0.3108 7791
50 0.4801 0070 0.4154 3541
0.5110 7887 0.3503 1008 0.3047 8227
80 0.4746 0700 0.4092 9597
0.5063 0319 0.3531 3025
01 0.4087 0874 0.4032 4720 0.2088 0014
0.4907 7077 0.3470 ο.2θ2Ω 4720
02 0.402θ 2222 0.3072 8704
0.4942 1000 03410 8772 0.2872 0314
M 0.4572 0713 0.3914
0.4887 1288 0.3352 2186 0.2815 7170
05 0,4515 0259 0.3860 3221
0.4832 7002 0.3294 5587 0.2700 5009
0.4450 8775 0.3700 3921
0.4778 0.3237 8056 0.2706 3793
07 8173 0.3743 1843
0.4725 8309 0.3182 2009 0.2063 3130
08 0.4350 4308 0.3687 8003
0.4073 2568 0.3127 4701 0.2601 2878
09 0.4290 7277 0.3033
0.4021 2075 0.3073 0.2650 2817
70 0.4243 0817 0.3570 0708
0.4509 8500 0.3020 8222 0.2500 2701
71 ο.4ΙθΙ 0.3520 7092
0.4519 0177 02008 8070 0.2451 2511
72 0.4408 7443
2905 0.8474 8495
73 0.4139 5402 0.8423 3000 0.2917 8054 0,2403 1874
0.4419 0302 0.2807 6221 0.2350 0061
0.4309 8002 0,4088 4407 0.8372 7093
75 0.2818 3018 0.2800 8087
0.4321 2551 0.4037 9001 0.3322 8ββ3
0,3088 1147 0.3273 7690 0.2709 8298 0.2204 5771
77 0.427.3 1818 0.2722 1914
78 0.3938 8787 0.3225 3703 0.2220 1737
0.4225 β433
79 0.8890 2500 0.3177 7130 0.2875 3724 0.2170 6408
0.4178 0337
80 0.3842 2228 0.3130 7523 0.2029 3580 0.2133 Ωβ10
0.4132 1470
0.3704 7870 0.3084 4850 0.2684 1362 0.2092 1192
81 0,4080 1775
0.3747 9387 0.3038 9015 0.2630 0.2051 0073
82 Ω.4Ο40 7104
0.3005 7-070 0.3701 ΟΩ1β 0.2010 8707
83 0.2093 9010 0.2490 0114
0,3051 3148 0.2040 7454 0.1071 4507
0.3055 9083
85 0,3907 3570 0.2463 0826 0.1032 7948
0.3010 8329 0.2900 1531
80 0,3803 8882 0.2863 2060 0.2410 8019 8908
0,3500 2547
87 0.3820 9031 0.3522 2208 0.2820 8017 0.2309 42θθ 0.1857 7420
88 0.3778 3001 0.3478 7420 0.2328 6751 0.1821 9157
0.2770 2086 0.2288 0242
89 0.3730 3821 0.1785 0036
0.2738 13 Ιβ
0F
1
ΟΙ 0.3004 7050 0.8436 7051 0.2βθ7 0.2240 2621 0.1750 5018
02 ο.3θ53 0,3303 3770 0.2657 7997 0.2210 6770 0.1716 2065
03 0.3851 4843 0.2018 6218 0.2172 5572 0.1082 0142
0.3013 0448
05 ().3572 8503 0.3310 1080 0.257Ω 8245 0.2135 1014 0.1840 0217
0.3538 1020 0.3202 2426 0.2541 0.2098 4082 0.1817 2762
07 0.3493 7070 0.2604 1300 0,2002 3760
0.3228 8814 0.158δ 50,19
08 υ.3464 9207 0.2467 1300 0.2026
0.3Ι8Ω 0187 0.1554 4754
99 0.3416 4041 0.3Ι4Ω 0481 0.2430 0057 0.1623 0066
100 0.3878 4801 0.3110 7636 0.2304 7487 0,1002
0450 0.1494 1132
0.3840 9010 0.3072 3601 0.2359 3583 Ο.Ιθδ7 0,1404 8100
0.3303 7340 0.2.324 4000
0.3034 4287 7837 0.1430
0.8200 0805 0.2200 1389
0.2000 α.1θ24 1118 ΟΩ5Ο
ΩΟ00 0.2250 2044
0.1891 0190 0.14υ7 0303
0.2Π5Ω 9070 0.1380 8297
0.1858 4053
0.2Ω23
0,1826 5310
4242
0.1795 1165
0.2887
0.1704 2422
3320
0F 1
TA.BLE VI--PRESENT

3%

1 0.0779 Ο.Ω7δ6 0970 0.9732 8001 Ο.Ω7Ο8 0.9001


2 9511 ο.Ω618 1440 0.9471 8838 7370 8857
0.9584 0.9286 9041 0.9218 8779 Ω.Ω425 0.0335
0.9059 5004 9501 0.9151
δ 7444
0.8838 6429
0.8071 0573
4100
1070
a 0.9354 2732 0.8731 5400 0.001θ
0.8622 0687 0.8884 8705
7 0.9148 4335 0.8407 8491 4271
0.8412 0524 0.8026 0878
8 0.8947 1232 0.8270 4128 0.8714
Ο 0,8750 0.8207 4857 0.8040 0635 0.8374 8420 4228
0.8007 2830 0.8130 0151 0.8410
10 2427 0.7833 0386
0.8567 0940 0.7811 0840 0.7023 θ7θ1 0.7804 0923 7817
11 0.83βθ 3836 0.7021 4478 0.7064 1 0,8185
12 0.7419 9310 (373
0.8186 2101 0.7435 0.7221 8440 0004
18 0.7440
0.8005 1013 5δ8Ω 0.7028 0720 0.7859
u 0.7828 9400 0.7254 2038 0.083θ 0728 0.7224 2128 0,7594
ο.7θ5β 6748 0.7077 2720 ο.β05β 0078 0.7013 7988 115a
10 0.0904 0550 5134 ο.αοι ι 0.7337
17 ο. 7488 δ.0478 7424
1005 0.7323 1781 0.0418 3007
18 0.0730 2493 0.0305 3464
4137 0195 0.7089
0.6571 0500
10 0.7162 2028 0.0130 68Ω2
ο.θ4ΙΙ 0591 0,0231 0004 1881
20 0.0255 2772 0.5072 3490
0.7004 6580 0.5812 5067 0.0060 1045 0,0849
21 0.6850 5212 0.0102 7004 0.5873 0401 4571
22 ο.βθθθ 7703 0.5953 8029 0.5702 8003
0.5505 6375 0.ββ17
28 0.8562 3484 0.5808 0407 0.5636 7576
0.5358 1874 8330
25 0.6408 1047 9724 0.5214 7800 0.5375 4928 0.6304
28 0.0207 1588 0.5528 7535 0.5218 9250 0,115
0.5076 2120
27 0.6120 2457 0.6803 0060 0.5010 9175 0.0177 8179
0.5994 3724 8790. 9062
28 0.5202 3472 0.4019 3374
0.5862 4068 0.4807 1821
29 0.5133 9078 0.4770 0557 0.5707
0.6738 4839 0.4078 5227
80 0.5008 7778 0.4563 3008 0.4830 0473 0.5572
0.5007 2997 0,4880 0125 0.4501 8908 0378
81 0.4481 4421
0.5483 9117 0.4707 4209 0.4370 7075 0.5883
82 0.4312 8901
0.6363 2388 0.4243 4036 8114
88 .0.40δ1 1481 0.4197 4103
0.5245 2213 0.4119 8676 0.5201
85 0.5129 8008 0.4537 7055 0,4086 0708 0.502δ
0,4427 0298 0.8975 7380 0.8099 8715
M 0.5016 9201 0688
0.431θ 0584 0.8800 8314 0.8883 8708
87 ο.4θ0θ 5238 0.4855
0.4213 7107 0.3770 2625
88 0.4798 5558 0.3705 7727 7090
0.4110 9372 0.3060
88 0.4692 9641 0.3064 0850 5063
0,4010 0705 0.8563 8340
0.4589 0960 0.3560 0.4632
0.3Ω12 8492 0.3471 4310
M 0.4-488 7002 0.8450 3243 8503
81
0F 1
12 0.4389 0268 0.8817 4130 0.3378 5222 0.8340 8294 0,4370
0.4293 3270 0.3724 3002 0.3288 0.3262 2015 5713
0.4108 8528 0.3083 4695 0.8200 0008 0.3167 53δδ 0.1231
0,4106 4576 0.8644 8483 0.8114 4405 0.3085 5684 4899
0.4018 0954 03458 3880 0.8031 0044 0.2078 2800 0.4088 8707
0.8927 7210 0.3874 0.2040 9702 0.2889 5922 0.3060 1224
0,8841 2926 0.3291 7440 0.2871 0172 0.2806 0.8810 5434
0.8750 7653 0.3211 4576 0.2704 1778 0.2723 7178 0,3687 4815
0.8874 0981 0,3662 7841
0.3133 12" 0.2719 3040 0.26"
0,8503 2600 0,3066 7116 ο 2640 0122 0.3442 8035
0.8614 1809 0.2982 1676 ο:2δ75 7783 8862 0.8325
0.8488 8518 0.2909 4221 0.2507 8663 0.3218 1271
0.8381 2242 0.9492 6870 0.3104 7005
0.8287 2608 0.2419 9880 0.2009 7080
0.2249 δ02θ
0.2281 0708 0.2898 3272
0.2800 3101
0.2705 0104
0.2614 1250
0.2525 7247
0,2440 8137
0.2367 7910
0,2278
0.2201 0231
0.2126 5924
0.2064 6787
.ο.Ι08δ 1988
ο.ΙθΙ8
0.1853 2024
ο.Ι7θ0 6337

82
TABLE νι —PRESENT VALUE

51 0.3214 9260 0.2838 4000 0.2500 8402 0.2214 6818 0.1729 9843
52 0.3144 1810 0.2769 22θ8 0.243θ 7471 0.2150 1280 0.1071 4824
0.3074 0030 0.2701 0870 0.2874 4497 0.2087 5029 0.1014 0589
0.3007 3287 0.263δ 7928 0.2810 0000 0.2020 7019 ο.Ι5θ0 3407
0.2941 1528 0.2671 5052 0.2240 0511 0,1007 0717 0.1507 5814

57 0.2870 4330 0.2508 7855 0.2188 8575 0.1910 8609 0,1456


58 0.2818 1374 0.2447 5966 0.2130 2740 0,1854 7193 600,1
0.2761-2347 0.2387 8082 0.2073 2008 0.1800 0084 0.1407 3433
80
0.2βθ0 ομο 0.2829 0508 0.2017 7710 0.1748 2508 0.1850 7520
01 0.2031 4850 0.2272 8359 0.1003 7079 0.1097 3309 0.1313 7701
02 0,1269 3431
0.2573 5801 0.2217 4009 0.1911 2097 0.1647 8941
83
0.2510 9487 0.2103 3170 0.18β0 0581 0.1599 8972 0.1220 4184
84
0.2401 5035 0.2110 5541 0.1810 2755 0.1558 2082 0.1184 9453
05
0,2407 3071 0.2059 0771 0.1701 8253 0.1508 0565 0.1144 8747
0.2354 4220 0.2008 8557 0.1714 0718 0.1464 1325 0.1106 1591
07
0.2302 8138 0.1950 8593 0.1068 7804 0.1008 7628
88 0.1421 4879
09 0.2251 9450 0.1012 0578 0.1024 1172 0.1880 0853 0.1032 0114
70 0.2202 8012 0.1865 4228 0.1580 0408 0.1330 8887 0,0997 0922
0.2153 0278 0.181θ 9241 0.1538 3448 0.1300 8028 ο.0θ03 9538
71 Ο.ΟΩ3Ι
0.2106 5309 0.1775 6358 0.1497 1720 0.12β2 0736
72 3503 0.089θ
0.2MO 1700 0.1732 2300 0.1457 1023
78 0,1220 1880 8012
74 0.2014 8420 0.1089 9805 0.1418 1044 0.1190 4737
75 0.1970 0.1048 7β15 0.1880 1503 0.0800 4311
0.1155 7998
0.1027 1458 0.1008 M78 0.1343 2119 0.0840 0300
70 0.1122 1357
0.1884 7391 Ο.ΙδΟθ 0.1307 2822 0.08Ι1 0232
77 3149 0.1089 4621
0.1848 2057 0.1272 2747 0.0784 1770
78 0.1057 7205
0.1802 7048 0.1581 0389 0.1238 2235 0.0757 6590
70 0.1026 9191
80 0.1703 0385 0.1498 6005 0.1206 0837 ο.0θ97 0030 0.0732 0370
0.1724 2411 0.1457 2049 0.1172 8809 0.0907 0.0707 2827
81 0.1886 2903 0.1421 7218 0.1141 4- Ωβ41 0.0083 3050
82 0.1387 0457 412 0.0939 7710 o.oooo
0.ΙΜθ 1925
88 21380
0.1012 9022 0.1368 2153 0.1110 8017 ο.0θ12 3900
84 0.0885 8243 ο.0β37 0285
0.1δ77 4105 0.1320 2101 0.1081 1698
85 0.0860 0230 0.0β10 3561
0.1642 0997 0.1288 0008 0.1052 2237
80 0.1508 7628 0.1258 5940 0.1024 0620 0.0834 2743 0.0595 6131
87 0.1225 9403 0.099β 0640 0.08Ι0 0647 0.0576 3750
0.1476 5528
88 0.0787 0434 0.0556 9178

83
0F 1
80 0.1443 0835 0.1100 0452 9795 0,0764 1108 0.0537 1187
0,1411 3288 8733 0,0944 0190 0.0741 8030 0.05Ι8 0553
91 0.1880 2724 0.1138 4130 0.0018 7533 0.0720 2502 0.0501 4000
0.1349 8997 0.1110 6468 0.0804 1038 2770
98 0.0484 4-
0.1320 1958 0.1083 5579 0.0870 2324 ο.0θ78 9105 503
0.1201 1446 0,1067 1296 0.084β 9415 ο.0ββθ 1804 0,0408 0079
M 0.1262 0.1081 3460 0.0824 0,0030 9383 0.0452 2896
0.1000 1012 2740 0,0021 2993
98 7881 'ο.04,30 9464
Ο.Ο8Ο2 ο.0β03 2032
99 0.1284 9468 0.0981 0500 0,0422 1704
0.1207 0.0957 7078 2131 ο.0δ85
100 0.0780 0.0407 8941
7719 8842
0.00" 7427 0.05β8 0.0304 1000
0.1181 0.07δΟ 8409 5769 0.0880
3480
1950 0.0739 0.0552 7735
0.09Ι1
0.1166 0,0889 3264 5104 0104 0.0307
2029 0.08β7 0.0719 0,0586 8971
0.1129 6355 7181 9383 0,0355
7828 0.0840 4737 0.0700 0.0520 4562
0.1104 0.0081 3284 0.0348
9221 7086 4850
0.1080 0.06θ8 0.0331
6084 8221
0,0320
6011
ΤΑΒΙ,Ε

0.9615 3840 0.θ5βθ 8780 0.9523 8005 0.9478 0780 0.9433 0023
1 0.9245 5621 0.9157 2995 0,0070 2948 0.8984 5242
2 0.8889 0.8762 0.8688 3760 0.8510 1.300
8 0.8548 0419 0.8385 6134 0.8227 0247 0.8072 1074 0. 8.3θΙ) 11128
0.8219 2711 0.8024 0.7835 2017 0.7051 3435 Ι).7Ω2Ο ΙΙ3Ι]ΙΙ
0.7-172 6817
8 0.7908 1453 0.7878 9574 0.7402 1540 0.7262 4583
0.7348 2840 Ω.β874 3081 0.7040
7 0.7599 1781 0.7106 8133
ο.β51δ 0887
0.7031 8513 0.6708 3930 571 Ι
8 0.7306 9021
0.6170 2θ20 0.Ιί274 1237
ιο 0.702δ 8674 0.6729 0443 ο.044θ 0802
0.6139 1325 0.6854 3068 0.5018 θ84ΙΙ
0.6755 6417 0.643θ 2768
11 0.5583 0478
0.6495 8093 0.6101 9874 ο.δ84θ 7929 0.6640 1050
12 0.52117 87Π3
0.6245 9705 0.5896 6,386 0.5508 3742 0.6260 8152 ιιιχια
18 0.6005 7409 0.5642 7184 0.5303 2135 0.4085 0.4088
0.5774 7508 ο.δ3θθ 7286 0.5050 0796 0.1726 00.37 31102
0.5167 2044

84
1
16 0.5662 0450 6932 0.4810 1710 0.4470 3305 0.4423
17 0.5339 0818 0.4731 7639 0.4245 8109 0.4172
0.4581 1162
18 0.5133 7325 0.4528 0037 0.4024 4053 0.3030
0.4802 9000
19 0.4936 2812 0.4333 0179 0.3814 0500 0,3713
0.4155 2005
20 0.4746 4242 0.4148 4286 0.3015 7900 0.3503
0.3957 3390
0.4563 8695 0.3θθ7 8748 0.8768 8048 0.3427 2800 4.371)
21
22 0.4388 0.3797 0089 0.8248 0158 0,3305 1301
0.3589 4230
0.4219 5589 0.3633 5013 0.3079 2507 0.8118 0473
24 0.8418 4987
0.4057 2633 0.3477 0847 0,2918 7267 0.2941 ή540
25 0.3327 3060 0.3255 7181
0.3901 2147 0.3100 0701 0.2760 5650 0.2776 0510
28 0.37δΙ 1880 0.3184 0248 0.2022 3870 0.2017 0720
27 0.2058 0277
0.3β0β 8923 0.3046 9137 0.2486 0275 0.2409 7855
28 0.2915 7069 0.2812 4073 0,2329 θ8ΙΒ
20 0.3468 1657 0.2078 4832 0.2350 0460
0.3834 7747 0.2790 1 δ02 0.2233 2181 0.2108
80 0.2670 0002 0.2650 0364
0.3206 5141 0.242θ 4632 0.2116 7044 ΙΩΩ.3
M 0.3083 1807 0.2555 0241 0.2000 4402 0.2073
0.2318 7745
88 0.2964 6026 0.24" 9991 0.1901 8390 07115
M 0.2203 5947 0.1066 3014
0.2850 5794 0.2389 7121 0.2098 6617 0.1802 0910
35 0.2740 0.2238 9589 0.1708 7119 0.1845
ο.1θθ8 7254
M 9417 0.2142 5444 0.1903 5480 0.1010 0821 507,1
37 0.263δ 5209 0.1812 9029 0.1635 1908 0.1741 1013
0.2050 2817
$8 0.2634 0.1901 0921 0.1042 5484
0.1456 1024
39 1547 o. 1877 0.1726 5741 0.137Ω 8008 0.1640 5740
40 0.164-4 3563
02436 6872 5044 0.17θθ 0.1307 3041 0.1401 8022
0.16β0. 0530
41 0.2342 0649 0.1230 0.1870
0.1401 4707
42 9885 0.1719 2870 2,302 0.1301 0522
0.1420 4568
0.2252 0.1846 2507 0.1174 0314 0,1227 4077
0.1362 8160
8543 0.1574 4026 0.1113 8047 0.1157 0318
0.1288 3θβ2
40 0.2Ιββ 2031 0.1506 0.1055 3504 Ο.ΙΟΩ2
0.1227 0440
47 0.2082 0.1441 7276 0.1000 3322 3885 0.1030
0.1168 0188
49 8904 0.1879 8437 Ο.Οθ48 1822 0,0972 2210
0.1112 9051 0.0808 7509
0.2002 0.1920 2982 0.0017 1005
0.1059 0868 0.08δ1 8005
7793 0.1263 8810 0.0805 2740
ο.Ι00θ 4921
0.1925 0.1208 9771 0.0807 4840 0.0810 2902
0,0961 4211
7493 0.1166 9158 0,0766 3885 0.077() 0ΩΙ)8
0,0915 β8θΙ
0.1851 0.1107 0965 0.0725 4867 0.072β 5007
0.0872 0878
6820 0.0β87 0052 0.0085 3781
0.1780 ο.0β40
4835 5831
0.1711 ο.0β00
9841 0840
0,0575
0.1646 450U

85
0F 1
1388 0,0542 8830
0.1682
8256
0.1521
0476
0.1463
4112
0.1407
1282
ΤΑΒΙ,Ε VI—PRESENT 09

6%

51 0.1353 0059 0.1059 4226 Ο.Ο83Ο 5117 0.005Ι 8153 0.0512


52 0.1300 0072 0.1013 8014 0.0790 0035 ο.0βΙ7 8344 0.04&3 Ιβ45
0.1250 9300 0.0070 14-40 0.0753 2080 0.0585 6250 0.0455 8150
53 0.07Ι7 4272 Ο.Ο43Ο
0.1202 8173 0.0928 3683 0.0555 0948
ο.Ι1δβ 5551 ο.0β83 2040 0.052β 1602 0147
0.0888 3007
50 0.00δ0 7270 0.0405 6742
0.1112 0722 0.0850 1347 0.0408 7283
58 0.1009 3002 Ο.ΟβΙθ 7400 0.0472 7203 0.0382 7115
0.0813 5200 ο.0δ00 2201
59 0.1028 1733 0.0448 0818 0.030Ι 0480
0.0778 4938 ο.0δβ2 1230
0.0988 6282 0.0424 7221 0.0340 0119
ο.0θδ0 0.07" 9701 0.0536 3552 0.0321 3320
81 0.0402 6802
0.0712 8001 Ο.Ο3Ο3
62 0.0914 0423 0.060θ 8021
0.0082 1915 0.038Ι 5920 1434
03 0.0878 8808 0.0485 5830
ο.0βδ2 8148 0.03βΙ
M 0.0845 0835 0.04β2 4000 0.028δ 9843
0.0024 7032 0.0342 8428
0.08Ι2 6803 0.0440 4881 0.02βθ 7θ05
0.0507 8021 0.0324
0.0781 3272 0.0419 4848 0.0254 5250
07 0.0572 0694 0.0308 0270
0.0890 4903 0.0240 117θ
68 0,0751 2702 0.0647 4253 0.0201 0.022β δ2β4
0.038Ο 4070
60 0.0722 3809 0.0523 8510 0.0270 7485
0.0302 3495 0.02Ι3 7041
70 0.0094 5070 0.0501 2037 0.0202
0.0345 0948 0,0201 0077
71 0.00θ7 8818 0.0479 7089 3208 ο.0Ι90 1950
0,0328 ββΙ7
0.0M2 1940 0.0450 0497 0.0248 0.0179 4301
72 0.0313 0111 0458 αο235
73 0.0βΙ7 4942 0.0439 2820 6828 0.01β0 2737
0.0208 1058
75 0.0503 7446 Ο.Ο42Ο 0.0159 0021
0.0283 9103 0.0223
0.0570 0081 0.0402 2037 0.0150 0580
70 0.0270 8908 ο.021Ι 7498
0.0548 0501 0.0384 0413 0.0142 1254
77 0.0257 5150 0.020D 7107
0.0δ27 8387 0.0368 3040 0.0134 0800
78 0.024δ 2524 ο.οιρο 2471
79 0.0507 5363 0.0352 6023 0.0180 3290 0.012β 4911
0.0238 5737
80 0,0488 0147 0.0337 3228 0.0222 4612 0.0119 8318
α.0Ι70
Ω.Ο4ΟΟ 0.0822 7θβΟ 0.0211 8582 0.0112 5707
0.01C2 0170
81 2449 0.0461 0.0308 8005 0.020Ι 7098

86
1
82 1070 0.0205 6948 ο.0ΙΏ2 1017 ο.0Ιδ8 5706 0.010β 2044
83 0.0488 8483 0.0183 0111 ο.0Ι4δ 5046 0.0100 1928
0.0282 8058
0.0174 2003 0.0137 9759 0.0094 δ2Ιδ
86 0.0417 1570 0.0270 0850
0.οιαδ
86 0.0401 ι 125 0.0250 0287 ο.0Ι30 7828 0.008θ 1718
0.01δ8 0910 0.0084 1238
87 0.0385 6861 0.0247 8744 0.0123 9648
0.0370 8610 0.0237 2003 0.0150 6687 0.01Ι7 5022 ο.007θ 8621
88 0,0071 8699
0.035β 5875 0.0148 0.01 ιι 3705
80 0.0220 9800 0.0070 6320
0.013β 5857 0.010δ 5701
0.0342 8720 0.02Ι7 2115
0.0329 0,0130 o.ooao 0840
M 0.0207 8579 0.0100 0004
0862 01320 0.00β2 8022
02 0.0198 0070 0.0094 8407
0,0317 0.0123 0.005Ω
98 0.0100 3417 0.0080 0049
M 0050 0.0117 0706 2180 80,10
Ο.Ο3Μ 0.0182 1451 0.0066, 9472
0.0174 3016 0.0112 3530 0.0080 7768
8125 Ο.ΩΙβ6 7Ω58 0.0107 0028 0.0052 7803
97 0.0208 0890 ο.0Ιδ0.β132 0.007β 5648
0.0101 0074 0.004D 7928
08 0.0281 8163 Ο.ΟΙδ2 0.0072 5728 θ743 0.0044
0.0097 0647
0.0970 9772 0.0068 7894 3154
100 0.014θ 1626 0.00Ω2 4331
0,0065 2032
0.0200 5550 α.013θ 8086 0.0088 0315 Ο.ΟΟ4Ι
0.0260 0.00βΙ 8040 8070 0,0039
- 0.0183 0.0083 8305
5.337 8454 4405
0.0070 8471
0.0240 8078 0.0128 0817 0,0070 0440 0.00,37 2081
0.0231 8325 0.0122 δββ8 0.0068 5820 0,0035 1019
0.0055 6279 0.0083 1160
0.0222 7236 0.0031 2406
0,0214 1672 0,0052 6381
ο.οο-2θ 4723
0.0206 9204 ο.004θ 8892
0.01θ8 0004 0,0047 2888

87
0F 1
TABLE νι — PRESENT VALUE

0.9389 0714 0.9346 Ο.Ω3Ω2 3260 Ι).025Ι) 2503 ΙΙΗΙΙ) (1,7215


1 0.8810 5928 7044 0.8053 0.8573 7,128
*15-12
2 0.8278 4009 0.8734 32111 (Ι.7Ω3Η
3 0.804Ω 0.7350
0.7773 2300 8873 0.7-188 (Η)53 ().ΙΜΙ5
0.7208 8084 0.81β2 ο.ΙΙΙΗΙ5 58113 (1.511,11) "Ι - 135
Ω. ιι:ςηι ΙΙΙΙΙί.•ι 0.47118 7ΙΝ!Η
0788
c 0.08δ3 3412
0.7028 9521
0,0495 0021
8 0.7120 8018 ().ΙΩ7ΙΙ
0.0042 3110 ().5215 Ω..37Π7
9 0.5673 5323 υ.001Ι3 4222 8817 —
Ο.ΙΥ227
10 0.5327 2004 4117.* 1).5820 0.4851 Ι5ΗΗ3
Ω.3Ι!
11 0.5002 1224 0.5430 Ω3ΙΙ3
ΙΙ .1Ι7Η
12 0.40θυ 8285 3371
0.4513
13 0.4410 1070 0.6083 Ω. 583,1
'1311) (1.2710 Ι'0(Ι7
4021) 0.2918 ιιο.Ι7
15 -0.4141 Ω.41ΙΙ8 Ω.2-1ΙΙ8
8.ΙΓ,Ω .
0025 0.4750 5413 0.3005 (Ι.Η!
10 Ι5 137Η
0.3888 2062 0280 0.363.3 1347
17 Ω,337θ ΙΗΙΙ)2
18 0.3650 0533 0.4440 ΙΙ.23Ι7
ΙΙΙ(ΙΩ
0.3428 1251 1ΙΩβ Ω.,3Ι.13 80Ι)Ω ()2Ι.16 0.ΙΙΙΙΙΙ 0738
10
0.8218 8009 0.4149 Ω.2Ω2-1
(Ι. 1531
Ω.272Ο 0.1080 5575
0.3022 4384 0445 ο.Ι.111
21 0.25.30 (ΙΩΙ3 Ω. 18311
0.2887 0703 0.3878 0.235-1 131Π Ω. 1703 ο. 9378
22
1724 1528 o. 1570
23 0.2004 7608 0.2Ι8Ω 8807 Ω.ιιηη
0.3024 ΙΙΙ)."
25 0.2502 1228 4002 Ω.2(Ι37 Ω.ΙOΙΗ 5Ι.ΙΗ
0.2340 4111 o. 180,1 Ι)83Ι) 1700 η.0Ω3Η 7233
26 0.3987 3- ο.Ι7ΙΥ2 77.ΗΙ
0.2206 0198 0.1352
27 100 ο.Ι03Ι)
0.2071 3801 0.1251
28 0.310δ 7-
0.1944 9570
Ω.ΙΠ2Ι1 38110 0. Ι 159
20 1.30 0.1-118 1.372 o.
80 0.1820 2516 0.2968 1073 2752
81 0.1714 7902 0302 0.131Ω
82 0.1810 1818 0.2705
0.1511 8607 0832
u ΙΗΙβ8 o,
85 0.1419 5875 0.2584 ιιιυο 1227 8701 1 H28
0.1332 9400 0.1142 2103 0,0382 0577
88 0.1251 6925 0.2415
37 0.1175 2042 1.300 0.1002 5212 l).lB25
83 0.1103 4781 0.2257 Ο.Ι)Ω88 0,0290
80 1.317 0.0010 4343 o. 0271'
0,1030 1207 0.2109 0.085Π 2877 4H4H
40 0.0θ72 8917 Ο.Ο7θΠ
7733
4688
0.00Ι3 5134 Ω.042ίΙ 2123 0,0234
0.1071 (Ι,Ω3Ω4 0,11 1

88
ΑΝΝΤΙΤΥ PER PERIOD

43 0.0857 7590 4002 0.0740 1083 ο.Ι)8θδ 5,182


0.0806 4075 0.1842 α.0088 ο.Ω338 3411
45 0.031.'3 278K
0.075β 2612 4018 4721)
47 0,0710 0950 0.1721 ο.ΩΜ0 0.02Ι>0 07.30
4,3110 0.02'*
49 0.0βθβ 7650 95,10 Ο.Ο5θ5
50 0.0β2β 0.1009 7580 6801
0819 3037 0.0564 0.0248
0,0587 8515 0.1504 1036 0,0230
0.05δ1 9733 0221
0.0515 5288 0.0213 2123
0.0618 2848 0.1405 0.047Ω 51117
0.048β 0524 0282 0.0440 1030
9506 0.1313 o.(M14
0712 9804 0.038β
0.0429
0288
0.1227 7301
0.1147 0.03δΟ 0001
4113 0,0334 0428
0.1072 0.03ΙΟ
3470 7.375
0.1002 0.028θ 0682
1034 0.02β8 8913
ο.0θ8β 6204
0.0875 3540
0.08Ι8 0884
0.0784
5080
0.0714
0.0θ07 8038
0.0β24 1157
0.0583
2857
0.0545
1208
0.050-θ 4643
0.047β
1340
0,0444
9850
0.041δ 8747
0,0388
0.03β3
2410
4770
TABLE — AMOUNT 0F 0F 1
ato —
89
8
η 1%

1.0000 0000 1.0000 0000 1.0000 1.οοοο 0000 1,0000 0000


1 2.004Ι 6067 2.0050 0000 2.0075 0000 2.οιοο 0000
2 3.0125 1736 3.01δ0 2500 2.0058 3333 3.0225 6625 8.0301 0000
8 4,0260 4.030Ι 0013 3.0175 3403 4.0452 2542 4.ΟβΟ4 0100
5 0Ω52 5.0502 4.035Ι 3β31 5.075δ 3461 5.1010 0501
5,0418 4064 5.058,0 7400
σ 0.076δ 0188 6.1186 .313 6.1520 1506
7 β.0028 4831 7.1058 7039 0.0881 8364 5 7.2135 3521
8 7,0881 1018 8.1414 0870 7.1280 7.1694 8368 8.2856 7056
8.1170 4397 0.1821 1583 9,3885 2727
ο 9.1514 6740
8.1652 5284 8.2131 7071
10.2280 2041 0.2128 8340 0.2747 7850 10.4622 1254
10 10.1806 2531 10.3443 11.5θθ8 3467
11 11.2701 6654
11,2320 5520 3940 12,6825 0301
12.3355 6237 11.3285
12 12.2788 5549 11.4219 13.8093 2804
13.3072 4018 1308
18 13.3300 1739 2194 14.θ474 2132
14.4642 203θ 12.8025
14 14,3855 5013 12.5075 10,0968 9654
15.6305 4752 8629
15.4464 0806 8036
15 10.0142 13.4048 17.2578 6440
10.5098 5520 7537 13.6013 18.4304 4314
16 17.5780 4027 8020
9325
17 17.007,3 0141 14.5434 10.6147 4757
18.β5Ι8 0063 18.7857 2048 14.7034 20.8108 0504
18 10.7200 0684 15.6282 0370 22.01θ0 0390
8701
20 20.8118 1353 5710 15.8136
10.8707 28.2391 0403
21 21.8986 2942 1685 7023
16.7104 24.4715 8598
22 22.θ8Ω7 7330 20.0791 1544 2193 16.0322
24.0855 6402 25.7163 0183
23 17.8109 8183
26.1859 2054 22.0840 1101 20.9784 6485
23.104-4 3107 5180 18.0592 28.2431 0050
28,2908 6187 7304
28 24.3104 0322 18.0208
27.4004 0713 10.1947 20.5256 8150
27 25.4810 5524 8411
28.5145 7540 1849 30.8208 8781
20.δ5θ1 1502 20.0312
29 20.β333 8622 20.3386 7888 32.1200
5503
80 30.7608 5887 27,6010 21.4012 1897 33.4603 8760
21.1481
31.8860 1224 1050 84.7848 9153
31 0493 22.6624 0312
32 33.0178 28.8303 28.8222 0614 86.1327 4045
22.2714
38 84.15Μ 4000 7015 25.0000 0330 87.4940 6785
0887
M 35.2077 5524 29.9745 2200 26.1884 7050 38.8βθ0 0853
23.4013
35 30.4448 2022 31.1243 27.8848 8412 40.2576 9862
8577
87.50θ0 8268 0401 41.6602 7560
38 24.5378 28.5902
32.2800 43.0768 7830
87 38.7533 0386 7075
1658 44.6070 4714
38 8552 25,0810 29.8046
30 30.0118 93.4414 3157 9778 45.9527 2361
40 0775 34.6080 26.8308 31.0282 47.4122
41.0811 2375 3750 3301 5085

90
ΑΝΝΤΙΤΥ PER PERIOD

41 1945 85.781θ 27.9873 82.2609 48.8863


42 42,2522 30.9605 5081 4476 7836
9078 7520 20.1506 33.5029 50.8752
48.4283 38.1458 1035 0184 3700
4190 7807 30.3206 34.7541 51.8780
40 5558
44.θΩ92 0342 39.8361 7361 8040
48 45.7951 31.4975
40.6327 88.0148 53.3077
50 6548 8540 2607 2991 7036
7806 41.7354 82.0812 37.2849 64.9317
48.1817 4042 33.8710 4113 5715
5358 42.0441 0233 38.5645 58.4810
4θ.3825 44.1588 85.0604 7810 7472
1088 4780 8843 39.8588 58.04δ8
50.5882 45.370β 36.2740 1253 8547
7134 6046 59.026,3
δΙ.7Ω9Ο 4153 41.1527
40.6065 37,4856 4432 θΙ.222β
5581 1612
8074 5013 0777
63,0148 42.4613
47.8395 88.7043 62.8848
8621 6140 8385
7244 2548
54.23δ7 43.7708 64.4031
8056 40.0787 30,0301 2170 8218
55.4017 7030 Μ71 45.1081
6298 60.3241 41.1630 7037
0415 2030 40.4464
51.5757 42.4031 8164
8497 4305
47.7948
62.8386 43.β5Μ 9502
3020
64.0078 44.0051
40.1582
3222 2352
0148
55.3683 46.1670 50.5219
2138 7007 4117
66.6451 47.4863 51.9008
7798 5573
48.7130 53.2001
0018 1215
49,9072
Μ.θ807
4θ88
8700
61.2880
0050
52.5880
58.9621
8575
1644
68.8048
60.3942
4059
5732
55,20θ2
3621
66.6312
0009

91
57.8610
5596
ΤΑΒΙ,Β VII—AMOUNT 0F ΑΙΤΝΤΙΤΥ 0F 1 PER PERIOD

η 1%

51 56.βθ28 57.028,3 8880 50.1985 01.8472 Ββ.ΙΩ78 1401


52 57.9290 59.2180 7877 1424 07.7ΙΙ88 0215
7388 3075 60.643Ω 03.31 ιο (]0.4405 8107
30.5141 0381 8835 71.1411) 4088
59.1704 2090 βΙ.81ββ βΙ.8Ω7Ο 64.7850 72.852,1 6735
56 4508 63.1257 7069
60.4169 ββ.27Ι7 74.5800 8102
58 7496 83.2581 Ω5Ι52 70.82157
8855 4287 07.7088
59 61.6887 0384 78.ηΩΟ0 5ΩβΛ
βδ.7β3β 1080 64.6271 3400
2600 4870
61 67,0024 2801 ΟΩ.277Ι 0035
62 82.9258 88.4278 9105 66,0041 83.4803 0055
7902 70.7900 7800
83 θθ.7700 3051 4040 72.3270 5360 85.3212 3022
64.1878
84 8935 71.1188 67.3801 73.8701 1109 87.174,1 4252
85 θ5.4δ58 72.4744 7507 6455 75.42-11 3603 80.04θΙ 80115
1881 68.7822 4882
86 78.8368 70.9898 1705
67 88.7280 75.2000 3108 6801 92.8460 1531
70.1834 78.5072 4150
88 4930 76.5820 8184 04.7744 75-10
68.00β0 9701 80.1564
89 81.7570 00.7222 2021
8284 77.9640 7215 71.5029
70 83.8708 5214 08.0804 4242
79.3547 9701 0105
θθ.28θ4 100.07(Β 308,1
71 4152 80.7515 7009 84.ΩΟβ1 3353
72 82.1553 2885 73.0105 80.0380 0453 102.0831 0021
70.5781 2001
73 4753 83.5661 0549 88.283,3 104.7000 3121
74 74.4364 6057 106.7570 3052
71.8722 84.9839 3602 2165
75 89.0454 108.8246 M83
2814 86.4088 5570 75.8706 3411 8174 θΙ.0200
76 73.1716 110,0128 4684
87.8408 9908 77.3132 1281 7286
77 9074 89.2801 0448 78.7642 113.02Ι0 7530
78 74.4765 93.3072 2840 115.1521 0506
90.7265 0500
7278 80.2236 6442 05.0070 2758 117.3037 1701
80 92.1801 81.βθ1β 3579 06.7105 8028
75.7868 3752 110.4707 6418
81 88.1681 7034 98.4440 7714 121.07Ιβ 2172
9184 θ3.β4ΙΟ
82 84.6533 1800 100.1833 1446
77.1026 3821 129.8882 8004
88 95.1022 86.1471 2902
7055 101,0340 8032 120.1271 1031
M 4340 87.&40θ 108.βθ01
78.939 3188 128.3883
85 96.6847 8082 5304 106.4700 4340
79.7506 130.0722 7440
88 98.0677 1357 8θ.1β0θ 4859 107.2β80 205β
9806 132.078Ω 0715
87 81.082θ οο.5δ80 5214 90.8810 100.0725 8072
88 101.0558 4240 02.2100 2188 135.3087 8712
9264 110.8Ω0δ 7470
92
ΑΝΝΤΙΤΥ PER PERIOD
89 82.4208 102.5611 2161 93.7479 1367 112,7222 5401 137.0018 7400
• 90
3844 104.0799 2722 95,2947 114.5β7β 7001 140.9384 0374
91 83.7842 105.5942 9685 7650 θβ.8506 110.42βθ 2845 142.4388 7808
92 δ8β0 107.1222 6834 6270 98.4158 118.3001 8041 144.8032 0740
98 85.1132 108.0578 7068 2490 θθ.98θ7 120.1873 8139 147.3110 ΩΟΙ4
7834 1604
110.2011 6008 122.Ω887 140.7850 1014
86.4879 101.5729
M 111.7521 7402 8038 8675 152,2828 "033
1500 113.3109 3580 124,0044 5265 .154.8Ω5β
98 87.8281 108.1664 9849 125.9344 8004
114.8774 0048 167.3537 6501
99 9797 104.7872 9728 127.8780
100 113.4518 7793 1δΩ.θ272 Ω2δβ
89.1941 108.3784 8980
118.0341 3732 1β2,52θδ θβ48
4880 Ι07.θθ8θ 8070 129.8380 8715
θ0.δβ57 119.aua 109.6289 7475 131.8118 7280 165.1518 3114
91.9431 0800 133.8004 βΙ8δ 187,8033 4945
4855 111.2βΜ 7710 185.8039 170.4818 8204
121.2224 2954
θ8.3282 112.9175 4322 187.8224
4500 122.8286 4169 114.5762 2889
124.4428 8440 130.8561 6377
94.7151 116.2445 9022
126.0648 9782 141.0050
0436 117.9226 8367
127.6952 2231 148.0693 7318
θθ.10θ7
129,3386 989 ΙΙθ.βΙ0δ 146.04θ1 4343
5062 121.3082 θ42θ 148.144δ 1201
97.5102 123.015θ 2601
0792 124.7885 18θΙ
98.9185 126,4611 3110
0045
100.8286 128.1988 2103
5254 Ι2θ.θ46θ 4749
181.7046
101.7466
183.472θ 4684
8859
135.2515 3903
108.170β
8312
104.6005
1078
106.0363
4622
107.4781
84,33
108.θ25θ
9002
110.8798
4831
111.8397
6434
113.8Οδ7 6336
114.7778
7071
116.2581

93
1184
117.7405
1230
119.2310
9777
120.7278
9401
122.2309
2690
123.7402
2248

1

ΤΑΒΙ,Ε VI—AMOUNT 0F 0F 1

η 1 1%

101 125.2658 130.ΩΜ3 137.Ω405 0034 150.2565 173.1881, 0677


102 ΟΟΟΩ 132.11.352 138,8390 0585 175.9180 5874
134.2ΩΜ 140.α.ΙΩ8 0870
103 120.7777 152.3825 178.0772 30,33
1-12,4702 β5Ω8
104 0680 460ΙΙ 1281 181.4040 1172
3732 144.3013
105 128.3050 4263 164.5263 181,2780 5184
1.37.ΙΙ4Ω7 8701
4033 150.8843 187.1214 3830
100 130.3380 140.14.31 0Ω30
120.8406 54- 2202 189.ΩΩ2Ο 5274
107 35114 0178
1-1 158.8504 1Ω2.8925 7927
108 141.113-17 2012 140.8580
131.3815 6444 105.8215 0506
100 142.7308 0040
5076 108.7707 2011
110 144.45.35 151.7330 101.050Ω 0035
132.Ω28Ο 7000
0026 8TM3 163.2687 201.7075 1731
111 134.4828 8210 204.7851 0248
140.1768 153.6181
112 5005 105,4832
0725 155.6143 207.833Ω
113 130.0431 2290
0226 4,141
114 0581] 147.9007 167.7243
157.4214 210.0113 7485
115 137.βΙΩ0 4058 1714
0001 214.0204 8800
110 4251 1.5Ω.33Ω7 1βθ.0822
1172 217.1006 0349
117 130.1834 15.3.1514 8428 001>1 7074
1700 220.3323 0042
118 4170 ιυι.2ΩΩ2
172.2571 223.5350 2343
110 140.7638 15".ΟΩ18 2791 4285
103.2000 4084 22β.770θ 7900
120 4800 158..1752 8704
8010 174.5-100 7544 230.038θ 80,16
121 142.34Ω8 "265 100.21370 0348
143.042Ω 102.00Ω0 0180 170.8581 233.3390 7035
122 165.1020 9361 236.6724 6712
94
ΑΝΝΤΙΤΥ PER PERIOD
123 145.5427 103.8703 3832 170.1846 240.0301 0179
124 4θ42 4081 167.1254 2000 243.4395 8370
125 147.1491 165.ΙΙΩ87 4354 8,354 181.5285 246.8739 7054
120 7754 107.5272 109.1003 1408 260.3427 1934
127 3726 8210 183.8800 253.8401 4053
148.7022
128 ΙΟΩ..3Ι548 171,0808 7854 267.3840 0800
0912
120 171.2116 0109 180.2001 200.9584 5408
150.3821
130 Ω781 173.0848 5338 204.51580 3882
4203
173.0077 0743 188.OC,01 7203
181 152.0087 208.2137 1000
5030 175.0Ω44 0881 101.08Ι1
182 3420 271.8058 5019
153.0421 174.0.330 0508 177.1168 0832 275.6148 1475
133 Ι7(Ι.8Ω77
0401 5321 103.5142 279.3700 6200
134 178.(ΙΩ17
155.2822 170.1490 7708 283.1646 7253
135 18<1.5862 5830
7045 182.4881 2002 105.9056
136 280.ΩΩ03 1020
8465 181.1040 3410 200.8802 8245
187 150.02Ω2 8805
0502 108.4363 204.7740 4527
138 158.5831 18,ι.40Ω0 2557
180.3226 183.2510 7642 208.7226 0473
130 6098
2870 3040 200.θ23β 302.7000 2167
140 100.2430
188.2542 185,31Ωθ 0474 203.4305
2415 300.7870 2080
141 101.011C 0717 418* 187.4010 0005
142 103.5802 2806 31Ω.8043 0110
100.1955 205.θ6β2
189.4042 314.0124 3501
143 3887 1306 0832
0071 310.0816 5080
146 105.2078 192.1404 ΙΩΙ.5ΩΩ5 208.5000 323.2621 7405
4810 9002 8355 7060
146 327.4846 0070
147 100.9.584 0423 104.1072 103.7172 211.0ΙΗ7
168.8521 4778 331.7505 4367
148 2307 2784 330.0771 3011
ΙβΙβ ΙΩ(Ι.0777 106.8472 213.6477
140 840.4370 1060
170.8648 Γ-,ΟΙΩ
0600 1330
150 ΙΏΒ.Ωδ8Ι 4708 344,8422 8000
3331 200.04&1 3872 ΙΩ7.ΙΝΩ7 210.2500
172.0β40 202.0-18(1 8002 0744 7115
4512 20-1.058Ω 2432 200.1446 218.8710
173.7815 18ΙΙ4 4740 4668
8114 208.1 (1011 202.3121
210.1501 6785 221.6134
175,505β 8028
7106 (ΒΙ1 204.4023
212.2000 1210 224.1748
177.230Ω 4400
178.Ω7δ4 3100 1303 3743
180.7211 200.0851 226.8501
214 2111 Ω 8302 4871
0203
1850 208.8ΩΟ8 4740 220.5575
182,4741 21(Ι 3332 2ΚΩΙ) 211.1ΩΟ3 7744
βΩ82
218. ΙΗ4Η 0423 213.3408
6777 232.2702
184.23-14 71581 220.500') 4881
180.ΩΟ2Ι 222.00Ω5 0354 5100
215.6853
187.7771 3700
2020
217,8420
180.55θ6 1822 236.0213
101.34Ω3 0530 220.11.36 0858 4598

95
Ιθ.Β.144'β 5441 222.3ΙΓΤΠ α,1Ω8 237.7840
104.061.* 3214
0608
100.7037
240.5073
2077
227.0057 8012
108.6835
0544 243,3710
7805
22Ω.32ΟΩ 0638 4162
200.4110 240.190Ω 2883
231.0β7β
1023 240.0434
202.24150 5010
0317
234.019') 0580
204,0887
6787 251.9112
4800
2345.3841 3134
205.9301 238.76.30 7000 254.8005
1770
0558
207.7071
257,7115
0744
6982
200.Μ44
203.5ΩΩ2 3064
200,6702
3304
200.6756
δ5βΩ
272.5973
7236
275.6418
5205

96
ТАВЕ VE—AMOVNT О? A.NNVITY 1 РЕП РЕМО)
(s at i) —

97
TABLE 0F ANNUITY PER

* 96

1.оооо 1.0000 1.оооо 1.0000 0000 l.OOOO 0000


1 0000 0000 0000 2.0175 0000 2.0200 0000
2 2.0112 2.0126 2.0150 3.0528 0625 3.0004
8 5000 0000 0000 4.1002 3030 0000
4 8.0338 3.0376 3.0462 6.1780 8938 4.1210
5 7056 5625 2600 в.2087 0693 0800
в 4.0080 4.0756 2095 4.опоо 7.37И 0831 6.2040
7 0767 6.1266 0338 8.5076 3046 4010
8 6.1137 7229 5.1522 0.0504 1224 0.3081
9 7276 8.1906 6893 10.8253 0946 2090
10 6.1713 5444 0.2296 12.0148 4304 7.4342
0270 7.2680 5003 8338
11 13.2251 0371
7.2407 3702 7.3220 14.4506 4303 8.5829
12 0905
2980 8.3588 8809 0419 16.7006 3263
13 0.7540
8.3221 9.4333 8.4328 3011 10.9844 4986
и 8807 7420 9.5593 2843
15 18.2816 7721 10.0407
9.4158 10.6810 3109 19.OOlO
18 1269 0637 10.7027 20.9446 3468 2100
17 10.6217 11.7130 2167 22.3111 6678 12.1687
18 4068 3720 11.8032 вио 23.7016 1110 1642
10 11.6401 12.8308 13.0412 13.4120
20 25.1103 8938
1013 6142 1143 20.6669 2620 8973
и 12.7710 14.0211 14.2368 28.0206 5400 14.0803
22 6140 1504 2000 29.6110 1037 3162
23 13.9147 16.1003 16.4503 31.0274 6916 16.9730
и. 3684 7088 8205 3815
25 10.3803 10.3821 32.5704 3900 17.2034
16.0712 34.1404 2238
7002 3463 3778 1002
27 36.7378 7077
10.2408 17.5011 17.9323 вои 18.6392
28 37.3032 0267
2848 0382 10.2013 8626
29 80.0171 6029
17.4236 18.8110 6530 20.0120
80 40.6999 6042 7096
„3780 5330 20.4893 7672
42.4121 9966 21.4123
81 18.0105 20.0461 21.7067
32 1305 1238
6230 9163 1ВЗВ
38 46.0271 1627 22.8405
19.8260 21.2067 23.1230 6710
и 35 47.7308 3979 6803
2267 6893 24.4706 2211
21.0620 22.6629 40.5061 2949 24.2973
86 26.8376 7904
9907 7854 ы.433Б 3675 0080
87 27.2261 53.8336 2365
38 22.2889 23.8450 4304 5Б.2ВВ9 6203
25.7833
80 3619 1677 28.6836 2080 57.2341 3390 1719
40 25.1430 27.2080 дзи
98
TABLE ΑΝΝΤΓΙΤΥ PER PER.IOD

23.6396 7847 80.оедо 2331 69.2357 31И 28.8449


8671 26.4673 31.5139 61.2723 5664 3321
24.8045 6695 32.9866 88.3446 2278 80.4218
0717 27.7880 7860 66.4631 6367 8247
им 26.08.35 6788 8403 87.5985 8380 32.0302
34.4814
48 27.8769 20.1354 вд,7816 5908 0072
7867
0700 3608 72.0027 3637
50 36.9087 0085 33.6709
28:3849 74.2027 8426
30.4006 37.6886 0672
8913 76.6623 8298
2802 8137 35.3443
30.0076 31.8808 78.9022 2468 2383
89.1017
9626 7337 87.06i2 1031
6159
31.3462 33.2703 38.7022
40.3882 8801
8183 8429 3461
42.2986 1233
32.0979 34.6953 40.6680
43.9380 9152
1625 7669 7021
46,5920 8789
34.0667 ЗВ.12ВО 6880
47.2750 8921 42.3794
6781 37.5806 48.9851 0874 4079
35.4490 8216 я.7198 8638 и.2270 2961
0769 39.0504 48.1116
62.4806 8368
86.8478 4089 54.2678 7020
0903 40.5386 9301 48.0338
88.2823 7120 0100
56.0819
4688 42.И53 0334 49.9044
1232
39.6927 43.5708 ввоз 7703
67.9281
9829 46.1156 61.9043
4100
41.1993 0650 6710
50.7919
4227 43.6794 64.0342
8812
42.6021 4932 5453
61.6888
6987 48.2926 56.1149
8794
44.0814 4243 63.6142 3062
3417 49.8862 0090 58.2372
46.8773 язо 2921 3841
66.5684
47.0000 61.4895 60.4019
1398
9549 5708 67.•6619 4018 8318
4&6198 5.3.1331 7654 69.6662 62.6100
6906. 54.7973 1929 2284
50.1668 4125 71.8086 9768 64.8022
3248 56.4823 78.6828 2804 2330
31.7312 0801 67.1694
„68,1883 0777
0934 3687 ы.91до вв.6020
53,3131 9108 6712
8545 71.8927
В1.6В46 3721
54.9129 1027
5879 63.4354

99
TABLE 0F ANNUITY PER

56.6307 4518 74.3305


2967 66.2283 0447
58.1067 8824 76.8171
0028 87.0437 7676
69.8210 4810 79.8635
7566 68.8817 1027
61.4940 6276 8980 81.9405
68.1868 7097 8966
и.89В7 1201 84.5704
66.6268 0145
0002
Π — ΑΜουτττ 0F

1*70 1*70
2%

100
TABLE ΑΝΝΤΓΙΤΥ PER PER.IOD

51 68.3708 5152 70.7428 1226 75.7880 7046 81.2830 1361 87.2709 8948
52 70.1455 8548 72.6270 9741 77.9248 9152 83.7054 6635 90.01β4 0927
71.9347 2332 74.5349 3013 80.003? 6489 86,1703 1201 92.8167
58 8746 958730
.73.7430 8895 76.40ββ 2283 82.2951 7136 88.6782 9247
75.5736 0883 78.4224 5502 84.5295 9803 91.2301 7221
θ8.δ8Ο5
77.4238 1193 80.4027 3031 86.7076 4292 03.82βθ 3865
57 9043 96.4680
70.2948 2081 82.4077 7052 89.00θ5
58 6752 οο.Ιδ08 101.5582 8432
81.1868 84.4378 0765 91,4352 9866
50 5002 104.5804 2θβΙ
83.1002 4023 86.4933 4000 93.8075 3863
96.2146 5171 101.8921 107.6812 1820
85.03δ1 2704 88.5745 0776 0405
61 110.8348 4257
8β.0917 7222 90.6816 8010 θ8.βδ78 7149 104.θ7δ2 1588
62 88.9704 101.1377 3056 114.0δΙδ 3042
63 92.8162 1022 107.5070 3215
20θβ 94.9754 0034 103.6548 0566 117.3825 7021
M 00.0713 110.3884 0522 120.β7θ2 2181
97.1625 0285 2774
4009 108.8027 7215 113.3202 0231 124.0928 0604
M 99.3771 2526 116.3038 0585
θΙ.ΟΟ47 127.5746 6216
07 7464 101.0103 3933 111.4348 1374 119.3386 1370 131.1261 5641
68 θάΟ4Οθ 103.8895 8107 114.ιοα3 8594
60 6586 106.1882 0083 116.8179 3098 122.4270 3944 134.7480 7852
70 108.5155 5334 119.5701 0096 125.5606 1263 138.4430 5209
07.1101 7872 122.3687 5295 128.70βθ 7010 142.2125 2613
71 00.202β 0021 110.8710 9776
182.0204 0124 140.05β7 7563
72 101.3186 9021 113.2578 9773 125.1992 0024 135.8807 5826
149.9779 1114
73 103 4585 3154 115.6730 2145 128,0771 9738
130.0θ83 138.6090 4653 153.9774 βθ37
74 105.'β224 4002 118.1195 4172
5534 142.Ι2β2 7984 158.0670 1875
75 107.8106 9247 120.6060 146.6184 8974
110.023δ 6276 123.1034 86" 133.9633 162.2181 5018
3067 140.1617 2581 166.4626 2231
77 112.2013 125.6422 8002
136.9727 152.7720 170.7917 7276
78 2784 128.2128
8063 5601
70 114.5242 0778 0852 175,2076 0821
80 116.8126 6579 130.8164 140.0273 156.4455 179.7117 6038
6863 7234 160.1833 184,3Οδθ
81 119.1288
133.4506 6441 Ιθ3.08θ5
143.1277 7320 9558
82 0828
186.1187 8292 167.8563 188.9021 Ιδ4θ
33 121.4800
0526 140.2746 3832 198,7719 5780
85 8487
138.8202 0067 171,7088 Ιθ8.β473 9θθβ
123.8334
M 8020 140.4688 2424 203.6203 4490
8845
87 141.5555 2010 208.βθ27 5180
126.2266 175.8002
88 3370 152.7108 218.8βθβ 0683
1520 1617
89 144.324θ 6247 210.1489 38θ7
128.64θβ 179.8767
90 7787 156.0016
6462 1006 224.508
91 147.1200 1525
131.Οθ3θ
4010 159.3415
184.0245 1775
08 8060 6255 230.0173 5411
14θ.068Ι 5310 8708 188.24-49 9239
95 183.5087 236.6177 0110
152.8427 102.7316 192.5302
4642 241.8300 δδ21
101
TABLE 0F ANNUITY PER

07 136.0713 5501 6105 7θ7θ 247.Ιδββ δβ82


98 0481 155.7532 160.1726
3507 106.9087 253.09θ7 8944
09 188.6021 9801 8045
158.7002 ισο.βθδ2 1716 259.1617 8528
100 141.1814 201.3546 265.8450 2094
0557 ιαι.083θ 2551
7273 1071 271.0δ1θ 2135
5814
143.74θδ 164.7060 173.2102 205.8783 278,0840 5078
8θ30 0702 0380 2556
173.8083 284.64βθ 5898
140.3607 210.4811 291.33θδ 0210
2102 167.7038 5605 0626 2θ8.1βθ8
140.0133 2021 180.4604 215.1Μβ 1718 8400
4724 170.8608 8230
173.9066 184.1673 210.02Οθ 805.1207
151.6897 0708 1108
4730 2881 8054
177.1716 187.0200 224.7787 312.2828
154.3θβ2 7296 0591
5705 8067 0038
180.3862 220.7124
157.1382 Ιθ1.7488 0148
3151
1404 Ιθ5.θ260 8102 234.7323
183.6410 190.5504
159.9000 6850
5040 5784
6861 239.401
186.2305 208.5528
Ιβ2.βΟθ8 8495
7264 4θ71
105.5302
100.2732 207.β0β1 246.0878
2276
7980 4246 8810
168.3024
198.051θ 2δΟ.82δ5
3770 211.7202
9580 4Μ8
171.2808 8459
197,0723 255.7062
52βθ 215.8000
4200 3947
174.2138 3811
261.1810 θ8θθ
2978 220.1844
266.7617
177.1737 78β8
6789
8537 224.4884
180.1069 0588
3θ8θ 228.8030
18.3.1938 1796 4330
m— AMOUNT 0F 1 PERIOD

η 3%

1.000Ο 0000 1.000-0 0000 1.0000 1.οοοο 0000 1.οοοο


1 2.022δ 0000 2.0250 0000 0000 2.0300 0000 0000
2 8.0680 0025 3.075β 2500 2.0275 8,0900 0000 2.0360
4.1370 3β3θ 4.1525 1583 0000 4.1830 2700 0000
5.2301 1971 5.2563 2852 3.0832 5.3001 3581 3.1002
5625 2500
102
TABLE ΑΝΝΤΓΙΤΥ PER PER.IOD

6.3477 9740 6.3877 3873 4.1680 6.4684 0988 4.2140


7.4θ00 2284 7.5474 3015 4680 7.0024 0218 4288
8.6591 0180 8.7301 1590 6.2826 8.8923 5.3024
0.8δ3θ 0300 9.9545 1880 0706 10.1501 0013 0588
7
11.0757 0784 11.2033 8177 0.4270 11.4638 7031
8 0.5501
9 12.3249 1127 12.4834 0631 4040 12.8077 0600 5218
10 13.6022 2177 13.7056 5297 7.8047 14.1920 2950 7.7704
14.2082 7176 15.1404 4170 0876 15.6177 0751 Ω.051β
11 10.2437 0788 16.5180 5284 8.8138 17.0808 2418 8077
12 17.6091 9180 17.9310 3825 18.5089 1380 10.3084
18 10.0602 0681
19.0058 9811 19.3802 2483 20.1508 8130
14 1880 11.7313
20.4330 1057 20.8047 3045 21.7615 8774
Ιδ 11.3327 0316
21.8027 8251 22.3888 4871 3637
16 23.3853 4966 23.Ο4βΟ 0743 0482 13.1410
25.1108 6844
17 24.0115 2008 25.5446 5701 1685 20.8703 7449 0ΙΩ2
1.3.ΩΩ2Ι 3720 14.001D 8104
18 26.4720 2923 27.1832 7405 28.β7Μ 8572 115.1130 3030
28.8628 5590 15.3700
19 28.0β76 30.5367 8030 17.07βΩ 8030
30.5844 2730 2107
20 29.βθθ1 7201 32.4528 8370 19.2θ5β
32.3490 3708 10.7007
21 81.3674 0338 34.4204 7022 8088
34.1577 0393 8039
22 33.0731 30.4502 04-32 20.0710
18.2617
28 84.8173 1628 36,0117 0803 8052 38.5630 4226 2971
24 36.6007 0590 37.9120 0073 10.7630 40.700β 3352 22.7050
25 38.4242 2178 39.8698 0075 7948 42.9300 2252 1575
20 40.2887 8677 41.8562 9577 21.3074 45.2188 5020 24.4000
27 42.1952 6402 43.9027 0316 8892 47.6754 1571 0130
28 46.0002 7074 20.3571
6740 22.8934 50.0Ω2θ 7818 8050
20 46.1379 1226 48.1502 7751 4487 52.5027 5852 28.2700
80 48.1760 1528 50.3540 24.5230 56.0778 4128 8181
60.2599 7563 62.6128 8531 1460 57.7801 7052
32 52.8908 2508 54.9282 0744 20.1078 60.4020 8181 30.2004
83 0750 7008
54.5698 1864 57.3014 1203 63.2750
32.3280
85 56.7974 3508 59.7339 4794 27.9178 4427
0215
86 59.0753 7786 62.2272 2503 60.1742
84.4004
2250
37 61.4045 7334 64.7829 7006 29.6855
1873
θθ.15θ4
38 63.7861 7624 67.4025 5854 ββΙ5
4027 30.θββ5
31.5010
89 68.2213 6521 70.087β 1737 72.2342 2821
1021
68.7118 4522 72.8398 0781 3275 88.θ408
41 33.3682
71.2578 5121 75.6608 0300 75.4012 6073
41.3131
42 2100
73.8806 4161 78.5623 2308 78.8632 9753 0168
35.2858
44 76.5225 0605 81.5161 3116 82.023Ι 0845 43.7590
45 4810
79.2442 6243 84.5540 8443 86.4838 9234 6024
82.0272 5834 87.6678 8580 37.2602 80.048-4 0911
40.2900
47 90.8595 8243 0802
103
TABLE 0F ANNUITY PER

48 84.8728 7165 94.1310 7199 30.2807 92.7198 0180 2734


87.7826 1126 97.4843 4879 5467 20.6014 5723 48.0107
90.7676 1776 41.8009 100.3006 0095 0030
7542 104.4083 9598 51.6228
43.4984 108.δ4Οθ 4785 7728
0224 112.7θβ8 6720 54.42 7008
45.ορ.-1β 0830
57.3345
47.9612 0247
1003 60.8412
50.2808 1006
6831 63.4531
52.6522 6240
8000 οο.β740
55.1002 2706 1274
57.6154 8801
οο.Ι909 0272 70.Μ7β 0818
62.8554 0724 73.4678
65.5830 6030
3004 77.0288
68.3874 0472
8904 80.7210
71.2681 0604
44Οθ 84.5502
7775
74.2280
1808 88.5005
77.2002 3747
8950 92.8073
80.3941 7128
83.6050 0β.Μ8β
8632 2928
86.0041 101.2883
7370 8130
θΟ.2Ω4Ο 105.78Ιβ
3867 93.7771 7290
2463 θ7.8δ5θ 110.4840
9556 3145
101.0832 7255
8544
120.3882
104.8117
125.6018
0079
4557
130.θ07θ
1016

104
TABLE ANNUITY PER PERIOD

VI—AMOUNT 0F 1
2*70 2*70 8*96
η 8%

51 03.7096 6416 100,0214 5751 108.6040 2256 117.1807 7331 136.5828 3702
52 ΩΟ.0ΙΟ1 9305 112.6831 0818 121.6001 9851 142.3632 3631
8513 108,0568 0029 110.7818 126.3470 8240 148.3450 4058
53 103.3426 7442 111.7560 0845 120,0933 0573 131.1374 0488 154.5380 5782
55 100.6078 8400 115.6600 2136 125.3207 1411 136.071C 160.04U8
50 110.0679 1200 110.43Ωβ 04-40 1072 8084
57 129.7670 3375
113.5444 4002 123.4256 8670 13-4.3356 2718 141.1537 8881 107.5800 3009
58 117.00θ1 8902 127.6113 2893 130.0208 5002 140.3883 8136 174.4453 3207
50 120.733θ 2160 131.000Ι 1215 143.8531 7700 151.7800 3280 181,5500 1800
60 124.4504 3403 135.091E 8005 148.8001 4038 157.3334 3379 188.9052 0085
61 128.2505 6972 140.3013 7070 153.0013 0174 163.0534 3680 10β.δΙ68 8288
62 132.1302 0754 144.00Ι1 6419 159.1336 8002 168.9450 3001 204.3049 7378
03 13θ.ΙΩΩ2 7221 149.6236 0330 164.5098 5022 175.0133 9110
M 212.5487 9786
140.1717 3083 164.2017 8503 170.0.338 7726 181..2037 0284 220,0880 0670
05 144.3255 9477 160.1183 3027 175.7098 0880 187.7017 0662 220.7225 8590
148.5720 2066 164,0962 8853 181,5418 2803 104.3327 5782 288.7028 7650
07
152.0158 1137 100.1θ8θ 0574 187.5342 201.1627 4055 248.1105 7718
88
157.8δβ4 1713 174.4286 0314 28Ω2 208.1976 2277 257.8037 8238
70 161.8009 3651 170.7893 7071 193.βΩΙ4 215.4485 5145 267.8268 9400
71 10β.5390 1758 185.2841 1421 2021 222.9008 5800 278.2008 3535
72 171,2867 5898 200.0179 397 230.6940 6874 288.0378
100.θΙθ2 1706 206.5184 2746
73 176.1407 1100 Ιθβ.β80Ι 2240 300.060θ 8085
238.5118 8565
75 181.1038 7705 2Ω2.β0β3 213.1976 8422 311.5524 6400
186.1787 1420 24β.ββ72 4222
5055 220.0006 2054 323.4608 0024
76 Ιθ1.8β77 3536 255.0672 5949
208.6715 0031 227.1122 8700 203.7102 7727 335.7777 8824
77
196.6735 0041 214.8882 9705 234.3578 7551 272.6308 5559 348.5300 1083
78
79 202.0θ8β 8837 221.2006 0447 241.8027 1709 281.8007 812β 381.7285 6121
80 207.8458 8329 227.7020 1709 249.4522 375.3800 6085
201,2840
213.3170 1507 234.4868 1751 9181 380.5276
81 74ΟΩ
21Ω.Ι175 241.3489 8705 257.3122 301.001Ω θ0θ3 77Ω8
82 6877 218.3827 1265 2083 311.0320 5084 401.1611 4671
83 265.3883 Ιβ15
225,0477 1407 255,5022 8047 321.3830 1855 419.3007 8685
84 273.6804
231,1112 8703 262.0820 332.0039 0010 434.0825 2430
85 9485
237,3112 9100 8748 842.9ΜΟ 451.200θ 1274
86 243.0607 9567 270.55ββ 282.2128
2638 467.00θΙ
87 250.1329 3857 278.3205 7345
364.2620 4717 486.3701 2510
88 286.2785 200.9737
256.7009 2980 365.8805 3658 503.3673 0*48
89 2747
203.5380 5000 204.4355 377.8δβ0 5185 521.9852 5320
270.4β7β 3370 209,0756
91 300.1026 541.2547 3716
6674 302.7064 0408
6020 561,1086 6295
105
0F A.NNUITY PER
277.5531 2213 300.2248 402.8084 4001 581,8406 0581
7002 811.8663 3137 416.9853 608.2050 2701
284.7081 32ββ 818.7285 9.321 825.3172 0295
2555 320.1504 1423 420.4640 6600 648.2033 0506
07 329.1542 828.4935 443.8480 0385 671,8904
202.2060
98 5328 4837 457.6498 7076 2073
8337 βθβ,40β5
100 299.7807 338.3831 338.5271 472.8788 6189
8540
2025 0061 2005 487.6502 17" 721.7808
307,5257 347.8420 348.8386 Ιβ78 503.1707 1595
8045 8735 359.4200 2397
315,4451 357.5387 2.374 510.2720 748,0431
1065 5453 370.813θ 2560 4451
323.6426 307.4772 3830 775.2246
535.8501
8177 2830 381.4θ75 5457
8645
377.θβ41 7170 808.3575
831.8223 552.025C
6308 1748
4090 302,0887 0205
832.4750
340.2883 388.1057 5492 570.5134
3059
4306 578,3 404.7060 6281
8β2.θ11β
348.9448 398,8084 4568 688.6288
3130 0177 416.0278 607.2877
3δ7.7ΩβΟ 9010 409.7788 ΙΙΒ2 3418 3270
3ββ.84θδ 421.0230 420.3983
0218 7711 4902
482.5486 442,2016
5404 0874
455.3022
1257
468.8840
7342
482.7720
ΩΙΜ
497.05δ4
2449
511,7244
4867
ΤΑΒΙ..Έ VII—AMOUNT + i)Tl — 1 0F Ι
UR.IOD (s αι ί) =
6%

1.0000 0000 1.0000 0000 1.0000 0000 1.0000 1.οοοο


1 2.0400 0000 2.045Ο 0000 2.0500 0000 0000 0000
3 8,1216 0000 3.1370 2500 3.1525 0000 2.0550 2.0β00
4 4.2404 6400 4.2781 0113 4.3101 2500 0000 0000
5.4163 2256 5.4707 0073 5.5256 3125 3.1680 2500 3.1830
0.6329 7546 0.7108 4.3422 6088 0000

106
0F 1 PER PERIOD
7 7.8082 0448 8.0191 5170 6.8010 1281 5.5810 9103 4.374β
8 θ.2Ι42 2020 9.3800 1302 8.1420 0845 1600
10.5827 0531 6.8880 6103
10.8021 1423 0.5401 0888 5.6370
10 12.00β1 0712 8.2608 9384
12.2882 0937 11.02β5 6432 6.075,3
11 9.7215 7300
13.4808 12.5778 0254 1854
13.8411 7870 11.2562 5051
12 5141 15,0258
15.4βδ0 3184 14.2007 8716 12.8763 5370 8.3038
0546 ια.β2β8
13 3768 18,2919 17.1δθ9 1327 16.0171 2052 3705
14.5834 0825
14 1119 18.9321 0θ37 17.7120 8285
10.3855 9065
9.8974
15 20,0235 8764 20.7840 5420 10.5080 3199 11.4013
18.2867 9814
22.7193 3678 21.5786 6359 1508
21.8245 3114 20.2θ25 7203
17 24.7417 0080 23.6574 9177 13.1807
23.0θ7δ 1239 22.4086 6350
18 28.8550 8370 26.8403 ββ3β 0494
25.6454 1288 24.6411 3θ9θ
20 27.0712 2940 20.0β3δ 6248 28.1323 8407 14.07Ιβ 4264
2β.θθβ4 0260
21 29.7780 7858 31.3714 2277 30.6390 0301 10.8600
29.4812 0488
22 83.7881 3080 33.0659 5410 4120
31.9092 0172 32.1026 7110
23 30.3033 7795 35.7102 5181 18.8821
84.2479 βθ7θ 34.8883 1801
24 38.9370 2096 38.5052 1440 3767
86.6178 8868 37.7860 7550
2-5 41.6891 θβ31 41.4304 7612 21.0150
30.082β 0412 40,8643
44.5652 1015 44.6010 0887 0503
26 41.645θ 0829 44.1118
47.7270 0882 23.2750
27 44.8117 4462 47.5706 4460 47.5379 9825 6988
47.0842 1440 50.7113 2301 51.1134 5370 51.1526 8816
64.0β01 2046 26,0725
40.0β75 8298 58.9θ33 3317 54.0050 8051
58.4025 8277 2808
52.9002 8630 57.4280 3816
81 02.3227 1191
58.08θ1 Οθ43 28.2128 7076
56.0840 3775 61.0070 63.2335 1045
32 66.4888 4750 80.005(1
88 59,3288 64.7623 8779 67.7113 5353 5255
3526 β8.θββ2 4524 70.7607 8988 72.4354 7797 83.7590
85 82,7014 72.7562 2628 75.2988 2937
77.4104 2926 0170
86 6807 77.0302 5646 80.0687 7084
ββ.209δ 82.6774 9787 80.785δ
87 81.40ββ 1800 5938
2742 0861 88.2217 6026 9120
88 86.1630 6681 90.3203 0735
73.6522 2480 04.0771 2207 39,0927 2068
80 θ1.Μ13 4427 96.8383 2272 100.2513 6378 43.3022
.40 77.6983 1385 98.1382 0476 101.8281 3886
81.7022 4640 101.4344 23θ8 106.7βδ1 8870 9028
107.7095 4580 46.0058
85.0703 8026 107.0303 2306 113.6372 7417
48 114,0960 2809 2760
00.40θ1 4971 ΙΙ2.Μβ6 8760 120.8873 2425
44 120.7907 7424 60.8155
05.02δ5 1570 118.9247 8854 128.5301 2708
127.83θ7 €295 136.6056 1407 7786
θθ.82βδ 8033 125.2764 0402 135.2317 5110 54.8645
104.8195 0778 181.9138 4220 142.9933 88ββ 145.1180 2285
1200
110.0123 8160 138.84θθ 6510 151.1430 0550 164.1004 6360
115.4128 146.0982 1953 168.575θ 8910 50.16β8•
159.7001 5587
121.0293 9204 153.8726 3314 178.5726 8272
168.6861 6366 184.11θΙ 6527 63,7057
126.8706 6772 161.6879 0168
178.1104 2186 6568
132.945,3 9043 169.8508 6720 196,2457 1986
188.0253 9294 68.5281
139.2632 0604 178.5080 2828 206.9842 8892
198,42βθ 6259 1162
0F A.NNUITY PER
145,8387 3429 209.3479 9572 219.3683 73.0397 0832
152.ββ70 8386 232.4386 2696 79.0581 8022
246.2174 7846 84.8016 7730
90.8807 7803
0471
104.1837
111.4347 7087
110.1208
127.2681 1800
13δ.θΟ42
0578
145,0584 5813
154.7610 6δβ2
Ιθδ.Ο47Ο
8366
175.0δ05 4457
187.5075 7724
199.7580 3188
212.7435 1370
226.5081 2462
241,0086 1210
256.6645 2882
272.9584
290.3869 0458

ΤΑΒΙ,Ε Π— ΑΜΟΤΝΤ

6%

51 150.7737 187.53δβ 220.8153 280.7594 308.7560 5880


52 6700 0455 9550 3765 328.2814 2230
53 167.1647 190.9747 232.85βΙ 270.1012 348.9783 0773
1708 206.8388 6528 0072 370.9170 0020
55
174.8518 3408 246.408θ 292.2807 394.1720 2057
56 0030 217.1463 7364 7309 418.8223 4810
57 182.8453 7202 268.7789 300.3026
58 444.9516 8905
5805 227.9179 2222 4501 472.6487 9040
50
191.1591 5038 272.7126 327.3774 502.0077 1782
οο
72θθ 280.1742 1833 8502 633.1281 8089
01 109.8055 0756 287.3482 346.3882
02 506.1158 7174
3001 250.9371 802.7156 4733 001.0828 2405
08 208.7977 26.3.2292 7953 0171 360.4343 638.1477 9849
M 218.1496 276.0745 9711 318.8514 2503 077.43θβ 0110
05 7107 289.4979 4479 387.6882 188β 719.0828 8070
227,8750 5885 5898 40θ.90δ5 0562
108
0F 1 PER PERIOD
07 287.θθ0β 8520 303.5253 335.7040 483.4503 7173 708.2278 3241
88 318.1840 1703
248.5103 1201 458.2001 4217 810.021δ 0230
80 259.4507 2511 031ρ 353.5837 869.6227 0260
484.4900
70 270.8287 338.5022 1788 512.1433 854θ 912.2001
71 5412 8333 372.2β2Ω 0378 541.3112 7170 907.9321 0085
282.6010 0428 342.5098 891.8700
78 572.0833 9104
2θ4.θβ83 8045 8008 4897 1027.0080 9983
74 300.2378 604.5479 7818 1089.6285 8682
412.4098
75 807.7671 3090 638.7981 115β.00β3
5141
76 1507 1098 0097
383.7185 484.0033
77 321.0778 074.9320 1341 1220.3ββ0
3835 4308
78 0080 713.0532 7415 7903
401.9868 456.7080
334.9209 753.2712 1800.9486 7977
80 6735 1118
1231 0423
421.0752 480.3379 1380.0056
81 840.3177 4890 796.7011
364.2904 3138 1174 0065
82 2046
5870 441.023θ 1670 605.3698 1463.8050
83 840.4β4Ο 8209
401.8090 0733 3059
85 379.8820 887.6902
7956 5.31.0632 0770 1552.6342
7711 550.5500 937.5132
86 483.0588 9278
0019 0258 0278
87 1618 990.078,4 2898
1046.7923
88 412.8088 688.5285 5085
2200 606.4182 1045.5300
89 1071 1740.6908
οο 430.4147 3081 8252
530.2070 018,θ54θ 1104.0348 9187
7550
91 5747 3025 1731 1852.8958
448.03Ι3
03 555.006,3 7505 060.002β Ι1βδ.7δ07 8485
407.δ7ββ 118 681.0±43 8306 322β 19β4.6300
M
487.27Μ 003 0198 084,4478 1230.8733 8704
507.7708 608.1918 1721 5254 2083.4120
00
7347 5822 710,0702 1299.5713 1022
07
529.0817 630.δ5θθ 0807 8693 2209.4107 3719
08
0841 0934 760.6537 1872.0478 2342.9817 4142
99
661.2442 οσο.2061 1848 1321
100 697.1844 2431.5006
7075 796.48Μ .1448.5104
0052 4591
574.2947 722.5570 0440 4204 2634,6342
7682 9854 830.2607 1529.1785 8400
598.2β05 2462 1730 2793.7123
0086 703.3877 1014.2833
87θ.0737 4174
623.1972 9407 3675
0085 2062.8350
2962 798.7402 1704.0β8θ
ΜΩ.12δΙ 924.0274 8226
4576 1θ21
1870 4889 3141.Ο7δΙ
835.0836
β70.οοοι 971.2288 1708.7927 8718
5080
2345 2134 0077 3330.53Ω0 9841
874.2803 1808.728.3 0881
1080 1020.7002 3531.8720
704.1337 2004.1502
914.034 3612 6240 8082
2839 5579
1072.8207 3744.2544
733.2020 050.7Ω07 0125
0F A.NNUITY PER
7753 1000.8463 7562 2115.3848 0514
763.0310 7085 1127.4712 4Ω8β 3θβθ.0ΟΟΟ 0944
4083 1046.8844 6430 2232.7310 4209.1042
795.1782 0381 11848448 1000 4402.β50δ
8226 1004.9042 2762 2350,5312 0459
827.9833 6468 1246,0870 2262 4781.4096
3354 1145.2000 6889 2487.1404 348€
862.1020 0059 1308.3414 8970 5010.2041
897.5807 1197.8001 2234 2024.9331 6318.2717
7.350 1180 1874.7584 0394 5337
θ34.40Ω2 1252.7073 9345 2770.3044 5638.8680
972.8098 8092 1444,4064 8796 5857
5428 1310.07θ2 1812 2923.0712
1012.7840 193,3 1517.7212 8480
4845 1370.0327 8903 8086,4731
1064.2θβ0 8420 1594.0073 5271
3430 1432.6842 0008 3256.1741
1007.4678 6049 1075.3870 7011
7577 1498.1550 0003 3436.2037
1142.3β05 5117 1760.1046 5580
9080 1500.5720 4033 3826.2582
1189.0012 2847 184θ.Ι007 8237
6443 16,38.0677 0976 7080 8826.7024
1237.0287 1712.7808 1042.δθ62 0080
0401 1939 2040.βθ35
1790,8660 2892
5627
2143.7282
0537
2261.014β
1504
2365.5103
4β42
2484.7858
6374
2010.02δ1
5093
58

110
0F ΟΙ 1 PER PERIOD

TABLE

ΤΙ 7% 8%

1.0000 0000 ι.οοοο 0000 1.0000 0000 1.0000 0000 1,0000


1 2.0βδ0 0000 2.0700 0000 2.0750 0000 2.0800 0000 0000
2 3.1992 2500 3.2149 0000 8.2300 2500 3.2464 2,0850 0000
8 4.4071 7483 4.43Ωθ 4800
4.4729 2188 4.5001 1200 3.2022
4 5.Οθ8β 4098 5.7507 3901 5.8088 9102 6.8806 2500
7.06.37 2764 7.1532 9074 4.6395
ο 7.2440 2034 7.3850 2004
8.5228 0994 8.6640 2109 1413
7 8.7873 2187 8.0228 0830
10.07θ8 5648 10.2508 0257 6,0253
10.4403 7101 10.0300 2703
8 11.7318 5215 11.9770 8875 7283
12.2208 4888 12.4875 5784
10 18.49" 13.8104 4700 7.4200 2052
14.1470 8750 14.4865
11 2264 15.7885 9982 0247
17.8884 5127 10.2081 1000
12 15.8715 6001 18.4237 27θθ 10.6454
18 17.3707 1141 20.140θ 4286 10,8300
22.5504 8780 20.8065 0769 8740
19.4998 0765 3027
14 25.1200 2201 23.3650 2000 18.9771
12.7512
21.7072 20.1183 0470 2640
24.1821 27.8880 6355 4301
20.0772 4208 21.4962 9058 1-1.83δ0 θθ32
17 80.8402 1780
26.7540 1034 82.2580 8521 24.2149 2030
18 29.4930 2101 33.9090 3251 27.1621 1393 8270
37.3789 6479 35.0773 8786 11).5402 4079
20 82.4100 6788
40.θθΜ 9232 89.8531 30.3242 8304 22.2100
21 85.5167 2176 0194 33.7502 3003
22 88.8253 0807 44.8651 7678 43.3046 8184 37.4602 4374 25.0Ω8Β (1560
23 42.3480 5378 49.00δ7 3910 41.4402 28.2822
47.5525 3244
24 46.1010 3673 53.4361 4090 δ2.Ι18θ 7237 8,324 ΟΩΙβ
25 60.0082 4205 58.1700 7070 57,0278 0530 45.7019 6430 31,0320
20 54.3546 2778 63.2490 3772 62.3049 874- 50.4220 2144 1204
27 δ8.887β 68.6764 7090 4 56.4567 5510 36.3207
28 63.7163 7760 74.4838 2328 87,0778 σο.8Ω32 3800
29 εο.8θ7β 9001 8150 Ω557
68.8568 7725 39.3229
87.3485 2927 00.7647 5022
80 74.3325 7427 0538
94.4007 8032 74.0702 0112 73.1050 3095
M 80.1041 9Ι5θ 80.6319 1020 43.0054
86.3748 102.0730 4187 87.0793 0001 70.0644 1515 4998
82 110.2181 5420 95.2552 6810 87.3507 0838
33 92.θ8θ2 8021 48.3770
118.9334 2500 103.3θθ4 0262 95.3388 2083 1323
M 100.0385 8017 128.2687 0481 103.Ωβ59 3022 53.4800 5930
35 107.5357 0903 138.2308 7835 112.1543 5771
115.δ2δ6 8076 121.6050 59,0850
80 148.9134 5984 346,1 2040
124.0340 9020
$7 100.3374 0202 131.0883 7003 05.0630 δ7ΩΩ
88 183.0θβ0 4613 172.5010 2017 128.3458 0800 71.58.32 1882
0F 1 PER PERIOD

39 142.7482 4050 186.6402 9158 142.5500 3310 184.2135 3744 78.0077


40 153.02β8 ΙΩθ.03δΙ 1190 154.2610 0568 145.9506 0242
8259 100,8204 7000 2044
41 214.β00δ 0983 80,3545
103.9730 2905 230.6822 3θ72 180.332Ω 1170 158,0200 7007 5478
175.6819 1590 247.7764 104.8600 1258 172.3108 0808 04.βθ4β
188.0479 9044 266,1208 5125 210.4711 8102 187.1021 0103
4-6 2012711 0981 286.7493 1084 227.2505 1000 4707 103.7437
47 215.3537 8196 245.8007 6857 203.070,3. 1081 4075
49 280,8517 2458 204.0983 1540 220.315Ω 4540 113.5010
50 240.8245 8602 285.660U 8912 238.9412 6871
800.7617 0080 2103 124.2147
208,3856 8475 Ο2βΩ
332,0045 1611 259,0606 2520
281.4525 0426 829,2243 8508 1871
800.7469 1704 358,2700 357.9803 5376 135.7720
821.2954 885.8170 5528 2Μ.78ΙΟ 4021 7084
878,θθ8θ θθ5Ι 804.2435 2342
843.1796 7198 408.5289 2047 416.7633 3442 148.8130
447.080 3451 320.5880 0580 7987
482.5209 101.9203
4,709 170.6835
380.6050 1788 7170
102.701U 7530
418.4200 6877
462.9001 210.0813
5211 1780
400.1321 3428 228.9382
530,3427 3742 2981
573.7701 5642 24Ω.3Ω7θ 7935
0759
2Ω5.β82δ 8024
821.8Ιδδ
5182
860,1008
7872
380,0343
1209
414.3137
2959
460,5308
θ0βΙ
489.8254
8032
532.4β0θ
4015
678,71θ8
0107
628.9109
8416
0F 1 PER PERIOD

β83.3β84
1782
54
ΤΑ.ΒΙ.,Ε VALUE 0F 1 PER. PERI@D
vm—PR.ESENT ΑΝΝΌΤΤΥ 0F

1 ο.0θ58 0.9960 2488 0.9042 0.0025 0.9900 0901


6082 1.θ85Ο 0050 5583 Ι.θ7Ο3
8 1.9875 9938 1.9820 1.θ777 2.θ40θ 8521
2.9761 7263 2.0702 3513 2.θδ55 5624 3.9019 0555
3.058U 7804 4.8534 3124
ο 3.θ504 2,0053 3.9201 ΙΜΙ
4.9381 0281
7 4.9258 6633 3733 4.8894 5.7954 7047
5.9134 8318 3.942,3 4034 3901 ο. 7281
8 0.88-17 5.8003 4.9130 0453 7.0610
ο 7.8520 5900 0.8020 7404 7723
5.8455
7775
10 8.8Ιδ3 2016 7.8229 5924 9703
5.8793 8.5000 1758
9.7740 8.7790 6302 0.7940
11 8084 0.4718 0453
9.7304 1180 3785
12 10.7298 υ.83θ4 7.7306 10.3070
0374 10.0770 8385 2825
13 11.0812 2073 7.7040
1325
8.071δ 7012 11.2550
14 2198 11.6189 1875 7747
8.7430 0.5005
15 12.028C 0280 3207
7068 12.1837
13.5720 12.5501 1781
10 4007
5267 6131 θ.υ805 10.6200 7452
17 18.0037 03M
14.5115 13.4887 1315 11.4340
18 1207
13.8050
8702 0777 10,6245 5262
19 14.4106 12.3423 4608
15.4472 11.5571
20 2406 13.2430 2242 14.7178
2418 2010
21 14.13θθ 0406 7378
16.3789 15.3390 12.4842
22 15.5022
7843 2502 13.4000 15.0243 1201
23 5127
17.30β8 10.2580 9201 15.9050 2402 16.3082
0048 17.1727 14.8225 10.7701 8107 6858
18.2309 8802 4473 17.041.18 2084
20 17.2200
0438 18.082,3 15.2330 18.5080
27 0850
28 10.1511 5024 8100 10.3027 9870 18.045E
29 20.0074 0352 18.9874 10.1395 20.2112 1459 5297
30 20.9800 7053 1915 3432 21.0533 1473
17.0401 21.8891 18.8669
31 21.8888 7289 10.8879
3354 22.7187 5547 8813
22.7938 9831 7025
32 17.9355 10.6008
23.βθ51 6843 20.7840 5800 28.5421 8005
33 Οθ74 7934
21.075β 8055 24.3594 0280
35 24.5020 9884 18.8260 20.4558 2113
22.5028 0022 25.1707 1251
25.4805 0500 0320 21.2433 8720
80 20.370U 0254 23.4450 3803 25.9758 9331 22.0231 5570
37 27.2030 0008 24.3240 1704 10.7107 20.7750 8021
38 1404 22.7952 0300
28.1457 3278 25.1080 2780 27.5083 1783 23.5590 0750
114
ΤΑ.ΒΙ.,Ε VALUE 0F 1 PER. PERI@D

30 20.0247 0012 20.007U 8036 20.5900 28,3550 5045 24,3104 4310


40 20.9002 1189 20.Ω330 0220 29.1371 2209 25.0857 8630
80.7710 0524 2423 21.4053 20.9127 7621 26.8077 0822
41 27.7940 5307
31.6401 0122 8745 80.6820 5820 26.5422 8537
43 32.5(Η7 22.3360
28.0507 9007 31.4408 0525 27.2005 8947
44 2480 20.50.32 8355 9038
32.2052 0570 27.98ΩΟ θ255
80.8516 23.1997 28.7020 0680
4-0 33.3067 32.9580 8010
25Ω2 0741 29.4086 8000
47 0100 33.7062
31.1956 4818 24.0594
48 34.2231 34.4409 3844 30.1075 0504
82.0353 7132 2079
40 0481 95.1830 6546 30.7905
50 36.070Ω 5084 32.8710 1024 24.0140 31.4840
35.1)272 5,304 35.9137
33.7025 0372 8802 82.1630 3298
30.7740 12130
34.6298 5445 25.7637 32.8340 8011
2881 30.0380 2070
35.35.30 8000 9970 39.4ΩΩ0 8022
30.1722 2786 37.3687 3022
37.0172 20.8085 34.1581
38.0731 8180
ΩΟOΩ *0.0872 8307 0814
38.4570 ΩΙ41 27.4484 98.7823 1401 34.8100
5236 37.7082 0702 39.4801 6774 08011
30.2033 ΩΟ91 28.2834 40,1847 8180 36.4554 5352
3013 38.0062 8000 40.8781 9642 0844
40.1201 39,4082 3238 2Ω.Ι18Ο 5044 41.5664 4707
29.9.300 0025 30.7272 8008
8788 40.2071 0040
30.7696 37.3630
40.05M
41.002Ι 8647 7640 9ΩΟΩ
8Ωθθ 37.Ω73Ω 5040
41.7032 1037 31.6753
41.7814 42.5803 1778 38 5880 7871
0081 32.3864 3011001
43.3036
42,0038 0403 1753
(Υ)28
8401 33.1028
44.1427 8035
43.422Ω 5602 3074
44.2380 3,3.0Ω45 3828
27ΩΩ 34.7915
45.050Ω 1582 8736
35.6840
13Ι)β
30.3718
4,187
37.1551
Ωθ7Ο
37.0838
2012
38.7080
2020
30.4777
4248

115
ΤΑ.ΒΙ.,Ε VALUE 0F 1 PER. PERI@D

40.2429
9170
41.0038
0287
41.71502 0170
42.5122
1380
43.2598
0400

116
VALTE 0F PER PERIOD

TABLE vm—PRESENT .ΑΝΠΙΤΥ 0F 1

1%

51 45.8598 3317 4-4.9Ι8Ι 9537 44.003Ι 7940 42.2495 7525 30.7081


58 46.6653 9401 45.0897 4604 44.7421 8385 42.9270 1812 3017
47.4676 1228 46.4574 5θ34 45.4709 0144 43.υοου 1351 40.3041
48.2ββδ 0184 47.2213 5258 40.2073 5853 44.2685 9902 40.9843
50 4θ.0β20 7651 47.9814 4535 40.9385 7θ33 44.9310 1108 5072
41.51580 ΜΟΒ
58 49.8543 5003 48.7377 5057 47.6655 8841 45.589C 8020
42.1471
50.6483 3012 49.4903 0505 48.3734 1020 40.2428 077υ
80 0211]
51.4290 4840 50.2391 0950 49.0870 0898 40.8011 8388
81 52.2115 0046 60.9841 8855 40.7θβδ 8889 47.5340 7382 42.7109
02 52.9907 0584 51.7255 6075 50.6019 9894 48.1783 7352 0224
08 δ3.7βββ 7800 43.2871
52.4632 4453 51.2038 0800 48.8078 1863
M M.5394 3036 49.4305 4456 2102
85 53.1972 5824 51.9005 5478
δδ.308θ 7627 δ0.0β10 8640 43.8480
53.927β 2014 52.5937 5787
66.0763 2905 44.4046
M 54.6648 4889 53.2829 4078 50.0809 7000
67 56.838δ 0194 δδ.3774 Μ.θβ81 2068 51.2902 6713 8870
68 δ7.δθ8δ 44.0550 38-
56.090θ 7621 54.6493 3888 51.0060 M97
69 0814 41
δθ.8Ι2θ ΙΙθδ 55.3206 0040 52.5131 0007
70 58.3δδ3 67.5252 8522 53.1147 4007 45.5000
6078 55.9θ9θ 3413 53.7110 0077 3803
71 58.2841 1405 56.0093 0287
59.1090 58.9894 17δβ δ4.3ΜΟ 40.030β
72 7296 57.3349 0925 2210 4101
78 δ9.8δ9β 69.6412 ΙΙδΙ δ7.θ9β5 9579 Μ.8Ο2θ 40.5730
74 5770 60.8395 1394 58.6544 4488 2516 0258
75 60.6071 61.0343 4222 59.3084 7877
2798 61.7257 56.4708 47.1028
δ9.Ο587 4880 δβ.0504
77 62.4136 4643 1069 60.6051 7385
61.3514 9672 2501 47.020υ
78 8984
62.0927 7680 63.0981 5460 56.8316 870J 0777
70
02.830θ 8108 63.7792 6830 61.2479 0088 57.2020 6704
80 48,1451
88.δββ1 2216 64.4569 7360 61.8869 0207
81 64.2982 1292 65.1313 1691 67.7693 0740 6021
62.5221 9021
82 66.8028 0638 58.3310 0815 48,0585
65,0272 63.1587 9310
88 58,8002 40.101JO 0149
85.7532 9388 63.7817 8801 9141 40.070Ι
66.4763 0924 64.4000 8118 50.4443 50.1085
67.1963 2453 66.469-9 5561 66.0207 0874 9842 1435
87 67.9183 5221 67.1842 8410 65.6437 8667 δΟ.θθ44
4012 50.6018
88 68.8274 0467 67.7958 0705 66.2572 8686 9539
89 69.3384 9426 68.4530 4244 66.8672 2705 60.5403 8722 51.1603
VALTE 0F PER PERIO

91 70.04ββ 3326 69.1075 0491 67.4736 3089 61.0822 7019 9148


70.7518 8393 68.07βδ 1780 01.0201 Ι 030 51.0340
18 69,7587 1136
71.4541 0846 70.4066 7796 68.6759 0846 62.1530 6450 52.1120
M 69.2718 2283 62.6888 8579 2175
72.1534 6898 71.0514 2086
72.8490 2769 71.6929 5008 69.8842 8121 03.2007 0257 52.5870
K 5124
97 73.5434 9633 72.3312 9058 70,4633 0363 03.73Ι7 7427
742341 8720 72.9θβ4 0725 71.0389 1001 64.2409 0002 53,0504
74.9220 1212 71.6211 2017 64.7041 0876 8637
73.5984 7487
72.1θθθ 5879 βδ.274β 0018 53.5212
75.8069 8800 74.2273 3818
76,2891 1168 74.8580 7282 72.77M 3047 66.7812 4981 7304
78.9684 0995 75.47δβ 9484 73.3475 66.2841 1892 53.0814
77.6448 8955 78.9163 9075 66.7832 4458 5005
76,0952 1825 54.4370
78.3185 6218 76.7ΙΙβ 74.4819 1294 67.2780 6467
75.0441 5639 67.7703 7685 8817
78,9894 8950 77.8250 8478 64.8882
79.6675 3808 77.9353 5799 7δ.Μ8Ι 8712 68,2584 8856 0β1Ι
80.8228 5450 78.6426 4477 68.7428 6705
80.θ8δ4 1624 69,2236 8938 55.8348
81.8452 2677 69.7009 3230 5759
70.1746 2272 66.7770
50.2140
8729
ao.M84
6270
δ7.Ο77β
7000
57.5020
4051
57.02,34
58,3400
11320
68.7524
9030
59.1008
8148
52,5052
2010
734
60.3619
5302
60.7644
0982
61.1429
8002
61,5277
VALTE 0F PER PERIOD

0299
01.θ08β
1082
62.2857
δθ23
62.6691
β7δ5
63.0288
7877
ΤΑΒΙ,Ε VA.LUE 0F PER PERIOD

wu—PRBSENT ANNUITY 1
(α ati) =
η 1 1%

101 82.3023 0049 70.1469 1021 70.1588 7702 70.6447 8882 63.3949 2947
102 82.9606 4777 79.7481 0037 70.711,3 9392 71.1114 δ0θ4 63.7673 5691
108 83.6082 7991 80.8404 3718 77.2β07 0648 71.5740 4113 64.1161 9397
77.80(58 3331
105 84.2572 0818 80.9417 2864 72.0343 8325 64.4714 7918
84.0084 4381 81.6340 5825 78.3407 9288 72.4007 0298 64.8232 4871
100 78.8800 0855
107 85.Μθθ 9796 72.9436 2579 85.1716 3140
88.1878 8175 79.4202 8350 73.3931 65.5103 0772
108 70.9508 5115
86.8201 0028 82.7098 9158 73.8393 8160 66.8577 8983
100 80.4003 2428 ββ.1958 3151
110 87.4016 8258 83.2934 2440 74.2822 6461
88.094θ 2163 83.8740 5419 81.0177 2003 74.7218 5073 68.5305 2626
111 84.4517 9522 81.5420 5895 οο.8β1θ 0718
112 88.7249 3437 75.1581 6450
85.0200 0101 82.0033 5600 • 67.1900
118 89.3520 3171 76.5912 3027
82.6810 29θΙ 0710
114 80.9777 2450 85.5980 0860 70.0210 7223
83.0θ08 9808 67.6148 5852
115 90.6002 2354 80.1078 2942 76.4477 1437
83.β0θΙ 7785 67.8364 9358
91.2201 3060 80.7341 5802 70.8711 8062
118 68.1640 44 Ι4
91.8374 8338 87.2970 7027 84.1184 8071 77.2914 9431
117 84.6248 4182 88.4702 4172
118 92.4522 0558 87.8588 7838 77.7080 7022 68.7824 1765
93.064-4 88.4102 86.1282 0033 78.1227 6853
110 86.0287 6920 69.0915 0252
03.β741 8767 88.9714 3070 78.5337 5636 69.3975 2726
120 04.2813 4869 86.1203
80.5238 2059 78.9416 9207 αρ.7005 2203
121 94.8859 90.0734 5338 80.6210 σαο2
122 79,3405 9322 70.0005 1686
0036 90.6203 5157 87.1129 0742
128 79.7484 7062 70.2976 4146
95.4881 87.6018 θβ88
2315 91,1045 2892 80.1473 7432 70.5916 2520
125 91.7069 98θ3 88.0880 4946 80.5482 9957
θθ.0877 88.5713 8308 70.8827 9722
120 5747 92.2447 7605 80.9362 7749 71.1710 8636
127 Θβ.β84θ 92.7808 7070 80.0510 1301 81.3203 3001
8θ.δ2θβ 6731 71.4565 2116
128 0307 93.3142 9920 81.7134 7892
97.2795 90.004β 3032 71.7391 2985
130 93.8450 7384 82.0977 4683 72.018θ 4046
7209 Μ,8732 0780 90.4768 4873
82.4791 5219 72.2969 8064
181 04.8087 1422
οο.θ4θ3 2861
97.8717 7301 82.8577 1929 72.5702 7786
182 95.4210 01.4130 8564
98.4015 83.2334 0828
138 01.8771 3601 72.8418 5927
9θ.Μ88 1324 95.9418 9071 83.00β4 2013
185 92.3384 9442 73.1107 5175
90.038β 7290 98.4595 9872 83.9785
92.7971 7768 73.8769 8198
180 1100.2161 0570 90.9747 2500 84,3440 1554
θ3.2δ32 73.6405 7617
137 100.7001 97.4872 8865 84.7087 0029
03.70θ6 7892 73.0015
188 2180 97.9973 0214
85.070β 7020 βΟδθ

120
ΤΑΒΙ,Ε VA.LUE 0F PER PERIOD

180 101.3737 08.5047 7826 94.1573 2787 85.4299 4567 74.1599 6095
11-0 3131 ΙΟΙ.Μ80 90.00θ7 2080 94.6054 85.7885 74.4158 0293
4-401 99.5121 087δ 0270 Ι1β57 74.0801 1181
141 102.6217 05.050θ
100.0121 0821 80.1404 9288 74.ριρρ 1268
19 0994
100.5096 0041 θ8δ7 80.4018 0484 75.1682 3038
108.0022 1800 θδ,4θ3θ
1" 103,6608 0104
101.0045 8772 6050 86.8405 0050
87.1800 0108
75.4140
8248
144 101.4070 5240
104.2260 2590 05.9343 3304
101.9871 1088 87.5301 2514 75.0675
104.7804 0338 90.3721 0272
14-0 102.4747 4310 87.8710 0105 1434
105.3504 4314 90.8074 5201
147 102.θδ90 4344 88,2005 2056 75.8986
105.0091 6496 97.2402 1804 2006
10 100.4656 4847 07.0704 7304 88.6464 2982
88.8788 8864 76.1371
149 107.019β 3330 103.4427 98,0082 3307 5747
80.2007 0530
107.5714 1002 103.0231 1422 98.5235 18Μ 89.5382 2858 70.3734
108.120θ 1517 104.4011 0808 2663 89.8642 4078 2824
108,0081 3120 104.8707 8705 70.6073
109.2130 7074 2505 90.7840 4974
100.7557 0103 105.3490 1078 70.8389
110.2901 9363 7618 0014
110.8343 83δβ 77,0682
111.3703 4044 7787
77.2958
2413
77.6201
2290

121
VALTE 0F PER PERIOD

TA.BLE vm—PRESENT ΑΝΝΤΙΤΥ 0F 1


(αλ-ι αι ί) =

η 2%

1 0.9888 7516 0.9878 5432 0.9852 2107 0.9828 0098 0.9803


1.9ββ7 1.9831 1.9558 8342 1.9486 9875 9216
8 4928 1588 2,9122 0042 2.8079 8403 ι.θ41δ
δ 2.9337 2.9205 3371 3.8543 8405 3.8300 4254 2.8838
8.8899 8280 3.8780 5798 4,7826 4497 4.7478 5508 8327
7 4.8355 8200 4.8178 8504 3.8077
5.6971 8717 5.6489 9702
8 δ.770θ 8205 5.7460 0992 0.5982 0.5340 4139 2870
9 6.6963 3948 6.6627 2585 7.4859 2508 7.4050 5297 4.7184
10 7.6097 3002 7.5081 2429 8,3805 1732 8.2004 04- 5051
8.6139 4810 8.4823 4498 9.2221 8466 32 5,0014
11 9.4081 0890
12 0.3455 2501 9.1012 2291 3080
10.2923 10.2178 10.071 ι 1779 9.9274 9181 0.4719
18
1832 0387 10.0075 0521 10.7306 0107
14
11.1θββ 11.0798 11.7315 8222 11.6376 4097 7.3254
15 12.6483 8150 8144
9302 1197 12.8220 0587
10 12.0318 40" 13.3482 3301 8.1022
17 11.9801 18.0928 8046
12.8863 8466 14.1312 0405 8071
18 13.8504 9677 8.9826
6880 12.7705 14,9070
19 14.5050 8282 8501
13.7318 5276 4031
20 15.3208 0272
8609 13.6005 15.6725 0089 10.040Ο 5073 0.7808
21 4592 16.4261 4805
14.6879 10.7528 8180
28 2514 6887 10.5753
u 14.4202 17.4475 4919
15.3948 9227 17.1680 3879 4122
18.1302 0948 11.348,3 7376
20 0860 15.22θθ 17.9001 3073 18.8012 4764
27 16.2124 12.1002
1829 18,0208 2437 19.4006 8566
1395 4877
16.029δ 19.3308 6146 20.1087 8100
80 17.020-9 2850 12.8402
4893 20.0304 0537
17.8204 20.7457 3106 0850
81 18.8103 20.71θβ 1120
4846 21.3717 13,6777
82 0759 21.3980 3172
18.0110 281,4 0981
88 17.5993 22.0670 1746
7387 21.9869 6474 14.2918
84 1613 22.7267 1071
19.8929 22.5010 0171 7188
85 18.3696 23.3760 7558 23.1858 4034
0371 24.0158 3801 149920
86 20.1660 19.1805 28.70Ω8 7650 8125
87 8580 0291 24.6461 4682 24.3438 5807 15.8784
88 20.9805 19.8820 25.2671 3874 24.0079 0061 8201
86 21.6865 3744 2δ.878θ 5442 25.4023 7789 16.3614
40 9276 20.6242 26.4817 2849 20.0072 5100 3884
8451 27.07δδ 9458
41 22.4342 28.6427 5283 17.0112
122
VALTE 0F PER PERIOD

48 0792 21.3572 27.8606 8431 27.0βρο 4455 0010


Α 28.1735 6885 28,2871 2740 27.5802 8467 17.0580
0698 22.0812 28.8050 6103 28.094β 2867 4820
28.9045 5299 29,3045 8288 28.5942 2955 18.2922
47 7940 20.9158 4620 0412
48 22.7962 θθ25 29.0882 8789
24.6275 23.5025 80.4589 0079 29.5078 0135 18.9139
1988 1778 30.9040 6004 30.042Ο 6522 2580
25.8424 24.2000 31.5212 8167 30.5081 7221 19.6284
1766 1756 32.040β 2223 30.9002 0201 5647
26.0493 24.8889 82.5523 8718 31.4164 7481 20.1210
8233 0023 38.0584 8983 31.8589 4281 3570
26.7484 25.5892 33.5531 θ1θ6 32.2938 0129 20.7068
4236 9010 34.0425 5305 32.7211 8088 0780
27.4397 26.2412 34.5246 8339 33.1412 0948 21.2812
4522 7418 8807 7236
28.1238 28.9049 21.8443
5745 8215 8400
28.7993 27.6804 22.8964
99.4678 5844 5δ5δ
5127 28.2078 22.9977
80.1289 6822 0152
0114 28.8472 28.4083
30.7825 8797 3482
31,4290 20.4787 23.088δ
2044 8259 0855
82Λ682 30.1026 24.408δ
6260 0138 0172
82.7903 80.7186 24.9980
7840 1983 1933
88.3254 31.8269 25.4888
8196 81.9278 4248
88.9435 8522 25.θβθ4
9049 32.5213 5841
84.5548 1874 26.4400
5488 33.1074 4060
8Μ598 1212 7530 20.9025
85.7570 38.8883 8883
4586 9538 27.8564
36.8481 84.2581 7024
2891 6825 27.7904
86.932β 84.8228 8045
3674 8222 28.2347
87.5108 2442 9858
4202 35.9314• 8091 28.0β1δ
123
VALTE 0F PER PERIOD

88.0822 38.4755 0233


1708 3870 29.079θ
87,0128 0807
7574 20.4901
5987
20.8923
80.2866
8196
30.8791
1067
31.0520
7801
31.423β
0689

124
ΤΑΒΙ,Ε VA.LUE 0F PER PERIOD

vm— PR.ESENT

η 2%

51 38.6474 3345 37.5435 8009 35.4076 7298 83.5540 1421 31.7878 4892
52 30.2063 0188 38.0β77 3431 35.9287 4185 38.9697 1913 32.1449 4992
53 7232 38.6864 30.3820 9090 34.3684 4033 32.4060 4804
40.3060 3304 39.0007 0770 30.8305 3882 84.7603 1679 82,8382 8327
55
40.8401 39.υοιο 8607 37.2714 0081 36.13M 4560 33.1747 8752
56
41.3805 8358 40.1004 3128 87.7058 7803 36.5130 5136 38.5040 0305
58 41.9091 0013 40.6930 1855 38.1338 7058 85.8850 4727 83.8281 8103
69 42.4317 4896 41.079δ 2449 88.5556 3751 30.2516 4528 84.1462 2060
οο
σι 42.0486 7740 41.5000 2410 38.θ7ΟΩ 7292 5520 34.4501 0441
48.4500 δβ38 42.034δ 0170 80.3802 0889 3β.9θ3θ 8552 34.7008 8068
43.θΟδΟ 4052 42.5033 0054 30.7885 1614 37.8110 4228 36.059β 0282
05 2029 42.Ωθθ2 2275 40.1808 0408 37,0521 3000 36.3528 4002
00 44.9500 8119 43.4234 2088 40.5722 2077 37,9873 5185 8δ.Ο3Ο8 4310
07 45.4477 4407 43.874-9 0247 40,0578 5298 38.3108 0728 35.9214 1480
08 45.9310 2000 44.3209 8022 41.3377 8018 38.0405 9678 36.1974
70 40.4080 1075 44.7014 0195 41.7121 0461 38.9588 1748 30.4081 0348
40.8815 0284 45.1906 05β3 42.0808 0125 39,2715 36.7334 3478
71 45,0201 7840 2783,1 39.5789 3875 6351
47.3488 2852
72 40.0606 42.8021 9490 30.8810 1597 37.2485 9108
47.8100 5527
73 4051J 43.1548 7183 40.1779 0207 87.4086 1020
48.2079
74 40.4000 7502 43.5023 3078
75 48.7108 4270 40.4090 8321 37.7437 4441
40.8830 8024 43.84-16 0077
49,1007 1714 40.7564 4642 37.0840 0314
70 49.13080 -47.2024 7431 4,1819 41.0382 7600 38.210U 0075
77 2010 δΟ.ΜδΟ 47.0θβ2 7093 3771 41.3152 5857 38.450β 5002
78 0708 50.4777 48.0060 8240 44.5112 2434 41,5874 7771 38.6771 1438
79 3250 48.4880 7027 44,8416 0084
80 41.8550 1496 38.8091 8170
50.9050 5077 48.8779 9633 45.1041 3820 42,1179 5081 39.1107 0578
81 51.327U 1611) 40.2022 1701 46.4810 0902 42.3703 0443 30.8301 9104
82 61.7454 7&17 45.7940 8485 42.15303 3350 3Ω.53Μ 0380
83 52.1580 0317 40.10ιΜ 3386 42.8709 8474 3θ.744δ 1359
85 52.5073 1092 50.0104 0027 40.4073 2349 43.1252 4298 30.0450
52,0713 8280 50.3800 6700 40,7007 2205 48.3β03 3217
80 40.1427
53.3709 5957 47,0010 0720 43,0082 7480
87 50.7622 5389 40.3360 2011
53.7600 0104 47.2023 1251 43.8301 4237
88 51.1133 3717 40.5255 1579
54.1508 2074 47,5780 3301 44.0050 0470
80 51.4099 40.7112 8090
54.6482 1557 47.8007 2218
51.8221 8632 44.2809 8009 40.8084 2150

125
ΤΑΒΙ,Ε VA.LUE 0F PER PERIOD

91 51.925,3 0688 52.1700 5958 48.1886 4264 44,5100 8809 41.0719 8102
02 65,3031 4540 48.4124 6571 44.7282 41.2470 4110
65.15707 52.5130 3000
08 52.8620 7088 48.6822 2237 44.9417 0855 41.4Ι8β 0774
94 50.0402 48,0480 0234 45.1510 1037 41.6869 2010
1)121) 53.1881 2531
53.5101 3011 40.2008 6452 45.8578 4803 41.7518 0133
50.4110 30-
11 53.8400 0035 49,4078 3090 45.5005 3800 41.9130
08 54.108Ω 40.7220 45,7507 4310 18115
56.7729 3400 01186 45.9556 2147 42.0721 7645
00 54.4878 6037
57,1302 1002 4Ω.0724 2066 40.1470 3266 42,2270
100 54.8028 ΙδΙ3
57.4835 3021 50,21 ΩΙ 22ΩΩ
67.832Ω ΩΩ07 65.1138 40.3370 3455
65.4211 2744 3855 413.5228 8408 42.3800 2264
58.1784 60,4022 0064
02114 56,7245 7031 40.7055 3718 42.6204 8880
50.0242 50.7010 7541 40.8880 4882 42.076D
58.5200 5235 60,0370 1124 47,0014 7304
50.3202 42.8195 2505
58.8679 0308 51.1700 42.9603 1807
50.1910 δβ.υ12β ΙΗ)84 43.098,3
60.5223 0610 61.3000 7422 5104
50.8400 50.9018 51.0247
ΙΥ2δ1 8931} 0307

126
ΤΑΒΙ,Ε VA.LUE 0F PER PERIOD

VII— PRESENT ΑΝΠΙΤΥ 1


2ξ7ο 2*70
3%

0.9779 9511 0.9756 0.0732 8001 0.9708 7379 ο.θββ1 8357


1 1.984 1.9274 2415 1.θ2Μ 2484 1.91.34 0970 1.8990
2 2.8560 2956 2.8422 6213 2.8280 1135 9428
2.8θθ8 9687
8 3.7β1θ 7421 3.7394 2787 3.7170 9840 2.8016 3098
3.7847 4021
4.6458 2850 4,6126 8180 4.5707 0719 3.0730 7021
5 4.β7θ4 5253
4.5150 5238
6 5.5544 7680 5.5081 2586 5.4628 6078 5.4171 9144
6.3498 0.2894 0800 6.2302 8206 5.8285 5902
8 6.4102 4626
7.1701 3717 7.0Μ3 1441 7.01Ωβ 9219 6.1145 4898
7.2471 8461
9 8.0657 0622 7.9708 6553 7.8770 7820 7.7801 0892 0.8780
10 8.8662 1635 8.7520 8.0400 7610 8.5302 0284 5664
9.2520 2411 7,0070 8051
11 θ.β4θΙ 1184 9.5142 0871 9.8820 6920
8.3100 0632
12 10.2577 10.1042 0360 ρ.θ540 0899
10.4147 7882 10.034θ 5683
13 11.1635 9787 10.9831 10.8070 1086 9.001δ 5164
8497 ιι.βθ09 11.2900 7314 9.0038
14 11.8959 8924 11.4910 0814
1217 11.9870 3509 3438
1.5 12.6121 12.1500 0892
12.8813 12.5011 0203 10,8027
1C 18.3126 7778 12.8045 7315
18.1001 1847 3842
17 8131 18.4351 0769
13.0560 13.7585 1308 10.θ20δ
18 13.θ97β 8843 14.0487 0061
13.7121 14.040Ο 0157 14.8237 9911 2028
19 14.β67β 0772 14.8774 7480 11.5174
15.3228 15.2272 5213
20 14.8588 1090
9δθΟ Ιά9θ87 Ιδ.792θ 4612 15.4150 2414
21 1237 6363 16.343-4 15.93βθ 1064 12.094Ι 1681
n 14.9788 9987 Ιβ.443β 0839 12.6513
28 16.5904 9134 10.8798 1801 2059
u 10.9355 4212
2775 15.5891 17.4007 13.1890
17.4131 4709
17.2033 6229 179088 1795 8178
28 17.8768 4242
5232 13,7098
27 16.1845 18.4022 6692 18.3270 3147
17.8027 8742
28 4857 18.8820 7413 18.7641 0823
8955 14.2124
80 16.7654 19.3508 2040 19.1884 5450
18.8890 0890
1324 19.8081 5708 Ιθ.βΟΜ 4135
81 8624
17.882Ι 1048 20.2493 0130 14.0970
82 18.9628 20.0004 2840
17.8849 7420
88 8268 20.β8Οδ 8520 20.8887 0553
8583 21.1008 2023 15.1071
35 19.5281 18.4248 20.7667 9178
21.5088 8882 2484
1260 7642 21.1318 3068
21.9064 0712 15.6204
87 20.0715 21.4872 2007
18.9506 22,2938 4020 1047
88 037β 21.8322 5250 10.0583
1114 22.6609 1753
89 20.6078 22.1872 3644 0760
ΙΘ.4840 23.0864 1609
2764 22.4924 6159 10.4815
1087 23.8931
127
ΤΑΒΙ,Ε VA.LUE 0F PER PERIOD

21.1328 Ιθ.θβ48 8866 05138 22.8082 1513 1450


4977 20.4535 23.7402 4884 23.1147 7107 10,8008
21.Μδ3 2985 4991 24.0781 0106 28.4123 9997 6220
22.1470 20.9302 24.4000 1101 23.7018 5920 17.2853
2186 9259 24.7269 2009 23.9810 0213 3451
22.6376 21.8964 25,0883 24.2542 7392 17.0β70
7419 0741 25.3414 7507 24.6187 1264 1885
23.117δ 21.8491 25.0804 7209 18.0357
24.7764 4007
2977 7796 26.2235 7381 6700
25.0247 0783
23.58β8 22,2918 26.2029 9154 18.8ύ20
25,2667
2618 8094 28.4749 4541
Οββ4
24.04δ7 9577 22.7287 26.7896 9216 18.7302
2δ.δΟ1θ
24.4946 8628 26.9971 6998 7670
25.7297 6401
6579 28.1451 19.0θ88
24.9336 5734 0547
5848 23.5662 19.3902
25.3629 5107 0818
9118 23.θδ73 19.7006
25.7828 1812 8423
7640 24.8486 20.οοοο
26.1936 0304 0110
2221 24.7808 20.2904
26.5961 1448 0381
8174 25.1027 20.5705
2β.θ87θ 7605 2542
0300 25.4661 20.8410
27.8720 2200 8730
8316 25.8206 21.1024
27.7477 0683 0087
28.1151 2β.1ββ4 21.8δδ0
1960 26.5038 7234
4-460 4945 21.δθΩ1
28,8258 26.8830 0371
6269 2386 21.8348
2θ.1βθδ 27.1641 8281
4777 6902 22.062Ο 8870
29.6056 27.4β74 22.2827
7019 8255 9102
29.8848 27.7731 22.4954
9627 5871 5028
28.07Ι3 6947 22.7000
28.8623 1813
1168 22.8θθ4

128
ΤΑΒΙ,Ε VA.LUE 0F PER PERIOD

3780
28.0θ12
4425
23.2765
23.4550
1757

129
ΤΑΒΙ,Ε vm—PRESENT 0FΑΝΤΙΥ 'PER PERIOD

1
8%
ΤΑΒΙ,Ε vm—PRESENT 0F 'PER PERIOD

5774 27.2478 6400 25.9612 2719 23.628β 1030


8072 27.4918 2871 26.16132 3900 23.7257 6464
4948 27.7202 7308 2β.374θ 9028 23.0572 6043
2876 27.0β03 20.5776 6047 24.1132 9610
7028 6,308 26.7744 2764 24.2640 5328
5784 28.1852 6870 26.0054 6373 24.4007 1327
1740 28.4041 5454 27.1500 24.5504 4700
0722 28.0171 8203 24.6864 2281
27.3310
290 28.8245 0800 24.8177 0981
27.5058 8058
20.0202 8522 24.9447 3412
27.6755 6307
9657 29.2226 β201 26.007,3 7596
2836 27.8403 5307
29,4137 8298 26.1868 7040
8377 28.0003 4270
2θ,5θθ7 8879 26.3003 5706
28.1556 72β1
20.7808 1834 26.4109 7388
28.3064 7820
2θ.0δβ9 9887 25.5178 4016
7705 28.4528 θΙδ2
30.1284 6005 25.6211 1030
0208 28.5960 4031
0878 30.2063 4400 28,7880 4884 25.7208 7051
10θθ 30.4577 5581 28.8670 3771 25.8172 7480
0340 80.βΙ58 2074 28.θ971 2399 25.9104 1052
30.7βθ0 5522 20.1234 2135 20.000,3
80.θ1θ3 7247 26.087.3 3975
7008 20.2400 4016
7803 31.0050 8270 20,3050 8752 20.1713 4275
5417 31.2068 θ314 29.4806 6750 26.2525 0608
0806 31.3"0 20.5928 8106 26.3300 2278
26.40ββ 8868
4044 0816 20.7018 2828
4433 31.4792 2930 29.8075 9833 26.4708 0244
1398 31.β0θ9 5658 29.θΙΟ2 8064 26,5506 2072
4047 30.000θ 8004 28.018θ 5721
31.7371 8804 26.6840 8281
1206 31.8010 0640 30.10β7 8036
1722 80.2007 6346 20.7487 7567
31.θ816 1377
32,0087 30.2020 0335 26.8104 1127
3875
32,2120 4008 30.3806 8577 20.8000 6258
070
80.4686 8813 20.9275 0008
8074 32.3240 8015
80.5500 8556 2β.θ830 9186
2028 32.4321 4618
80.6811 5103 27.0308 0373
Ι48β 32.5378 8860
82.β3θ7 74βθ 80.7008 5537 27.088β 0026
1038
82.7304 4000 30.7862 6735 27.1388 3080
0671
30.8004 5374 27.1872 8480
4801 32.8364 8804
30.9324 7086 27.2340 9108
12βθ 32.9308 3904
31.0024 0714 27.2703 1664
0848 33.0227 1527
81.0702 0820 27.3230 1028
38.1121 3165
ΤΑΒΙ,Ε vm—PRESENT 0F 'PER PERIOD

8144 33.1921 6480 81.1302 1184 27.3652 2732


1604 33.2838 4005 81.2002 0507 27.4000 1673
3510 33.3602 7844 81.282,3 3560 27,4454 2680
0010 33,4464 θ77θ 81.3226 5502 27.4835 0415
7080 83.5245 7202 81.3812 1934 27.5202 9387
0574 33.θΟ0δ 31.4380 7703 27.5668 3048
βΙ70 6671 81.4032 7807 27.5001 8308
0434 31.5408 7260 27.02-83 6520
33.8745 0775
6700 31.508θ 0534 27.6654 2640
33.7464 7056
0520 33.8105 2512
33,8846 0508
33,0510 4232
vm—PRESENT VALUE ANNUITY PER PERIOD
TABLE 0F 1

6%

ο.9β1δ 3846 0.95βθ 3780 0.9523 8095 0.9478 0730 0.9438


1 1.8860 9467 1.8720 1.8594 1043 1.8463 1971 0623
2 2.7750 9103 6775 2.7232 4803 2.697Ω 3338 1.8333
8 3.6298 9522 2.7489 6435 3.5450 5050 3.6051 5012 9207
4.4518 2233 3.5875 2570 4.3294 7607 4.2702 8448 2.0730
5.2421 3686 4.3899 7074 5.07δβ 0200 4.095δ ΙΙΩ5
8 6.0020 5407 3.4051
5.1578 7248 5.7863 7340 30,31
7 0501
6.7327 4487 5.8927 0094 6.4632 1276 5.8820 0712
8 7.4353 8161 8.5958 8007 7.1078 2168 6.3345 0500 4.2123
10 8.1108 9578 7.2887 0050 7.7217 3493 6.0521 0525 6370
11 8.7604 7671 7.9127 1818 8.3064 1422 7.5370 2583 4.9173
12 9.3850 7376 8.5280 1092 8.8632 5104 8.0925 3633 2433
vm—PRESENT VALUE 0F ANNUITY PER PERIOD
18 0.9856 4785 9.1185 8078 9.3935 7299 8.6185 1785 5.6823
14 10.5831 2298 9.6828 5242 9.898β 4094 9.1170 7858 8144
11.1183 8743 10.2228 10.3796 5804 0.5806 4790 0.2007
15 2528 10.0375 8004 0881
11.8522 10.8877
17 9561 10.7895 11.2740 0.8010
10.4β21 6203
12.1656 4578 ΙΙ.6έ95 8600 0227
18 10.8648 0860
7.3000
6885 11.2340 12.0853 2086 11.2400 7447
19 8705
12.6592 1505 12.4622 1034 11.007β 6352
20
9697 11.7071 12.8211 5271 11.9503 8240 7.8808
21 13.1339 8940 9143 7458
22 13.1630 0258 12.2752 4406
13.5903 12.169θ 13.4885 7388 8.3838
28 12.6831 8073
2684 9180 13.7086 4170 4304
12.8750 4240
25 12.5932 8.8520
14.0291 14.0939 4457 13.1516 0806
28 9359 8200
5995 18.007D 3645 14.3761 8530 13.4139 3206
27 0.2049
14.4511 14.6430 3362 13.θβ24 9541
28 18.4047 8803
1583 14.8981 2726 13.8980
20 2388 0.7122
14.8508 15.1410 7858 14.1214 2172
80 13.7844 4800
4167 15.3724 5103 14.3331 011β
31 15.24θθ 6814 2476 10.1058
14.1477 15.5928 1050 14.5387 4617
32 15.6220 0527
7489 15.8026 14.7280 2007 10.4772
88 7994
14.4964 7667 14.0041 9817 5069
84 15.9827 7837 Ιβ.0025 4921 15.0750 10.8276
85 6918 14.8282 10.1929 0401 15.2370 3257 0848
86 16.8295 0896 16.3741 9420 15.8005 5220 11.1581
87 8575 1649 ΙΙ,4βθ9
15.1486 16.5468 15.5380 6843
88 6822 2122
1145 5171 15.0739 9851
89 16.9837
15.4613 16.7112 15.8047 11.7640
1463
0282 8784 3793 7ββ2
41 17.2920
15.7428 10.8678 15,0286 12,0415
3330
7351 0271 6154 8172
44 17.5884 16,0218 17.0170 Ιθ.Ο4β1 24βθ 12.30.33 7808
9356 8853 4067 12.5503
46 16.1574
17.8735 16.2888 17.1590 5753
6416
5150 8854 8635 12.7833
49 16.2629
18.1476 17.2943 5610
50 Ιβ.6443 9095 9020
4567 17.4282
Ιβ.7888 16.8680 18.0081
18.4111 0758
9086 8242 8619
9776 17.54δθ 1108
17.0228 16.4578 13.2105
18.θ64θ 17.ββ27
6207 5008 3414
1828 7331
17.2467 16.5477 2572 13.4061
18,0082 5796 17.7740 6428
8195 1537
17.4010 6982 Ιβ.7Ι8β 6.380 13.5007
19.1425 1240 17.8800 10.7902 2102
7880 17.0810 0271 13.7048
17.6660
10.3878 1571 16.8027 3115
4058
6428 18.0771 6139
17.8622 13.9290
19.58" 3970 6782 16.9315 8699
8484 18.049θ 18.1687 1790 14,0840
19.7927 0023 2173 4339
7888 18.2298 18.2559 14.2802
5572 2646 2061

134
vm—PRESENT VALUE ANNUITY PER PERIOD
10.9930 18.4016 14.3081
5181 8442 4114
20.1856 18.5661 14.4082
2674 0049 4636
20.3707 18.7236 14.β2ΟΩ 8713
9494 4075 14.7807
20.5488 18.8742 8031
4129 1029 14.8460
20.7200 10.0183 1010
8970 8805 14.θ4θΟ
20.8846 19.193 7468
5356 15.0462
4742 0087
21.042θ
19.2888
8612 15.1880
7074
21.1951 1592
19.4147
8088 15.2245
0884
21.8414 4332 Ιδ.30βΙ
Ιθ.δ35θ
7200 7204
0654
21.4821 19.6512 15.3831
8462 9813 8202
19.7820 15.4568
0778 3200
16.5243
16.5800
2821
16.7075
7227
16.7618
6004
TABLE OF OF 1
vm—PRESENT VALUE 0F ANNUITY PER PERIOD
(ail at i) — i
n 4% 590

51 21.6174 10.8670 18.3380 7663 10.0066 15.8130


52 8521 5003 18.4180 7298 9948 7607
63 21.7476 10.0693 18.4034 0284 17.0584 15.8618
54 8103 3017 18.5061 4550 8287 0252
21.8720 20.0003 18.0334 7100 17.1170 15.DOß9 7408
55
7403 4408 4538 15.9409
50 18.6086 4473
21.0020 20.1601 17.1725 7554
67 18.7005 1870
5067 8140 6486 15.0005
18.8196 4170
59 22.1080 20.2480 17.2261 4297
18.8767 6400
00 1218 2057 7048 16.0288
18.9292 8952
61 22.2180 IMD 20.3330 18.9802 7674 17.2760 1412
02 22.8267 3404 4311 10.0640
19.0288 3404
83 4043 20.4143 17.3223 1808
19.0750 8003
22.4205 20.4922 1675 10.0080
65 19.1191 2384
0076 3002 17.3071 8017
00 10.1010 7033
22.5284 20.6007 2a93 10.1311
67 2067 10.2010 1930 1337
3303 17.4005
68 22.0234 10.2300 10.1614
20.0380 9014
60 8907 10.2753 0101 2771
2204 17.4408
70 10.3008 1048
22.7148 20.7062 6410 16.1900
71 0421 10.3420 7006 2614
4118 17.4880
72 22.8027 19.8730 7776 10.2170
73 u 20.7716 1343
8280 2268 19.4037 8884 17.5241 0579
76
22.8872 20.8330 10.4321 7037 83.34 1ß.2424
76 0124 9208 10.4602 1845 17.6584 6829
77 22.9086 20.8037 19.4840 0702 16.2004
78 4927 7810 19.5004 0610 17.5900 7009
70 23.0400 20.0500 10.6328 5257 6467 10.2801
80 8109 7013 19.5660 9768 17.6217 2272
81 23.1218 21.0057 10.5702 8361 0737 10.3104
82 0061 2105 10.5004 6048 17.6600 0314
83 28.1040 21.0581 10.6160 7866 0438 16.8300
84 4770 0684 10.0330 7776 17.6780 5300
85 23.2035 21.1082 10.0514 0730 3017 10.3406
0740 3021 10.0080 0704 17.7048 7840
88 23.3802 10.3676
87 21.1502 10.0838 1023 7125
0658 0000 17.7297 1850
88 23.3045 10.0088 7200 16.3845
80 21.2021 19.7132 1200 3570
1408 1187 17.7538 4387
19.7268 6867
91 23.4602 10.7308 7483 0400 16.4006
21.2400
92 6440 4007 19.7522 6174 17.7750 1308
08 23.5166 21.2880 4300 10.4155
3885 10.7040 6880 7888
7062 17.7008
06 23.6727 10.7762 0410 10.4207
21.3283 1804
06 2060 10.7860 0438 9093
0298 17.8168
07 23.0270 10.70M 8612 16.4431
21.3067 8970
08 2408 10.8058 0050 9800
0711 17.8350

136
vm—PRESENT VALUE ANNUITY PER PERIOD
100 23.6804 21.4030 10.8151 3300 17.8530 10.4558
0834 8300 10 8230 8706 4731 4810
23.7311 21.4388 10 : 17.8710 16.4077
6187 8383 8,323• 4010 8123
23.7700 21.4720 17.8872 16.4700
0333 1011 2100 4180 3889
23.8268 21.5048 10.8403 0671 17.0025 16.4806
8782 0570 10.8470 1020 9887 5083
28.8720 21.6357 17.9171 16.4006
0752 8546 6532 7802
23.0163 21.5663 17.0300 18.5091
9185 4403 5201 3077
23.0671 21.5936 17.0440 16.5180
0754 8151 3120 4700
23.0072 21.6207 17.06M 2708 16.6264
1870 0001 17.0081 0028
24.0367 21.6466 7789 16.5343
8780 0288 17.0703 0640
24.0728 21.6713 1654 10.5418
7240 9032 17.0898 8348
24.1085 21.0051 7256 10.5480
3110 1036 17.0008 4068
24.1428 21.7178 7010 16.5560
1842 0806 18.0003 1008
24.1757 21.7805 6410 10.5618
8094 3000 18.0183 0030
24.2074 21.7003 6400 16.5078
8745 1588 18.0208 2670
24.2370 21.7802 7045 18.6734
6870 0658 18.0340 2141
24.2672 21.7002 6308 16.5780
7750 4076 18.0426 9944
24.2964 21.8174 1041 10.6830
6023 6520 18.0498 7872
24.3225 21.8348 0769 16.6883
6005 8542 18.0607 7616
24.3480 21.8516 4002 16.5928
1245 0490 18.0682 0709
24.3730 21.8075 0094 10.5960
0582 2631 18.0604 8830
24.9077 21.8828 4784 10.6009
6560 21.8974 18.0753 3244
24.4200 1055 0558 10.0040
1884 21.0114 18.0808 5326
24.4431 0340 5833 16.30S1 0344
0110 21.0247 18.0801 10.6114
24.4640 8704 2104 10.0145
0002 21.0376 18.0011 9000
24.4861 0012 1055 10.6175
0896 21.0498 18.0968 4628
24.6049 5274 3039
vm—PRESENT VALUE 0F ANNUITY PER PERIOD
0000
$3

138
VALUE ANNVITY PER

ТАВПЕ PRESENT ОР ор 1 РЕМО)


(ат at

770

0.9389 8714 0.9345 7044 0.0302 3250 0.9250 2503 ОЛИВ 5800
1 1.8200 2642 1.8080 1817 1.7065 0517 1.7832 6475 1.7711 И27
2 2.В484 7561 2.6243 1004 2.0006 2574 2.5770 0090 2.6540 2237
8 3.4267 0800 3.3872 1120 3.3403 2027 3.3121 2084 3.2756 пвов
4.1566 7944 4.1001 9744 4.0458 8490 3.9927 1004 3.0400 4208
5 4.55.36
4.8410 1356 4.7065 зоов 4.6038 4042 4.6228 7030
7 5.2000 0182 6.2003 7006 8717
6.4845 1977 6.3892 8040
8 5.8673 0365 Г,.1185
0.0887 5000 5.0712 9851 5.7400 3804 1362
D 0.B5B1 6.5152 3225 в.3788 8703 0.2408 8701
10 0.8040 8000 5.6301
7.1888 3022 7.02.35 8154 0.7100 8140 8207
7.6800 4240 7.4980 7484 7.3154 2416 7.1380 6420 0.1100 6204
ш 7.9420 8030 7.7362 7827 6.5013 4800
8.1587 2532 7.6300 7802
13 8.3576 5074 8.1258 4020 7.9037 7504
8.6907 4208 0.9080 изо
14 8.7454 0799 8.4801 5373 8.2442 3608
9.0138 4233 7.3440 8007
15 0.4026 6886 0,1079 1401 8.8271 1074 8.6604 7800 7.опоп 6400
16 9.4466 4860 0.1415 ови 8.0100 0008
9.7077 6418 8.8613 6016
17 9.7032 2200 9.4330 5076 8.3042 3058
10.1106 7670 0.1210 3811
18 10.4324 0038 10.0590 8691 0.7000 0008 0.3718 8714 8.5753 3326
19 10.8366 0524 9.0590 7821 9.00.36 0020 8.8251 0104
10.7347 1022
20 10.5940 1426 10.1044 0130 0.8181 4741 о.огм 7044
11.0186 0725
21 10.0108 0316 0.2077
11.2840 8333 10.8355 2733 10.4134 8033
2022
22 11.6361 9562 11.0612 4050 10.6171 0101 10.2007 4306
0.4033
23 11.7701 3073 11.2721 8738 10.8060 8031 10.3710 5805 3601
24 11.9007 3871 11.4093 3400 10.9829 6080 10.5287 5828
25 12.1078 7672 11.6535 8318 11.1400 4586 10.6747 7010 о.изв 2821
10.8000 7705 9.8097 0650
26 12.3023 7251 11.8267 7867 11.2994 8462
10.0361 0477 0.9029 4624
27 12.6749 9706 11,0807 0904 11.4413 8005
11.0510 10.1040 9700
23 12.7404 7668 12.1371 1125 11.5733 поз
7849 ll.i6U 10.2341 0078
29 12.9074 8084 12.2776 7407 11.6901 0624
0001
30 13.058В 7691 12.4000 4118 11.8103 8027 10.3540 0288
11.2577 вз:и
31 11.010B 3830 10.4640 0174
18.2000 8465 12.6318 1410 11.8407 0030
82 12.0164 7767 10.5604 6321
13.3839 2925 12.6465 5632 11.4340 9044
33 и 12.1074 2000 10.0003 2564
13.4590 8860 12.7537 0002 11.6138 8837
35 12.1929 4970 10.7408 4382
13.6706 0892 12.8540 0936 11.5800 3307
30 13.6869 5673 12.9476 7230 12.2725 1141 10.8265 8410
11.0И5 10.0000 7767
87 18.7005 6970 13.0362 0776 12.3405 2224
38 0822
13.8878 6887 13.1170 1060 12.4163 0953

139
VALUE ANNVITY PER

89 13.9792 1021 13.1934 7346 12.4704 1361 11.7i71 0270 10.9078


14.0640 8611 13.2649 2846 12.6380 8931 11.7751 7851 1343
41 14.1456 2087 13.3317 0884 12.5944 0800 11.8288 0899 11.0302
11.8785 8240 4270 l1.0877
14.2211 6190 13.8941 2041 12.6460 0165 8137
13.4524 4898 12.09ЗО 1772 11.0246 1333
14.2921 0140
14.3688 3708 13.5009 12.7885 2811 11.OB72 3457 11.1408 1233
12.7800 2615 12.OOOO 0867 11.1800 8878
14.4214 4327 6167
-13.b579 0810 12.8186 2898 12.0432 3951 11.2347 3020
14.4802 2842
13.6055 12.0770 7302 11.2702 6467
14.6354 2675 12.8646 3868 12.1084 0160
2169 11.3145 2034
14.5872 5422 12.8879 4287
13.6600 12.9190 1662 12.1374 0880 11.3407 8833
14.6359 1946
2018 12.9479 12,1042 11.3822 оз.зв
14.8810 1451
13.ВДIВ 0784 2244 (3741 11.4122 6107
14.7246
18.7304 12.9748 12.1891 3649 11.4308
2007
7443 1157 12.2121 6341 0357
13.7687 12.2334 ИИ 11,4053
986,3 1206
13,8007
4029 11.4887
овдо
11.5103
8420
11.5308
0802
11.5480
7009
11.565П 0538

140
ΡΑΥΜΒΝΤ 0F WHOSE
ΙΧ— PERIODICA.L ANNUITY
PRESENT VALUE Ις Ι

8
1%

1.0041 6687 1.0050 0000 1.0058 3333 1.0075 0000 1.οιοο 0000
1 0.5031 2717 0.5037 5312 0.5043 7924 0.5050 32M Ο.Μ7δ 1244
2 0.3400 2211
8 0.3361 1406 0.3366 7221 0.8372 2078 0.3388 4579
0.2520 0058 0.2531 3279 0.2536 0.2647 0501 0.2562 8109
0.2025 0693 0.2080 0097 0.2035 1357 0.2045 2242 0.2060 3980
ο ο.1β0Ι 0564 ο.Ιβθ5 0.1700 8504 0.1710 6801 0.1725 4837
78 0.1452 4800 0.1467 2854 0.1486 2828
0.1462 0086 0.1471 7488
ο 0.1273 5512 0.1806 9029
0.1278 2886 0.1283 0851 0.1202 5552
10 0.1134 3876 0.1167 4037
0.1139 0736 0.1143 7698 0.1153 1020
11 0.1023 0506 0.1027 7057 0.1032 3632 0.1041 7123 0.1055 8208
12 Ο.ΟΩ31 0.003β 5908 0.0941 2175 0.0950 5094 Ο.Οθβ4 5408
13 9757 0.0888 4879
0.08β0 ββ43 0.0865 2675 0.0874 5148
0.08δ6 0.0824 1482
14 0.07οσ 4224 0,0801 0004 0.0810 2188
0748 ο.07θθ 0117
15 0.0741 0.0745 0205 0.0755 1148
Ο.Ο7Ω1 0.0721 2378
3θΟΟ 0.0β08 1999 0.0707 3639
10 8532
ο.0β03 6436 0.0679 4460
17 0.0730 8082 ο.0β56 4401 0.0ββ5 6870
0.0651 8937 0.0θ1θ ο.0θ42 δ80β
18 0.0680 1045 0.0628 7321 ο.0β09
0.0β15 0570 0.058β 8499 0.05θ5 0766
19 0.0647 3055 8206
0.0582 8173 0.0657 5532 0.0566 6740 Ο.Οδ8Ο
20 0.0β10 5387
0.0553 0253 0.0531 1889 0.0540 3063 5175 1532
21 0,0577 8053
0.052σ
22 0.0648 51θ1 0.0507 3383 0.051β 4548 0.0530 3075
0.0622 1030 0.0502 8163 0.0486 0585 0.04θ4 7748
23 0.0608 6371
0.0481 1380 0.04θ5 8063 0.0474 9846
24 0.0498 3183 0.0488 8684
0.04β1 3465 0.0417 7258 0.045β 8474
0.047β 6427 7347
0.0443 2061 0.0431 0388 0.0440 1660
26 0,0456 8531 0.0464 0675
0.042β 5180
27 0.0438 7180 0.0416 6376 0.0424 7003 0.0438 6888
28 0,0422 0270 0.0411 1163 0.0401 3793
Ω.03Ωβ 8505
0.0410 5178 0.0424 4
30 0.040β 6247 0.0388 1415 0.0397 2871 0.04Ι1
0.0383 0167 0.0375 8186
0.08θ2 3645 0,0384 0723 0.0398 9502
81 0.0371 2014 Ο.Ο3Μ
0,0370 1230 0.0373 0.0387 4811
32 0.0359 7802 3191
0.03ββ 7974 0.03β2 7352 0.037θ 7578
83 0.0356 2036 0.0349 0304 0.0363 5633 0.0352 0634 o.oaaa 7080
0.0338 0453 0.0343 4816 0,0843 2048
Ω.0844 6330 0.0357 2744
0.0334 4458 0.032θ 4727 0.0334 0124 0.0384 3053
80 0.0348 8907
0.0324 9708 0.0320 6586 0.0325 1020 0.032δ 0170
87 Ο.Ο3Ι2 0.0340 0368
0.03Ι6 0540 0,0316 7024
ΤΑΒΙ,Ε —PERIODICAL ΡΑΥΜΕΝΤ WHOSE
PRESENT
1
88 0.0307 6476 1550 0.0308 7710 0.0817 0973 0.0332 1431
80 0.02Ω9 7000 0.0304 2104 0.0301 2098 0.0310 5082 0.0924 6805
0.02Ω2 2003
0.02Ωθ 0.0294 1640 0.0308 4157 0.0317 6150
0.0285 0875 7130 0.0287 4258 0.029β 6803 0.0310 9180
0.0278 3402 0,0280 0045 0.0281 0.02θ0 3016 0.0804 6560
0.0271 0.0282 8607 0261 0.0284 2270 0.0298 5102
0810 0.0270 4552 0.0274 0.0278 4452 0.0202 7568
0379 0.0272 9338 0,0287 2787
0.0270 3031 0.0282
47 0.0269 0.0207 0751
0.02M 5022 0.0277
48 0.0268 1420 0.02β2 6521
0,0250 0320 0505
8352 0.02β3 0,0257 8406
0.0253 7541
0.02β0 6170 0.0253 0.0272
0.0248
0303 0.0258 2532 2775
7117
0.02" 3443 0.0248 0.0267
0.0243 0.0268 7111
4961 8504
8804 3073 0.0208
0.0249 0.0244
0.023θ 2733 8384
2141 0.0248 6202
0.02" 4905 0.0240 0,0250
0.02" 8603 o.ou,3 8708 1474
5787
1075 0.0230 0.0230 0.0255
0.0239 6087 4624 1278
3400 0.0226 0.0235
0.0234 6376 2265
7204 0,0231
0.0230 1611
2920
0.0226
0468
0.0221
9711
65
IX ΟΡ ΑΝΙΠΠΤΥ
VALUE ις 1

η 1 1%
51 0.0218 0557 0,0222 6269 0.0227 2563 0.0230 0888 0.0251 2880
52 0.0214 291β ο.02Ι8 8075 0.0223 5027 0.0232 Ω 0.0247 5003
ΡΑΥΜΒΝΤ 0F WHOSE
53 0.02Ι0 6700 0.0216 2507 0,0219 8019 503 0.0243 9050
0.0207 18.30 0.0211 7086 0.0210 4157 0.0220 3540 0.0240 5068
55
0.0203 8234 0.0208 4189 0.0213 0071 0.0225 8038 0.0237 2637
56 0.0222 5005
57 0.0200 5843 0.0205 1797 0.0200 8390 0.0234 0823
58 0.0197 4503 0.0202 0698 0.020β 7251 0.0210 3478 0.02.31 0156
0,0194 4426 0.0109 0481 Ο.Ο2Ο3 0.021β 2400 0.0228 0573
50 7100
0.0191 5287 0.0100 1392 0.0213 2507 0.022δ 2020
0.0188 7123 0.0103 3280 0.0200 8170 0.0210 3727 0.0222 4446
81 0.01θ8 0120
82 0.0190 0006 0.0207 5836 0.0210 7800
0.0185 9888
83 0.0188 8636 0,0187 0706 0.01θδ 2900 0.0204 8873 0,0217 2041
0.01Ω2 6762 0.0214 7125
M 0.0180 8025 0.0185 4337 0.0202 2795
0.01ΟΟ 1306
0,0178 3315 0.0182 9681 0.01Ω0 7500 0.0212 3013
0.0187 0773
88 0.0176 9371 0.0180 6789 0.01θ7 3127 0.0200
0,0185 2948 Ο.Ο1θ4 9460
67 0.0173 0.0178 2627 0.0207 7062
88 0.0182 0848 0.0192 6524
0.0171 8639 0.017β 0163 0.0205 5130
89 0.0180 7440 0.0Ιθ0 4280
ο.01β9 1788 0.0178 83θθ 0.0203 3888
70 0.0171 7200 0.0178 6710 0.0188 2716
0.01θ7 0574 0.020Ι 3280
ο.0Ι0θ ββ57 0.017β 4622 0,0186 1785 Ο.ΟΙΟ9
71 0.0184 9971 0.0174 4138 0.0184 1464 3282
72 0.01θ2 9952 0.0107
78 0.0172 4239 0.0182 1728 0.0107 3870
0.0161 0493 0.01β5 7289
0.0170 4001 0.0180 2654 0.01θδ 5010
75 0.0159 1572 0.01β3 8422
0.01β8
0.0157 31βδ 0.01β2 0070 0.0178 3017 0.0103 8700
76 0100 7814
0.0155 5258 0.01β0 2214 0.017β 5796 0.0191 8910
77 0.01θ5
0.0158 4832 0.0174 8170 0.0100
78 0,0153 7816 0024.
0.0152 0886 0.015θ 7908 0,0103 2709 0.0173 1020 0.0188 4784
80 ο•0Ι50 4205 0.0155 1423 0.0181 6851 0.0171 4328 0.018β 8416
81 0.0148 8177 0.0153 5360 0.01βθ 8074 0.0185 2488
ο.0Ι47 2464 0.0150 0482
82 0.0151 9704 0.0158 3436 0.01β8 2244 0.0183 8984
88 0.0145 7144 0.01δ0 4430 7847 0.0Ι0β 6821 0.0182 1886
84 0.01" 0.0148 0652 0.01β6 1700 0.0180 7180
85 0,0155 2650
2200 0,0147 5028 0.01β3 7186 0.0170 2851
0,0153 7830
M 0.0142 7020 0.0140 0856 0,0102 2847 0.0177 8880
0.0152 3378
87 0.0144 7021 8908 0.017β 5273
0.0141 3391 0.0150 0208 0.017δ 1008
89 0.0139 9500 0.0148 8513 0.015θ 6308
0,0140 5601
00 0.0142 0320 0.0158 2034 ο.σ173 0050
0.0188 5935 0.0148 2000
0.0140 7431 0.015β 9070 0.0172 6417
91 0.0137 2885 0.014β 8935
0.0155 6423 0.0171 4080
92 0.013δ 9740 0.013θ 4837 0.0145 6115
0.0138 Ο.ΟΙδ4 0.0170 2050
98 0.0134 7088 0.0144 3588 4064 0.010Θ 0300
94 0.0188 4721 2527 0.0143 1347 0,0158 Ιθ8θ 0.01β7 8882
05 0.0132 2029 0.0137
0.0136 0.0141 9380 0.0152 0190 0.01ββ 7624
M 0.0181 0803 0.0140 7679 0.0150 8657 0.01β6
27 0.0129 9234 8724 (3678
0.0134 0.0189 6238 0.0140 7382
0.0128 7915 0,0188 5042 0.0148 Ο.ΟΙβ4
99 7213
0.0127 6887 6071
100
ΤΑΒΙ,Ε —PERIODICAL ΡΑΥΜΕΝΤ WHOSE
PRESENT
1
0.012β 5992 0.0133 0.0137 40M 0.0147 5571 0.01β3
0,0125 5374 5950 5511
0.018β 3372 0.0148 6020
0.0124 4976 0.0132 0.0185 2880 0.0145 0.0102 6284
0.0123 0.0131 0,0184 2608 0.01βΙ 5284
47θΟ
0.01"
0.0130 0.0133 2549 4692 0.01β0 5508
0.0122 8583- 0.0182 2βΜ 0.015θ
0.0143 4701
0.0129 0.01δ8 6574
0.0142 5017
3242
0.0128
8115
0.0127
8194
ΤΑΒΙ,Ε ΡΑΥΜΕΝΤ 0F
WHOSE ις

IX—PERIODICAL A.NNUITY
PRESENT VAI,UB 1

η 1%

101 0.0121 5033 Ο.Ο12θ 3478 0.0131 3046 0.0141 5533 0.0157 7413
102 0.0120 5440 0.0125 3947 0.0130 3587 ο.οιιο 0248 0.015θ 8440
103 0.0119 6054 0.0124 4β11 0.0120 4310 Ο.ΟΙ3Ω 0.0155 0008
0.01Ι8 6842 0.0123 6467 0.0128 5234 7143 ο.0Ι5δ 1073
105 0.0122 6481 ο.0Ι38 8220 0.0154 2056
0.0117 7800 0.0127 6288
100 0.0137 0487
0.01Ι0 8M8 0.012Ι 7670 0.012β 0.0153 4412
107 ο.οιισ 0250 0.0120 0.012δ 0029 0.0187 0022 0.0152 0336
108 0,0115 0.0120 0575 0.0125 0028 0.0130 2524 0.0151 81123
100 1727 0.0110 2264 ο.0ΙΜ 2385 0.0135 4201 0.01δ1
110 0.0114 3358 0,0118 4107 0.0123 4208 0.1134 6217 0.0150 30β9
111 0.0ΙΙ3 5143 0.0133 8206
0.01Ι7 6102 0.0122 6361 0.014θ 5620
112 0.0112 7070 0.01Ιβ 8242 Ο.Ω12Ι 0.0133 0527 0.0148 8317
113 0.01Ι1 9161 0,0116 0626 8571 0.0Ι21 0.0132 2005 0•0148 1156
114 0,0111 1386 0,0115 2948 0023 0.0Ι31 5426 0.0147 4133
115 0.0110 3750 0.0114 ο.0Ι20 8414 0.0130 8084 0.0Ι4β 7245
0.0109 6249 0.0110 6041
116 0,0113 8195 0.0130 0878 0.014β 0488
117 0.0108 8880 0.0113 1013 0.0118 8790 0.0129 3808 0.0145 3860
118 Ω.Ο1Ο8 0.0112 8060 0.01Ι8 1080 0.0128 6867 0.0144 7356
119 1030 0.0107 0.01 ιι 7021 0.0Ι17 4098 0.0Ι28 0087 0.0144 0973
120 4524 0.011β 7832 0.0127 3388 0.0143 4700
0.0111 0205
0.010β 7580 0.011β 1086 0.012β 6758
121 0,0106 0665 0.0110 8505 0.0142 8561
0,0109 0918 0.01Ι5 44M 0.0120 0204 0.0142 2525
122
0.0105 3890 0,0100 0441 0.0114 7θ3β 0.012δ 3042 0.0141 •
128
0.01M 7251 0.01Ι4 1528 0.0124 7702 0.0141 0780
124 0.0108 4072 0.0113 5228
0,0104 0716 0.0107 7808 0.0124 1668 0.0140 6005
125 ο.0ΙΙ2 9033
0,0108 4288 0.0123 6540
120 0.0107 1047 0.013θ 0452
0.0102 7965 0.01Ι2 2040 0.0130 3030
127 0.0Ι0β 5680 0.0122 0614
0.0102 1745 ο.οιοσ 9028 0.0ΙIΙ 0.0122 3788 ο.0Ι38 8624
10 0,0101 5025 0.0Ι0δ 3755 θθ48 0.012Ι 8000 0.0138
120 0.0100 9603 0,0104 7081 0.0111 3203
0,0121 2428
130 0.0100 3677 1054 0.0137
0.0104 2208 0,0120 6888 7076
0.0090 7844 α,Ω103 0.0110
131 6255 0.0120 1440
ο.0Ι37 2837
0.00ΟΩ 2102 0,0103 1107 0,0110 6080
132 ο.οιοο 0,0130 7788
0,0008 6440 0.0102 5775 ο.011θ 0808
133 0560 0,0136 2825
0.0008 0883 0,0102 043β ο.0ΙΙ8 δβ21
134 0.0135 7047
0,0007 5403 0,0100 3085 ο.0ΙΙ8 0610
135 0,0101 5170 Ο.ΟΙ3δ 3151
0.0007 0005 0,0108 8410
130 0.010Ι 0002 0.0117 5493
0.0108 2072 0.0134 8437
ΤΑΒΙ,Ε ΡΑΥΜΕΝΤ 0F
WHOSE ις

137 4β80 ο.οιοο 4002 0.0107 7019 ο.0ΙΙ7 0.0134 3801


188 0.000ΰ 0.0Ω09 0.0107 2340 0.0ΙΙβ 6684 0.0133 0242
130 Ο.ΟΩ05 0870 4080 ο•0Ι1β 0804 0.0133 4750
0,0100 0.01Ι5 0179
140 4206 0,0004 ο•00θ9 0055 7101 0.0133 0840
0213 0.00θ8 5260 ο.οιοσ 0,0116 1536
141 0.00θ4 4206 0.0132 6012
142 0.00θ8 0510 2052 0,0114 0.0132 1746
0.0Ωθ3 9271 0,0007 5850 0.0105 0.0Ι14 2464 0.0131 7640
19 0,0093 4408 0.0007 1262 7021 0.0113 8031 0.0Ι3Ι 8410
144 Ο,ΟΟΩ2 0615
0.0105 2007 0,0113 3684
0.0002 4800 o.oooa 0710 0.0130 0356
O.MOO 0.0104 7187
0.0092 0233 0.01Ι2 0364 0.0130 6358
148 2250 Ο.ΟΙΩ4 2380
0,0001 5041 0.01Ω3 7044 0.0112 5127 0.0180 1423
147 0.0096 7844
0.0001 1114 0.0103 2078 0.0112 0053 0.0120 7651
148 3500
6050 0.0102 8881 0.0111 0841 0.012θ 3789
140 0.0004 0217
0.00θΟ 2247 0.0102 38δΙ 0.01Ι1 2790 0.0128 0088
160
0.0089 7905
0.0101
93813
Ω.αιοι
40815
Ω.ΟΙOΙ
0.01Ω0 8373
0.0100 21δθ
67
ΤΑ.ΒΙΕ PERIODICAL ΡΑΥΜΈΝΤ 0F WBOSE PRESENT ις

1
IX ΑΠΤΙΤΫ
VAI,UE 1

1*70 1*70 1*0/0


2%

1.0112 5000 1.0125 0000 1.0150 0000 1.01'75 1,0200 0000


1 0.5084 5323 0.Μ03 9-441 0.5112 7792 0000 0.6160 4950
2 0.3408 6130 0,3417 0117 0.3433 8206 0.6131 • 0.3467 5467
0.2570 7058 0.2578 0102 0.2604 4478 0206 ο.2β2θ 2376
8 0.2008 0034 0.2075 0211 0.2000 8032 0.8450 0.2121 5830
5 0.1732 0.1740 3381 0.1765 2521 0.2610 3237 0.1785 2681
6 0034 O. 0.1500 8872 0.1516 5016 0.2106 2142 0.1545 1106
7 1403 5762 0.1321 3314 0.1335 8402 0.1770 2256 0.1366 0080
8 0.1814 0.1181 7055 0.1106 0082 0.1530 3050 ο. 1225
1071 1644 0.1113
ο 0.1070 0307 0.1084 3418 0.1350 4202 2653
0.1174 0.1210 5813
10 6432 Ο.Οθ78 0.0002 9384
6839 0.001θ 7000 0.1008 7534 0,1021 7794
11 ο. 1062 9131 0.0002 0.0945
0.0852 4030 0.1007 3038
12 0.0θ71 6084 5831 0.0797 2332 0.0031 1377 5θθΟ
13 0.0805 5203 0.0838 0.0881 1835
2100 0.0740 4430 0.0800 7283
14 0,0881 1620 0.0811 5502 0.0820 0107
ο•0783 0.0707 8608
0.0770 0138 0.0703 7730 0.0778 2547
0515 0.0670
0.0728 2321 0.0735 0.0θ38 0578 0.0721 9958 0.0730 5013
17
0,0680 4363 2646 0.0β08 7847 0,0685 1023 0.0θθθ 0984
18 0.0662 4402 0.0βθ7 0210
0.0640 5098 0.0003 4072 0.0582 4574
10 0.081β 8113 ο.0θ23 2061 0.0687 8177
20 0.0650 0023 0.0558 0550 0.0590 0122
0.0587 5120 0.001Ι 5072
0.0023 8479 0.0537 0331
21 0.0561 1531 0.0504 6648 0.0617 3075 0.0573 1404 0.0587 8477
22 0.0637 3145 0.06β8 0.040θ 2410 0.0551 ο.05ββ 3140
23 0.0615 6526 203D 0.0482 β34δ 0.0581 8706 ο.054β 6810
24 0.0513 8565 0.0528 7110
0.04θ5 8833 0.0544 3748 Ο.Ο4β7 Ο.Οδ12 2044
25 0.0477 7701 0.0622 7238 0.0453 1527 0.04θ7 2052
0.04Ο8 0028
0.04β1 0.0502 0.0440 0108 0.0482 0269
27 0.0482
0.0446 7470 0.0484 8665 0.0427 0.04β7
28 0.04β0
0.0431 5278 0.04θ8 2247 7878 0079
20 8067
0.04Ι8 3290 0.0462 8720 0.041θ 0.0464 8161
8-0 0.0467
0.040β 0498 0.0438 6677 8910 0.0442 6424
7836
31 Ο.Ο8Ο4 5063 0.0425 4868 0.04Ο5 0.043Ι 2076
0.044β
82 0.0413 2228 7430 0.0420 7005
0.0383 8866 4022
38 0.0401 7854 0.0305 0.0410 7812
0.0973 8596 00435
36 0.0364 4349 0.03θ1 0942 7710 0.0401 4770 0.042β
86 0.0355 6763 0.088θ 0.03θ2 7803 1061 ο.04Ιβ
0,0381 0701 0.0377 0.0384 5082 8053
PERIODICAL ANNUITY WHOSE
PRESENT
1 1
87 0.0347 2200 0.0371 6180 0.0370 0,0408
88 0.0862 8387 ο.ωσο 3363 7507 1807
0.0330 3520 0,0400
89 0.0354 5111 0.038θ
0.038Ι 0072 0221
10 0.0340 0533 4257
0.0324 8589
0.033Ω 2270 0.03β2 0.08θ2
0,0318 1773 0.08βΙ 5240
0.0332 1083 0.0355 0300 3286
0,0811 0.0864 Ο.ΟΜθ
0.0326 6805 0.038δ
0.0305 8060 0.031θ 1437 7200 Ωβ78
0.0300 0709 2141 0.0347 0.0378
40 0.0343
0.02θ4 1613 2067
47 0.0313 8170
0.028θ 8940 0.0340 0.0871
2068 0.0338
0.02" 6468 7114
60 0.08Ο7 4θ0β 2067
4197 0.0334 0,0305
0.0302 0.0382
2710 6676
0.0279 8652 0466 8θββ
0.0276 0.020β 0.0328 0.0327 0.035Ω 7188
1173 8557 3106 7810 0.03Μ 1720
0.0270 0.02θ1 0.0322 0.0322 0.0348
7032 9012 6426 0821 0.0343
0.02ββ 0.08Ι7 2465 8794
0.0287 0.0918
5910 0.08Ι2 1038 0.033θ
1675 3043
0.02β2 0.0807 0062
0.0282 0.0313
5898 1076
0.0278 8830 0,0334
3075 0.0802 0.0300 5842
0.0274 5126 0660 0.0330 Ι
1563 0.0-298 0342
0.030δ 702
0.0270 0.0293 6124 0.032β
1768 7500 0.0301 0184
0.028θ 7891 0.0322
8478
0.0818
0.0285
2321
7168
ΙΧ nTODlCAL ΑΝ1ΠΙΤΥ
PRESENT VALUE Ι

1*70 1*70
η 2%

51 0.0258 0.02β6 0.0281 0.0298 0.0314 5866


62 7494 3571 0.0278 0209 0.0311 0909
0.02δ5 0.0202 8287 0.02θ4 Ο.Ο3Ο7 7392
68
0.025Ι 6897 0.0274 4065 0.0304 5220
M 0.0301 4337
6149 0.02δ9 8637 0.0291
0,0248 1658 0.0271 0492 0,0298 4656
87 1048 0.02δ6 7760 5138 0.0287 0,0295 6120
58 0.0244 0.0252 5146 0.02θ8 7072 0.0202 8607
59 8213 8018 0.0284 0.0290 2243
0.0249 3739

148

ΡΑΥΜ
ΕΝΤ
0F ις
οο Μ241 0.0246 3478 0.0205 β129 0.0287 6797
βδθ2 0.0248 4803 2100 0.0281 0.0285 2278
81 0.0240 6158 0.0202
02 0.0238 5795 0.0282 8643
0.0235 0.0237 8993 2341 0.0278 0.0280 5848
68
M 0720 0.0235 2768 0.02δ0 0.0275 0.0278 3855
0.0232 0.0232 7410 3661 8603 0.027β 2624
8386 0.0230 2904 0.0250 0.0273
M 6012 0.0274 2122
88 0.0280 0.0227 0203 1480 0.0272 2316
0985 0.0225 6268 0.0253 0.0270
60 9843 0.0270 3173
70 0.0227 0.0223 4065 5336 0.02β8 4665
4684 0.0221 2560 0.0261 0.0208 0.02θ0 6765
71 ο.021θ 1724 3804 0172
0.0224 0.0264 9440
72 0.02Ι7 1527 0.0248
0.0222 0.02β5 0.02θ3 2683
78 0.02Ι5 1941 8751
4247 6892 ο.02β1 6464
74 0,0213.2941 0.024β 0.02θ0 0786
0.0220 0.02β3
76 0.0211 4501 4741 0,0268 5608
0329 2455
70 0.0209 0.02" 1534 0.02β0
0.0217 Ο.Ο24Ι 0,0257 0751
78 7178 0.0207 9216 9821 0.025δ 6447
90θ4
0.020β 2325 0,0258 Ο.Ο2δ4 2576
80 0.0216 0.0239
0.02M 7252 0.0252 9123
81 4758 7880
0.0202 9953 0.02βθ 0,0251 6071
0.02Ι8 0.0237
82 0.0201 "36 6813 0.0250 3405
3037 8370
88 0.019θ 9341 0.02M 0.0249 1110
0.021Ι Ιθ85 0.0235
84 0.0198 4652 6372 0.0247 9178
0.0209 6033
85 0.0252 0.0246 7681
1571 0,0107 0366 0.0283
86 0.0207 0.0195 6437 6329 0.0260 0.0245 0321
87 1709 0.0104 2881 0.0281 7459
88
0.02"
0.0102 9β76 7235 0.0248
80 0.020δ
8930 5381
2552 0.0101 6808 ο.022θ 8727 0.0248 4760
M Ο.Ο2Ο3 3890 0.0190 4207 0.0228 0770 0.0247 0.0242 4410
0.0201 0.0189 2041 0,0226 0985 0.024Ι 4970
92 0.0245
5779 0.0188 0119 3308 0.0240 4602
93 3000
0.0199 0.0180 0.0224
8177 0.0243 0.0299 5101
0,0185 7140 6473
0.0198 0750 0.0238 5859
0.0184 8070 0.0223
1072 0,0242 0.0237 8868
0.0183 6271 0072
0418 0.0236 8118
0.0198 0.0182 4724 0.022Ι 4140
0.0240 4- 0,0285
4442
100
0,0194
.0Ι81 8 0.0219 570 0.0285 1313
4425 8076
82βθ 0.0238 0,0234 3242
Ο.Ο1θ3 .0180 4300 0.0218
9200 0.0288 6383
2536 8645
0.0179 4640 0.0237 0.0232 7729
0.01θ1 0.02Ι6 9036
0,0178 4941 4284 0.0282_0274
7226 0,0215
0.0177 5580 0,0286
PERIODICAL ANNUITY WHOSE
PRESENT
1 1
0.0190 0.0170 0391 4882 9806
2828 0.0176 7428 0.0214 0,0234
0.0188 1019 5748
7812 0.0212 0.0233
0.0187 7588 2003
3678 0,0211 0,0281
0.018δ 4509 8828
9208 0,0210 0.0230
0.0184 1784 0,0229
6489 0,0208 3406
0,0183 0.0207 0,0228
8400 7383 1223
0.0182 0,0208 0.022β
0654 5584 9375
0,0180 0.0205 0,0226
8215 4138 7860
0.0179 0,0204 0.0224
6081 2084 0636
0.0178 0.0203 0,0223
0.0177 2113 6724
2684 0.0202 0.0222
0.0Ι76 1510 5102
1403 0.0201 0.0221
ο.0Ι7δ 1182 4700
0387 0,0200 0,0220
0,0173
9020 1104 4000
0,0172 0,0199 0.0219
9110 1273 4882
-0,0171 0,0198 0.0218
8851 1681 6327
0.0107 0,0217
0.0170 0017
8818 2321
8186 0.021θ
ο.0Ιβ0 6944
0007 0.01θ5 4268
0.01θ8 0.01θ4 0.0215
9418 0.01θ3 7057 8101
0,0108 0,0214
0041 9480
0.01β7 0,0214
0870 1074
0,0213
2876
0.0212
4880
TABLE rx — PAYMENT OF

150

ΡΑΥΜ
ΕΝΤ
(Sil
0F ις
VALUE IS 1
i
(ail ati) = 1 —
n 3%

1.0226 1.0260 1.0275 1.0300 0000 1.0350 moo


1 0000 0000 0.5207 1825 0.5226 1084 0.5264 0049
2 0.5169 3768 0.6188 0.3618 3243 0.3535 3030 0.3600 3418
3 0.3484 4468 2716 0.2074 2069 0.2000 2705 0.2722 6114
4 0.2042 1893 0.3501 0.2107 9832 0.2183 5457 0.2214 8137
5 0.2137 0021 3717 0.1846 0750 0.1876
0.1830 7083
0.2658
6 0.1800 0.1580 9747 0.1006 0035 (1821
34Gß 1788 0.1035 4449
7 0.2152 4086
0.1409 5705 0.1424 5030
8 0.1600 0025 0.1200 4006 0.1284 3380 0.1454 7005
o 0.1370 8402 0.1815 4997 0.1157 3972 0.1172 3061 0.13M 4001
0.1239 8170 0.1674 0.1202 4137
10 0.1006 8029 0.1080 7746
0.1127 8788 9543 0.1004 0200 0.1110 0107
11 0.1394 6735
0.0989 0871
0.1036 8649 0.0925 3262 0.0040 2054 0.1034 8306
18 0.0900 1740 0.1254 5089 0.0070 0167
0.0870 2457 0.0886 2034
15 0.0895 7086 0.1142 6870 0.0837 0058 0.0916 7073
0.0822 6017
16 0.0840 0.1051 0596 0.0808 2607
0.0780 0710 0.0700 1085
17 6230 0.0974 8713 0.0826 8488
0.0744 0.0750 5258
0.0792 8852 0.0910 4827 0.0700 4313
19 318B 0.0727 0870
0.0751 1668 0.0855 0.0758 1084
0.0711 8063 0.0098 1888
0.0714 3653 0.0720 4038
0.0082 7802 0.0072 1671
n 4039 0.0807 6ß4ß
0.0056 7173
0.0648 7178
0.0703 0108
28 0.0681 0.0765 9809 0.0080 3069
0.0633 1041 0.0027 4730
24 7720 0.0729 2777 0.0060 3207
0.0011 8040 0.0008 1300
0.0062 0.0090 0.0040 1880
0.0502 4410 0.0600
26 0182 7008 0.0022 7283
0.0574 0808 0.0574 2787
27 0.0026 0.0607 0.0000 7404
0.0568 3907
4207 6082 0.0550 8820
29 0.0543 4116 0.0502 0540
0.0002 0.0641 0.0546 6421
80 0.0529 ma 0.0678 6241
7572 4713 0.0532 0323 0.0500
81 0.0581 0.0617
0.0610 7738 0.0521 1407 0205
82 0.0604 0.0510 1020
2821 8738 0.0564 4538
83 8035
0.0561 0.0690 0.0400 9803 0.0543
84 0.0493
7097 4661 0.0400 4002 7133
85 8442
0.0543 0.0570 0.0481 5612
0.0483 0.0533
86 8023 9638 0.0478 2106
6463 7240
87 0.0627 0.0569 0.0405 8929
0.0473 0.0524
PERIODICAL ANNUITY WHOSE
PRESENT
1 1
88 3599 1282 0203 0.0468 0370 4150
3D 0.0512 0.0542 0.0464 9253 0.0451 1162 0.0516
2134 7502 0.0450 0.0444 5034 7242
41 0.0498 0.0527 4875 0.0438 4885 0.0507
2188 6875 0.0448 0.0482 0238 0.0490
0.0485 0.0513 5045 0.0427 1241 0816
2525 7687 0.0441 0.0421 0.0402
0.0473 0.0500 1132 0167 8410
2081 8793 0.0434 0.0410 0811 0.0480
49 0.0401 0.0488 0958 0.0412 1325
50 9934 9127 0.0427 2085 0.0470 82M
0.0477 4704 0.0407 8518 0.0473
0.0461
7764 0.0421 8775
5280 0.0403
2250 0.0488
0.0441 0.0467 0254
0.0415 2728
7416 3900 0.0300
0.0432 0.0467. 6831 3161 6051 0.0402
5722 0.0448 0.0396 0822
0.0400 0.0457
0.0423 5988 7777
7200 0828
9655 0.0440 0.0892
0.0404 o.(Y1fi3
0.0415 0076 1814 2M) 0.0148
4175
8731 0.0432 0.0388 7788
0.0300
0658 0550 0.0444
0.0408 3871
2522 00424 5158 0.0394 6.343
0.0401' 0043 o:0417 6100
4090 0.0440
0.0894 0.0890 6108
2753 0.0410
0603 0.0430
0.0987 7012
0.0404 0.0885 0919
8648 0.0483
3615 7403
0.0881 0040
0.0308 0.0381
7738 0.0420
3628 8858
0.0878 0.0377 0107
0.0392 0.0426
0087 7158
d78d 8871
0.0370 0.0373
0.0387
6864 9773
2870
0.0305 0.0870
0.0882
3364 4092
1688
0.0880
0.0377
8901
3087
0.0865
0.0372
0805
0752
0.0851 0.0908
1921 2676
0.0346 0.0364
9107 0669
0.0342 0.0360
8233 0599
152

ΡΑΥΜ
ΕΝΤ
0F ις
0.0838 0.0356
9179 2348
0.0385 0.0852
1886 5806
— PERIODICAL 0F ANNUITY WHOSE
PRESENT

1 1
TABLE ΙΧ ΡΑΥΜΕΝΤ

2*70
η

51 0.0331 6102 0.0340 0870 0.0307 0014 0.0386 3382 0.0423


0.0328 1884 0.031δ 7440 0.0308 7444 0.0382 1718 0.0420 2429
0.0824 9004 0.0842 5449 0.0300 0207 0.0379 1471 0.0417 4100
0.0921 7061 0.0330 4790 0,0357 0401 0,0876 2658 0.0414 7090
0.03Ι8 7480 0.033β 5410 0.03δ4 7953 0.0378 4907 0.0412 1323
57 0.031δ 8530 0,0833 72*3 0,0352 0012 0.0370 8447 0.0409 0730
58 0.0813 0712 0.083Ι 0204 0.034θ 4404 0.03β8 8114 0,0407 8246
80 0.0310 3θ77 0,0328 4244 0.0340 0270 0.08β5 8848 0.0-405 0810
0.08Ο7 8208 0.0826 0307 0.0344 5153 Ο.Ο3Ο8 5503 0.0402 9306
61 0.0805 3583 0.03Ο1 3290
0,0323 5340 0.0342 2002 0.0400 8802
03 0.0802 9724 0,0350 1008
0.032Ι 2294 0.033θ 9707 0.0398 9240
M 0.0800 0,0357 1385
05 0.03Ιθ 0126 0.0337 8402 0.03θ7 0480
0.0208 4704 0.031β 8790 0.0335 7860 0.03δδ 1082 0.03θδ 2513
0.02θβ 3411 0.0314 8249 0.0333 8118 0,0853 2700 0.03θ3 5308
08 0.02θ4 2878 0.0312 8463 0.0331 9120 0.0361 4681 0.03θ1 8826
οο 0.0202 8070 0.0310 9308 0.0380 0837 0.0349 7110 0.03θ0 3031
70 0.02θ0 8956 0,0348 0313 0.0388 7802
0.030θ 1021 0,0328 3236
71 0.0288 6500 0.0307 3300 0.0320 6285 0,0348 4150 0.0387 3875
72 0.028β 7677 0.0305 0206 0.0324 9965 0.03" 8018 0.088δ 0463
78 0.0285 0458 0.03Ο3 0712 0.0323 4218 0.0348 8603 0.0384
75 0.0283 3810 0.0302 3790 0.032Ι 9048 0.084Ι 9266 0,0383 3277
0.0281 7728 0.0300 8417 0.0320 4420 0.0340 5404 0.0882 0973
76
0.0280 2169 0.0209 3568 0,0310 0311 0.0339 2053 0,0380 91β0
78 0.0278 7118 0.0207 0222 0.0317 6098 0.0337 9191 0.087Ω 7810
70 0.0277 2554 0.02Ωβ 5358 0.08Ι0 3500 0.0330 0700 0.0378 6919
80
0.0276 8457 0.0205 0.031δ 0878 0.0886 4840 0.0877 8460
81 0.0274 4808 Ο.Ο2Ω9 8007 0.0313 8638 0,0370 ωρο
82 0.0292 0403 0.02" 3331
0.0273 1589 0.03Ι2 0800 0.0883 2224 0.0875 0721
88 0.0271 8784 0.029Ι 4338 0.03Ι1 5382 0.0874 7420
0.0200 2005 0.0332 1510
85 0.0270 0370 0,0310 4342 0,0331 1175 0.0878 8480
0.020Ω 4350 0.028θ 1248 0.03ΟΩ 3074 0.0372 9804
88 Ω.Ο2β8 2092 0.0380 1201
0.0288 0254 0,0308 3381 0.0372
87 0.0207 1387 0.0320 1576
0.028β 9008 0,0307 3380 1628
88 0.020β 0423 0.0828 2284
0,0286 0298 0.030θ 8747 0,0371
89 0.0204 0787 0.0327 3813
0.0284 ΜΙΟ 0,0805 4420 3076
0,0203 9407 0.0320 4660 Ο.Ο37Ο
M 0.0288 9033 0.03M 5307
0,0202 9452 0,0825 6284 0025
02 0.0283 0255 0.0303 0007
0.02θΙ 9730 0.0324 8202 0.08βθ 8002

154
— PERIODICAL 0F ANNUITY WHOSE
PRESENT

1 1
98 0.0201 0291 0.0282 1165 Ο.Ω302 8210 0,0324 0398 0.0309 1570
M 0.0200 1126 0.0281 2363 o•03D2 0041 0,0323 2848 0.0368 4760
0.0280 380θ 0.03ΟΙ 2125 0.0822 0.0807 8190
0.02ΰ0 2224
0.0268 3677 α.0279 6523 0.03M 4400 Ω,0821 8508 0.03β7 1868
0,0257 5176 0.0278 7480 0.0209 7038 0.0821 1094 0,0300 5781
99 0,0250 7012 0.0277 9690 Ω.02Ω8 9850 0,0320 5107 Ο.ΟΒβ6
100 0.0256 9078 0.0277 2120 0.02Ο8 2887 0.0319 8737 0019 0.03βδ
0.027β 4780 0.02Ω7 0141 4273
0.02δ5 1300 0,0310 2577
0.027δ 7802 0.0200 Ω.Ο318 0.03M 8834
0.02" 0.02θβ 3272 0.03β4 8694
0,0276 0747 0,0318 0850
3808 0.0274 4084
Ω.02θ6 7134
0.03Ι7 0.0303 8640
0.0268 0.0206
0.0278 7617 Π2Β1 0.03Ο3 8082
0.02δ2 0489 1185 Ο.Ο3Ο2 8θΩδ
0.0278 1188 0.0310 0.03β2 4478
0,0262 2594 0.0294 6418 Ο.Ο3ΙΟ 4067
0.08θ2 0124
0.0361
6927

155
PERIODICAL ANNUITY WHOSE
PRESENT
1 1
ΤΑ-ΒΙ.,Ε IX— ΡΑΥΜΕΝΤ 0F
VALUE ις 1

4*70
η 4% 6%

156
— PERIODICAL ANNUITY
PRESENT
1
1.0400 0000 1.0450 0000 1.0500 0000 1.0δ50 0000 1.0800 0000
1 0.5801 9608 0.5339 0.5378 0488 0.54Ιβ 1800 0.5454 3680
2 0.3003 4864 0.8087 7336 0.3672 085β 0.3706 5407 0.3741 0981
8 02754 9005 0.2787 4305 0.2820 1183 0.2852 9449 0.2885 9149
δ 0.2246 2711 0.2277 9164 0.2309 7480 0.2341 7644 0.2373 0640
o 0.1907 0.1988 7839 0.1970 1747 0.2001 7805 0.2038 0203
8 ο.1βββ 0961 0.1697 0147 0.1728 1982 0.1759 6442 0.1701 3502
0.1485 2783 0.1516 0.1610 3504
9 0.1547 2181 0.1578 6401
0.1344 9299 0.1375 7447 0.1400 9008 0.1438 0.1470 2224
ι 0.1282 θ094 0.1283 7882 0.1296 0458 0.1326 6777 0.1358 0700
ο
0.1141 4904 0.1172 4818 0.1203 8889 0.1235 7005 0.1207 0294
11 0.1065 5217 0.1096 0,1128 2541 0.1100 2023 0.1192 7708
12 0.1001 4373 0.1032 7535 0.1084 5577 0.1096 842β 0.1120 6011
18 u 0.094θ 6807
Ιδ 0.0978 2032 0.1010 2307 0.1042 7012 0.1075 8401
0.089θ 4110 0.0931 1381 0.0963'4229 0.0θ9θ 2500 0.1029 0270
18 0.0858 2000 0.0980 5214
17 0.08θ0 1637 0.0955 8254
0.082Ι 9852 0.0θ20 4197 0.0954 4480
18 0.08δ4 1758
0.078θ 9333 0.088θ 1992 0.0023 5064
19 0.0822 3890 0.0022 6991
0.07β1 88β2 0.08βΙ 5008 0.0800 2080
20 0.0704 0734 0.088θ 9914
0.0735 8175 0.0830 7033 0.0871 8460
0.0708 7614 0.08δ5 4022
21 0.0712 8011 0,0827 4501 0.0814 0478 0.0850 0455
22 0.0091 9881 0.0740 0057
28 u 0.0725 4505 0.0802 4259 0.0794 7123 0.0830 4567
0.0β73 0θ0β 0,0812 7848
25 ο.0β55 8683 0.070β 8249 0.077θ 9811 0.077θ
ο.0β8θ 8708 0.075θ 0.07β0 3580 0.0700 7900
26 η 0.0640 1196
ο.0β74 3903 7051 0.0745 4935 0.0782 2072
28 0.0625 8738
0.0660 2137 0.074Ι 8682 0.0731 9307 0.07β0 0435
80 ο.0θ12 3854 0.0724 7090
0.0β00 1298 0.0647 0,0719 5228 0.0750 0717
81 ο.0β35 2081 0.0709 5246 0,0708 14-40 0.0745 9255
0,0588 7θθ8 Ο.Ω735 7901
82 0.0578 9010 ο.0β24 Ι4βΙ ο.0βθ6 0482 0,0607 β8δ7
88 ο.0β13 0154 0.0082 9186 0.0β88 063θ 0.072β 4801
0.05β8 5535 0.0717 0222
85 0.0604 4846 0.0θ71 2258 0.0070 1066
0.0559 485θ 0.0710 0234
88 Ο.Ο5θδ 6320 0.00β0 0.0670 0519
0.0551 0357 0.0702 7208
87 0.0687 4453 0.0β50 5144 0.0βθ3 3409
0.0543 1477 0.06Ω5 9843
88 0.0579 8191 Ο.Ο64Ι 3212 0.005β 2968
0.0635 7732 ο.0β80 7380
89 0.0572 7045 ο.0β32 8042 0.064θ 7403
0.0528 8888 0.0083 0483
0.056β 0578 ο.0θ24 0004 0.0643 6036
0.0522 8957 0.0078 5743
0.0550 8402 ο.0β17 5545 0.0687 0903
0.05Ιβ 8192 0,0673 5812
0.05M 0.0610 7171 0,0032 7217
48 0.0510 6083 0.0008 9377
0.0548 5567 0.0004 0,0027 7001
44 0.050δ 234θ ο.0θ04 0164
0.0643 4315 0.05θ8 3079 0.0623 2034
0.0500 1788 o.oooo 5880
0.0538 0168 0.0502 8423 ο.0β18 0000 ο.οοσο 8842
0.04θδ 4020
0.0534 0808 0.0587 0482 0,0014 8027 0,0663 3812
0.0400 8089

157
— PERIODIbAL ANNVITY WEOSE
PEESENT
1
0.048β 6454 0.0520 8235 0.0δ82 7816 0,0011 1337 0.0θ50 0000
0.0482 6248 0,0525 8071 0.0578 2220 0.0007 0128 0.0047 0050
0.0478 8205 0.0522 0202 0.0673 0471 0,0004 3127 0,0044
0.0475 2180 0.0518 4471 0.Ω50θ 0333 0.0601 2175 1485 ο.0β4Ι
Ω.0500 1025 4708
0,0471 8065 0,0515 0784 0.0562 8173 0.05θ8 3120
0.04β8 5712 0.0511 8868 0.0505 5854 0.0038
0.0559 2820 0706
0.04θδ 5020 0.0508 8722 0.0598 0280
0.06δβ 1421 0.083β 035β
0.050θ 0215 0.0590 0146
0.05δ3 1843 0,0634 4429
0.05δ0
0.0547 7674
TABLE rx PAYMENT OF VÄOSE
VALUE IS 1

(ail at i) I — (1 + i)—n (Sil at i)


6%
n 5%
5 0.0402 6885 0.0503 0.0645 2867 0.0588 84M 0.0032
1 0.0450 8212 3232 0.0542 9-150 0.058B 3880
5 0.0457 1015 0.0500 0.0640 218b ().0B30
2 0.0454 0010 7070 0.0538 2130 4017
63 0.0452 3124 0.0408 0.053" 0.0582 0.0028 0551
64 3400 32,16 0.0020
0.0450 0487 0.0634 8010
55 0.0490 0.0580 0.0026
0.0447 8032 0.0533 0343
0510 5,168 31100
66 0.0446 8401 0.0631
57 0.0443 8830 0.0493 0.0520 7802 0.0578 8008 0.0023 8706
58 0.0442 0185 8764 0.0528 2818 0.0577 2000 0.01122 4744
59 0.0401 0.0676 8000 0.0021 1574
0.0440 2808 0.0520 8027 D.OUID
00 8105 :10fil'
0.0438 5430 0.0525 5183 0200
0.0480 8600 0.0673 0707
61 0.0436 9237 0.0624 2442 0.0018 7672
02 0.0435 3780 0.0487 0807 0.0523 0305 0.0b71 8202
0.01117
0.0433 0019 0.0480 2221 0.0521 8016 0.0670
04 0.0010
65 0.0484 5420 0.06M) 6268
0.0482 4921 0.0520 8067 0.05118 4737 0.0016
0.0431 1451 0.0482 0402 0.0510 7757 11704
cc 0.0607 4800
87 0.0429 8678 0.0481 4284 0.0518 7080 0.0014
0.0470 0848 0.0600
09 0.0428 6272 0.0617 8715 71316
0.0478 011b t9L13
n 0.0427 4500 0.0610 991b 0.0013 0000
0.0477 3047
78 0.0426 325.9 0.0510 1509 0.0013 1022
0.0426 2480 0.0470 0808 0.0515 3033 0.0012 3454
74 0.05M 8103
0.0424 2100 0.0474 8705 0.0614 0103 0.0011
75 0.06M 02-12
0.0473 7487

158
— PERIODICAL ANNUITY
PRESENT
1
76 0.0423 2334 0.0472 0745 0.0613 8953 0.05(13 27M 0.0010
77 0.0422 2900 0.0471 0611 0.0618 2101 n.0fi(i2 5076 0.0010 331.3
78 0.0421 3869 0.0470 0760 0.0612 5700 0.06451 0.0009 n70
79 0.0420 5221 0.0400 7406 8082 o.ouon +7-1
0.0611 0580 0.05111 2052
80 0.0419 0930 0.0408 8606 0.0611 3760 0.0008 (150b
0.0560 0.0008 15,12
81 0.0418 9007 0.0468 0169 0.0510 8222 0.06M)
82 0.0418 1408 0.0407 2104 0.0510 2002 0.0007 0807
0.0550 6045
83 0.0417 4127 0.0406 4422 0.0609 7009 0.0007 2403
0.0660 0577
85 0.0410 7150 0.0466 7004 0.0609 321 t 0.0000 8.316
0.0558 6781
0.0410 0403 0.0406 0104 0.0608 8111M 0.0000 4407
80 0.0558 12113
0.0416 4054 0.0464 3434 0.0608 4.300 0.0000 0724
87 0.0667 0.0605 72M
0.0414 7909 0.0403 706B 0.0508 0810
88 0.0567 2884 0.0606 3084
80 0.0414 2018 0.0403 0096 0.0607 04.33 0.0M! 0.0005 0003
0.0413 0870 0.0402 6107 0.0507 27-10 0.0M) 6/05 0.0004 7008
91 0.0413 0953 0.0401 0.0500 0228 (1.0650 1047
0.0604 r,2tsr
92 0.0412 00401 0.0500 5888 0.0655
0.0004 2081
98 u 0.0412 0775 4370 0.0600 2711 0.0665 6603
85 0.0604 0249
0.0411 6005 0.0400 0334 0.0505 0081) 0.0656 2/(57
0.0003 70511
0.0411 1410 0.0400 4516 0.0505 (1815 0.0554 0800
97 5705
0.0410 7010 0.0460 0015 0.0605 4080 0.0654 7273
08 w 0.0003 3767
0.0410 2789 0.0459 6622 0.0605 1478 0.0554 4788
100 0.0003 18.30
0.0409 8738 0.0450 132b 0.0504 0.0654 2436
0.0008 0025
0.0400 0.0458 7310 0,0504 0207
0.0002 8318
0.0409 1119 0.0458 3480 0.0504 0107 0.0563
0.0002 (1708
0.0408 7588 0.0467 9827 0.0604 2274 0.0663 0.0602 51M)
0.0408 4100 0.0467 0381 0.0504 0245 0.06M 4204 0.0602 3758
0.0408 0800 0.0467 20G1 0.0503 8314 0.0553 2410 0.0002 2408
0.0466 070B 0.0553 0711 0.0002 1136
0.0552 0101 0.01m 9096
0.0552 7577 0.0001 8803
0.0450 0740 0.0662 0182 0.0001 7730
0.0458 8834
• 0.0450 1048
0.045b 8385
0.0466 6830

159
— PERIODIbAL ANNVITY WEOSE
PEESENT
1
ТАВО rx РАУМЕНТ ор
VA.LUE IS 1

1
(а att) ¯ 1

п
1 1.00бО 0000 1.0700 1.0750 1.одоо 1.0860
2 0.5492 0000 0000 0000 0000
з В150 0.6680 0.5669 0.5007 0923 0.5040 Ш31
5 0.8775 9170 2771 0.3880 .336 0.3016
7670 0.3810 0.3846 1 3026
7 0.2919 5100 3703 0.3010 2080 0.3052
8 0274 0.2962 0.2085 0.2604 8780
9 0.2400 2812 0761 г,046 0.2637 0675
3464 0.2438 0.2471
10 0.2103 1639 0.2100 0708
0.2005 9000 6472
11 0.1020 7240 0.1053 0022
6831 0.2097 0.2130 0.1740 0.1773
12 0.1823 4480
0580 1470 3006
18 3137 0.1866 0.1888 0.1000 7071 0.1084 2872
0.1042
1' 3730
5322
0.1074
0032
0.1707
0.1400
2049
0.1624
0771
13 0.1602 0770 2702
3803 0.1400 0.1434 0203
17 0.1534 0.1607
0.1301 7034 0.1301
18 8B47 0710
0.1320 6280
19 0469 0.1423 0.1450
0502 0.1300 2287
20 0.1300 7760 8693
0.1206 0.1248
21 5621 0.1333 0.1300 2181 4244
22 0.1226 6090 9747 0.1212 о«86 0.1204 2040
6817
и 0.1162
0.1269
0190
0.1202
7783
0.1108 205'
о. 1120
0.1100 1354
8250 0.1133
0.1100 0.1230 7087 0.1000 1108
0.1109 6085 6420 2043
4048 0.1104 3041
0.1143 0.1177 0.1007
0.1063 0.1070
4494 0737 0210
6278 0140
0.1007 0.1132 0.1041
30 0.1060
0.1023 0462 8724 2703
31 7007
7767 0.1093 0.1018
0.1058 0.1030 0641
33 0.0980 9110 6221
5706 0.1010
и 0633 0.1024 0.1000 0.0008 3226
35 0.0058 0.0080 3207 3802
2610 0003 0.1003 7193
за 5401 0.1030 о.опи 2217

160
— PERIODICAL ANNUITY
PRESENT
1
37 0.0931 0.0094 2800 0.0040 7700 0.0080 0076
38 5675 1200 0.1004 0.0030 0.0077 1108
89 0.0907 0.0007 1000 7878
о.оовп
6040 5301 0.0080 0.0026 8010
0,0880 0.0943 0219 0713 о.оог,б 0026
48 1333 9298 0,0900 0,0014 0.0040 3014
и 0.0800 0.0922 2937 4800 0.0938 0677
0120 8900 0.0941 0.0004 0.00.30
0.0840 0.0004 8087 8801 5058
6078 0577 0.0Р26 0.08П0 1854
0.0023 оли
0,0833 0.088'7 1393 3628 0.0888
0.0017 4247
50 0770 0.0871 0.0910 2743 0.01)11 7588
0.0819 8902 5Ш8 0.0881 0728 ().оооо лом
8148 0.0858 0.0807 0.0874 6081
1062 1007 0.0808 лоз
0.0806
0480 0.0846 0.0884 0.080.з
0.07О6 2288 6103 0001 0411 0,0807 0000
0.07И 6305 0.0884 0.08Д8 0.0RIi3 0700
0.0874
0,0774 2673 0320 0.0800 ооон
0204
7440 0.0823 0.0804 0620 0.0880 8103
0.0863 4407 8201
0.0766 9103 0.08Љ4 0.0840 2440
7744 0.0814 • 0811 0,0881 0737
0.0846 3804
0.0757 4866 о.оио 0.0841 8613 0.0878 5670
5393 0.0805 7124 0.0838 0010 0.0870 2512
0.0740 8340 о.од.зп 0.0874 1363
0.0336 0,0872 1901
9666 0,0727 1028 0.0832
2509 ВИО
0.0742 9001 0.0832 8084 0.0870 4154
9924 0.0700 0,0826 0.0808 7807
9307 0.0830 3414
0.0730 7202 0.0828 0162 0.0807 2796
6010 0.0784 0.0820 1461 0,0806 9006
0.0814 0.0825 8728
0.0730 0807 0.08И взи
0220 0,0777 8291 0.0823 8991
0674 0.0800 0447 0.0822 0709
0.0726 0,0820 4027
1332 0.0772 0.0806 4533
3300 0.0801 3107 0.0818 8667
0.0720 0.0817 4280
0634 0.0707 0.0797 5124
00716 3480 1631 0.0794 0031
0,0710 0.0702 0.0790
9854 омов 8086 766.3
9373 0.0767 0,0787 7789
0.0703 0506 0,0786 0201
1770 0.07В3 8070 0.0782 4710
0.0099 0,07Я 0014 0.0780 1146

161
— PERIODIbAL ANNVITY WEOSE
PEESENT
1
0842 0.074B 6962 0.0777 9363
0.039B 0.0748 0.0776 9190
4862 3691 0.0774 0627
0.0093 0.0740 0.0772 3247
4119 3600 0.0770 7241
0.0090 0.0737
6068 6709
0.0734 9967
0.0687
9743 0.0732
0.0B86 6300 6993 отзо
0.0683 3744
2506 0.0728
0.06Д1 1240 3070
0.0679 0.0726
1398 3863 оми
6985

162

ΤΑ.ΒΙ,Ε Χ COMPOTND AMOUNT ΟΡ 1 FOR FRACTIONAL


PERIODS
1
1 1%

8 1.0020 8117 1.0Ω24 0088 1.ΩΩ29 1248 1.0037 4200 1.004Ω 8750
1.απα Ι.ΟΟ1θ 4068 1.Ω024 11378 1.W33 2228
4 1,0013
1.001C) 4004 1.0012 1,0014 5615 1.0018 (1075 1.0024
12 0.324 (1087
1.0012 5977
47011 8182
18 1.0003 1.000” 1.ΩΟΟ8 2064
3100 1.0ΙΚ).ι 4751
20 1.000,3
1.0Ω04 1571 1.ΟΩ05 7404 (E570
1.000*2 2373 1.0ΙΚΙ3 8270
1.0001 5094 8873 1.0002 8743
1.0001 0185
1*0/0 1*70
2%

2 t.0050 0027 1.00β2 3060 1.0074 7208 1.0087 1205 1.00θ9 5050
3 1.0037 31102 Ι.Ο0-1Ι 4043 Ι.(ΗΜΩ 1.0Ωΰ7 1.0ΩΟθ 2271
1.0028 0081 1.0031 1040 7521 1.0037 1.004.9 4058 1.00,10 0293
1.0018 0627 1.0020 7257 ι.ουη 8452 1.0028 0502 1,0033 0580
12
ι.οοοο 3270 1.0010 3575 1.ΩΩΙ2 4140 Ι.ΟΩ“ 4077 1,0016 6168
13 1.001.3 36-40
1.0008 0092 1.οαοο 5004 1.0Ω11 4604 1.0015 2444
20 1.0000 0748
1.0004 3037 1.0004 7700 1.0005 7280 1.0007 0193
2*70

2 1.0111 8742 1.012,1 2284 1.0130 5075 110148 8010 1.017.3 4060
8 1.0074 1.0082 1,0090 8300 1.0090 0163 1.01Ι5 3314
Ο 1.0055 7815 0484 0522 1.0074 1707 1.0080 87-15
12 Ι .οοοι 1.00.ΙΠ 3108 1.00,19 3862
1.0037 1532 0225 1.0057
1.ΩΟΙ8 5504 1.0Ω22 υ.328 1.0024 0027 1.0028 7090
52 1,0041 1.ΩΩΙΟ 4306
1.0008 1.001Ι 3752 1,0013 2401
2302 21&1
1.0005
1.0004 2709 1.ΙΗ)2Ω 5ΩΗ ι.οοου 0170
1.000Ω δΩ17
1.0004 7407
4*70
4% 5% 6%

163

2 1.0198 0301) 1.0222 5242 1,0240 1.0271 3103 1.02Ω5


3 1.013Ι 1.0147 8040 1.010.3 '10.30 1.018Ω 0713 α,302
ο 1.0098 5341 1.ΟΙΙΟ 1,0122 722- 7518 1282
1.007.3 0312 1 1.009Ω 1.01-10 7385
12 1.0005 5820
20 - 1.0032 7374
Ι.Ω030 7481 1,0081 tM85 7170 1.Ω0θ7 5880
1.001β 1.(ΥΗΩ 7-112 1.0Ω2Ω 0138 1.0048
52 1.Μ1δ Ι]43Ω 1.00Ι8 7831 1.0010 3010 1.0022
1.0007 6453 4Ω84 1,0000 3871 43113
1.0ΩΙΙ 2118

7% 8%

2 1.0310 u,37 1.0344 0804 1.0308 2207 1.0302 3Μ8 1,0410 3339
1.0212 1347 1.0228 0912 1.024,3 "081 1.025θ 8557 1,0276 ΟβΜ
8 1,0182 4460 1.01Ω4 2655
4 1.0158 0828 1,0170 5863 1,0200 0440
1,0106 5107 -1.0113 4020 1.012Ι 2638 1.012D
1.013') 8952
12 ι.οοαο 4402 1.0064 3408
1.0052 1 5415 1.0027 8544 1.0M8 2141)
21 611.]Ω 1:tY2G 1,0029 6443 1,0031 4232
52 1.001.? 0107 1.0013 Ω17δ 1.0014 8112
1.0024 2504 1,0015 7008
1.0012 1179

164
TABLE ΧΙ —ΝΟΜΙΝΑΙ RATE j ΙΈ CONT,TRTED ΤΙΜΕς PER
YEAR GIVES EFFECTIVE RATE i
1

165
1%

.0041 6234 .0049 9877 .0058 2485 .0074 8599 .oooo 7512
2 .0041 8089 .004θ .0058 2203 .0074 8133 .0009 0885
.004Ι 0017 .0Ο4θ .0058 2082 .0074 7900 .0009 6272
8 .004Ι 5945 .0049 8962 .0058 1921 .0074 7007 .οοοο 5850
.ΟΟ49 8858 .ΟΟ9Ω 514U
12 .0041 5873 .0058 1780 .0074 7434
.0041 5868 .0Μθ 8850 .0068 1709 .0074 7410 .ΟΟθ9 5414
18 .Ο0±Ι 5884
.ΟΟ4θ 8802
.0058 Ι .0074 7309 .0099 5224
704

2 .0112 1854 .0124 0118 .0149 4417 .0174 2410 .01Ω9 0099
3 .0112 0807 .0Ι24 4828 .0149 2562 .0173 .0198 0813
a .0112 0285 .0124 4183 .014θ 1636 .0178 8631 .01θ8 5173
12 .01D8 3534
.0111 9763 .0124 3539 .0140 0710 .0173 7374
.0198 18θ8
18 .0111 9241 .0124 2895 .0148 9785 .0173 β11θ
.0Ι24 2846 .0148 .0173 6022 .0Ι98 1772
20 .οιιι .0198 1017
.0124 2649 9714 .0178 5443
9200 .0148 9288
.0111 8980

8%

2 .0223 7484 .0248 4567 .0273 .0207 7831 .Ο34β 0899


.0223 3333 .0247 9451 1349 .0297 0490 .0345
8 Ωθ43
.0228 1281 .0247 6899 .0272 .029β 0829
12 .0222 9102 .0247 4349 5170 .029β 3178 .0346 4978
.0222 7125 .0247 1804 .0272 .029δ 9624 .0346 0024
52 .Ο344 5078
.0222 0013 .0247 0434 2087 .0295 7501
.0222 5637 .024β 9848 .0271 .02θ5 0721 .0344 2420
.0271 .0344 1281
5036
.0271
4283
.0271
3575

4% 5% 6%

2 .0398 0781 .044δ 0483 .04θ3 .0542 3380 .060Ι 2003


3 .03θ4 7821 .0443 4138 9016 .0640 2130 .0588 3847

166
ο .08θ4 1383 .0442 .0491 .058θ 0070 .0580
12 .0393 7874 8007 .0537 05.38
4918 .0440 9771 .04θ0 8036 .0585
52 .Ο4-4Ο 5417
.0302 8894 .0δ3υ 5277
8488 .M40 3552 .0480 .0635 .0584
.0392 8908 Ω5θ8 1061
5031 .0488 .0535 .0583 3425
.0892 8949 6834 .0583
3551 .0488 0157
3507
.0488
1800

7% 8%

2 .0β3θ .0Ι88 1β09 .073β .07M 0007 .0832


8 7874 .oau 4414 .077Ω 5670 0067
a .0β3β .0781 .0777 .082β
2737 0619
12 4042 .0β82 3410 9942 99,38
.0θ34 .072θ .0774 .0824
20 .0680 4158
7814 7840 5074 1768
.0θ78 4974 ,0772
.0638 .0β77 4676 .0727 .0821
0044 5827 0888 3712
.0θ77 0268 .0770
.M31 4088 .0725 .0818
.ωεο 3903 7606 5792
.0724 .0770 .08Ι7 0811
5113 1802
.0β30 2134 .081β
1296 .0723 4401
7098

167
ΤΑΒΙΕVALUE 0F τη CONVERSION FACTOR
(8

1%
2 1.0010 4068 1.0012 4844 1.001,1 5621 1.0018 7150 1.002,1 0378
a 1.Μ13 8761 1.001β 0482 Ι.00ΙΩ 4103 1.0024 0585 1.0033 2500
4 1.0015 6115 1,0018 1.0021 8485 1.ΩΟ28 0812 1.0Ω37 4223
l.oou 2781 1,0031 2040
12 1.0017 3471 7Β05 1.002υ 7080 1.0041
1.0010 0820 1,0020 8131 1.Μ34 3286 1.0045 7510
13 1.001D 2104 1.0022 8000
1.002d 8060 Ι.Ω034 Ι .00-1ΙΙ 0714
1.0020 0170 1.0028 1.003β 0111 0011 Ι
1.0023 0603
1.0024 2182

1*0/ο
2%

2 1.0028 0463 1.0031 1520 1.0037 3004 1.0043 6176 1.ΩΟ40 7525
1.00ΙΙ(Ι 3733
1.0037 4008 1.004Ι 5510 1.0049 8340 1.0068 1081
1.0042 0802 1.ω4Ι5 7537 1.0050 0755 1.0005 8878 1.0074
12 Ι.004β 7730 1.006Ι 0575 1.0Ω02 3ΙΩ1 1.0072 0707 1.008,3 0125
1.00(18 6052 1.001) 1 3380
13 1,0051 4583 1.0057 ΙΙΒ2 1.007D 9571
1,0051 8188 1.000D 0458 1.008Ω 5177 9706
20 Ι.ΟΟ57 5037
1.0053 0818 1.0050 1.0071 1.0083 8820 1.0005 8243

8%

2 1.τη55 0871 1.00β2 1142 1.0068 2837 1.0074 4458 1.0080 7475
3 1.0074 β'2θ2 1.0082 8761 1.001>1 1141 1.00Ι)0 3431 1.0115 7748
4 1.0083 0839 1.ΩΟΟ3 2677 • 1.0102 6422 1.0111 8072 1,0130
1.010,3 6005 1.0113 0780
1.0003 3444 1.011,1 0725
1.ΩΙ24 2810 30114
12 1.0102 7107 1.012% 4243
1.013β 7002
26 1,0110 0780 1.01.31 5008
1.014,3 4020
1.01"
1.0107 7506 1.0122 0810 1.0134 2348
Ι.0Ι09 1.014β 3757 8678
1.ΩΙ5Ι) 4203
θΙΟ5 2θ7,1
1.0170 0810

5% 6%
2 1.ΟΟΟΟ 0105 1.0111 2021 1.012,3 4754 1,0185 1.014? 8151
3 1.01.32 1713 1.0148 5828 1.0Ιθ4 8,507 1.Ω18Ι Ι 522 1,0107 4101
1.0148 77" 1.018,5 5042 1.Ω222 2088
ο 2020
1.020β 3570
1.020.3
12 1.0Ιθ5 1.018Π 8053 1,0220 7810 Ι.Ω247 1070

168
20 3057 1.02M 1.0227 1470 1.024θ 0405 1,0272 1070
62 1.0182 Ω351 1.0214 1.0238 3548 1.0201 1.028δ 552β
1.0219 0231 1.024,3 1002 Ω72Ω 1.020Ι
1.ΩΙ01 0023
1.0104 8470 1.0207 2686 31815

8%

2 1,0152 ΜΙΟ 1.0172 0402 1.01&4 1103 1.010d 1524 1.02Ω8 1607
3 1,0213 0348 1.0220 8264 1,0245 0826 1.02β2 1066 1.0278 1074
1,0240 5523 1.0258 8002 1.0277 0120 1.020δ 1004 1.031,3 3332
6 1.0308
1.0267 5172 1.0287 82θ8 1.0328 1.0348 6402
12 1.0339 2617
1.0204 5294 1.031β 9143 1,0361 5721 1.0388 8456
20 1.03δ6 1.037D 4927 1,0402 8846
1,0309 0941 1.0332 5978
52 1.086,3 2705 1.0887 1704 1.0411
1.03Ι5 1.083θ 3242

169
ΤΑΒΙ,Ε χπ — AMERICAN TABLE 0F MORTALITY
Νιιπιbe Numbe Yearly Yeari Num- Numba ΥοατΙΥ probubillt
Ag r living r probabilit y ber r of probllbill y Of living
e ΙΧ deathB y of dying proba living deathH t of
- dyMg
bilit>,• of
χ l.ivmg χ

10 100,000 0.007 490 0.992 53 66,79 1,001 0.01G 8,33 0.083 007
ΙΙ 99,251 746 0•007 516 510 54 7 1,143 0.017 0.982 004
12 98,505 743 0.007 543 0.992 55 65,70 1,190 0.018 571 0.081 420
18 97,762 740 Ο.ΟΟ7
484 0 1,200 0.010 885 0.080 115
97,022 737 0.007 0.992 64,66 1,325 0.021 335 0.978 605
15 96,285 735 0.007 634 457 58 3 0.022
50 63,36 1,394 0.077 064
16 95,550 732 0.007 ββ1 0.992 1,468 0.024 720 ο.Ω75 280
17 94,818 729 0.007 688 431 61 4 1,540 0.02θ 803 0.973 307
727 0.007 727 0.992 62 62,10 0.028 880 0.071 120
1 94,089 1,628
725 0.007 765 404 88 4 0.03Ι 202 Ο.θβ8 708
8 93,362 1,713
0.007 805 0.902 0.00β 057
19 92,637 723 65 60,77 1.800 0.033 943
0.007 855 386 0.003 127
20 91,914
722
0.992 66 0 1,880 0.03β 873
0.050 871
0.007
21 91,192
721
339 67 59,38 1,980 0,040 120
ο.0δ0 298
720 0.007 968 5 0.043 707
22 90,471 719 0.008 011 ο.θθ2 88 57,91 2,070
0.047 047
0.062 353
23 89,751 312 69 7 2,158
0.047 008
718 0.008 0.992 0.052 002
89,032 70 56,37 2,243 0.943 238
718 0.008 130 273 Ο.Οδθ
25 88,314 71 1 2,321 o. 038 007
718 0.008 107 0.992 702
26 87,596 72 64,74 2,301 0.032 385
718 0,008 264 235 ο.0θ1 003
27 86,878 2,448 0.920 267
0.008 345 78 3 0.007 βθ5
η 86,160 719 0.992 2,487
29 74 53,03 0.073 783 0.01θ 822
85,441 720 0.008 427 195 2,505 0.912 972
0.008 ο.θθ2 76 0 0.080 178
80 84,721 721 2,501 ο.Ω87 0.0Ω5 029
723 0.008 807 145 77 51,23 η.807 080
81 84,000 0.992 2,476 028 0.094
82 83,277 726 0.008 718 78 0 2,431 871 0.888 930
094
83 82,551 729 0.008 831
Ο.9θ2 70 49,34 2,360 0.102 311 0.87Ω 173
732 0.008 940 042 80 1 0.111 Ι).8β8
81,822 47,36 2,291
85 81,090 737 0.00θ 089 0.991 8 1 2,100 0.120 827 0.856 5,34
80 80,853 742 0.009 234 1 45,29 2,091 0.131 784 0.841 305
0.991
87 79,611 749 0.00θ 408 82 1 1,001 0.825 703
935
88 756 0.009 δ8β

170
89 78,882 0.00θ 704 0.991 83 43,13
1,816 0.808
0,010 008 870 84 3 0.788 ΜΙ
40 78,106 1,048 α. 764
77,341 0.010 252 0.991 85 40,80
1,470 ο. 158
0.010 517 808 86 0 448 0.734
43 78,567 1,202 605 310 υ.β00
0.010 829 0.991 87 38,56
75,782 1,114 0.174 207 080
4-5 74,985 ο.οιι 0.0Ι1 738 88 9 933
4-6 0.991 38,17 0.101 0,053 308
562 80
47 74.173 855 8 744 0.211 350 Ω.0Ι)4 137
0,012 οοο
48 78,345 91 555
33,73 0.235 552 0.545 455
0.012 509 0.991
72,497 02 0 885 0.285 0.467 5,34
0.013 573
50 71,827 240 0.803 020 0.8Ι)δ 741
70,781 0.013 781 0.991 93 31,248 137
51 0.340 823
0.0Ι4 641 490 05 28,73
52 58 βΩ2 0,142 857
68,842 0.015 389 0.θθΙ 8
398 26,237 18 0.395 863 ο.οοοοοο
67,841 0.464 645
0.991 23,76 3
282 1 0.582 408
0.991 21,33 0.684 2δθ
0.991 0 0.734 177
0.9φο 18,96 0.857 143
0.900 1 1.000 000
778 ισ,07
ο.θθ0 Ω
592 14,47
0.990 4
414 12,38
765 0.990 3
774 208 10,41
785 0.089 0
797 992 8,603
812 0.980 0,955
828 748 5,485
848 0.989 4,193
870 483 3,070
8θβ 0.989 2,140
927 171 1,402
062 0.988 847
1,001 837 402
1,044 0.988 210

171
438 79
0.088 21
οοο 8
0.987
491
0.986
804
0.986
219
0.985
450
0.084

172
ΤΑΒΙ,Ε — COMMUTATION COLUMNS, AMERICA.N
EXPERONCE ΤΑΒΙ,Ε,

173
Age

ΙΟ 70 801.0 1 675 535,3 17 53 ιο 787.4 915.7 5 853.005


11 67 Ι 504 012.01 ιο 252.4 185 128.2 5 682.801
1 430 601.0 17 50 θ 733.40 124 875,8 5 510.644
081.5 οοο.8θ
12 180.0 1 371 472.0 57 220.00 116 142.4 5 335.808
ιο 8 740.17 δ 158.573
1 308 θβ3.5 806.20 58 106 012.8
13 02 1 240 025,ο 50 8 204.44 4 078.405
500.4 ιο 97 172.04
Ι 101 563,4 131.12 οο 7 801.83 88 008.20
4 705.260
14 50 1 15 01 7 81 106.38
4 608.926
038.413U ,14θ.2 073.0β 82 351.65 73 754.78
4 410.322
1 083 616.2 6 4 226.413
54 471.0 1 032 θθ2.4 Ιδ 234.05 03 013.44 86841.28
18 104.2 14 810.17 64 β 4 030.200
984 390.6 854.54
52 832.0 14 402.3Ω 85 480.75 8 831.187
037 843.4 283.27
50 653.0 14 000.83 07 0 071.27 3 020.300
803 212.8 48 610.41
562.8 13 031.β8 08 5 οοο.85 3 424.843
850 420.0 43 343.08
8 218.321
40 550,2 809 420.6 18 207.32 οο 5 273.38 38 452.53
800.55 3 010.2θ0
44 770 113.6 12 010.25 70 88 033.88
518.06 2 801.80θ
42 782.8 732 480.0 12 577.53 71 29 77β.0β
41 ΟΟθ.2 12 250.71 2 502.538
833.8 4 157.82 25 067.74
39 307.1 ΙΙ 935.38 13 3 808.82 2 384.657
782.4 22 497.07
11 74 3 470.67 Ιθ 351.β4 2 170.018
37 073.β 628 575.0
30 106.1 031.14 75 3 145.43 1 Ω77.1Ο7
808.6 11 10 518.22
34 601.5 77 2 833,42 18 982.47 1 780.781
566 302.0 387.59 1 501.240
83 157.4
537 199.3 ΙΙ 78 2 δ8δ.7δ 11 728,00
1 400.088
31 771.3
201.8 053.07 79 2 253.57 0 741.028
1 238.047
30 440.8 10 770.04 1 987.87 8 001.833
482 501.3 1 070.158
29 103.5 10 515.18 81 1 780.39 492.000
45β 871.2 82 1 508.63 δ 197.271 024.808 7
27 987.σ ιο 260.02
432 326.5 784.804
26 780.δ ιο 011.17 83 1 205.73 8 173.286
408 824.0 065.024 5
25 830.1 9 2 410.062
880 322.0 85 1 100.65 538.066 7
24 544.7 771.375 023.338 1 780.687
364 782.0 80 484.477 β
28 502.5 87 703.234 1 204.592
344 Ιβ7.4 342.862 4
22 601.4 620.405 907.051 3
324 440.0 88 203.006 ο
21 539.7 0 404.995 613.341 7
305 506.3 80
20 615.5 318.β38 38θ.Μ1 305.748 8
287 513.4
10 7274 204,610 241.360 0
270 240.8 141.000 3
18 878.6 217.598 137.807 8
253 745,5 08 05,ΒΟΙ 07
18 052,ο 8 154.383 71.774 70
237 071.θ 60.070 82
17 203.6 882.708 103,983 33.470 01
222 οοι.ρ 85.877 52
Ιβ 504.4 8 65.028 1 13.283 00
208 500.8 10.056 οο
15 778.6 676,415 38.304 7 8.000
771.8
15 070.ο 8 20.186 0
181 063.4 3.081 546
14 302.1 47Π58 0.118 80 4.164 21
160 104.7 .706 702
13 738.5 8 8.222 80 0,041 84 .110 300
167 256.2
18 107.0 270.860 0.827 011 0.11423
12 408.0 8 0.114 232
11 θ0θ.β 088,θ16
ΙΙ 380.δ 7
902.231
7
710.738
7
540.910
7
365.480 174
7
102.800
17
18
10
20
21 22

u
20
η 29
80
31
33
M
85
86
38
89

41
42
48

45
47

50
61 52

175
TABLE χν — SQUARBS — SQUARB ROOTS — R.ECIPROCALS

νη η

1.00 1.0000 1.00000 3.16228 1.000000 1.50 2.2500 1.22474 3.87298 .8θθθβ7
1.01 1.0201 1.004θθ 3.17805 .θθ0099 1.51 2.2801 1.22882 8.88587 .θθ2252
1.02 1.0404 1.00θθ5 3.19374 980392 1.52 2.3104 1.23288 3.89872 .βδ7805
1.03 Ι.0β0θ 1.0148θ 3.20030 .970874 1.53 2.8409 1.23003 3.91152 .β535θ5
1.04 1,0816 1.01980 3.22490 .θβ1δ38 1.54 2.37Ιθ 1.24097 3.02428 .β40351
1.05 1.1025 1.02470 3.24037 .θ5238Ι 1.65 2.4025 1.24409 3.037ΟΟ .Μ61βΙ
1.06 1.128d 1.029K 3.25576 943398 1.56 2.4338 1.24900 3.04068 041020
1.07 1.1449 1.03441 3.27100 .θ34δ79 1.57 2.4β4θ 1.26300 3.00232 .03βθ43
1.08 1.1664 1.03923 3.28684 .02592β 1.68 2.40β4 1.250Ω 3.97402 882011
1.09 1.1881 1.04403 3.30161 .917431 1.50 2.5281 8 3.08748 828081
1.10 1.2100 1.04881 3.81662 .Ω0θ091 1.80 2.5800 1.26005 4.00000 .625000
1.11 12321 1.0δ3δ7 8.33167 .οοοροι. 1.61 2.5921 1.26401 4.01248 .β211Ι8
1.12 1.2544 1.05830 3.34684 .892857 1.62 2.6244 1,26886 4,02402 .θ172Μ
1.13 1.2760 1.06301 3.36155 .8849δ6 1 83 2.θ5βθ 1.27279 4.08733 113407
1.14 1.2996 1.06771 3.37639 877193 ι:64 2.θ8θθ 1.27671 4.040β0 .θ0θ7δ6
1.15 1.8225 1.07238 8,39116 .8θ9565 1.66 2.722δ 1.28002 4.06202 .β0β061
1.18 1.8889 1-07703 3.40588 .8θ2069 1.66 2.7558 1.28452 4.0743Ι 302410
1.17 1.3924 1.08187 8.42053 .85470Ι 1.87 2.7889 1.28841 4.08β50 .5θ8802
1.18 1.4161 1.08628 3.43511 .847458 1.68 2.8224 1.29228 4,09878 .595238
1.10 1.4-400 1.09087 3.44964 .84083β 1.69 2.8581 Ι.2θβ15 4.110Ωβ .59171θ
1.30000
1.20 1.4641 1.09545 3.46410 .833383 1.70 2.890 4.12311 .588235
1.21 1.4884 1.10000 3.47851 .826446 1.71 0 1.30384 4.13521 .6847θδ
1.22 1.5129 1.10464 3.49286 .818008 1.72 2.9241 1.30767 4.14720 .581395
1.23 1.5876 1.10905 3.50714 .80β452 1.73 2.958 1.3114θ 4.15033 .67803δ
1.24 1.5625 1.11355 3.52186 .800000 1.74 4 1.31529 4.17133 .δ74718
1.31900 4.18330
1.25 1.5876 1.11808 3.58558 .793θ5Ι 1.75 2,0029 .δ71429
1.26 1.6129 1.12250 8.64θθ5 .78740 1.76 8.027β 1.32288 4,1952 .508182
1.27 1.β384 1.12θθ4 8.56371 2 1.77 8.0β2 1.32ββ5 4 .5Μ072
1.28 1.6841 1.18137 3.57771 .7812δ 1.78 5 1.33041 4.2071 .δθ17θ8
1.29 1.6900 1.13578 8.59166 0 1.79 3.097 1.33417 4 .5δ8β6θ
1.7161 β 1.33791 4.2190
1.80 1.14018 8.80556 .7751θ 1.80 .5δ555θ
1 81 1.7424 1.14-455 8,61939 4 1.8 3.132 1.341U 0 .55248β
1.32 1.7689 1.14891 3.68318 .76θ23 1
9 1.3453 4.2308 .649461
1.33 1.7θ6β 1.15326 1.82 3.1β8 0 4
3.64692 1 .64β448
1.15758 4 1.3490 4.2423
1.34 1.8225 3.β60θ0 .76885 1.88 .δ43478
1.8496 1.16190 1.8 3.204 7 4 .
1.35 3,67423 9 1 1.3527 4.2544
1.87βθ 1.1θβΙθ 4
1.36 3.68782 .7δ757 7 1
.540541
1.37 1.00" 1.17047 3.70185 β 1.8 3.240 .537β34
1.17473 .7δ188 5 0 1.3564 4.2ββ1
1.38 1.932 3.7148 .δ3475θ
1.17898 0 1.8 3.276 7 5
1.39 1 4 .53Ιθ16
1.18922 .74θ2θ 6 1 1.8601 4.2778 .520101
1.40 1.960 1.18743 3.72827 θ 1.8 3.312 5 5 .δ2θ31β
1.41 0 1.19164 8.7416 .74074 7 4 1.3638 4.2805 .5235β0
1.42 1.988 1.1958 6 1 1.8 3.348 2 2 .520833
1.4 1 3 3.7550 .73529 8 9 1.3674 4.8011 .518185
3 2.01" 1.2000 0 4 1.8 3.38δ 8 6 .51Μθ4
1.4 2.04-49 0 8.7882 .72992 9 θ 1.8711 4.3127 .δ12821
4 2.078 1.2041 9 7 1.ρο 3.422 3 7 510204
1.4 β 6 3.7816 .724β3 ι.θ1 5 1.3747 4.8248 .507θ14
5 2.102 1.2088 3 8 1.02 3.46θ 7 5 . 505051
1.4 6 0 8.7947 .71942 1.θ3 β 1.8784 4.3359 .502518
6 2.131 1.2124 3 4 1,9 3.496 0 0 .δ00000
1.4 6 4 4 9 1.3820 4.8474
8.8078 .7142Μ
7 2.160 1.2165 9 .70θ22 1.9 3.534 3 1
1.4 9 5 3.8209 0 5 4 1.3856 4.8689
8 2,190 1.220K 9 .70422 1.0 8.572 4 0
1.4 4 1.2247 8.8840 5 6 1 1.3892 4.8708
9 22201 4 6 .βθ930 1.2 3,610 4 5
1.5 2.250 8.8470 1 7 0 1.3928 4.3817
0 0 8 .θθ44-44 1.0 3.848 4 8
3.8600 8 1 1.3964 4.3031
.68θθδ
5 1.0 3.686 2 8
5
9 4 1.4000 4.4045
8.8729 184932
8 880272 2,00 3.724 0 4
.θ75β7 9 1.4035 4.4158
θ 3.763 7 8
.87114 6 1.4071 4.4271
1 3.802 2 9
5 1.4106 4.4384
.θβ68β 7
7 8.841 7
6 1.4142 4.4497
3.880 1 2
9 4.4609
8.920 4
4 4.4721
3.960 4
1
4,000
0

ТАВО ROOTS—REOROCLS
и; и;
2.0 4.000 1.4142 4.47214 .600000 2.5 02600 1.6811 б.ооооо .400000
0 0 1 4.48330 .407612 0 8.300 4 6.00990 .39840
2.0 4.040 1.4177 4.49444 .406060 2.5 1 1.6843 5.01 0
1 1 4 4.50665 .402011 1 6.350 0 оев 806825
2.0 4.080 1.42127 4.51664 -400196 2.624 1.6874 6.02001 .396267
2 4 1.42478 . 2.636.400 6 5.03984 .303701
.487806
2.0 4.120 1.42820 4.62760 .485437 2.3 0 1.6006 6.04075 .392167
3 9 1.43178 4.53872 .48.300 2.65 6.461 0 6.0Б9В4
.390025
2.0 4.181 5.00062
1.43527 4.54973 2 2.50 0 1.6037 ..38010Б
4 6 4 6.07937 .387607
1.48875 4.60070 .480760 2.57 0.6026
2.0 4.2026 4.67105 .478439 в.5530 1.6008 6.08020 .386100
1.44222 2.68 6.00-40
6 4.2436 1.44568 4.58258 .47BlOO 2.60 0.0604 7 6.00002 .384015
2.06 4.2840
4.69347 .473034 1.60000 6.10882 .383142
1.44914 2.00 6.7081
2.07 4.3264 .471098 1.60312 6.11869 „381670
1.46258 4.60435 2.61 0.7000
2.08 4.3681 .469434 1.00В24 5.12835 .380228
1.45802 4.61510 2.02 0.8121
2.00 4.4100 .437290 1.80936 6.13809 .378788
1.46945 4.62601 2.63 6
2.10 4.4521 1.40287 .43511B 2.64 Вб44 1.61246 6.14782 .377358
4.63081
2.11 4.5369 1.4B62 .432063 1.61565 6.16752 .376040
4.64768 2.66 6.9160
4.5796 9 .430820 1.61804 6.1B72 .З74бЗ2
2.13 4.66833 2.66 6.0696
.4б87Ш 1.82173 0 .373134
2.14 4.3225 1.43069 4.66005 2.87 7.0226
-466621 1.02481 6.17687 .371747
2.16 4.8056 1.47309 4.67974 2.08 7.0763
1.62788 5.18662 .370370
2.10 4.7089 1.47648 4.69042 .454645 2.60 7.1289
4.7524 1.47080 .452480 1.63096 5.10816 .звоои
2.17 4.70106 2.70 7.1824
4.7061 1.48324 .450450 1.63401 5.20677 .307647
2.18 4.71160 2.71 7.2361 1.63707 5.21633
2.10 4.8400 1.48661 4.72220 .мизо 2.72 7.2900 1.64012 5.22404
.зеозоо
220 4.8841 1.48907 4.73286 .446429 7.73 7.3441 .364064
4.0284 1.40332 1.04317 6.23450
2.21 4.74342 2.74 .зозвзв
4.0720 1.49666 .444444 7.зои 1.34021 5.24404
2.22 4.76396 .442478 2.75 7.4520 .302310
6.0176 1.64024 5.26367 .361011
2.23 1.50000 4.70445 .440529 2.76 7.6076
6.0626 1.65227 6.26308 .360712
2.24 1.50333 4.77493 .438600 2.77
6.1073 1.60a05 7.6626 1.66620 6.27257 .358423
2.26 4.78530 .43В6Д1 2.78
5.1629 1.60007 7.0170 1.65831 6.28206
2.26 6.1984 4.79583 -434783 2.79 7.072D .357143
1.68132 6.20160
2.27 6.2441 1.61327 4.80625 .4З20ОО 2.80 7.7284 1.60433 5.30004
.Зб5872
2.28 1.51658 4.81664 .431034 .З54В1О
6.2900 2.81 7.7841 186733
2.20 4.82701 5.31037 .353367
6.3301 1.61987 .429185 2.82 1.67033 6.31977 • .35211
2.80 5.3824 1.62315 4.83736 .427350 2.83 7.8961
1.07332 6.32017 3
2.31 6.4280 1.62643 4.84738 .426632 2.84 7.0624
6.4760 1.62971 1.67631 6.33854 .Зб0877
2.32 4.86798 .423720 2.85 8,0080 1.67929
2.33 5.6226 1.63297 4.8B82B 8.065B 6.34700 .340660
.421D41 2.86 1.68228
2.34 4.87852 5.86724 .34,S4.32
5.6139 1.63023 .4201ав 2.87 8.1226 1.68523
Б.веи 1.53048 4.88876 5.36056 .347222
2.36 .418410 2.88 8.1703
6.87687 .346021
2.30 6,7121 1.54272 4.89898 .413B67 2.89 82369 1.68819
2.37 5.70(Ю
1.54600 4.00918 .414Я38 2.00 8,2944 1.60116 5.38616 .зиви
2.38 6.8081 l.54019 4.01936 .413223 2.01 8.3621 1.60411 6.30444 .343643

178
2.30 6.8664 1.56242 4,02060 .411523 2,02 8.4100 1.60700 6.40370 .3424B6
6.9049 1.66663 4.93004 .40083B .341207
293 8.4681 1.70000 6.41296
2.41 6.9636 1.56885 4,04076• .408133 2.04 8.6264 1.70204 6.42218 .340130
242 а.0026 1.66205 4.96984 .406604 8.5849 1.70587 6.43130 .338083
6.0616 2.05 8.04.30
2.48 6.1009 1.66626 4.06091 .404858 2.06 1.70880 6.44060 .S37888
2.u 0.1504 1.60844 4.97903 .40322а 2,97 8.7026 1.71172 6.44977 336700
2.45 6.2001 1.67162 4,08999 .40160В 2.98 8,7616 1.71434 6.46894 .885670
2.46 6,2500 1.67480 б.оошо 2,99 8.8209 6.46800 .334448
.400000 1.71766
2.47 1.67797 8.8804 .зззззз
3.00 1.72047 6.47723
2.48 1.68114 8.0401 1.72387
2.49 о.оооо 1.72627
2.50 1,72916
1.78206
П

81
ΤΑΒΙΕ XV —SQUARES—SQUA.RE

1/η η

3.00 9.0000 1.73205 5.4772 .33333 8.5 12.250 1.8708 6.0180 .28571
3.01 9.0601 1.73404 8 3 0 0 3 8 4
3.02 9.1204 1.73781 5.4888 .83222 3.5 12.320 1.8785 5.9245 .284ΩO
Ο
3.03 0.1809 1.740θθ 5 β 1 1 0 8 .2840Ω1
3.04 9.2416 1.74866 5,4054 .33112 3.5 12.390 1.87β1 5.032Ω0
5.0413 .28328
3.05 0.3025 1.74642 5 θ 2 4 7 0
5.5046 .83ΟΟ33 3.5 12.460 1.8788 8
3.08 9.3836 1.74029 .28248
4 .328θ4 3 9 3 5.9407
3.07 9.4240 1.75214 0
5.5136 7 8.5 12.531 1.8814 0
3.08 9.4864 1.75409 .281βΩΟ
3.09 9.6481 1.75784 2 .32786Ω 4 6 0 5.9581Ω
δ.ΩΟΟ57 .28080
5.5226 .82β70 3.5 12.002 1.8841
8.10 9.6100 1.76068 5.Ω74Ω5 0
8 7 6 δ 4 5.118331
3.11 9.8721 1.76352 .280Ι12
5.5317 .32573 8,5 12.673 1.8868
5.00100
.27Ω33Ο
3.12 0.73“ 1.76635 3
1.7βθ18 3 6 0 0 ο.οοοο .27855
3.13 0.7960 .324β7 ο
3.14 9.8506 1.77200 5.ΜΟ7β 3.5 12.744 1.880* 2
5.6407 5 7 0 4 0.0083
3.15 9.9225 1.77482 7 .32382 3 .27777
3.5 12.816 1.8020 Β
3.10 9.9856 1.77764 5.5587 6 8 4 9 0.0100
.2770Ω8
8.17 10.048 1.78045 8 .32258 3.6 12.888 1.80473 4
.27β24
3.18 9 1.78326 1 0 1 0.024θ
5.5677 3
3.1 10.112 1.78β0β 6 .32Ι543 3.6 12.θ0ΟΟ 1.80737 5 .27648
13.032
0 4 1.78885 5.5767 .32Ο5Ι 0 1.00000 0.0332 2
4 1
8.20 10.170 1.70105 5.5857 3 3.6 1.00203 4 .27472
13.104
3.2 1 1.794-44
0 δ.
.31θ48 1 1.0062β 0,04152 δ
10.2400 1.79722 50464 0 3.0 4 1.90788 β.Ο4Ο7
1 Ω .27307
.31847Ι 2 13.170
3.2 ιο.3ΜΙ 1.80000 5,6035 1,01050 β.Ω58Ο5 3
10.3684 3.0 0 6.000,30
2 1.80278 7 .3174β 1.01311 .27322
13.240 6.0745
3.2 10.482 1.80555 5.6124 0 3 1,91572 4
9 3.6 0 4
3 1.80831 9 .31β4δ 1.91833 .27248
3.2 10.497 1.81108 5.6213 β 4 13.322 1.92004 6.0827 0
4 8 1.81884 9 .31545 3.6 5 0 ,27Ι73
1.92354
13.305 6.0000
3.2 10.562 1.81859 5,6302 7 5 1.02β14 Ω
5 .3144θ 3.0 0 8 .27100
5 1,81934 8 1,02878
13.468 8.0001
3.2 10.627 1.82209 5,6391 5 6 1.93132 3
6 .31348 3.6 0 8
6 1.82483 5 1.03301 .27027Ω
3.27 10.θθ2 0 7 13.542 6.1073 .2β0Μ2
1.82757 5.6480 4 1.03640
3.2 θ .31250 3.0 7 .2β881
8 10.758 1.83030 1 8 13.616 1.93007 6.1155 7
0 1.041Ο
3.2 4 1.83308 5. 3.6 1 5 .2080Ω7
.31152 5
9 10.824 1,83570 βδβ8δ 0 13.600 .2β738
β 1.04422 0.1287
1.83848 5.8656 0
8.3 1 1.84120 9
.810δ5θ 8.7 0
1.94079 2
0 10.890 .8095θ 0 13.764 6.1318 .200ββ7
3.3 0 1.843θΙ 5,6745 8 3.7 1 1.0403β 8 .2650δ
1.Ω5ΙΩ2
1 10,956 1.84662 0 .308β4 1 13.838 6.1400 7
1.8493 5.8833 4 1.06448
3.3 1 2 3.7 1.05704 3 .2β526
2 1 13.012 6.1481 2
2 11.022 .3070θ 2 1.05050
3.3 4 1.8520 5.8921 3.7 θ 7 .2G4560
0 2 1.06214
3 11.088 8 3 13.987 0.1563 .2β3862
.30θ74 1.ΩΜβ
3.3 9 1.8547 5.7008 3.7 6 0 .203158
8 9
4 2 8 4 14.062 .2β24β7
.30581 1.Ωβ723 6.1644
3.3 11.222 1.8574 5.7096 0 3.7 E 1.90077 1 .2β1780
5 2 4 14.137 6.1725 .2β1ΟΩ7
5 .30487 5 1,07231
3.3 11.289 1.8601 5.7183 8 3.7 6 2 .2βΟ4Ι
6 1 9 14.212 1.07484 7
6 ,3Ο3θδ1 6 0.1808
1.8627 5.7271 1.07737
3.3 11.350 .30303 3.7 9 1.07000
1 .25θ740
9 9 3 14 0.1887 .26Ω0β7
7 0 7 1.98242
3.3 11,192 1.'8654 5.7858 3.7 2884 0 .2583Ω8
5 .30211 1.98494 .267732
8 1 8 8 14:364 6.1967
5 .26700
3.3 11.560 1.8681 5,7445 3.7 1 1.08740 7
.30120 0
9 5 8 9 14.440 1.9809
0 5 8.2048
1.8708 5.7582 7 .25β41
3.4 11,628 .30030 3.80 0 4
0
8 6 1.9024
0 1 0 8.8 14.510 6.2128
.2657Μ
5.7619 1 9
3.4 11.θ9β 299401 1 0
4 1,9940 .25610
1 4 .2θ85Ο7 3.8 14.502 0.2200
2
5.7700 4 9
3.4 11.764 .207β1 2 3
2 1.9976 .2Μ4δ.
2 0 θ 3.8 14,068 0.228Ωβ
3
5.7792 0 0 0.2800
4.4 11.833 298736 3 268807
7 2,00000
3 6 • .29δ86 8.8 14.745 0

180
3.4 11.902 5.7879 8 4 6 • .2δ31θ6
4 5 2 .2θ498 0.24500 .2δ252
ιι.θ71β 3.85 14.822
3.4 5.7985 6 8.8 5 0.2580 5
12,040 5 0 .2δ188
5 -294118 0 14.800
9 6.2009
3.4 5.8051 .20325 3.8 6 0
12.110 0
8 7 5 7 14.θ7β .2δ125
4 6.2689
3.4 5.8137 .2θ23θ 8.8 θ β
12,180 7
7 8 8 8 15.064 250027
1 θ.27βΜ
3.4 5.8223 .29154 3.8 4 .2Β000Ο
12.250 7 5 6.2840
8 9 15.132
0 0
• 5.8309 .2θ0β0 1
3,49 5 8 8.0 8.2928
15.210 6
8.5 5.8305 .28985 1 0
2 3.0 0.3007
0 5 15.288
5.8480 .28901 2 0
1 6.3087
8 7 3.0 15.30Μ
5.85ββ .2881 8 2
15.444 6.3106
2 Μ 8.9 0
5.8β51 .2873 4 4
15.623
5 M 8.9 6 6.3245
5.8736 286638 δ 6
15.602
7 .28571 3.0 5
5,8821 4 6 16.681
8 3.0 0
5.890β 7 15.760
7 8.9 9
5.89915 8 15.840
5.90762 8.9 8
δ.θ1β0 9 15,920
8 4.0 1
0 16,000
0

1/η

TABLE XV— —SQUARE ROOTS —RECIPROCALS


ν•Πλ

4.0 16.000 2.00ΟΟ 6.32456 .25ΟΟO 4.5 20.260 2.1213 8.7082 .222222
Ο 0.33240 Ο
0 0 0 0 2 0 221729
2.0025Ω a.M035 .24937
4.0 10.080 2.Μ4ΩΩ 4.6 20.340 2.1236 θ.71δβ5 .22123θ
1 1 (3.3482 7 1 1 8 6.72309 .220751
2.0074 3
4.0 16.100 9 0.35010 .24875 4.5 20.430 2.1260 6.7305 .220264
2 4 2.ωθ08 β.3θ3Ω0 β 2 4 3 3 .219780
4.0 • 2.0124 0.37181 .24813 4.5 20.520 2.1283 6.7379
0,37000 .2Ι029
3 10.2400 0 0 3 9 8 5
4.0 10.321 2.014Ω4 0,38749 .24762 4.5 20.611 2.1307 6.74537
8
4 0 2.0174 0,30531 5 4 8 3 6.7527 .2Ι881
4.0 16.402 2 0.40312 .240ΩΙ 4.5 20.702 2.1330 8 8
5 5 2.01 β.4ΙΟΩ 4 6 5 7 6.7801 .2Ι834
4.0 16.4836 2.0223 3 .24β30 4.5 20.703 2.1354 8
ιο.5ΙΗθ 0.41872 1
0 7 5 0 0 2 8.7075 .2178β5
Ιβ.04θ4 a.u051 7
4.07 2,0248 σ,43428 .245008 4.5 20.884 2.1877
16.7281 6.7749 .2178θ1
4.08 6 σ."205 .244400 7 9 6
5 .21θθ20
4.00 10.81Μ 2.0273 4.68 20.978 2.1400
10.8Ω21 0.46756 .2439Ο2 .2Ιβ450
4.10 1 .24330Ω 4.5 4 0 6.7823
ΙΩ.Ω744 0.40520 .215θ83
4.11 Ι7.Ο5(5Ω 2.0297 .2427Ι8 0 21.0β8 2.1424 3
17.1390 8
6.47302
1 3 8.7897
.2ΙδδΙ7
4.12 .242Ι31 4.00
2,03224 0.48074 0 .2Ι5064
4.1.3 17.2225 241540 4.01 21.160 2.14476
6.48846 .214592
17.30.ΡΑ] 2,0347 β.4θ015 .240064 4.02 0 2.1470 8.7070
4.15 0 6 .2Ι4Ι83
17.3880 0.50.384.
.240385 4.03 21.252 9
4.10 17.4724 2,0371 6.51153 .23Ω8Ο8 1 2.1494 8.80441 .213θ76
4.17 17.5501 8.51020 .23θ234 4.05 21 34,14 2 6.81175 213220
5
4.18 17.0400 2.0300 8.52087 238063 4.βθ 2124309 2.15174 β.8Ιθ09 .2127βθ
4,10 σ.5'ωΙ52 4.07 21.5206 2.16407 8.82642 .2Ι281
17,7241 1 238095
4.20 8.54217 4.68 21.6225 2.Ι5θ3θ 6.83374 4
17.8084 2.Μ46 .237530
4.21 17.892Ω
6.54081 4.09 21.7166 2.16870 6.84105
.2300β7
6.84836 211864
Ο
4.22 17.077β 2.0460δ 6.56744 4.70 21.8089 2.16102
.23θ407
4.23 18.0β2δ 2,0403
6,50500
.235840 4.71 21.9024 2.103,38 6.85565 211416
β.572θ7
4.24 18.1470 0 4.72 21.9θβ1 2.16564 6.86204 .210070
8.58027 .2352Π4
4.26 18.2320 2.0518 .234742 4.73 22.0000 2.16795 6.87023 .210526
0.58787
4.20 18.3184 3 .2341Ω2 4.74 22.1841 2.17025 6.87750 .210084
6.50545 6.88477 .209β44
4.27 18.4041 2.054-20 .23364δ 4.75 22.2784 2.17256
6.80.303
4.28 18.4ΩΟΟ 2.05670 ο.βΙ0θ0
.233Ι00 4.76 22.3722 2.17486 6.80202 209205
4.20 18.5701 2.05Ω13 α.0181β .232δ58 4.77 22.4076 2.17715 6.89028 .2087β8
4.30 18.0624 2,0016 σ.θ2δ71 .2320Ι9 4.78 22.5625 2.17045 6.90662 .208333
4.31 18.7480 5 6,63325 .23Ι481 4.79 22.6576 2.18174 6.91375 .207000
4.32 18.8850 2.003Ω8 β.β407Β .230Ο47 4.80 22.7529 2.18403 6.92008 .2074θ9
.230Ι±1δ
4.33 18.9225 2.0064 6.648,31 4.81 22.8484 2.18632 6.92820 .20703θ
ΙΩ.ΟΟΩβ 0,05582 .22Ω885 .20ββ12
4.34
10.00β0
0 4.82 22.Μ41 2.18861 β.Ο3Μ
4.35 2.00882 ο.θβ333 .220358 4.83 23,0400 2.19089 2 .20β186
ΙΩ.Ι844 6.94262 .2057β1
4.30 10.2721 2.07123 0.07ΟΜ .228838 4.84 23.1361 2.19817
β. .22831Ι 6.94982
4.37 2.07364 4.8δ 23.2824 2.19545 .206339
19.3600 67832 .227700 6.25701
4,38 2,07605
10.4481 2.078,40 4.86 23.3289 2.19773 204918
0.88681 .227273 β.9β41θ
4.30
10.5304 2,08087 4.87 23.4256 2.20000 204490
β. 60328 220757 β.θ7137
4.40 19.024Ω 4.88 23.6225 2.20227 .204082
2.08827 β.7007δ .'220244 6.97864
4.42 10.7136 4.8θ 23,6106 2.20454 .203ββ6
2.08507 0,70820 .225734 6.98570
4.44 10.802δ 2.0880 .22δ225 4.90 23.71βθ 2.20081 6.99285 .203252
19.8010 4.01 23,8144 220907 .202840
ΙΩ.Ο8ΟΟ β 224710 7.00000
4.02 23.9121
182
4.40 20.07Μ 2.000-15 .224215 4.03 24.οιοο 2,21133 7.00714 .202429
4,47 20.1β0Ι 2.00284 .223714 4.04 24.1081 2,21359 7.01427 .202020
4.48 20.2600 2.00523 .223214 4.05 24.2064 2,21685 7,02140
.222717
.20Ιβ1
2.007β 4.Ωθ 24.3042 2.21811 7.02851
4.50 2 4.07 24.1086
3
.222222 2.22036 7.03δβ2 .20Ι207
2.1000 4.08 24,5025 2.22261 7.04278 .200803
0 4.θ0 24.6016
2.1023 2.22486 7,04082 .2ΟΟ4Ο1
5.00 24.7009 7•05βθ1
2.22711
8 24.8004 7,06399 .200000
2.10476 2.22936
24,0001 2.23159 7,07107
2.10713
25.0000 2.23388
2.1005Ω
2.1118 2.28607
7
2.1142
4
2.11080
2.118Ωβ

2.1213
2
1/η
88
—SQUAE
XV—SQVARES ROOTS—RECTROCALS
И10 п цип

184
XV—

5.00 26.0000 2.23607 7.07107 .200000 6.61 30.2500 2.34521 7.41620 .181818
*6.01 25.1001 2.23830 7.07814 .199B01 5.62 30.3601 2.34734 7.42294 .181488
5.02 26.200' 2.24054 7.08520 .199203 5.53 30.4704 2.34947 7.42067 .181169
5.03 26.3009 2.24277 7.09226 .198807 5.54 30.5809 2.35100 7.43640 .180832
5.04 26.4016 2.24499 7.00030 .198413 30.6016 2.35372 7.44312 .18ОбО6
5.56
6.05 26.5025 2.24722 7.10034 .198020 5.56 30.8025 2.36584 7.44983 .180180
6.06 25.0036 2.24044 7.11337 .197628 6.57 30.01.36 2.35707 7.46654 .170866
6.07 26.7040 2.25107 7.12030 .107239 5.58 31.0249 2.30008 7.46324 .170533
5.08 26.806' .1D6850 31.1.364
2.26389 7.12741 5.50 2.36220 7.40994 .170211
6.00 26.9081 7.13442 .1004В4 31.2481 2.33432 7.47663 .178891
2.26610 5.00
5.10 26.0100 7.14143 .106078 6.61 31.3600 2.30643 7.48331 .178671
2.25832
5.11 20.1121 7.14843 .196005 5,02 31.4721 2.30854 7.489D9 • 178253
2.26063
5.12 26.2144 7.16542 .196312 6.03 31.6844 2.37065 7.49667 .177936
2.26274
6.13 26.316D 7.16240 .104032 5.64 31.600D 2.37276 7.50333 .177620
2.20405
5.14 26.4196 7.169.38 .104653 31.8006 2.37487 7.60900 .177306
2.26716 6.05
6.16 26.522E
7.170.35 .104176 5.00 31.9225 2.37607 7.61665 .176991
2.26036
6.16 26.026€ 7.18331 .103798 5.07 32.0350 2.37008 7.52330 .176678
2.271бВ
5.17 26.7289 7.10027 .193424 6.08 32.1489 2.38118 7.62994 .170307
2.27376
6.18 26.8324 7.10722 .1D3050 6.00 32.262,1 2.38328 7.53068 .170056
2.27596
6.19 26.0331 7.20417 .102078 32.3701 2.38537 7.64321 .176747
2.27813 5.70
5.20 27.0400 7.21110 .102308 6.71 32.4000 2.38747 7.64983 .175439
2.28035
6.21 27.1441 7.21803 .191939 6.72 32.6041 2.38056 7.56046 .176131
2.28264
6.22 27,2484 7.22400 .101671 6.73 32.7184 2.39105 7.60807 .174826
2.28473
6.23 17.3629 7.23187 .101205 6.74 32.8329 2.30374 7.60908 .174620
228692
5.24 27.4576 7.23878 .190840 32.0470 2.30683 7.67628 .174216
2.28910 5.76
6.26 27.605 7.24669 .100476 6.70 33.0025 2.30792 7.58288 .173013
2.29120
5.26 27.G07B 7.26259 .100114 6.77 33,1776 2.40000 7.58947 .173611
2.29347
6.27 27.7729 2.20565 7.25048 .180763 6.78 33.2920 2.40208 7.59005 .173310
5.28 27.8784 .180394 33.4084
2.29783 7,20036 6.79 2.40410 7.00263 .173010
33.6241
5.29 27.9841 2.30000 7.27324 .18003
5.80 2.40624 7.60020 .172712
B зз.иоо
5.30 28.0900 2.30217 7.28011 5.81 2.40832 7,01577 .1724И
33.7561
5.31 28.1001 2.30434 7.280D7 .188070 5.82 2.41030 7.622.34 .172117
33.8724
5.32 28.8024 2.30661 7.29383 .188324 5.83 2.412,17 7.82880 .171821
33.088D
5.33 28.4089 2.30868 7.зоовд .187070 6.84 2,41464 7.63644 .171627
34.1050
5.34 28.6150 2.31084 7.80753 .187017
5.85 34.2225
2.41601 7.0419D .171233
.1872B6
5.36 28,6225 2.31301 7.31487 6.80 34.3306 2.41808 7.а4863 .170940
6.30 28.7206 2.31617 7.32120
.18В01В
6.87 34.460D 2.42074 7.66ЯО .170040
6.37 28.8360 .180607 2.42281 .170368
2.31733 7.32803 5.88 34.5744 7.00150
5.3 288444 .180220 2,42487 .170008
2.31048 7.33485 6.80 34.021 7.00812
8 20.0621 .185874 2.42093 .160770
2.32164 7.34100 5.09 34.8100 7.07403
6.3 20.160 .185629
2.3287П 7.348,1 6.01 34,9281 2.428пп 7,08116 .1В049
0 0 2.326D4 .186186 2.43106 2
7 5.02 35.04И 7.08705
6.40 20.268 2.32800 7.36627 .18484 2.43311 .16О2О6
5.0 36.104 7.00416
6.4 1 2.33024 7.30200 3 2.4351B .168019
3 D 7.70005
1 29.376 2.3323 7.30886 .184602 2.43721 .108034
.1&4102 5.0 35,2836 7.70714
6.4 4 8 7.37664 .18882 4 2.43920 7.71302
.1B886
29.484 36,4025

185
XV— —SQUARE

2 9 2.3345 7.38241 4 36.6210 2.441.3 7.72010


6.4 29,503 2 7.38018 .1Њ3480 36.0409 1 7.72358
3 6 2.3306 7.30504 .18316 86.7004 2.4-1330 7.73306
6.4 6 7,40270 0 35.8801 2.44540 77305
29.702
4 2.3388 7,40045 .18281 2.44746
6
6.96
80.0000 1
6.4 29,811 0 7.41620 6 2.44949 0
5.03 7.74697
6 6 2.3400 .18248 .168067
4 д.97
6.4 20,020 2 6.08 .107786
0 9 2.3430 .18214 .167604
5.0
6.4 30,030 7 9 .167224
9
7 4 2.3452 .18181 .160946
0.0
6.4 80.140 1 8 .1В06В7
0
8 1
6.4 30.260
9 0
5.5
0

4/10 п Vn пя
С; Чоп
ΑΒΙΒ SQUAR.ES—SQUAR.E ROOTS--RECIPROCALS

3Ι'.οοοο 2.44940 7.74507 .1ΟOΩ67 0.50 42.2500 2.64951 8.00220 .158&46


311.1201 2.45153 7.75242 .ιαο.380 0.51 42.3801 2.651,17 8.0θ84Ο .Ι53β10
e .45357 7.76887 .ιιιυιι, 0.52 42.6104 2.65Μ3 8.07405 .Ι53374
2.455111 7.706,3 3 (5.5 42.ΜΟΙ) 2.65539 8.080K .Ι6313θ
30..Ι8Ι0 2.457114
1 .10Γ,837 3 42.7716 2.66734 8.087Ο3 .1δ2905
2.45,ηα7 7,77174 .1055β3 υ.δ4 42.0025 2.569,30 8,09321 .152072
311. ΗΜΙ'
2..10171 7.77817 .105289 0.55 43.11140 2.50125 8.0θ038 .Ι52439
37.0881 2.4ΙΙΠ77 7.7Μ80 .ΙΙ)ΛΩ17 Ω.6(Ι 43.200.* 2.60:120
8.10655 .152207
0.57
37.2100 2.40770 7.7ΙΙ102 .1ΙΗ7.ι.π υ.68 4Β.428Ι 2.611710 8.11172 .15Ιθ7β
.1Ι54474
37.3321 2,411082 7.707.14
.1134204 . 43.5000 2.66006 8.11788 .151746
37.4544 2.,Ι7 Ι 7.80385 4-3.00'21 2.570ΙΙΩ 8.12404 .151515
103934 0,00
37.67110 2.,Ι738ΙΙ
7.81025 0.01 43.8244 2.67294 8.130Ιθ .1δ128β
.Ί7.υυοο 2.475ΗΗ 7.81005 .ιυ:- 11.02 43.060') 2.57488 8.13634 .1δ1057
υ.υ.3
37.8225 2.47700 7.82304 Ι300 0.64
44,0890 2.57882 8.14248 .16Ο83Ο
37,η.ιπυ 2.47002 7.829-13 .103132 0.1.15 44.2225 2.Λ7870 8.1480 -150002
.3Η.1ΙΙ2,Ι 2..ΙΒ.3Ω5 7.83682 .1028ου υ.οο 44.3650 2.68070 2 .15037β
38.3161 .1β2ΙΚ)2 0.07 44.488%) 2.582153
7,84210 0.118 4,1.0224 8.1547 .150Ι50
2.58-167
38.4400 2..18707 7.8-1857 .1112076 σ.υθ *1.7501 2.68060 5 .14θ925
7.85-103 ,101812
38.088,Ι 2.48008 7.81113Ω .ιυΙ6Γ,1 6.70 44.8900 8.1008 .14θ701
38.8120 2..ΜΙ.3ΙΙΙΙ 7.80700 υ.71 45,0241 2.588" 8 .149477
.1012ΩΩ '2.6ΩΩ37 8.1670
:j8.0370 '2.401500 0.72 .14θ2Μ
2.40800 7.87401 .ιοιο,Β1 0.73 45.1584 2.50230 1
.149031
39.0025 7.880,30 .1Ι]Ω772 46.2029 2.59422 8.1731

186
XV—

311.1871 2.60000 7.88ΙΙ7() .1U0614 0.76 45.4270 2.60016 3 .148810


1 7.80303 .10Ω26Ο
2.504( 0.70 45.5025 2.59808 8.1792 .148688
:ιιι.312Ι)
7.80037 .ιοοοοο (Ι.77 4 .1483θ8
311.438-1 2.θ0Ιθ2
.3θ.5ΙΗ1
2,50790 7.ΩΩ5ΟΩ .1ΛΩ7-14 0.78 46.8320
2.60384 8.18685 .148148
2.6ΩΩΩ8 0.79 46.008,
7.01&3.3 1 2.00676 8.10146 -147920
311.8101
2.511 7.1)2405 .168θ8 0.80 .147710
40.1041 8.1976
Ω7 7.030θ5 3 0.81 2.60708 -147493
8
2.51.300 46.2400 2.00Ι)β .14727δ
40.1050
2.51606 7.Ω872 .Ι68730 8.82 8.20886
0.83 40,3781 0 8.20075 .
.10.3225 2.51704 5 .15847
7.9-1355 8.84 40.6124 2.01161 .147060
40.,Ι40β θ 8.2158
4β.Μ8Ο 2.81348
40.6700 2.51 7.041>84 ,158'228
0,85 4
.14ΟΒ43
40.711,14 ΩΩ2
7.Ω5Ο13 .157Ω78 40.78δβ 2.01634 .14θ028
7.ΩΙΙ241 0,80 8.221θ2
40.8321 2.521 .15772 2.01725 .14β4Ι8
0,87 40.9225 8.2280
2.Γ,23ΒΩ 7.00809 0 0.88 47.οποιι 2.010Ι6 .14β10θ
0
41.08Β 2.52587 7.07-1Ω(Ι
.1574Μ 6.80 47.1ΩΟΩ 2,02107 823408 .145985
7.118123 47,3344 2.112298
1 7.087-10 .15723.3
8.2401 .145773
.1Ι.21Μ 2.62082 .15βΩ8ΙΙ 0.00 47.4721 2.112488
7.ΩΙ)375 5 .1455θ0
2.5318() .1δΙΙ74Ι)
0.92 47.Ω1ΩΩ 2.62070 .146349
2.53377 .1504θ 0,03 47.7481 8.2402
5 2.ΙΙ.305Ω .145138
41.002δ 2.5357.& 8.01240 0.04 47,8804 2,08241)
1
41.73113 2.53772 .1Μ25Ω 8.2522 .144θ28
8.01873 48,0240 2.034,39
41 Β.ΙΥ24Ω 7 .144718
0 .1557(13 0.00 48.1030 2.03020
41.ΙIΙΝ)1 2.5411E5 8.2583 .14450θ
42.1201 2.54.302 .1δ552Ι 0.07 48.3026 2,03818 .14-4300
8.031 .1δ528Ο 0,08 48.4410 3
42,2500 2.51558 ΙΩ 2,ΙΗ008 8.2043 .1440θ2
2.547Π .15Μ30 48.5800 2,04107 .14388δ
8.03741 8
Π 8.11-1303
,ΙΜ7Ω 7.00 48.7201 2,04880 8.2704 .143θ78
2.Π4Οδ Β.Ω4ΙΙ84 0 48.8001 2.64576 .14Μ72
3
1 Η.0Μ0δ .1Μ5(Κ) 4θ.0ΟΟΟ .143280
.1M321 8.2784
8,00220 .143002
.16408„g 7
8.2825 .1428δ7
.1Μ84β
1
8.2885
5
8.2045
8
8.8000
0
8.3000
2
8.3128
4
8.3180
5
8.3240
0
8.3300
7

187
XV— —SQUARE

8.3360
7
8.3420
0
8.3480
5
8.3540
4
8.3600
2
8.3066
0

νη 1.,Ιη

TABLE sQUAREs ROOTS — R.ECIPROCALS

1/71

7.00 4 8.3βββ0 .142857 7.50 50.2500 2.73801 8.66025 .133338


7.01 8.37257 .142β53 7.51 δβ.4001 2.74044 8.β(5β03 .133150
7.02 4θ.2 , 8,37854 .142450 7.52 50.5504 2.74226 8.67179 .132Ω7θ
7.03 49.4209 .38451 .142248 7.53 66.7000 2.74408 8.07756 .132802
7.04 49.5010 o 39047 .142045 7.54 56.8510 2.74591 8.083.32 .132026
7.05 49.7025 2. 643 .141844 7.55 57.0025 2.74773 8.08907 .132450
7.οσ 42.84.30 7 814 38 .141643 7.66 57.1636 2.74955 8.0Ω483 .132275
7.07 49.084θ 5 8.4 .141443 7.57 57.3040 2.75130 8.7ΩΟ57 .132100
7.08 50.1204 2.060 8. 7 .141243 7.68 57.4564 2.75318 8.70632 .13192G
7.09 50.2081 2.662 21 7.59 57.6081 2.75500 8.71200 .131752

7.10 50.4100 2.66458 8.42θ1δ .140845 7.60 57.7600 2.75081 8.71780 .131570
7.11 50.5521 2.θββ4β 8.43208 .140M7 7.01 57.9121 2.75802 8.72353 .ι.314ου
7.12 50.6044 2.66833 8.43801 .140449 7.02 58.0&44 2.70043 8.72020 .131234
7.13 50.8369 2.67021 8.44393 .140252 7.03 58.2100 2.7β22δ 8.73409 .1310(Ι2
7.14 50.9796 2.67208 8.44985 .14006β 7.04 58.30θΟ 2.76405 8.74071 .130800

7.15 51.1225 2.67395 8.45577 .1308β0 7.05 58.5225 2.76580 8.74043 .18071Ω
7.10 51.265β 2.67582 8.40188 .13θβ0δ 7.06 58.β750 2.70707 8.75214 .Ι30548
7.17 51.4089 2.077θθ 8.46759 .139470 7.07 68.8280 2.76948 8.75785 .130378
7.18 51.5524 2.67955 8.47349 .13927β 7.68 58.9824 2.77128 8.70356 .130208
7.19 51.8961 2,88142 8.47939 .139082 7.09 59.1361 2.77308 8.70020 .130089

7.20 61.8400 2.68.328 8.48528 .138889 7,70 59.2000 2.77480 8.774Ωβ .120870
7.21 51.9841 2.68514 8.49117 .138096 7.71 2.77600 8.78000 .129702
7.22 52.1284 2.88701 8.49700 .138504 7.72 59.5984 2.77849 8.78635 .12Ω534
188
XV—
7.23 52.272θ 2.08887 8.50204 .Ι38313 7.73 59.7529 2.78029 8.70204 .1293β0
7.24 52.4176 2.69072 8.60882 .138122 7.74 59.9070 2.78209 8.79773 .12010θ

7.25 52.5β2δ 2.89258 8.51489 .13793Ι- 7.75 60.0625 2.78388 8.80341 .12θΩ32
7.26 52.7076 2.69444 8.52056 .137741 7.70 60.2170 2.78508 8.80909 .128860
7.27 52.8529 2.09629 8.52043 .Ι37552 7.77 60.3729 2.78747 8.81476 .128700
7.28 62,9984 2.09816 8.63220 .137363 7.78 60.5284 2.78927 8.82043 .128535
7.29 53.1441 2.70000 8.53815 .137174 7.79 00.6841 2.79100 8.82010 .128370
7.30 63.2900 2.70185 8.54400 .136θ86 7.80 60.8400 2.70285 8.83170 .12820δ
7.81 53.4361 2.70370 8.54085 .13θ7θ9 7.81 2.70464 8.&3742 .128041
7.32 53.5824 2.70555 8.55570 .13βθΙ2 7.82 81.1524 2.79643 8.84308 .127877
7.33 53.7289 2.70740 8.56154 .13Μ2β 7.83 61.3089 2.79821 8.84873 .127714
7.34 53.8750 2.70924 8.56788 .13β240 7.84 β1.4β5β 2.80000 8.85488 .12755Ι

7.35 54.0225 2.71109 8.57321 .13θ0Μ 7.85 01.β22δ 2.80179 8.86002 .Ι27389
7.30 54.1096 2.71293 8.57904 .13δ870 7.86 01.77θθ 2.80857 8.80500 .12722β
7.37 64.3169 2.71477 8.68487 .135085 7.87 β1.θ3βθ 2.80635 8.87130 .127005
7.38 2.71θβ2 8.59069 .135501 7.88 02.0044 2.80713 8.87βΠ4 .120004
7.39 64.6121 2.71846 8.50651 .135318 7.80 02.2521 2.80891 8.88257 .12β743
7.40 M. 7600 2.72029 8.60233 .135135 02.4100 2.81009 8.88819 .12β582
7.41 Μ.9Ο8Ι 2.72213 8.60814 .Ι84063 7.91 02.δβ81 2.81247 8.80882 .126422
7.42 55.0564 2.72307 8.61394 .Ι34771 7.92 62.7284 2.81425 8.8 θ944 .120263
7.43 55.2049 2.72580 8.61974 .1345Ω0 7.93 02.884θ 2.81603 8.90505 • .12β103
7.44 55.3530 2.72764 8.02554 .13440θ 7.94 63.048d 2.81780 8.91067 .125046

7.45 55.5025 2.72947 8.63134 .134228 7.95 63.2025 2.81957 8.91628 .125780
7.46 2.73130 8.63713 .134048 7.00 θ3.3βΙβ 2.82135 8.92188 .125628
7.47 55.8009 2.73313 8.64292 .1338θ9 7.97 63.6209 2.82312 8.92749 .125471
7.48 55.9504 2.73496 8.64870 .183βθ0 7.08 63.8804 2.82489 8.93308 .125313
7.49 56.1001 2.73879 8.65448 .133511 7.99 63.8401 2.82βθβ 8.038θ8 .125Ι60

7.50 56.2500 2.73881 8.66025 .Ι33333 8.00 64.0000 2.82843 έ.Μ427 .125000
η 1/η ΤΑΈΙ,Έ SQUA.RES—SQTIA.RE

8.00 64.1001
2.82843 8.04427 .12δ000 8.50 72.2500
2.91548 0.2k954 .117647
8.01 04.3204
2.88010 8.94080 .124844 8.51 72.4201
2.91710' 9.22407 .1Ι7δ0θ
8.02 64.4800
2.83100 8.95545 .124β88 8.52 72.5904
2.θΙ8οο: 9,23038 .117371
8.03 θ4.ΜΙβ2.83373 8.90103 .Ι24533 8.53 2.Ω2Ο02
72.7609 0.23580 .117233
8.04 Μ.8025 2.83549 8.9σαοο .124378 8.54 2.92233 9.24121 .1170ίΙ0
72.9310
8.05 64.9030 2.83725 8.97218 .12-124 8.55 73.1025 2.92404 0.24002 .11ΟΩ5ΙΙ
189
XV— —SQUARE

8.00 2.83001 8.97775 .ΙΙ2-40Ι]Ω 8.50 73.2736 2.92676 0.25203 .116822


8.08 05.1240 2.84077 8.083.32 .Ι239]0 8.57 73.4449 2.92740 0.25743 .110080
8.09 05.2804 2.84253 8.08888 .1237β2 8.58 73.0Ι04 2.0201β 9.20283 .110550
,β5.4481 2.84429 8.90444 .Ι23009 8.50 73.7881 2.03087 0.20823 .Ι10414
8.10 Ω.οοοοο
8.11 05.0100 2.81005 9.005δ6 .Ι23457 8.00 73.0000 2.93258 Ω.27362 .110270
0.27901
8.12 05.772Ι 2.84781 0.0ΙIΙ0 .123306 8.01 74.1321 2.93428 -110144
9.28440
8.18 05.9844 2.84060 0.01005 .123153 8.62 74.8044 2.93508 .ιιοοοο
0.28078
8.14
00.090Ι) 2.85132 9.022Ι0 .12300Ι 8.03 74.47βΩ 2.93700 .1Ι5875
06.25θ0 .122850 74.0490 9.29510
2.85307 8.04 2.93039 .115741
8.16 9.02774 9.30054
00.4225 2.86482 0.03327 .Ι220Ω9 8.05 74.8225 2.9410Ω .115β07
8.10 0.30591
00.5850 2.85657 9,08881 .122540 8.06 74.Ω956 2.0427Ο
8.17
80.7489 75.108Ω 2.0444θ 9.31128 .ΙIΜ73
8.18 2.85832 9.04434 .122300 8.07 75.3424 .Ι15340
60.9124 2.94018 0.31605
8.19 2.86007 0.04θ8Ο .122240 8.08 75.5101 .116207
07.0701 .122100 2.04788 0.32202
2.80182 8.00 .1Ι5075
8.20 0,05530 75.0000 2.04Ω58 0.32738
67.2400 2,80350 ο.ουοοι .121051 8.70 75.8641
8.21 2.95127 9.33274 .114Ο43
07.4041 2.80531 9.006-42 .Ι21803 8.71 70.0384 .1148ΙΙ
8.22 2.05290 0.33809
07.5084 2.8070δ θ.Ο71Ω3 .12Ι05ΰ 8.72 70.2120 .114079
8.28 2.Ωδ40Ο 0.34345
07.7329 2.80880 9.077U .121507 8.73 7Ω.3870 .114648
8.24 2.06035 0.34880
67.8970 2.87054 9.082Ω5 .121359 8,74 .11M10
8.25 9.08845 70.5025 2.95804 0,85414
68.0β25 2.87228 θ.003Ω5 .Ι2Ι212 8.75 70.7370 2.Ω5Ο73 .1Ι4280
8.20 ρ.35θ4Ω
082270 2.87402 θ.ΟΩΩ45 .121005 8,70 70.9120 2.00142 9.38483 .114155
8.27 08.892Ω 0.10494
2.87570 .12ηοιο 8.77 77.0884 2.Ωβ3ΙΙ 9.37017 .114025
8.28 68.5584
8.29 68.7241 2.87760 0.11043 .120773 8.78 77.2041 2.0047θ 9.37550 .1Ι38Ωδ
2.87024 9.11502 .120027 8.70 2.00048 0.38083 .Ι13700
8.80 88.8000 77.4400
8.31 οο.05θ1 2.88007 0.12140 .120482 8.80 77.6101 2.90810 0.38610 .113030
8,32 00.2224 2.88271 0.12088 .120337 8.81 77.7024 2.00985 0.30140 .113δ07
8.33 89.3889 2.88444 0.18236 .Ι2Ω102 8.82 77.9080 2.Ω71Δ3 9.39081 .1Ι837Ω
2.Ω7321
8.84 09.5550 2.88017 9.13783 .12Ω048 8.83 0.40213 .ΙΙ3250
2.88701 0.143.30
.110ΙΚ)4
8.84 78.1450 2.07480 .113122
8.35 βθ.7225 Ω.14877 78.3225 0.40744
8.30 09.8800 2.88934 Ω. 15423 .110700 g. 25 78.4900 2,97668 0.41270 .1129M
.1ΙΩOΙ7 2.07825
8.37 70.0500 2.801,37 Ω.Ι59ΟΟ 8.80 0.41807 .112807
.ΙΙΩ474 78.6700
8.38 70.2244 2.80.310 0.10515 8.87 78.85-14 2.97903 0.42888 .112740
.1Ι0332
8.30 70.8921 2.80482 0.1700(5 .11Ω1θ0 8.88 70,0321 2.98101 0.42868 .1Ι2βΙ3
2.80055 8.80 2.98320 Ω.433Ω8 .1Ι2480
8.40 70.5000 2.80828 0.18150 .ΙIΟΟ48 70.2100 2.08-100 Ω.43Ω28
8.41 70.7281 2.9Ω00Ω 0.18005 .118000 8.00 70.8881 9.4-4468 .112.goo
8.01 70.560, 2.08004 .112233
8.42 70.8964 2.οοιη Ω.1θ23Ο .Ι18706 2.98831
Ω.44Ω87
71.004Ω 0.10783 8.02 1 Ω.4551Ο
.112108
8.43
71.2880 2, ΩΩ.345 9.20326 .1Ι8024 8.03 70.744Ω 2.98908 9,40044 .Ι11Ω82
8.44 2.00δΙ7
71.4025 0.20800 .ΙΙ8483 70.9230 2,001130 Ω.4β573 .111857
8.45
71.5716 2.00680 0.21412 9.47101
.11Μ4-3 2.90.333
8.05 80.1025 2.Ω95ΟΟ 9.47θ2Ο .111782
8.40 .Ι182Ο8
71.7409 2.008υι 0.21054 8.00 80.281U
.ιιιυο7
8.47 2.910,33
71.9104 2.01204
.118064 8.07 80,4009 2.99833 0,48158
8.48 .1Ι7θ2δ .1Ι1483
72.080Ι 2.91370
.Ι1778β
8.98 80.6404 3,00000 0.48083 .1Ι1359
8.40 8.99 80.8201
?2.2500 2.01548 .11Ι235
8.50 .Ι17047 9.00 81.οοω

190
XV—

TABLE SQUARBS — ROOTS —RECIPROCAI,S

1/η η

9.00 81.000-0 3.οοοοο 9.48083 .110θ88 9.50 90.2500 3.08221 9.74679 .1052β3
0.01 81.1801 3.001θ7 ρ.4Ω210 .1108β5 0.51 90.4401 3.08383 9.75192 .105152
81.3604 9.40737
9.02 3.00833 9.50263 .110742 0.52 90.6304 ,3.08645 0.75705 .105042
9.03 81.5409 3.00500 9.50789 .110βΙ9 0.53 90.8209 3.08707 0.70217 .104θ32
9.04 81.7216 3.00βθβ θ.7β72θ
.110497 9.64 91.0110 3.088β9 .104822
81.9025 9.51315 9.77241
9.05 3.00832 9.61840 .110375 0.56 91.2025 3.09031 .104712
9.06 82.083θ .110254 9.50 91.3930 0.77763 .104β0.3
3.00998 0.52365 3.00192
9.07 82.2640 01.5849 o. 78264 .104493
3.01164 9.52890 .110132 9.57 3.0Ω364
0.78775
82.4-464 91.7764 .104384
9.08
9.0θ 82.6281
3.01330 0.53415 .ιιοοιι 9.58
θ.δθ θ1.0β81
3.0051β
3.09β77
0.79286 .104275
8.0149β .10θ890
82.8100 0.53939 0.79790
0.10 3.0Ι6β2 0.54403 .1097θ9 o.ao 02.1600 3.09839
9.80306
.104107
9.11 82.9921 9.01 02.3521 3.10000 .104068
3.01828 9.54987 .109049 9.62 9.80810
9.12 83.1744 92.5444 8.10Ιβ1 .108260
3.01993 9.55510 .109520 ρ.β8 0.81320
9.18 83.35β9 92.7969 3.10322 .103842
8.02159 9,56033 • 0.04 0.81885
9.14 83.6896 θ2.θ2θβ 3.10483 .108784
3.02324 109409 .
83.7225 9.56556 9.05 93.1225 3.10044 .103627
9.15 8.02490 9.57079 .1092θ0 2.06 0.82844
θ. Ιθ 88.9066 93.3156 8.10806 .108520
3.02β55 -0.57001 .100170 9.07 9.82863
0.17 84.0889 03.6080 3.10966 .103413
3.02820 θ.58123 -109051 0.68 0.83302
9.18 84.2724 93.7024 3.11127 .10330β
3.02985 9.58645 .108932 9.83870
191
XV— —SQUARE

9.19 84.4581 3.031δ0 9.501ββ .Ι08814 9.69 93.8901 3.11288 0.84378 .1031θ9
0.20 84.6400 3.03315 ρ.6θβ87 .108β9β 9.70 94.0900 3.11448 0.84886 .1030θ3
9.60208
0.21 84.8241 8.03480 108578 9.71 94.2841 3.11600 9.85393 .102087
9.60729
9.22 85.0084 3.03645 :ιοκοο 9.72 94.4784 3.11760 0.86001 •102881
0.61249
9.23 85.1929 3.038ορ .108342 9.78 94.6729 3.11920 9.86408 .10277δ
9.24 85.8776 3.03974 9.617βθ .10822δ 9.74 94.8β7θ 3.12090 9.86014 .1026βθ
9.62289
9.26 85.5625 3.04188 .108108 0.75 05.0026 3.12250 9.87421 .1025θ4
9.02808
9.20 85.7476 3.04302 .1079θ1 9.76 06.2576 3.12410 9.87927 .Ι0245θ
9.θ3328
0.27 85.9329 3.044β7 .107875 9,77 95.4520 3.12570 9.88438 .102864
9.83840
9.28 86.1184 304631 .10775θ 9.78 95.6484 3.12730 0.88939 .102240
9.29 80.8041 8:04795 9.64365 .1Ο7843 9.79 95.8441 3.12890 0.89444 .Ι02145
9.64883
0.80 86.4900 8.049δ9 9.65401 .107δ27 0.80 90.0400 3.13050 9.89949. .102041
9.31 8β.β7β1 3.05128 985919 .107411 9.81 96.2361 3.13209 0.90454 .101037
9.32 86.8624 3.05287 θ.β&48 .107296 9.82 96.4324 3. Ι33βθ 9.00059 .101833
0.83 87.0489 3,05450 7 θθ. 0289 3.13528 9.01404 .10172D
.107181 9.88
9.34 87.2358 3.05614 96.8250 3.18688 0,01008 .101β2β
.1070ββ 2.84
0.68964 .101523
9.85 87.4225 3.0δ778
9.67471 -100952 0.85 97.0225 3.13847 9.92472
9.36 87.6090 3,05941 Ω.80 97.210θ 9.14006 θ.Ω2Ω75 .101420
9.67988 .10β838 0.98470
9.37 87.7θβθ 3.08105 9.68504 9,87 97.41βΩ 3.14160 .101317
-106724 9.93082
0.88 97.6144
9.38 87.9844 3.06268 0.69020 .100β10 3.14326 Ω.θΜ8δ .101215
9.39 88.1721 3.06431 9.89 97.8121 3.14484 .101ΙΙ2
.10849β
ρ.0953θ 9.90 98.0100 0.94987
0.40 88.3800 3.06594 9.70052 .100383 9.01 98.2081
8.14643 9.θ64θΟ .ιοιοιο
9.41 88.5481 3.00757 9.70567 .100270 3.14802 0.95002 .100908
9.42 88.7364 3.00920 9.71082 0.02 08.4064
.100157 0.08 08.
3.14060 0.08494 .10080σ
0.48 88.9249 3.07083 0.71597 .106045 3,15119 9.90095 .100705
9.44 89.1136 3.072-46 9.04 98.8030 3.16278 .100604
0,72111 .106θ32 0.97497
9.45 89.3026 3.07409 9.72626 0.95 99,0025 3.16430
.105820 θ.9β 99.2010 θ.θ79θ8 .Ι00603
9.46 89.4010 3.07571 9.73189 .105708 8,15505 9.98490 .100402
9.47 89.6809 3.07784 9.73β53 θ.θ7 99.4009
.1055θ7 3.15758 θ.98θθθ ,100301
9.08 90.β004
9.48 80.8704 3.078θβ 9.741θβ 405485 3.15911 9.99500 .100200
9.09 90.8001
9.49 90,0601 8.08058 .106874 8.16070 .ιοοιοο
9.74679 10.00 100.000 10.0000
0.50 90.2500 8.08221 ,1052β3 3.16228 .100000

η 1/η 1/η

13841 Ιδώ

192

You might also like